Вы находитесь на странице: 1из 245

MATABABE FILES

Corporation Law

Republic of the Philippines


SUPREME COURT 1. That prior to the presentation of the petition the petitioners had been associated together
Manila as partners, which partnership was known as "mercantil regular colectiva, under the style and
EN BANC firm name of "Siuliong y Cia.;"

G.R. No. L-15429 December 1, 1919 2. That the petitioners herein, who had theretofore been members of said partnership of
"Siuliong y Cia.," desired to dissolve said partnership and to form a corporation composed of
UY SIULIONG, MARIANO LIMJAP, GACU UNG JIENG, EDILBERTO CALIXTO and UY CHO the same persons as incorporators, to be known as "Siulong y Compaia, Incorporada;"
YEE, petitioners,
vs. 3. That the purpose of said corporation, "Siuliong y Cia., Inc.," is (a) to acquire the business of
THE DIRECTOR OF COMMERCE AND INDUSTRY, respondent. the partnership theretofore known as Siuliong & Co., and (b) to continue said business with
some of its objects or purposes;
MANDAMUS TO REQUIRE THE DIRECTOR OF COMMERCE AND INDUSTRY TO FILE AND
REGISTER ARTICLES OF INCORPORATION UPON PAYMENT OF THE LAWFUL FEES.Held: That 4. That an examination of the articles of incorporation of the said "Siuliong y Compaia,
under the laws of the Philippine Islands, a corporation may be organized for "mercantile Incorporada" (Exhibit A) shows that it is to be organized for the following purposes:
purposes" and to engage in such incidental business as may be necessary and advisable to
give effect to, and aid in, the successful operation and conduct of the principal business; that (a) The purchase and sale, importation and exportation, of the products of the
all of the power and authority included in the articles of incorporation of Siuliong & Co., Inc., country as well as of foreign countries;
were only incidental to the' principal purpose of its proposed incorporation, to wit:
"mercantile business." (b) To discount promissory notes, bills of exchange, and other negotiable
instruments;
ORIGINAL ACTION in the Supreme Court. Mandamus.
(c) The purchase and sale of bills of exchange, bonds, stocks, or "participaciones de
The facts are stated in the opinion of the court. sociedades mercantiles e industriales [joint account of mercantile and industrial
Kincaid and Perkins for petitioners. associations]," and of all classes of mercantile documents; "comisiones
Attorney-General Paredes for respondent. [commissions];" "consignaciones [consignments];"

JOHNSON, J.: (d) To act as agents for life, marine and fire insurance companies;

The purpose of this action is to obtain the writ of mandamus to require the (e) To purchase and sell boats of all classes "y fletamento de los mismos [and
respondent to file and register, upon the payment of the lawful fee, articles of incorporation, charterage of same];" and
and to issue to the petitioners as the incorporators of a certain corporation to be known as
"Siuliong y Compaia, Inc.," a certificate under the seal of the office of said respondent, (f) To purchase and sell industrial and mercantile establishments.
certifying that the articles of incorporation have been duly filed and registered in his office in
accordance with the law. While the articles of incorporation of "Siuliong y Cia., Inc." states that its purpose is to
acquire and continue the business, with some of its objects or purposes, of Siuliong & Co., it
To the petition the respondent demurred and the cause was finally submitted upon will be found upon an examination of the purposes enumerated in the proposed articles of
the petition and demurrer. incorporation of "Siuliong y Cia., Inc.," that some of the purposes of the original partnership
of "Siuliong y Cia." have been omitted. For example, the articles of partnership of "Siuliong y
The important facts necessary for the solution of the question presented, which are Cia." gave said company the authority to purchase and sell all classes "de fincas rusticas y
found in the petition, may be stated as follows: urbanas [of rural and city real estate]" as well as the right to act as agents for the

1
MATABABE FILES
Corporation Law

establishment of any other business which it might esteem convenient for the interests of "la Without discussing or deciding at this time whether a corporation organized under
compaia [the company]." (Exhibit C). the laws of the Philippine Islands may be organized for more than one purpose, we are of the
opinion and so decide that a corporation may be organized under the laws of the Philippine
The respondent in his argument in support of the demurrer contends (a) that the Islands for mercantile purposes, and to engage in such incidental business as may be
proposed articles of incorporation presented for file and registry permitted the petitioners to necessary and advisable to give effect to, and aid in, the successful operation and conduct of the
engage in a business which had for its end more than one purpose; (b) that it permitted the principal business.
petitioners to engage in the banking business, and (c) to deal in real estate, in violation of the
Act of Congress of July 1, 1902. In the present case we are fully persuaded that all of the power and authority included
in the articles of incorporation of "Siuliong y Cia., Inc.," enumerated above in paragraph 4
The petitioners, in reply to said argument of the respondent, while insisting that said (Exhibit A) are only incidental to the principal purpose of said proposed incorporation, to
proposed articles of incorporation do not permit it to enter into the banking business nor to wit: "mercantile business." The purchase and sale, importation and exportation of the
engage in the purchase and sale of real estate in violation of said Act of Congress, expressly products of the country, as well as of foreign countries, might make it necessary to purchase
renounced in open court their right to engage in such business under their articles of and discount promissory notes, bills of exchange, bonds, negotiable instruments, stock, and
incorporation, even though said articles might be interpreted in a way to authorize them to so interest in other mercantile and industrial associations. It might also become important and
to do. That renouncement on the part of the petitioners eliminates from the purposes of said advisable for the successful operation of the corporation to act as agent for insurance
proposed corporation (of "Siuliong y Cia., Inc.") any right to engage in the banking business as companies as well as to buy, sell and equip boats and to buy and sell other establishments,
such, or in the purchase and sale of real estate. and industrial and mercantile businesses.

We come now to the consideration of the principal question raised by the While we have arrived at the conclusion that the proposed articles of incorporation
respondent, to wit: that the proposed articles of incorporation of "Siuliong y Cia., Inc.," do not authorize the petitioners to engage in a business with more than one purpose, we do
permits it to engage in a business with more than one purpose. not mean to be understood as having decided that corporations under the laws of the
Philippine Islands may not engage in a business with more than one purpose. Such an
If upon an examination of the articles of incorporation we find that its purpose is to interpretation might work a great injustice to corporations organized under the Philippine
engage in a business with but one principal purpose, then that contention of the respondent laws. Such an interpretation would give foreign corporations, which are permitted to be
will have been answered and it will be unnecessary to discuss at length the question whether registered under the laws here and which may be organized for more than one purpose, a
or not a corporation organized for commercial purposes in the Philippine Islands can be great advantage over domestic corporations. We do not believe that it was the intention of
organized for more than one purpose. the legislature to give foreign corporations such an advantage over domestic corporations.

The attorney for the respondent, at the time of the argument, admitted in open court Considering the particular purposes and objects of the proposed articles of
that corporations in the Philippine Islands might be organized for both the "importation and incorporation which are specially enumerated above, we are of the opinion that it contains
exportation" of merchandise and that there might be no relation between the kind of nothing which violates in the slightest degree any of the provisions of the laws of the
merchandise imported with the class of merchandise exported. Philippine Islands, and the petitioners are, therefore, entitled to have such articles of
incorporation filed and registered as prayed for by them and to have issued to them
Referring again to be proposed articles of incorporation, a copy of which is united a certificate under the seal of the office of the respondent, setting forth that such articles of
with the original petition and marked Exhibit A, it will be seen that the only purpose of said incorporation have been duly filed in his office. (Sec. 11, Act No. 1459.)
corporation are those enumerated in subparagraphs (a), (b), (c), (d), (e) and ( f ) of paragraph
4 above. While said articles of incorporation are somewhat loosely drawn, it is clear from a Therefore, the petition prayed for is hereby granted, and without any finding as to
reading of the same that the principal purpose of said corporation is to engage in a mercantile costs, it is so ordered.
business, with the power to do and perform the particular acts enumerated in said
subparagraphs above referred to. Arellano, C.J., Torres and Avancea, JJ., concur.

2
MATABABE FILES
Corporation Law

Republic of the Philippines Petitioner Alhambra Cigar and Cigarette Manufacturing Company, Inc. (hereinafter
SUPREME COURT referred to simply as Alhambra) was duly incorporated under Philippine laws on January 15,
Manila 1912. By its corporate articles it was to exist for fifty (50) years from incorporation. Its term of
EN BANC existence expired on January 15, 1962. On that date, it ceased transacting business, entered
into a state of liquidation.
G.R. No. L-23606 July 29, 1968
Thereafter, a new corporation. Alhambra Industries, Inc. was formed to carry
ALHAMBRA CIGAR & CIGARETTE MANUFACTURING COMPANY, INC., petitioner, on the business of Alhambra.
vs.
SECURITIES & EXCHANGE COMMISSION, respondent. On May 1, 1962, Alhambra's stockholders, by resolution named Angel S. Gamboa
trustee to take charge of its liquidation.
Corporation law; Term of existence; Amendment of articles of incorporation after
expiration of its corporate life.A corporation cannot extend its life by amendment of its On June 20, 1963 within Alhambra's three-year statutory period for liquidation -
articles of incorporation effected during the three-year statutory period for liquidation when Republic Act 3531 was enacted into law. It amended Section 18 of the Corporation Law; it
its original term of existence had already expired. empowered domestic private corporations to extend their corporate life beyond the period
fixed by the articles of incorporation for a term not to exceed fifty years in any one instance.
Since the privilege of extension is purely statutory, all of the statutory conditions Previous to Republic Act 3531, the maximum non-extendible term of such corporations was
precedent must be complied with in order that the extension may be effectuated. And, fifty years.
generally, these conditions must be complied with, and the steps necessary to effect the
extension must be taken, during the life of the corporation, and before the expiration of its On July 15, 1963, at a special meeting, Alhambra's board of directors resolved to
term of existence as originally fixed by its charter or the general law, since, as a rule, the amend paragraph "Fourth" of its articles of incorporation to extend its corporate life for an
corporation is ipso facto dissolved as soon as that time expires (8 Fletcher, Cyclopedia of additional fifty years, or a total of 100 years from its incorporation.
Corporations, Perm. ed., 1931, pp. 559-560).
On August 26, 1963, Alhambra's stockholders, representing more than two-thirds of
REVIEW of a ruling of the Securities and Exchange Commission. its subscribed capital stock, voted to approve the foregoing resolution. The "Fourth"
paragraph of Alhambra's articles of incorporation was thus altered to read:
The facts are stated in the opinion of the Court
FOURTH. That the term for which said corporation is to exist is fifty (50) years from
Gamboa and Gamboa for petitioner. and after the date of incorporation, and for an additional period of fifty (50) years
Office of the Solicitor General for respondent. thereafter.

SANCHEZ, J.: On October 28, 1963, Alhambra's articles of incorporation as so amended certified
correct by its president and secretary and a majority of its board of directors, were filed with
To the question May a corporation extend its life by amendment of its articles of respondent Securities and Exchange Commission (SEC).
incorporation effected during the three-year statutory period for liquidation when its original
term of existence had already expired? the answer of the Securities and Exchange On November 18, 1963, SEC, however, returned said amended articles of
Commissioner was in the negative. Offshoot is this appeal. incorporation to Alhambra's counsel with the ruling that Republic Act 3531 "which took effect
only on June 20, 1963, cannot be availed of by the said corporation, for the reason that its
That problem emerged out of the following controlling facts: term of existence had already expired when the said law took effect in short, said law has no
retroactive effect."

3
MATABABE FILES
Corporation Law

On December 3, 1963, Alhambra's counsel sought reconsideration of SEC's ruling and close its affairs, to dispose of and convey its property and to divide its capital stock,
aforesaid, refiled the amended articles of incorporation. but not for the purpose of continuing the business for which it was established.2

On September 8, 1964, SEC, after a conference hearing, issued an order denying the Plain from the language of the provision is its meaning: continuance of a "dissolved"
reconsideration sought. corporation as a body corporate for three years has for its purpose the final closure of its
affairs, and no other; the corporation is specifically enjoined from "continuing the business for
Alhambra now invokes the jurisdiction of this Court to overturn the conclusion which it was established". The liquidation of the corporation's affairs set forth in Section 77
below.1 became necessary precisely because its life had ended. For this reason alone, the corporate
existence and juridical personality of that corporation to do business may no longer be
1. Alhambra relies on Republic Act 3531, which amended Section 18 of the Corporation extended.
Law. Well it is to take note of the old and the new statutes as they are framed. Section 18,
prior to and after its modification by Republic Act 3531, covers the subject of amendment of Worth bearing in mind, at this juncture, is the basic development of corporation law.
the articles of incorporation of private corporations. A provision thereof which remains
unaltered is that a corporation may amend its articles of incorporation "by a majority vote of The common law rule, at the beginning, was rigid and inflexible in that upon its
its board of directors or trustees and ... by the vote or written assent of the stockholders dissolution, a corporation became legally dead for all purposes. Statutory authorizations had
representing at least two-thirds of the subscribed capital stock ... " to be provided for its continuance after dissolution "for limited and specified purposes
incident to complete liquidation of its affairs".3 Thus, the moment a corporation's right to
But prior to amendment by Republic Act 3531, an explicit prohibition existed in exist as an "artificial person" ceases, its corporate powers are terminated "just as the powers
Section 18, thus: of a natural person to take part in mundane affairs cease to exist upon his death". 4 There is
nothing left but to conduct, as it were, the settlement of the estate of a deceased juridical
... Provided, however, That the life of said corporation shall not be extended by said person.
amendment beyond the time fixed in the original articles: ...
2. Republic Act 3531, amending Section 18 of the Corporation Law, is silent, it is true,
This was displaced by Republic Act 3531 which enfranchises all private corporations to as to when such act of extension may be made. But even with a superficial knowledge of
extend their corporate existence. Thus incorporated into the structure of Section 18 are the corporate principles, it does not take much effort to reach a correct conclusion. For, implicit in
following: Section 77 heretofore quoted is that the privilege given to prolong corporate life under the
amendment must be exercised before the expiry of the term fixed in the articles of
... Provided, however, That should the amendment consist in extending the corporate incorporation.
life, the extension shall not exceed fifty years in any one instance: Provided, further, Silence of the law on the matter is not hard to understand. Specificity is not really necessary.
That the original articles, and amended articles together shall contain all provisions The authority to prolong corporate life was inserted by Republic Act 3531 into a section of the
required by law to be set out in the articles of incorporation: ... law that deals with the power of a corporation to amend its articles of incorporation. (For, the
As we look in retrospect at the facts, we find these: From July 15 to October 28, 1963, manner of prolongation is through an amendment of the articles.) And it should be clearly
when Alhambra made its attempt to extend its corporate existence, its original term of fifty evident that under Section 77 no corporation in a state of liquidation can act in any way, much
years had already expired (January 15, 1962); it was in the midst of the three-year grace period less amend its articles, "for the purpose of continuing the business for which it was
statutorily fixed in Section 77 of the Corporation Law, thus: established".
All these dilute Alhambra's position that it could revivify its corporate life simply
SEC. 77. Every corporation whose charter expires by its own limitation or is annulled because when it attempted to do so, Alhambra was still in the process of liquidation. It is
by forfeiture or otherwise, or whose corporate existence for other purposes is surely impermissible for us to stretch the law that merely empowers a corporation to act in
terminated in any other manner, shall nevertheless be continued as a body corporate liquidation to inject therein the power to extend its corporate existence.
for three years after the time when it would have been so dissolved, for the purpose
of prosecuting and defending suits by or against it and of enabling it gradually to settle

4
MATABABE FILES
Corporation Law

3. Not that we are alone in this view. Fletcher has written: "Since the privilege of True it is, that the Alabama Supreme Court has stated in one case. 8 that a corporation
extension is purely statutory, all of the statutory conditions precedent must be complied with empowered by statute to renew its corporate existence may do so even after the expiration
in order that the extension may be effectuated. And, generally these conditions must be of its corporate life, provided renewal is taken advantage of within the extended statutory
complied with, and the steps necessary to effect the extension must be taken, during the life period for purposes of liquidation. That ruling, however, is inherently weak as persuasive
of the corporation, and before the expiration of the term of existence as original fixed by its authority for the situation at bar for at least two reasons: First. That case was a suit for
charter or the general law, since, as a rule, the corporation is ipso facto dissolved as soon as that mandamus to compel a former corporate officer to turn over books and records that came
time expires. So where the extension is by amendment of the articles of incorporation, the into his possession and control by virtue of his office. It was there held that such officer was
amendment must be adopted before that time. And, similarly, the filing and recording of a obliged to surrender his books and records even if the corporation had already expired. The
certificate of extension after that time cannot relate back to the date of the passage of a holding on the continued existence of the corporation was a mere dictum. Second. Alabama's
resolution by the stockholders in favor of the extension so as to save the life of the law is different. Corporations in that state were authorized not only to extend but also
corporation. The contrary is true, however, and the doctrine of relation will apply, where the to renew their corporate existence.That very case defined the word "renew" as follows; "To
delay is due to the neglect of the officer with whom the certificate is required to be filed, or to make new again; to restore to freshness; to make new spiritually; to regenerate; to begin
a wrongful refusal on his part to receive it. And statutes in some states specifically provide again; to recommence; to resume; to restore to existence, to revive; to re-establish; to
that a renewal may be had within a specified time before or after the time fixed for the recreate; to replace; to grant or obtain an extension of Webster's New International Dict.; 34
termination of the corporate existence".5 Cyc. 1330; Carter v. Brooklyn Life Ins. Co., 110 N.Y. 15, 21, 22, 17 N.E. 396; 54 C.J. 379. Sec".9

The logic of this position is well expressed in a foursquare case decided by the Court On this point, we again draw from Fletcher: "There is a broad distinction between the
of Appeals of Kentucky.6There, pronouncement was made as follows: extension of a charter and the grant of a new one. To renew a charter is to revive a charter
... But section 561 (section 2147) provides that, when any corporation expires by the which has expired, or, in other words, "to give a new existence to one which has been
terms of its articles of incorporation, it may be thereafter continued to act for the forfeited, or which has lost its vitality by lapse of time". To "extend" a charter is "to increase
purpose of closing up its business, but for no other purpose. The corporate life of the the time for the existence of one which would otherwise reach its limit at an earlier
Home Building Association expired on May 3, 1905. After that date, by the mandate period".10Nowhere in our statute Section 18, Corporation Law, as amended by Republic Act
of the statute, it could continue to act for the purpose of closing up its business, but 3531 do we find the word "renew" in reference to the authority given to corporations to
for no other purpose. The proposed amendment was not made until January 16, protract their lives. Our law limits itself to extension of corporate existence. And, as so
1908, or nearly three years after the corporation expired by the terms of the articles understood, extension may be made only before the term provided in the corporate charter
of incorporation. When the corporate life of the corporation was ended, there was expires.
nothing to extend. Here it was proposed nearly three years after the corporate life of
the association had expired to revivify the dead body, and to make that relate back Alhambra draws attention to another case11 which declares that until the end of the
some two years and eight months. In other words, the association for two years and extended period for liquidation, a dissolved corporation "does not become an extinguished
eight months had only existed for the purpose of winding up its business, and, after entity". But this statement was obviously lifted out of context. That case dissected the
this length of time, it was proposed to revivify it and make it a live corporation for the question whether or not suits can be commenced by or against a corporation within its
two years and eight months daring which it had not been such. liquidation period. Which was answered in the affirmative. For, the corporation still exists for
the settlement of its affairs.
The law gives a certain length of time for the filing of records in this court,
and provides that the time may be extended by the court, but under this provision it People, ex rel. vs. Green,12 also invoked by Alhambra, is as unavailing. There, although
has uniformly been held that when the time was expired, there is nothing to extend, the corporation amended its articles to extend its existence at a time when it had no legal
and that the appeal must be dismissed... So, when the articles of a corporation have authority yet, it adopted the amended articles later on when it had the power to extend
expired, it is too late to adopt an amendment extending the life of a corporation; for, its life and during its original term when it could amend its articles.
the corporation having expired, this is in effect to create a new corporation ..."7
The foregoing notwithstanding, Alhambra falls back on the contention that its case is
arguably within the purview of the law. It says that before cessation of its corporate life, it

5
MATABABE FILES
Corporation Law

could not have extended the same, for the simple reason that Republic Act 3531 had not then conceptual scheme under which corporations operate, and with Section 77 of the Corporation
become law. It must be remembered that Republic Act 3531 took effect on June 20, 1963, Law particularly in mind, we find no vagueness in Section 18, as amended by Republic Act
while the original term of Alhambra's existence expired before that date on January 15, 3531. As we view it, by directing attention to Republic Act 1932, Alhambra would seek to
1962. The mischief that flows from this theory is at once apparent. It would certainly open the create obscurity in the law; and, with that, ask of us a ruling that such obscurity be explained.
gates for all defunct corporations whose charters have expired even long before Republic This, we dare say, cannot be done.
Act 3531 came into being to resuscitate their corporate existence.
The pari materia rule of statutory construction, in fact, commands that statutes must
4. Alhambra brings into argument Republic Act 1932, which amends Section 196 of the be harmonized with each other.14 So harmonizing, the conclusion is clear that Section 18 of
Insurance Act, now reading as follows: the Corporation Law, as amended by Republic Act 3531 in reference to extensions of
corporate existence, is to be read in the same light as Republic Act 1932. Which means that
SEC. 196. Any provision of law to the contrary notwithstanding, every domestic life domestic corporations in general, as with domestic insurance companies, can extend
insurance corporation, formed for a limited period under the provisions of its articles corporate existence only on or before the expiration of the term fixed in their charters.
of incorporation, may extend its corporate existence for a period not exceeding fifty
years in any one instance by amendment to its articles of incorporation on or before 5. Alhambra pleads for munificence in interpretation, one which brushes
the expiration of the term so fixed in said articles ... technicalities aside. Bases for this posture are that Republic Act 3531 is a remedial statute, and
that extension of corporate life is beneficial to the economy.
To be observed is that the foregoing statute unlike Republic Act 3531 expressly
authorizes domestic insurance corporations to extend their corporate existence "on or Alhambra's stance does not induce assent. Expansive construction is possible only
before the expiration of the term" fixed in their articles of incorporation. Republic Act 1932 when there is something to expand. At the time of the passage of Republic Act 3531,
was approved on June 22, 1957, long before the passage of Republic Act 3531 in 1963. Alhambra's corporate life had already expired. It had overstepped the limits of its limited
Congress, Alhambra points out, must have been aware of Republic Act 1932 when it passed existence. No life there is to prolong.
Republic Act 3531. Since the phrase "on or before", etc., was omitted in Republic Act 3531, Besides, a new corporation Alhambra Industries, Inc., with but slight change in
which contains no similar limitation, it follows, according to Alhambra, that it is not necessary stockholdings15 has already been established. Its purpose is to carry on, and it actually does
to extend corporate existence on or before the expiration of its original term. carry on,16 the business of the dissolved entity. The beneficial-effects argument is off the
mark.
That Republic Act 3531 stands mute as to when extention of corporate existence may
be made, assumes no relevance. We have already said, in the face of a familiar precept, that a The way the whole case shapes up then, the only possible drawbacks of Alhambra
defunct corporation is bereft of any legal faculty not otherwise expressly sanctioned by law. might be that, instead of the new corporation (Alhambra Industries, Inc.) being written off,
the old one (Alhambra Cigar & Cigarette Manufacturing Company, Inc.) has to be wound up;
Illuminating here is the explanatory note of H.B. 1774, later Republic Act 3531 now and that the old corporate name cannot be retained fully in its exact form. 17 What is important
in dispute. Its first paragraph states that "Republic Act No. 1932 allows the automatic though is that the word Alhambra, the name that counts [it has goodwill], remains.
extension of the corporate existence of domestic life insurance corporations upon
amendment of their articles of incorporation on or before the expiration of the terms fixed by FOR THE REASONS GIVEN, the ruling of the Securities and Exchange Commission of
said articles". The succeeding lines are decisive: "This is a good law, a sane and sound November 18, 1963, and its order of September 8, 1964, both here under review, are hereby
one. There appears to be no valid reason why it should not be made to apply to other private affirmed.
corporations.13
Costs against petitioner Alhambra Cigar & Cigarette Manufacturing Company, Inc. So
The situation here presented is not one where the law under consideration is ordered.
ambiguous, where courts have to put in harness extrinsic aids such as a look at another
statute to disentangle doubts. It is an elementary rule in legal hermeneutics that where the Concepcion, C.J., Reyes, J.B.L., Dizon, Makalintal, Zaldivar, Castro, Angeles and Fernando, JJ.,
terms of the law are clear, no statutory construction may be permitted. Upon the basic concur.

6
MATABABE FILES
Corporation Law

Republic of the Philippines


SUPREME COURT This is an appeal from an order of the Court of First Instance of Misamis Oriental
Manila dismissing the petition of the Clavecilla Radio System to prohibit the City Judge of Cagayan de
EN BANC Oro from taking cognizance of Civil Case No. 1048 for damages.

G.R. No. L-22238 February 18, 1967 It appears that on June 22, 1963, the New Cagayan Grocery filed a complaint against
the Clavecilla Radio System alleging, in effect, that on March 12, 1963, the following message,
CLAVECILLIA RADIO SYSTEM, petitioner-appellant, addressed to the former, was filed at the latter's Bacolod Branch Office for transmittal thru its
vs. branch office at Cagayan de Oro:
HON. AGUSTIN ANTILLON, as City Judge of the Municipal Court of Cagayan de Oro City
and NEW CAGAYAN GROCERY, respondents-appellees. NECAGRO CAGAYAN DE ORO (CLAVECILLA)
REURTEL WASHED NOT AVAILABLE REFINED TWENTY FIFTY IF AGREEABLE SHALL
Corporation Law; Domicile of a corporation.The residence of a corporation is the SHIP LATER REPLY POHANG
place where its principal office is established. It can be sued in that place, not in the place
where its branch office is located. The Cagayan de Oro branch office having received the said message omitted,
in delivering the same to the New Cagayan Grocery, the word "NOT" between the
Actions; Venue; Venue of a tort action against a, corporation in inferior court.Where words "WASHED" and "AVAILABLE," thus changing entirely the contents and purport
the action filed against a corporation in the inferior court is based on tort, it should be filed in of the same and causing the said addressee to suffer damages. After service of
the place where the corporation has its principal office, not in the place where it has its summons, the Clavecilla Radio System filed a motion to dismiss the complaint on the
branch office. To allow an action against a corporation to be instituted in any place where a grounds that it states no cause of action and that the venue is improperly laid. The
corporate entity has its branch offices would create confusion and work untold inconvenience New Cagayan Grocery interposed an opposition to which the Clavecilla Radio System
to the corporation. filed its rejoinder. Thereafter, the City Judge, on September 18, 1963, denied the
motion to dismiss for lack of merit and set the case for hearing.
Same; When provision, may be served with summons, applies.The phrase may
be served with summons in section 1, Rule 4 of the Revised Rules of Court does not apply Hence, the Clavecilla Radio System filed a petition for prohibition with preliminary
when the defendant resides in the Philippines, for, in such a case, he may be sued only in the injunction with the Court of First Instance praying that the City Judge, Honorable Agustin
municipality of his residence, regardless of the place where he may be found and served with Antillon, be enjoined from further proceeding with the case on the ground of improper venue.
summons. The respondents filed a motion to dismiss the petition but this was opposed by the petitioner.
Later, the motion was submitted for resolution on the pleadings.
Same; Plaintiff may not choose venue of action.The laying of the venue of an action
is not left to plaintiffs caprice because the matter is regulated by the Rules of Court. In dismissing the case, the lower court held that the Clavecilla Radio System may be
sued either in Manila where it has its principal office or in Cagayan de Oro City where it may be
APPEAL from an order of dismissal rendered by the Court of First Instance of Misamis served, as in fact it was served, with summons through the Manager of its branch office in
Oriental. said city. In other words, the court upheld the authority of the city court to take cognizance of
the case.
The facts are stated in the opinion of the Court
In appealing, the Clavecilla Radio System contends that the suit against it should be
B. C. Padua for petitioner and appellant. filed in Manila where it holds its principal office.
Pablo S. Reyes for respondents and appellees.
It is clear that the case for damages filed with the city court is based upon tort and
REGALA, J.: not upon a written contract. Section 1 of Rule 4 of the New Rules of Court, governing venue

7
MATABABE FILES
Corporation Law

of actions in inferior courts, provides in its paragraph (b) (3) that when "the action is not upon
a written contract, then in the municipality where the defendant or any of the defendants
resides or may be served with summons." (Emphasis supplied)

Settled is the principle in corporation law that the residence of a corporation is the
place where its principal office is established. Since it is not disputed that the Clavecilla Radio
System has its principal office in Manila, it follows that the suit against it may properly be filed
in the City of Manila.

The appellee maintain, however, that with the filing of the action in Cagayan de Oro
City, venue was properly laid on the principle that the appellant may also be served with
summons in that city where it maintains a branch office. This Court has already held in the
case of Cohen vs. Benguet Commercial Co., Ltd., 34 Phil. 526; that the term "may be served with
summons" does not apply when the defendant resides in the Philippines for, in such case, he
may be sued only in the municipality of his residence, regardless of the place where he may be
found and served with summons. As any other corporation, the Clavecilla Radio System
maintains a residence which is Manila in this case, and a person can have only one residence
at a time (See Alcantara vs. Secretary of the Interior, 61 Phil. 459; Evangelists vs. Santos, 86
Phil. 387). The fact that it maintains branch offices in some parts of the country does not
mean that it can be sued in any of these places. To allow an action to be instituted in any place
where a corporate entity has its branch offices would create confusion and work untold
inconvenience to the corporation.

It is important to remember, as was stated by this Court in Evangelista vs. Santos, et


al., supra, that the laying of the venue of an action is not left to plaintiff's caprice because the
matter is regulated by the Rules of Court. Applying the provision of the Rules of Court, the
venue in this case was improperly laid.

The order appealed from is therefore reversed, but without prejudice to the filing of
the action in Which the venue shall be laid properly. With costs against the respondents-
appellees.

Concepcion, C.J., Reyes, J.B.L., Dizon, Makalintal, Bengzon, J.P., Zaldivar, Sanchez and
Castro, JJ., concur.

8
MATABABE FILES
Corporation Law

Republic of the Philippines Petitioners seek to set aside the decision of respondent Court of Appeals in CA-G.R.
SUPREME COURT SP No. 25237, which reversed the Order dated February 8, 1991 issued by the Regional Trial
Manila Court, Branch 11, Cebu City in Civil Case No. CEB 6967. The order of the trial court denied the
FIRST DIVISION motion to dismiss filed by respondent George C. Roxas of the complaint for collection filed by
petitioners.
G.R. No. 104175 June 25, 1993
It appears that sometime on October 28, 1987, Young Auto Supply Co. Inc. (YASCO)
YOUNG AUTO SUPPLY CO. AND NEMESIO GARCIA, petitioners, represented by Nemesio Garcia, its president, Nelson Garcia and Vicente Sy, sold all of their
vs. shares of stock in Consolidated Marketing & Development Corporation (CMDC) to Roxas. The
THE HONORABLE COURT OF APPEALS (THIRTEENTH DIVISION) AND GEORGE CHIONG purchase price was P8,000,000.00 payable as follows: a downpayment of P4,000,000.00 and
ROXAS, respondents. the balance of P4,000,000.00 in four post dated checks of P1,000,000.00 each.

Remedial Law; Actions; Venue; A corporation is in a metaphysical sense a resident of Immediately after the execution of the agreement, Roxas took full control of the four
the place where its principal office is located as stated in the articles of incorporation.A markets of CMDC. However, the vendors held on to the stock certificates of CMDC as security
corporation has no residence in the same sense in which this term is applied to a natural pending full payment of the balance of the purchase price.
person. But for practical purposes, a corporation is in a metaphysical sense a resident of the
place where its principal office is located as stated in the articles of incorporation (Cohen v. The first check of P4,000,000.00, representing the down-payment, was honored by
Benguet Commercial Co., Ltd., 34 Phil. 526 [1916] Clavecilla Radio System v. Antillon, 19 SCRA the drawee bank but the four other checks representing the balance of P4,000,000.00 were
379 [1967]). The Corporation Code precisely requires each corporation to specify in its articles dishonored. In the meantime, Roxas sold one of the markets to a third party. Out of the
of incorporation the place where the principal office of the corporation is to be located proceeds of the sale, YASCO received P600,000.00, leaving a balance of P3,400,000.00 (Rollo,
which must be within the Philippines (Sec. 14 [3]). The purpose of this requirement is to fix p. 176).
the residence of a corporation in a definite place, instead of allowing it to be ambulatory.
Subsequently, Nelson Garcia and Vicente Sy assigned all their rights and title to the
Same; Same; Same; A corporation cannot be allowed to file personal actions in a place proceeds of the sale of the CMDC shares to Nemesio Garcia.
other than its principal place of business unless such a place is also the residence of a co-
plaintiff or a defendant.In Clavecilla Radio System v. Antillon, 19 SCRA 379 ([1967]), this On June 10, 1988, petitioners filed a complaint against Roxas in the Regional Trial
Court explained why actions cannot be filed against a corporation in any place where the Court, Branch 11, Cebu City, praying that Roxas be ordered to pay petitioners the sum of
corporation maintains its branch offices. The Court ruled that to allow an action to be P3,400,00.00 or that full control of the three markets be turned over to YASCO and Garcia.
instituted in any place where the corporation has branch offices, would create confusion and The complaint also prayed for the forfeiture of the partial payment of P4,600,000.00 and the
work untold inconvenience to said entity. By the same token, a corporation cannot be payment of attorney's fees and costs (Rollo, p. 290).
allowed to file personal actions in a place other than its principal place of business unless such
a place is also the residence of a co-plaintiff or a defendant. Roxas filed two motions for extension of time to submit his answer. But despite said
motion, he failed to do so causing petitioners to file a motion to have him declared in default.
PETITION for review of the decision of the Court of Appeals. Roxas then filed, through a new counsel, a third motion for extension of time to submit a
responsive pleading.
The facts are stated in the opinion of the Court. On August 19, 1988, the trial court declared Roxas in default. The order of default
Angara, Abello, Concepcion, Regala & Cruz for petitioners. was, however, lifted upon motion of Roxas.
Antonio Nuyles for private respondent.
QUIASON, J.: On August 22, 1988, Roxas filed a motion to dismiss on the grounds that:

9
MATABABE FILES
Corporation Law

1. The complaint did not state a cause of action due to non-joinder of


indispensable parties; In the Regional Trial Courts, all personal actions are commenced and tried in the
2. The claim or demand set forth in the complaint had been waived, province or city where the defendant or any of the defendants resides or may be found, or
abandoned or otherwise extinguished; and where the plaintiff or any of the plaintiffs resides, at the election of the plaintiff [Sec. 2(b)
3. The venue was improperly laid (Rollo, p. 299). Rule 4, Revised Rules of Court].
There are two plaintiffs in the case at bench: a natural person and a domestic corporation.
After a hearing, wherein testimonial and documentary evidence were presented by Both plaintiffs aver in their complaint that they are residents of Cebu City, thus:
both parties, the trial court in an Order dated February 8, 1991 denied Roxas' motion to
dismiss. After receiving said order, Roxas filed another motion for extension of time to submit 1.1. Plaintiff Young Auto Supply Co., Inc., ("YASCO") is a domestic
his answer. He also filed a motion for reconsideration, which the trial court denied in its Order corporation duly organized and existing under Philippine laws with principal
dated April 10, 1991 for being pro-forma (Rollo, p. 17). Roxas was again declared in default, on place of business at M. J. Cuenco Avenue, Cebu City. It also has a branch
the ground that his motion for reconsideration did not toll the running of the period to file his office at 1708 Dominga Street, Pasay City, Metro Manila.
answer.
Plaintiff Nemesio Garcia is of legal age, married, Filipino citizen and with
On May 3, 1991, Roxas filed an unverified Motion to Lift the Order of Default which business address at Young Auto Supply Co., Inc., M. J. Cuenco Avenue, Cebu
was not accompanied with the required affidavit or merit. But without waiting for the City. . . . (Complaint, p. 1; Rollo, p. 81).
resolution of the motion, he filed a petition for certiorari with the Court of Appeals.
The Article of Incorporation of YASCO (SEC Reg. No. 22083) states:
The Court of Appeals sustained the findings of the trial court with regard to the first
two grounds raised in the motion to dismiss but ordered the dismissal of the complaint on the THIRD That the place where the principal office of the corporation is to be
ground of improper venue (Rollo, p. 49). established or located is at Cebu City, Philippines (as amended on December
20, 1980 and further amended on December 20, 1984) (Rollo, p. 273).
A subsequent motion for reconsideration by petitioner was to no avail.
Petitioners now come before us, alleging that the Court of Appeals A corporation has no residence in the same sense in which this term is applied to a
erred in: natural person. But for practical purposes, a corporation is in a metaphysical sense a resident
1. holding the venue should be in Pasay City, and not in Cebu City (where of the place where its principal office is located as stated in the articles of incorporation
both petitioners/plaintiffs are residents; (Cohen v. Benguet Commercial Co., Ltd., 34 Phil. 256 [1916] Clavecilla Radio System v. Antillon,
2. not finding that Roxas is estopped from questioning the choice of venue 19 SCRA 379 [1967]). The Corporation Code precisely requires each corporation to specify in
(Rollo, p. 19). its articles of incorporation the "place where the principal office of the corporation is to be
The petition is meritorious. located which must be within the Philippines" (Sec. 14 [3]). The purpose of this requirement is
to fix the residence of a corporation in a definite place, instead of allowing it to be
In holding that the venue was improperly laid in Cebu City, the Court of Appeals relied ambulatory.
on the address of YASCO, as appearing in the Deed of Sale dated October 28, 1987, which is
"No. 1708 Dominga Street, Pasay City." This was the same address written in YASCO's letters In Clavencilla Radio System v. Antillon, 19 SCRA 379 ([1967]), this Court explained why
and several commercial documents in the possession of Roxas (Decision, p. 12; Rollo, p. 48). actions cannot be filed against a corporation in any place where the corporation maintains its
branch offices. The Court ruled that to allow an action to be instituted in any place where the
In the case of Garcia, the Court of Appeals said that he gave Pasay City as his address corporation has branch offices, would create confusion and work untold inconvenience to
in three letters which he sent to Roxas' brothers and sisters (Decision, p. 12; Rollo, p. 47). The said entity. By the same token, a corporation cannot be allowed to file personal actions in a
appellate court held that Roxas was led by petitioners to believe that their residence is in place other than its principal place of business unless such a place is also the residence of a co-
Pasay City and that he had relied upon those representations (Decision, p. 12, Rollo, p. 47). plaintiff or a defendant.
The Court of Appeals erred in holding that the venue was improperly laid in Cebu City.

10
MATABABE FILES
Corporation Law

If it was Roxas who sued YASCO in Pasay City and the latter questioned the venue on
the ground that its principal place of business was in Cebu City, Roxas could argue that YASCO
was in estoppel because it misled Roxas to believe that Pasay City was its principal place of
business. But this is not the case before us.

With the finding that the residence of YASCO for purposes of venue is in Cebu City,
where its principal place of business is located, it becomes unnecessary to decide whether
Garcia is also a resident of Cebu City and whether Roxas was in estoppel from questioning the
choice of Cebu City as the venue.

WHEREFORE, the petition is GRANTED. The decision of the Court of Appeals


appealed from is SET ASIDE and the Order dated February 8, 1991 of the Regional Trial Court is
REINSTATED.

SO ORDERED.

Cruz, Grio-Aquino and Bellosillo, JJ., concur.

11
MATABABE FILES
Corporation Law

Republic of the Philippines The facts are stated in the opinion of the court
SUPREME COURT
Manila Modesto Reyes for appellants.
EN BANC Attorney-General Villamor for appellee.

G.R. No. 9321 September 24, 1914 MORELAND, J.:

NORBERTO ASUNCION, ET AL., petitioners-appellants, This is an action to obtain a writ of mandamus to compel the chief of the division of
vs. achieves of the Executive Bureau to file a certain articles of incorporation.
MANUEL DE YRIARTE, respondent-appellee.
The chief of the division of archives, the respondent, refused to file the articles of
1. CORPORATION LAW; POWERS AND DUTIES OF CHIEF OF DlVISION OF ARCHIVES, incorporation, hereinafter referred to, upon the ground that the object of the corporation, as
EXECUTIVE BUREAU.The chief of the division of archives, for and on behalf of the division, stated in the articles, was not lawful and that, in pursuance of section 6 of Act No. 1459, they
has authority under the Corporation Law (Act No. 1459) to determine the sufficiency of the were not registerable.
form of articles of incorporation offered for registration with the division.
The proposed incorporators began an action in the Court of First Instance of the city
2. ID.; ID.The chief of the division of archives, on behalf of the division, has also the of Manila to compel the chief of the division of archives to receive and register said articles of
power and duty to determine from the articles of incorporation presented for registration the incorporation and to do any and all acts necessary for the complete incorporation of the
lawfulness of the purposes of the proposed corporation and whether or not those purposes persons named in the articles. The court below found in favor of the defendant and refused to
bring the proposed corporation within the purview of the law authorizing corporations for order the registration of the articles mentioned, maintaining ad holding that the defendant,
given purposes. under the Corporation Law, had authority to determine both the sufficiency of the form of
the articles and the legality of the object of the proposed corporation. This appeal is taken
3. ID.; ID.; MANDAMUS TO COMPEL HIM TO PERFORM DUTIES.The duties of the from that judgment.
chief of the division of archives, so f ar as relates to the registration of articles of
incorporation, are purely ministerial and not discretional; and mandamus will lie to compel The first question that arises is whether or not the chief of the division of archives has
him to perform his duties under the Corporation Law if, in violation of law, he refuse to authority, under the Corporation for registration, to decide not only as to the sufficiency of
perform them. the form of the articles, but also as to the lawfulness of the purpose of the proposed
corporation.
4. ID.; MUNICIPALITIES; ORGANIZATION OF BARRIO INTO SEPARATE CORPORATION.
When articles of incorporation presented for registration show that the object of It is strongly urged on the part of the appellants that the duties of the defendant are
incorporators is to organize a pueblo or barrio of a given municipality into a separate purely ministerial and that he has no authority to pass upon the lawfulness of the object for
corporation for the purpose of taking possession and having control of all municipal property which the incorporators propose to organize. No authorities are cited to support this
within the pueblo or barrio so incorporated, and administer it exclusively for the benefit of proposition and we are of the opinion that it is not sound.
the residents of that pueblo or barrio, said articles of incorporation show upon their face that
the object of the incorporation is unlawful in that it seeks to deprive the municipality in which Section 6 of the Corporation Law reads in part as follows:
the pueblo or barrio is situated of its property and its citizens of the right of enjoying the
same and would, if permitted, disrupt and destroy the government of the municipalities of Five or more persons, not exceeding fifteen, a majority of whom are
the Islands and abrogate the laws relating- to the formation and government of residents of the Philippine Islands, may form a private corporation for any lawful
municipalities. purpose by filing with the division of archives, patents, copyrights, and trademarks if
APPEAL from a judgment of the Court of First Instance of Manila. Crossfield, J. the Executive Bureau articles of incorporation duly executed and acknowledged
before a notary public, . . . .

12
MATABABE FILES
Corporation Law

the court can decide only one way and be right. As a matter of law, there is only one way and
Simply because the duties of an official happens to be ministerial, it does not be right. As a matter of law, there is only one course to pursue. In a case where the court or
necessarily follow that he may not, in the administration of his office, determine questions of other official has discretion in the resolution of a question, then, within certain limitations, he
law. We are of the opinion that it is the duty of the division of archives, when articles of may decide the question either way and still be right. Discretion, it may be said generally, is a
incorporation are presented for registration, to determine whether the objects of the faculty conferred upon a court or other official by which he may decide a question either way
corporation as expressed in the articles are lawful. We do not believe that, simply because and still be right. The power conferred upon the division of archives with respect to the
articles of incorporation presented foe registration are perfect in form, the division of registration of articles of incorporation is not of that character. It is of the same character as
archives must accept and register them and issue the corresponding certificate of the determination of a lawsuit by a court upon the merits. It can be decided only one way
incorporation no matter what the purpose of the corporation may be as expressed in the correctly.
articles. We do not believe it was intended that the division of archives should issue a
certificate of incorporation to, and thereby put the seal of approval of the Government upon, If, therefore, the defendant erred in determining the question presented when the
a corporation which was organized for base of immoral purposes. That such corporation articles were offered for registration, then that error will be corrected by this court in this
might later, if it sought to carry out such purposes, be dissolved, or its officials imprisoned or action and he will be compelled to register the articles as offered. If, however, he did not
itself heavily fined furnished no reason why it should have been created in the first instance. It commit an error, but decided that question correctly, then, of course, his action will be
seems to us to be not only the right but the duty of the divisions of archives to determine the affirmed to the extent that we will deny the relief prayed for.
lawfulness of the objects and purposes of the corporation before it issues a certificate of
incorporation. The next question leads us to the determination of whether or not the purposes of
the corporation as stated in the articles of incorporation are lawful within the meaning of the
It having determined that the division of archives, through its officials, has authority Corporation Law.
to determine not only the sufficiency as to form of the articles of incorporation offered for
registration, but also the lawfulness of the purposes of leads us to the determination of the The purpose of the incorporation as stated in the articles is: "That the object of the
question whether or not the chief of the division of archives, who is the representative corporation is (a) to organize and regulate the management, disposition, administration and
thereof and clothed by it with authority to deal subject to mandamus in the performance of control which the barrio of Pulo or San Miguel or its inhabitants or residents have over the
his duties. common property of said residents or inhabitants or property belonging to the whole barrio
as such; and (b) to use the natural products of the said property for institutions, foundations,
We are of the opinion that he may be mandamused if he act in violation of law or if he and charitable works of common utility and advantage to the barrio or its inhabitants."
refuses, unduly, to comply with the law. While we have held that defendant has power to The municipality of Pasig as recognized by law contains within its limits several
pass upon the lawfulness of the purposes of the proposed corporation and that he may, in barrios or small settlements, like Pulo or San Miguel, which have no local government of their
the fulfillment of his duties, determine the question of law whether or not those purposes are own but are governed by the municipality of Pasig through its municipal president and
lawful and embraced within that class concerning which the law permits corporations to be council. The president and members of the municipal council are elected by a general vote of
formed, that does not necessarily mean, as we have already intimated, that his duties are not the municipality, the qualified electors of all the barrios having the right to participate.
ministerial. On the contrary, there is no incompatibility in holding, as we do hold, that his
duties are ministerial and that he has no authority to exercise discretion in receiving and The municipality of Pasig is a municipal corporation organized by law. It has the
registering articles of incorporation. He may exercise judgment that is, the judicial function control of all property of the municipality. The various barrios of the municipality have no
in the determination of the question of law referred to, but he may not use discretion. The right to own or hold property, they not being recognized as legal entities by any law. The
question whether or not the objects of a proposed corporation are lawful is one that can be residents of the barrios participate in the advantages which accrue to the municipality from
decided one way only. If he err in the determination of that question and refuse to file articles public property and receive all the benefits incident to residence in a municipality organized
which should be filed under the law, the decision is subject to review and correction and, by law. If there is any public property situated in the barrio of Pulo or San Miguel not
upon proper showing, he will be ordered to file the articles. This is the same kind of belonging to the general government or the province, it belongs to the municipality of Pasig
determination which a court makes when it decides a case upon the merits, the court makes and the sole authority to manage and administer the same resides in that municipality. Until
when it decides a case upon the merits. When a case is presented to a court upon the merits,

13
MATABABE FILES
Corporation Law

the present laws upon the subject are charged no other entity can be the owner of such
property or control or administer it.

The object of the proposed corporation, as appears from the articles offered for
registration, is to make of the barrio of Pulo or San Miguel a corporation which will become
the owner of and have the right to control and administer any property belonging to the
municipality of Pasig found within the limits of that barrio. This clearly cannot be permitted.
Otherwise municipalities as now established by law could be deprived of the property which
they now own and administer. Each barrio of the municipality would become under the
scheme proposed, a separate corporation, would take over the ownership, administration,
and control of that portion of the municipal territory within its limits. This would disrupt, in a
sense, the municipalities of the Islands by dividing them into a series of smaller municipalities
entirely independent of the original municipality.

What the law does not permit cannot be obtained by indirection. The object of the
proposed corporation is clearly repugnant to the provisions of the Municipal Code and the
governments of municipalities as they have been organized thereunder. (Act No. 82,
Philippine Commission.)

The judgment appealed from is affirmed, with costs against appellants.


Arellano, C.J., Torres, Johnson, Carson and Araullo, JJ., concur.

14
MATABABE FILES
Corporation Law

Republic of the Philippines addition to that specified by section 30 of the Corporation Law, which provides that every
SUPREME COURT director must own in his right at least one share of the capital stock of the stock corporation
Manila of which he is a director * * *.
EN BANC
Same; Stockholder has no vested right to be elected as stockholder.Any person
G.R. No. L-45911 April 11, 1979 who buys stock in a corporation does so with the knowledge that its affairs are dominated
by a majority of the stockholders and that he implied contracts that the will of the majority
JOHN GOKONGWEI, JR., petitioner, shall govern in all matters within the limits of the act of incorporation and lawfully enacted by-
vs. laws and not forbidden by law. To this extent, therefore, the stockholder may be considered
SECURITIES AND EXCHANGE COMMISSION, ANDRES M. SORIANO, JOSE M. SORIANO, to have parted with his personal right or privilege to regulate the disposition of his property
ENRIQUE ZOBEL, ANTONIO ROXAS, EMETERIO BUNAO, WALTHRODE B. CONDE, MIGUEL which he has invested in the capital stock of the corporation and surrendered it to the will of
ORTIGAS, ANTONIO PRIETO, SAN MIGUEL CORPORATION, EMIGDIO TANJUATCO, SR., and the majority or his fellow incorporators. **** It can not therefore be justly said that the
EDUARDO R. VISAYA, respondents. contract, express or implied, between the corporation and the stockholders is infringed ***
by any act of the former which is authorized by a majority, ***.
Supreme Court; Judgments; Securities and Exchange Commission; Corporation Law;
Supreme Court always strives to settle a legal controversy in a single proceeding.xxx In the Same; A director stands in a fiduciary relation to the competition and its stockholders.
case at bar, there are facts which cannot be denied, viz.: that the amended by-laws were The disqualification of a competition from being elected to the board of directors is a
adopted by the Board of Directors of the San Miguel Corporation in the exercise of the power reasonable exercise of corporate authority. Although in the strict and technical sense,
delegated by the stockholders ostensibly pursuant to section 22 of the Corporation Law; that directors of a private corporation are not regarded as trustees, there cannot be any doubt
in a special meeting on February 10, 1977 held specially for that purpose, the amended by-laws that their character is that of a fiduciary insofar as the corporation for the collective benefit of
were ratified by more than 80% of the stockholders of record; that the foreign investment in the stockholders, they occupy a fiduciary relation, and in these sense the relation is one of
the Hongkong Brewery and Distillery, a beer manufacturing company in Hongkong, was made trust.
by the San Miguel Corporation in 1948; and that in the stockholders annual meeting held in
1972 and 1977, all foreign investments and operations of San Miguel Corporation were ratified Same; Same.It is obviously to prevent the creation of an opportunity for an officer
by the stockholders. or director of San Miguel Corporation, who is also the officer or owner of competing
corporation, from taking advantage of the information which he acquires as director to
Corporation Law; While reasonableness of a by-law is a legal question, where promote his individual or corporate interests to the prejudice of San Miguel Corporation and
reasonableness of a by-law provision is one in which reasonable minds may differ a court will its stockholders, that the questioned amendment of the by-laws was made. Certainly, where
not be justified in subsisting its judgment for those authorized to make the by-laws.The two corporations are competitive in a substantial sense, it would seem improbable, if not
validity or reasonableness of a by-law of a corporation is purely a question of law. Whether impossible, for the director, if he were to discharge effectively his duty, to satisfy his loyalty to
the by-law is in conflict with the law of the land, or with the charter of the corporation, or is in both corporations and place the performance of his corporate duties above his personal
a legal sense unreasonable and therefore unlawful is a question of law. This rule is subject, concerns.
however, to the limitation that where the reasonableness of a by-law is a mere matter of
judgment, and one upon which reasonable minds must necessarily differ, a court would not Same; Same.Sound principles of corporate management counsel against sharing
be warranted in substituting its judgment instead of the judgment of those who are sensitive information with a director whose fiduciary duty to loyalty may well require that he
authorized to make by-laws and who have exercised their authority. disclose this information to a competitive rival. These dangers are enhanced considerably
where the common director such as the petitioner is a controlling stockholder of two of the
Same; Under the Corporation Law a corporation is authorized to prescribe the competing corporations. It would seem manifest that in such situations, the director has an
qualification of its directors.In this jurisdiction, under Section 21 of the Corporation Law, a economic incentive to appropriate for the benefit of his own corporation the corporate plans
corporation may prescribed in its by-laws the qualifications, duties and compensation of and policies of the corporation where he sits as director.
directors, officers and employees ***. This must necessarily refer to a qualification in

15
MATABABE FILES
Corporation Law

Same; Another reason for upholding a by-law provision that forbids a competitor to be consequence that petitioner is ipso facto disqualified. Consonant with the requirement of
elected as corporate director are the laws prohibiting cartels.There is another important due process, there must be due hearing at which the petitioner must be given the fullest
consideration in determining whether or not the amended by-laws are reasonable. The opportunity to show that he is not covered by the disqualification. As trustees of the
Constitution and the law prohibit combinations in restraint of trade or unfair competition. corporation and of the stockholders, it is the responsibility of directors to act with fairness to
Thus, Section 2 of Article XIV of the Constitution provides: That State shall regulate or the stockholders. Pursuant to this obligation and to remove any suspicion that this power
prohibit private monopolies when the public interest so requires. No combinations in restraint may be utilized by the incumbent members of the Board to perpetuate themselves in power,
of trade or unfair competition shall be allowed. any decision of the Board to disqualify a candidate for the Board of Directors should be
reviewed by the Securities and Exchange Commission en banc and its decision shall be final
Same; Same.Basically, these anti-trust laws or laws against monopolies or unless reversed by this Court on certiorari.
combinations in restraint of trade are aimed at raising levels of competition by improving the
consumers effectiveness as the final arbiter in free markets. These laws are designed to Same; Every stockholder has the right to inspect corporate books and records.The
preserve free and unfettered competition as the rule of trade. It rests on the premise that stockholders right of inspection of the corporations books and records is based upon their
the unrestrained interaction of competitive forces will yield the best allocation of our ownership of the assets and property of the corporation. It is, therefore, an incident of
economic resources, the lowest prices and the highest quality ***. They operate to forestall ownership of the corporate property, whether this ownership or interest be termed an
concentration of economic power. The law against monopolies and combinations in restraint equitable ownership, a beneficial ownership, or a quasi-ownership. This right is predicated
of trade is aimed at contracts and combinations that, by reason of the inherent nature of the upon the necessity of selfprotection. It is generally held by majority of the courts that where
contemplated acts, prejudice the public interest by unduly restraining competition or unduly the right is granted by statute to the stockholder, it is given to him as such and must be
obstructing the course of trade. exercised by him with respect to his interest as a stockholder and for some purpose germane
thereto or in the interest of the corporation. In other words, the inspection has to germane to
Same; Election of petitioner as San Miguel Corporation Director may run counter to the petitioners interest as a stockholder, and has to be proper and lawful in character and not
the prohibition contained in Section 13(5) of Corporation Law on investments in corporations inimical to the interest of the corporation.
engaged in agriculture.Finally, considering that both Robina and SMC are, to a certain
extent, engaged in agriculture, then the election of petitioner to the Board of SMC may Same; The right of stockholder to inspect corporate books extends to a wholly-owned
constitute a violation of the prohibition contained in Section 13(5) of the Corporation Law. subsidiary.In the case at bar, considering that the foreign subsidiary is wholly owned by
Said section provides in part that any stockholder of more than one corporation organized respondent San Miguel Corporation and, therefore, under its control, it would be more in
for the purpose of engaging in agriculture may hold his stock in such corporations solely for accord with equity, good faith and fair dealing to construe the statutory right of petitioner as
investment and not for the purpose of bringing about or attempting to bring about a stockholder to inspect the books and records of the corporation as extending to books and
combination to exercise control of such corporations. ***. records of such wholly owned subsidiary which are in respondent corporations possession
and control.
Same; The by-law amendment of SMC applies equally to all and does not discriminate Same; Purely ultra vires corporate acts of corporate officers to invest corporate funds
against petitioner only.However, the by-law, by its terms, applies to all stockholders. The in another business or corporation, i.e., acts not contrary to law, morals, public order as public
equal protection clause of the Constitution requires only that the by-laws operate equally policy, may be ratified by the stockholders holding 2/3 of the voting power.Assuming
upon all persons of a class. Besides, before petitioner can be declared ineligible to run for arguendo that the Board of Directors of San Miguel Corporation had no authority to make the
director, there must be hearing and evidence must be submitted to bring his case within the assailed investment, there is no question that a corporation, like an individual, may ratify and
ambit of the disqualification. Sound principles of public policy and management, therefore, thereby render binding upon it the originally unauthorized acts of its officers or other agents.
support the view that a by-law which disqualifies a competitor from election to the Board of This is true because the questioned investment is neither contrary to law, morals, public order
Directors of another corporation is valid and reasonable. or public policy. It is a corporate transaction or contract which is within the corporate powers,
but which is defective from a purported failure to observe in its execution the requirement of
Same; Petitioner is not ipso facto disqualified to run on SMC director. He must be given the law that the investment must be authorized by the affirmative vote of the stockholders
full opportunity by the SEC to show that he is not covered by the disqualification.While We holding twothirds of the voting power. This requirement is for the benefit of the
here sustain the validity of the amended by-laws, it does not follow as a necessary stockholders. The stockholders for whose benefit the requirement was enacted may,

16
MATABABE FILES
Corporation Law

therefore, ratify the investment and its ratification by said stockholders obliterates any defect Same; Same; Reservation of the vote of the Chief Justice.As expressly stated in the
which it may have had at the outset. Mere ultra vires acts, said this Court in Pirovano, or Chief Justices reservation of his vote, the matter of the question of the applicability of the
those which are not illegal and void ab initio, but are not merely within the scope of the said section 13(5) to petitioner would be heard by this Court at the appropriate time after the
articles of incorporation, are merely voidable and may become binding and enforceable when proceedings below (and necessarily the question of the validity of the amended by-laws
ratified by the stockholders. would be taken up anew and the Court would at that time be able to reach a final and
conclusive vote).
Corporation Law; Judgment; The doctrine of the law of the case.We hold on our
part that the doctrine of the law of the case invoked by Mr. Justice Barredo has no Same; Same; Validity of the amended by-laws.The six votes cast by Justices
applicability for the following reasons: a) Our jurisprudence is quite clear that this doctrine Makasiar, Antonio, Santos, Abad Santos, De Castro and this writer in favor of validity of the
may be invoked only where there has been a final and conclusive determination of an issue in amended by-laws in question, with only four members of this Court, namely, Justices
the first case later invoked as the law of the case. Teehankee, Concepcion Jr., Fernandez and Guerrero opining otherwise, and with Chief Justice
Castro and Justice Fernando reserving their votes thereon and Justice Aquino and Melencio
Same; Same; When doctrine of the law of the case not applicable.The doctrine of Herrera not voting, thereby resulting in the dismissal of the petition insofar as it assails the
the law of the case, therefore, has no applicability whatsoever herein insofar as the question validity of the amended by-laws . . . . for lack of necessary votes, has no other legal
of the validity or invalidity of the amended by-laws is concerned. The Courts judgment of consequence than that it is the law of the case far as the parties herein are concerned, albeit
April 11, 1979 clearly shows that the voting on this question inconclusive with six against four the majority opinion of six against four Justices is not doctrinal in the sense that it cannot be
Justices and two other Justices (the Chief Justice and Mr. Justice Fernando) expressly cited as necessarily a precedent for subsequent cases. This means that petitioner Gokongwei
reserving their votes thereon, and Mr. Justice Aquino while taking no part in effect likewise and the respondents, including the Securities and Exchange Commission, are bound by the
expressly reserved his vote thereon. No final aad conclusive determination could be reached foregoing result, namely, that the Court en banc has not found merit in the claim that the
on the issue and pursuant to the provisions of Rule 56, section 11, since this special civil action amended by-laws in question are invalid. Indeed, it is one thing to say that dismissal of the
originally commenced in this Court, the action was simply dismissed with the result that no case is not doctrinal and entirely another thing to maintain that such dismissal leaves the issue
law of the case was laid down insofar as the issue of the validity or invalidity of the questioned unsettled.
by-laws is concerned, and the relief sought herein by petitioner that this Court bypass the SEC Same; Same; Where petitioner can no longer revive the issue validity of the amended
which has yet to hear and determine the same issue pending before it below and that this by-laws.I reiterate, therefore, that as between the parties herein, the issue of validity of the
Court itself directly resolve the said issue stands denied. challenged bylaws is already settled. From which it follows that the same are already
Same; Same; Constitutional Law; Due Process; When procedural due process was not enforceable insofar as they are concerned. Petitioner Gokongwei may not hereafter act on
observed.The entire Court, therefore, recognized that petitioner had not been given the assumption that he can revive the issue of validity whether in the Securities Exchange
procedural due process by the SMC board on the matter of his disqualification and that he Commission, in this Court or in any other forum, unless he proceeds on the basis of a factual
was entitled to a new and proper hearing. It stands to reason that in such hearing, milieu different from the setting of this case. Not even the Securities and Exchange
petitioner could raise not only questions of fact but questions of law, particularly questions Commission may pass on such question anymore at the instance of herein petitioner or
of law affecting the investing public and their right to representation on the board as anyone acting in his stead or on his behalf. The vote of four justices to remand the case
provided by lawnot to mention that as borne out by the fact that no restriction whatsoever thereto cannot alter the situation.
appears in the Courts decision, it was never contemplated that petitioner was to be limited
questions of fact and could not raise the fundamental question of law bearing on the Same; Same; Where Court has not found merit in the claim that the amended by-laws
invalidity of the questioned amended by-laws at such hearing before the SMC board. in question are valid.I concur in Justice Barredos statement that the dismissal (for lack of
Furthermore, it was expressly provided unanimously in the Courts decision that the SMC necessary votes) of the petition to the extent that it assails the validity of the amended by-
boards decision on the disqualification of petitioner (assuming the board of directors of San laws, is the law of the case at bar, which means in effect that as far and only in so far as the
Miguel Corporation should, after the proper hearing, disqualify him as qualified in Mr. Justice parties and the Securities and Exchange Commission are concerned, the Court has not found
Barredos own separate opinion, at page 2) shall be appealable to respondent Securities and merit in the claim that the amended by-laws in question are valid.
Exchange Commission deliberating and acting en banc and ultimately to this Court.

17
MATABABE FILES
Corporation Law

Same; Same; Term and meaning of farming.This is my view, even as I am for a Zobel, Antonio Roxas, Emeterio Bunao, Walthrode B. Conde, Miguel Ortigas, Antonio Prieto
restrictive interpretation of Section 13(5) of the Philippine Corporation Law, under which I and San Miguel Corporation", was docketed as SEC Case No. 1375.
would limit the scope of the provision to corporations engaged in agriculture, but only as the As a first cause of action, petitioner alleged that on September 18, 1976, individual
word agriculture refers to its more limited meaning as distinguished from its general and respondents amended by bylaws of the corporation, basing their authority to do so on a
broad connotation. The term would then mean farming or raising the natural products of resolution of the stockholders adopted on March 13, 1961, when the outstanding capital stock
the soil, such as by cultivation, in the acquisition of agricultural land such as by homestead, of respondent corporation was only P70,139.740.00, divided into 5,513,974 common shares at
before the patent may be issued. P10.00 per share and 150,000 preferred shares at P100.00 per share. At the time of the
amendment, the outstanding and paid up shares totalled 30,127,047 with a total par value of
Same; Same; Poultry raising or piggery is included in the term agriculture.It is my P301,270,430.00. It was contended that according to section 22 of the Corporation Law and
opinion that under the public land statute, the development of a certain portion of the land Article VIII of the by-laws of the corporation, the power to amend, modify, repeal or adopt
applied for a specified in the law as a condition precedent before the applicant may obtain a new by-laws may be delegated to the Board of Directors only by the affirmative vote of
patent, is cultivation, not let us say, poultry raising or piggery, which may be included in the stockholders representing not less than 2/3 of the subscribed and paid up capital stock of the
term Agriculture in its broad sense. For under Section 13(5) of the Philippine Corporation corporation, which 2/3 should have been computed on the basis of the capitalization at the
Law, construed not in the strict way as I believe it should because the provision is in time of the amendment. Since the amendment was based on the 1961 authorization,
derogation of property rights, the petitioner in this case would be disqualified from becoming petitioner contended that the Board acted without authority and in usurpation of the power
an officer of either the San Miguel Corporation or his own supposedly agricultural of the stockholders.
corporations.
As a second cause of action, it was alleged that the authority granted in 1961 had
ORIGINAL ACTION in the Supreme Court. Certiorari, mandamus and injunction. already been exercised in 1962 and 1963, after which the authority of the Board ceased to
exist.
The facts are stated in the opinion of the Court. Gokongwei, Jr. vs. Securities and Exchange
Commission, 89 SCRA 336, No. L-45911 April 11, 1979 As a third cause of action, petitioner averred that the membership of the Board of
De Santos, Balgos & Perez for petitioner. Directors had changed since the authority was given in 1961, there being six (6) new directors.

Angara, Abello, Concepcion, Regala, Cruz Law Offices for respondents Sorianos As a fourth cause of action, it was claimed that prior to the questioned amendment,
Siguion Reyna, Montecillo & Ongsiako for respondent San Miguel Corporation. petitioner had all the qualifications to be a director of respondent corporation, being a
R. T Capulong for respondent Eduardo R. Visaya. Substantial stockholder thereof; that as a stockholder, petitioner had acquired rights inherent
in stock ownership, such as the rights to vote and to be voted upon in the election of
ANTONIO, J.: directors; and that in amending the by-laws, respondents purposely provided for petitioner's
disqualification and deprived him of his vested right as afore-mentioned hence the amended
The instant petition for certiorari, mandamus and injunction, with prayer for issuance by-laws are null and void. 1
of writ of preliminary injunction, arose out of two cases filed by petitioner with the Securities
and Exchange Commission, as follows: As additional causes of action, it was alleged that corporations have no inherent
SEC CASE NO 1375 power to disqualify a stockholder from being elected as a director and, therefore, the
questioned act is ultra vires and void; that Andres M. Soriano, Jr. and/or Jose M. Soriano, while
On October 22, 1976, petitioner, as stockholder of respondent San Miguel representing other corporations, entered into contracts (specifically a management contract)
Corporation, filed with the Securities and Exchange Commission (SEC) a petition for with respondent corporation, which was allowed because the questioned amendment gave
"declaration of nullity of amended by-laws, cancellation of certificate of filing of amended by- the Board itself the prerogative of determining whether they or other persons are engaged in
laws, injunction and damages with prayer for a preliminary injunction" against the majority of competitive or antagonistic business; that the portion of the amended bylaws which states
the members of the Board of Directors and San Miguel Corporation as an unwilling petitioner. that in determining whether or not a person is engaged in competitive business, the Board
The petition, entitled "John Gokongwei Jr. vs. Andres Soriano, Jr., Jose M. Soriano, Enrique may consider such factors as business and family relationship, is unreasonable and oppressive

18
MATABABE FILES
Corporation Law

and, therefore, void; and that the portion of the amended by-laws which requires that "all of the stockholders representing a majority of the subscribed capital stock at any regular or
nominations for election of directors ... shall be submitted in writing to the Board of Directors special meeting, as provided in Article VIII, section I of the by-laws and section 22 of the
at least five (5) working days before the date of the Annual Meeting" is likewise unreasonable Corporation law, hence the, petition is premature; that petitioner is estopped from
and oppressive. questioning the amendments on the ground of lack of authority of the Board. since he failed,
to object to other amendments made on the basis of the same 1961 authorization: that the
It was, therefore, prayed that the amended by-laws be declared null and void and the power of the corporation to amend its by-laws is broad, subject only to the condition that the
certificate of filing thereof be cancelled, and that individual respondents be made to pay by-laws adopted should not be respondent corporation inconsistent with any existing law;
damages, in specified amounts, to petitioner. that respondent corporation should not be precluded from adopting protective measures to
minimize or eliminate situations where its directors might be tempted to put their personal
On October 28, 1976, in connection with the same case, petitioner filed with the interests over t I hat of the corporation; that the questioned amended by-laws is a matter of
Securities and Exchange Commission an "Urgent Motion for Production and Inspection of internal policy and the judgment of the board should not be interfered with: That the by-laws,
Documents", alleging that the Secretary of respondent corporation refused to allow him to as amended, are valid and binding and are intended to prevent the possibility of violation of
inspect its records despite request made by petitioner for production of certain documents criminal and civil laws prohibiting combinations in restraint of trade; and that the petition
enumerated in the request, and that respondent corporation had been attempting to states no cause of action. It was, therefore, prayed that the petition be dismissed and that
suppress information from its stockholders despite a negative reply by the SEC to its query petitioner be ordered to pay damages and attorney's fees to respondents. The application for
regarding their authority to do so. Among the documents requested to be copied were (a) writ of preliminary injunction was likewise on various grounds.
minutes of the stockholder's meeting field on March 13, 1961, (b) copy of the management
contract between San Miguel Corporation and A. Soriano Corporation (ANSCOR); (c) latest Respondents Andres M. Soriano, Jr. and Jose M. Soriano filed their opposition to the
balance sheet of San Miguel International, Inc.; (d) authority of the stockholders to invest the petition, denying the material averments thereof and stating, as part of their affirmative
funds of respondent corporation in San Miguel International, Inc.; and (e) lists of salaries, defenses, that in August 1972, the Universal Robina Corporation (Robina), a corporation
allowances, bonuses, and other compensation, if any, received by Andres M. Soriano, Jr. engaged in business competitive to that of respondent corporation, began acquiring shares
and/or its successor-in-interest. therein. until September 1976 when its total holding amounted to 622,987 shares: that in
October 1972, the Consolidated Foods Corporation (CFC) likewise began acquiring shares in
The "Urgent Motion for Production and Inspection of Documents" was opposed by respondent (corporation. until its total holdings amounted to P543,959.00 in September 1976;
respondents, alleging, among others that the motion has no legal basis; that the demand is that on January 12, 1976, petitioner, who is president and controlling shareholder of Robina
not based on good faith; that the motion is premature since the materiality or relevance of and CFC (both closed corporations) purchased 5,000 shares of stock of respondent
the evidence sought cannot be determined until the issues are joined, that it fails to show corporation, and thereafter, in behalf of himself, CFC and Robina, "conducted malevolent and
good cause and constitutes continued harrasment, and that some of the information sought malicious publicity campaign against SMC" to generate support from the stockholder "in his
are not part of the records of the corporation and, therefore, privileged. effort to secure for himself and in representation of Robina and CFC interests, a seat in the
Board of Directors of SMC", that in the stockholders' meeting of March 18, 1976, petitioner
During the pendency of the motion for production, respondents San Miguel was rejected by the stockholders in his bid to secure a seat in the Board of Directors on the
Corporation, Enrique Conde, Miguel Ortigas and Antonio Prieto filed their answer to the basic issue that petitioner was engaged in a competitive business and his securing a seat
petition, denying the substantial allegations therein and stating, by way of affirmative would have subjected respondent corporation to grave disadvantages; that "petitioner
defenses that "the action taken by the Board of Directors on September 18, 1976 resulting in nevertheless vowed to secure a seat in the Board of Directors at the next annual meeting;
the ... amendments is valid and legal because the power to "amend, modify, repeal or adopt that thereafter the Board of Directors amended the by-laws as afore-stated.
new By-laws" delegated to said Board on March 13, 1961 and long prior thereto has never
been revoked of SMC"; that contrary to petitioner's claim, "the vote requirement for a valid As counterclaims, actual damages, moral damages, exemplary damages, expenses of
delegation of the power to amend, repeal or adopt new by-laws is determined in relation to litigation and attorney's fees were presented against petitioner.
the total subscribed capital stock at the time the delegation of said power is made, not when
the Board opts to exercise said delegated power"; that petitioner has not availed of his intra- Subsequently, a Joint Omnibus Motion for the striking out of the motion for
corporate remedy for the nullification of the amendment, which is to secure its repeal by vote production and inspection of documents was filed by all the respondents. This was duly

19
MATABABE FILES
Corporation Law

opposed by petitioner. At this juncture, respondents Emigdio Tanjuatco, Sr. and Eduardo R. 4. Finally, the Commission holds in abeyance the resolution on the matter of
Visaya were allowed to intervene as oppositors and they accordingly filed their oppositions- production and inspection of the authority of the stockholders of San Miguel
intervention to the petition. Corporation to invest the funds of respondent corporation in San Miguel
On December 29, 1976, the Securities and Exchange Commission resolved the motion for International, Inc., until after the hearing on the merits of the principal issues
production and inspection of documents by issuing Order No. 26, Series of 1977, stating, in in the above-entitled case.
part as follows:
This Order is immediately executory upon its approval. 2
Considering the evidence submitted before the Commission by the
petitioner and respondents in the above-entitled case, it is hereby ordered: Dissatisfied with the foregoing Order, petitioner moved for its reconsideration.

1. That respondents produce and permit the inspection, copying and Meanwhile, on December 10, 1976, while the petition was yet to be heard,
photographing, by or on behalf of the petitioner-movant, John Gokongwei, respondent corporation issued a notice of special stockholders' meeting for the purpose of
Jr., of the minutes of the stockholders' meeting of the respondent San "ratification and confirmation of the amendment to the By-laws", setting such meeting for
Miguel Corporation held on March 13, 1961, which are in the possession, February 10, 1977. This prompted petitioner to ask respondent Commission for a summary
custody and control of the said corporation, it appearing that the same is judgment insofar as the first cause of action is concerned, for the alleged reason that by
material and relevant to the issues involved in the main case. Accordingly, calling a special stockholders' meeting for the aforesaid purpose, private respondents
the respondents should allow petitioner-movant entry in the principal office admitted the invalidity of the amendments of September 18, 1976. The motion for summary
of the respondent Corporation, San Miguel Corporation on January 14, 1977, judgment was opposed by private respondents. Pending action on the motion, petitioner filed
at 9:30 o'clock in the morning for purposes of enforcing the rights herein an "Urgent Motion for the Issuance of a Temporary Restraining Order", praying that pending
granted; it being understood that the inspection, copying and the determination of petitioner's application for the issuance of a preliminary injunction
photographing of the said documents shall be undertaken under the direct and/or petitioner's motion for summary judgment, a temporary restraining order be issued,
and strict supervision of this Commission. Provided, however, that other restraining respondents from holding the special stockholder's meeting as scheduled. This
documents and/or papers not heretofore included are not covered by this motion was duly opposed by respondents.
Order and any inspection thereof shall require the prior permission of this
Commission; On February 10, 1977, respondent Commission issued an order denying the motion for
issuance of temporary restraining order. After receipt of the order of denial, respondents
2. As to the Balance Sheet of San Miguel International, Inc. as well as the list conducted the special stockholders' meeting wherein the amendments to the by-laws were
of salaries, allowances, bonuses, compensation and/or remuneration ratified. On February 14, 1977, petitioner filed a consolidated motion for contempt and for
received by respondent Jose M. Soriano, Jr. and Andres Soriano from San nullification of the special stockholders' meeting.
Miguel International, Inc. and/or its successors-in- interest, the Petition to
produce and inspect the same is hereby DENIED, as petitioner-movant is not A motion for reconsideration of the order denying petitioner's motion for summary
a stockholder of San Miguel International, Inc. and has, therefore, no judgment was filed by petitioner before respondent Commission on March 10, 1977. Petitioner
inherent right to inspect said documents; alleges that up to the time of the filing of the instant petition, the said motion had not yet
been scheduled for hearing. Likewise, the motion for reconsideration of the order granting in
3. In view of the Manifestation of petitioner-movant dated November 29, part and denying in part petitioner's motion for production of record had not yet been
1976, withdrawing his request to copy and inspect the management contract resolved.
between San Miguel Corporation and A. Soriano Corporation and the
renewal and amendments thereof for the reason that he had already In view of the fact that the annul stockholders' meeting of respondent corporation
obtained the same, the Commission takes note thereof; and had been scheduled for May 10, 1977, petitioner filed with respondent Commission a
Manifestation stating that he intended to run for the position of director of respondent
corporation. Thereafter, respondents filed a Manifestation with respondent Commission,

20
MATABABE FILES
Corporation Law

submitting a Resolution of the Board of Directors of respondent corporation disqualifying and from taking up Item 6 of the Agenda at the annual stockholders' meeting, requesting that the
precluding petitioner from being a candidate for director unless he could submit evidence on same be set for hearing on May 3, 1977, the date set for the second hearing of the case on the
May 3, 1977 that he does not come within the disqualifications specified in the amendment to merits. Respondent Commission, however, cancelled the dates of hearing originally scheduled
the by-laws, subject matter of SEC Case No. 1375. By reason thereof, petitioner filed a and reset the same to May 16 and 17, 1977, or after the scheduled annual stockholders'
manifestation and motion to resolve pending incidents in the case and to issue a writ of meeting. For the purpose of urging the Commission to act, petitioner filed an urgent
injunction, alleging that private respondents were seeking to nullify and render ineffectual the manifestation on May 3, 1977, but this notwithstanding, no action has been taken up to the
exercise of jurisdiction by the respondent Commission, to petitioner's irreparable damage and date of the filing of the instant petition.
prejudice, Allegedly despite a subsequent Manifestation to prod respondent Commission to
act, petitioner was not heard prior to the date of the stockholders' meeting. With respect to the afore-mentioned SEC cases, it is petitioner's contention before
this Court that respondent Commission gravely abused its discretion when it failed to act with
Petitioner alleges that there appears a deliberate and concerted inability on the part deliberate dispatch on the motions of petitioner seeking to prevent illegal and/or arbitrary
of the SEC to act hence petitioner came to this Court. impositions or limitations upon his rights as stockholder of respondent corporation, and that
respondent are acting oppressively against petitioner, in gross derogation of petitioner's
SEC. CASE NO. 1423 rights to property and due process. He prayed that this Court direct respondent SEC to act on
collateral incidents pending before it.
Petitioner likewise alleges that, having discovered that respondent corporation has
been investing corporate funds in other corporations and businesses outside of the primary On May 6, 1977, this Court issued a temporary restraining order restraining private
purpose clause of the corporation, in violation of section 17 1/2 of the Corporation Law, he respondents from disqualifying or preventing petitioner from running or from being voted as
filed with respondent Commission, on January 20, 1977, a petition seeking to have private director of respondent corporation and from submitting for ratification or confirmation or
respondents Andres M. Soriano, Jr. and Jose M. Soriano, as well as the respondent from causing the ratification or confirmation of Item 6 of the Agenda of the annual
corporation declared guilty of such violation, and ordered to account for such investments stockholders' meeting on May 10, 1977, or from Making effective the amended by-laws of
and to answer for damages. respondent corporation, until further orders from this Court or until the Securities and Ex-
change Commission acts on the matters complained of in the instant petition.
On February 4, 1977, motions to dismiss were filed by private respondents, to which a
consolidated motion to strike and to declare individual respondents in default and an On May 14, 1977, petitioner filed a Supplemental Petition, alleging that after a
opposition ad abundantiorem cautelam were filed by petitioner. Despite the fact that said restraining order had been issued by this Court, or on May 9, 1977, the respondent
motions were filed as early as February 4, 1977, the commission acted thereon only on April Commission served upon petitioner copies of the following orders:
25, 1977, when it denied respondents' motion to dismiss and gave them two (2) days within
which to file their answer, and set the case for hearing on April 29 and May 3, 1977. (1) Order No. 449, Series of 1977 (SEC Case No. 1375); denying petitioner's motion for
reconsideration, with its supplement, of the order of the Commission denying in part
Respondents issued notices of the annual stockholders' meeting, including in the petitioner's motion for production of documents, petitioner's motion for reconsideration of
Agenda thereof, the following: the order denying the issuance of a temporary restraining order denying the issuance of a
temporary restraining order, and petitioner's consolidated motion to declare respondents in
6. Re-affirmation of the authorization to the Board of Directors by the contempt and to nullify the stockholders' meeting;
stockholders at the meeting on March 20, 1972 to invest corporate funds in
other companies or businesses or for purposes other than the main purpose (2) Order No. 450, Series of 1977 (SEC Case No. 1375), allowing petitioner to run as a
for which the Corporation has been organized, and ratification of the director of respondent corporation but stating that he should not sit as such if elected, until
investments thereafter made pursuant thereto. such time that the Commission has decided the validity of the bylaws in dispute, and denying
deferment of Item 6 of the Agenda for the annual stockholders' meeting; and
By reason of the foregoing, on April 28, 1977, petitioner filed with the SEC an urgent
motion for the issuance of a writ of preliminary injunction to restrain private respondents

21
MATABABE FILES
Corporation Law

(3) Order No. 451, Series of 1977 (SEC Case No. 1375), denying petitioner's motion for (3) that by laws are valid and binding since a corporation has the inherent right and
reconsideration of the order of respondent Commission denying petitioner's motion for duty to preserve and protect itself by excluding competitors and antogonistic parties, under
summary judgment; the law of self-preservation, and it should be allowed a wide latitude in the selection of means
to preserve itself;
It is petitioner's assertions, anent the foregoing orders, (1) that respondent
Commission acted with indecent haste and without circumspection in issuing the aforesaid (4) that the delay in the resolution and disposition of SEC Cases Nos. 1375 and 1423
orders to petitioner's irreparable damage and injury; (2) that it acted without jurisdiction and was due to petitioner's own acts or omissions, since he failed to have the petition to
in violation of petitioner's right to due process when it decided en banc an issue not raised suspend, pendente lite the amended by-laws calendared for hearing. It was emphasized that it
before it and still pending before one of its Commissioners, and without hearing petitioner was only on April 29, 1977 that petitioner calendared the aforesaid petition for suspension
thereon despite petitioner's request to have the same calendared for hearing , and (3) that (preliminary injunction) for hearing on May 3, 1977. The instant petition being dated May 4,
the respondents acted oppressively against the petitioner in violation of his rights as a 1977, it is apparent that respondent Commission was not given a chance to act "with
stockholder, warranting immediate judicial intervention. deliberate dispatch", and

It is prayed in the supplemental petition that the SEC orders complained of be (5) that, even assuming that the petition was meritorious was, it has become moot
declared null and void and that respondent Commission be ordered to allow petitioner to and academic because respondent Commission has acted on the pending incidents,
undertake discovery proceedings relative to San Miguel International. Inc. and thereafter to complained of. It was, therefore, prayed that the petition be dismissed.
decide SEC Cases No. 1375 and 1423 on the merits.
On May 21, 1977, respondent Emigdio G, Tanjuatco, Sr. filed his comment, alleging
On May 17, 1977, respondent SEC, Andres M. Soriano, Jr. and Jose M. Soriano filed that the petition has become moot and academic for the reason, among others that the acts
their comment, alleging that the petition is without merit for the following reasons: of private respondent sought to be enjoined have reference to the annual meeting of the
stockholders of respondent San Miguel Corporation, which was held on may 10, 1977; that in
(1) that the petitioner the interest he represents are engaged in business competitive said meeting, in compliance with the order of respondent Commission, petitioner was
and antagonistic to that of respondent San Miguel Corporation, it appearing that the owns allowed to run and be voted for as director; and that in the same meeting, Item 6 of the
and controls a greater portion of his SMC stock thru the Universal Robina Corporation and the Agenda was discussed, voted upon, ratified and confirmed. Further it was averred that the
Consolidated Foods Corporation, which corporations are engaged in business directly and questions and issues raised by petitioner are pending in the Securities and Exchange
substantially competing with the allied businesses of respondent SMC and of corporations in Commission which has acquired jurisdiction over the case, and no hearing on the merits has
which SMC has substantial investments. Further, when CFC and Robina had accumulated been had; hence the elevation of these issues before the Supreme Court is premature.
investments. Further, when CFC and Robina had accumulated shares in SMC, the Board of
Directors of SMC realized the clear and present danger that competitors or antagonistic Petitioner filed a reply to the aforesaid comments, stating that the petition presents
parties may be elected directors and thereby have easy and direct access to SMC's business justiciable questions for the determination of this Court because (1) the respondent
and trade secrets and plans; Commission acted without circumspection, unfairly and oppresively against petitioner,
warranting the intervention of this Court; (2) a derivative suit, such as the instant case, is not
(2) that the amended by law were adopted to preserve and protect respondent SMC rendered academic by the act of a majority of stockholders, such that the discussion,
from the clear and present danger that business competitors, if allowed to become directors, ratification and confirmation of Item 6 of the Agenda of the annual stockholders' meeting of
will illegally and unfairly utilize their direct access to its business secrets and plans for their May 10, 1977 did not render the case moot; that the amendment to the bylaws which
own private gain to the irreparable prejudice of respondent SMC, and, ultimately, its specifically bars petitioner from being a director is void since it deprives him of his vested
stockholders. Further, it is asserted that membership of a competitor in the Board of rights.
Directors is a blatant disregard of no less that the Constitution and pertinent laws against
combinations in restraint of trade; Respondent Commission, thru the Solicitor General, filed a separate comment,
alleging that after receiving a copy of the restraining order issued by this Court and noting

22
MATABABE FILES
Corporation Law

that the restraining order did not foreclose action by it, the Commission en banc issued Orders There is no factual dispute as to what the provisions are and evidence is not necessary to
Nos. 449, 450 and 451 in SEC Case No. 1375. determine whether such amended by-laws are valid as framed and approved ... "; second: "it
In answer to the allegation in the supplemental petition, it states that Order No. 450 which is for the interest and guidance of the public that an immediate and final ruling on the
denied deferment of Item 6 of the Agenda of the annual stockholders' meeting of respondent question be made ... "; third: "petitioner was denied due process by SEC" when
corporation, took into consideration an urgent manifestation filed with the Commission by "Commissioner de Guzman had openly shown prejudice against petitioner ... ", and
petitioner on May 3, 1977 which prayed, among others, that the discussion of Item 6 of the "Commissioner Sulit ... approved the amended by-laws ex-parte and obviously found the same
Agenda be deferred. The reason given for denial of deferment was that "such action is within intrinsically valid; and finally: "to remand the case to SEC would only entail delay rather than
the authority of the corporation as well as falling within the sphere of stockholders' right to serve the ends of justice."
know, deliberate upon and/or to express their wishes regarding disposition of corporate
funds considering that their investments are the ones directly affected." It was alleged that Respondents Andres M. Soriano, Jr. and Jose M. Soriano similarly pray that this Court
the main petition has, therefore, become moot and academic. resolve the legal issues raised by the parties in keeping with the "cherished rules of
procedure" that "a court should always strive to settle the entire controversy in a single
On September 29,1977, petitioner filed a second supplemental petition with prayer proceeding leaving no root or branch to bear the seeds of future ligiation", citing Gayong v.
for preliminary injunction, alleging that the actuations of respondent SEC tended to deprive Gayos. 3 To the same effect is the prayer of San Miguel Corporation that this Court resolve on
him of his right to due process, and "that all possible questions on the facts now pending the merits the validity of its amended by laws and the rights and obligations of the parties
before the respondent Commission are now before this Honorable Court which has the thereunder, otherwise "the time spent and effort exerted by the parties concerned and, more
authority and the competence to act on them as it may see fit." (Reno, pp. 927-928.) importantly, by this Honorable Court, would have been for naught because the main question
will come back to this Honorable Court for final resolution." Respondent Eduardo R. Visaya
Petitioner, in his memorandum, submits the following issues for resolution; submits a similar appeal.

(1) whether or not the provisions of the amended by-laws of respondent corporation, It is only the Solicitor General who contends that the case should be remanded to the
disqualifying a competitor from nomination or election to the Board of Directors are valid and SEC for hearing and decision of the issues involved, invoking the latter's primary jurisdiction to
reasonable; hear and decide case involving intra-corporate controversies.

(2) whether or not respondent SEC gravely abused its discretion in denying petitioner's It is an accepted rule of procedure that the Supreme Court should always strive to
request for an examination of the records of San Miguel International, Inc., a fully owned settle the entire controversy in a single proceeding, leaving nor root or branch to bear the
subsidiary of San Miguel Corporation; and seeds of future litigation. 4 Thus, in Francisco v. City of Davao, 5 this Court resolved to decide
the case on the merits instead of remanding it to the trial court for further proceedings since
(3) whether or not respondent SEC committed grave abuse of discretion in allowing the ends of justice would not be subserved by the remand of the case. In Republic v. Security
discussion of Item 6 of the Agenda of the Annual Stockholders' Meeting on May 10, 1977, and Credit and Acceptance Corporation, et al., 6 this Court, finding that the main issue is one of law,
the ratification of the investment in a foreign corporation of the corporate funds, allegedly in resolved to decide the case on the merits "because public interest demands an early
violation of section 17-1/2 of the Corporation Law. disposition of the case", and in Republic v. Central Surety and Insurance Company, 7 this Court
denied remand of the third-party complaint to the trial court for further proceedings, citing
I precedent where this Court, in similar situations resolved to decide the cases on the merits,
Whether or not amended by-laws are valid is purely a legal question which public interest instead of remanding them to the trial court where (a) the ends of justice would not be
requires to be resolved subserved by the remand of the case; or (b) where public interest demand an early disposition
of the case; or (c) where the trial court had already received all the evidence presented by
It is the position of the petitioner that "it is not necessary to remand the case to respondent both parties and the Supreme Court is now in a position, based upon said evidence, to decide
SEC for an appropriate ruling on the intrinsic validity of the amended by-laws in compliance the case on its merits. 8 It is settled that the doctrine of primary jurisdiction has no application
with the principle of exhaustion of administrative remedies", considering that: first: "whether where only a question of law is involved. 8a Because uniformity may be secured through
or not the provisions of the amended by-laws are intrinsically valid ... is purely a legal question. review by a single Supreme Court, questions of law may appropriately be determined in the

23
MATABABE FILES
Corporation Law

first instance by courts. 8b In the case at bar, there are facts which cannot be denied, viz.: that exercised, personally or thru two corporations owned or controlled by him, control over the
the amended by-laws were adopted by the Board of Directors of the San Miguel Corporation following shareholdings in San Miguel Corporation, vis.: (a) John Gokongwei, Jr. 6,325
in the exercise of the power delegated by the stockholders ostensibly pursuant to section 22 shares; (b) Universal Robina Corporation 738,647 shares; (c) CFC Corporation 658,313
of the Corporation Law; that in a special meeting on February 10, 1977 held specially for that shares, or a total of 1,403,285 shares. Since the outstanding capital stock of San Miguel
purpose, the amended by-laws were ratified by more than 80% of the stockholders of record; Corporation, as of the present date, is represented by 33,139,749 shares with a par value of
that the foreign investment in the Hongkong Brewery and Distellery, a beer manufacturing P10.00, the total shares owned or controlled by petitioner represents 4.2344% of the total
company in Hongkong, was made by the San Miguel Corporation in 1948; and that in the outstanding capital stock of San Miguel Corporation. It is also contended that petitioner is the
stockholders' annual meeting held in 1972 and 1977, all foreign investments and operations of president and substantial stockholder of Universal Robina Corporation and CFC Corporation,
San Miguel Corporation were ratified by the stockholders. both of which are allegedly controlled by petitioner and members of his family. It is also
claimed that both the Universal Robina Corporation and the CFC Corporation are engaged in
II businesses directly and substantially competing with the alleged businesses of San Miguel
Corporation, and of corporations in which SMC has substantial investments.
Whether or not the amended by-laws of SMC of disqualifying a competitor from
nomination or election to the Board of Directors of SMC are valid and reasonable ALLEGED AREAS OF COMPETITION BETWEEN PETITIONER'S CORPORATIONS AND SAN MIGUEL
CORPORATION
The validity or reasonableness of a by-law of a corporation in purely a question of
law. 9 Whether the by-law is in conflict with the law of the land, or with the charter of the According to respondent San Miguel Corporation, the areas of, competition are enumerated
corporation, or is in a legal sense unreasonable and therefore unlawful is a question of in its Board the areas of competition are enumerated in its Board Resolution dated April 28,
law. 10 This rule is subject, however, to the limitation that where the reasonableness of a by- 1978, thus:
law is a mere matter of judgment, and one upon which reasonable minds must necessarily
differ, a court would not be warranted in substituting its judgment instead of the judgment of Product Line Estimated Market Share Total
those who are authorized to make by-laws and who have exercised their authority. 11 1977 SMC Robina-CFC
Table Eggs 0.6% 10.0% 10.6%
Petitioner claims that the amended by-laws are invalid and unreasonable because Layer Pullets 33.0% 24.0% 57.0%
they were tailored to suppress the minority and prevent them from having representation in Dressed Chicken 35.0% 14.0% 49.0%
the Board", at the same time depriving petitioner of his "vested right" to be voted for and to Poultry & Hog Feeds 40.0% 12.0% 52.0%
vote for a person of his choice as director. Ice Cream 70.0% 13.0% 83.0%
Instant Coffee 45.0% 40.0% 85.0%
Upon the other hand, respondents Andres M. Soriano, Jr., Jose M. Soriano and San Woven Fabrics 17.5% 9.1% 26.6%
Miguel Corporation content that ex. conclusion of a competitor from the Board is legitimate
corporate purpose, considering that being a competitor, petitioner cannot devote an Thus, according to respondent SMC, in 1976, the areas of competition affecting SMC
unselfish and undivided Loyalty to the corporation; that it is essentially a preventive measure involved product sales of over P400 million or more than 20% of the P2 billion total product
to assure stockholders of San Miguel Corporation of reasonable protective from the sales of SMC. Significantly, the combined market shares of SMC and CFC-Robina in layer
unrestrained self-interest of those charged with the promotion of the corporate enterprise; pullets dressed chicken, poultry and hog feeds ice cream, instant coffee and woven fabrics
that access to confidential information by a competitor may result either in the promotion of would result in a position of such dominance as to affect the prevailing market factors.
the interest of the competitor at the expense of the San Miguel Corporation, or the
promotion of both the interests of petitioner and respondent San Miguel Corporation, which It is further asserted that in 1977, the CFC-Robina group was in direct competition on
may, therefore, result in a combination or agreement in violation of Article 186 of the Revised product lines which, for SMC, represented sales amounting to more than ?478 million. In
Penal Code by destroying free competition to the detriment of the consuming public. It is addition, CFC-Robina was directly competing in the sale of coffee with Filipro, a subsidiary of
further argued that there is not vested right of any stockholder under Philippine Law to be SMC, which product line represented sales for SMC amounting to more than P275 million. The
voted as director of a corporation. It is alleged that petitioner, as of May 6, 1978, has CFC-Robina group (Robitex, excluding Litton Mills recently acquired by petitioner) is

24
MATABABE FILES
Corporation Law

purportedly also in direct competition with Ramie Textile, Inc., subsidiary of SMC, in product In this jurisdiction, under section 21 of the Corporation Law, a corporation may
sales amounting to more than P95 million. The areas of competition between SMC and CFC- prescribe in its by-laws "the qualifications, duties and compensation of directors, officers and
Robina in 1977 represented, therefore, for SMC, product sales of more than P849 million. employees ... " This must necessarily refer to a qualification in addition to that specified by
section 30 of the Corporation Law, which provides that "every director must own in his right
According to private respondents, at the Annual Stockholders' Meeting of March 18, at least one share of the capital stock of the stock corporation of which he is a director ... "
1976, 9,894 stockholders, in person or by proxy, owning 23,436,754 shares in SMC, or more In Government v. El Hogar, 14 the Court sustained the validity of a provision in the corporate by-
than 90% of the total outstanding shares of SMC, rejected petitioner's candidacy for the Board law requiring that persons elected to the Board of Directors must be holders of shares of the
of Directors because they "realized the grave dangers to the corporation in the event a paid up value of P5,000.00, which shall be held as security for their action, on the ground that
competitor gets a board seat in SMC." On September 18, 1978, the Board of Directors of SMC, section 21 of the Corporation Law expressly gives the power to the corporation to provide in
by "virtue of powers delegated to it by the stockholders," approved the amendment to ' he its by-laws for the qualifications of directors and is "highly prudent and in conformity with
by-laws in question. At the meeting of February 10, 1977, these amendments were confirmed good practice. "
and ratified by 5,716 shareholders owning 24,283,945 shares, or more than 80% of the total
outstanding shares. Only 12 shareholders, representing 7,005 shares, opposed the NO VESTED RIGHT OF STOCKHOLDER TO BE ELECTED DIRECTOR
confirmation and ratification. At the Annual Stockholders' Meeting of May 10, 1977, 11,349 Any person "who buys stock in a corporation does so with the knowledge that its
shareholders, owning 27,257.014 shares, or more than 90% of the outstanding shares, rejected affairs are dominated by a majority of the stockholders and that he impliedly contracts that the
petitioner's candidacy, while 946 stockholders, representing 1,648,801 shares voted for him. will of the majority shall govern in all matters within the limits of the act of incorporation and
On the May 9, 1978 Annual Stockholders' Meeting, 12,480 shareholders, owning more than 30 lawfully enacted by-laws and not forbidden by law." 15 To this extent, therefore, the
million shares, or more than 90% of the total outstanding shares. voted against petitioner. stockholder may be considered to have "parted with his personal right or privilege to regulate
the disposition of his property which he has invested in the capital stock of the corporation,
AUTHORITY OF CORPORATION TO PRESCRIBE QUALIFICATIONS OF DIRECTORS and surrendered it to the will of the majority of his fellow incorporators. ... It cannot therefore
EXPRESSLY CONFERRED BY LAW be justly said that the contract, express or implied, between the corporation and the
stockholders is infringed ... by any act of the former which is authorized by a majority ... ." 16
Private respondents contend that the disputed amended by laws were adopted by
the Board of Directors of San Miguel Corporation a-, a measure of self-defense to protect the Pursuant to section 18 of the Corporation Law, any corporation may amend its articles
corporation from the clear and present danger that the election of a business competitor to of incorporation by a vote or written assent of the stockholders representing at least two-thirds
the Board may cause upon the corporation and the other stockholders inseparable prejudice. of the subscribed capital stock of the corporation If the amendment changes, diminishes or
Submitted for resolution, therefore, is the issue whether or not respondent San Miguel restricts the rights of the existing shareholders then the disenting minority has only one right,
Corporation could, as a measure of self- protection, disqualify a competitor from nomination viz.: "to object thereto in writing and demand payment for his share." Under section 22 of the
and election to its Board of Directors. same law, the owners of the majority of the subscribed capital stock may amend or repeal any
by-law or adopt new by-laws. It cannot be said, therefore, that petitioner has a vested right to
It is recognized by an authorities that 'every corporation has the inherent power to be elected director, in the face of the fact that the law at the time such right as stockholder
adopt by-laws 'for its internal government, and to regulate the conduct and prescribe the was acquired contained the prescription that the corporate charter and the by-law shall be
rights and duties of its members towards itself and among themselves in reference to the subject to amendment, alteration and modification. 17
management of its affairs. 12 At common law, the rule was "that the power to make and adopt
by-laws was inherent in every corporation as one of its necessary and inseparable legal It being settled that the corporation has the power to provide for the qualifications
incidents. And it is settled throughout the United States that in the absence of positive of its directors, the next question that must be considered is whether the disqualification of a
legislative provisions limiting it, every private corporation has this inherent power as one of its competitor from being elected to the Board of Directors is a reasonable exercise of corporate
necessary and inseparable legal incidents, independent of any specific enabling provision in its authority.
charter or in general law, such power of self-government being essential to enable the
corporation to accomplish the purposes of its creation. 13 A DIRECTOR STANDS IN A FIDUCIARY RELATION TO THE CORPORATION AND ITS
SHAREHOLDERS

25
MATABABE FILES
Corporation Law

interest at heart, and it would simply be going far to deny by mere


Although in the strict and technical sense, directors of a private corporation are not implication the existence of such a salutary power
regarded as trustees, there cannot be any doubt that their character is that of a fiduciary
insofar as the corporation and the stockholders as a body are concerned. As agents entrusted ... If the by-law is to be held reasonable in disqualifying a stockholder in a competing
with the management of the corporation for the collective benefit of the stockholders, "they company from being a director, the same reasoning would apply to disqualify the wife and
occupy a fiduciary relation, and in this sense the relation is one of trust." 18 "The ordinary trust immediate member of the family of such stockholder, on account of the supposed interest of
relationship of directors of a corporation and stockholders", according to Ashaman v. the wife in her husband's affairs, and his suppose influence over her. It is perhaps true that
Miller, 19 "is not a matter of statutory or technical law. It springs from the fact that directors such stockholders ought not to be condemned as selfish and dangerous to the best interest
have the control and guidance of corporate affairs and property and hence of the property of the corporation until tried and tested. So it is also true that we cannot condemn as selfish
interests of the stockholders. Equity recognizes that stockholders are the proprietors of the and dangerous and unreasonable the action of the board in passing the by-law. The strife over
corporate interests and are ultimately the only beneficiaries thereof * * *. the matter of control in this corporation as in many others is perhaps carried on not
altogether in the spirit of brotherly love and affection. The only test that we can apply is as to
Justice Douglas, in Pepper v. Litton, 20 emphatically restated the standard of fiduciary whether or not the action of the Board is authorized and sanctioned by law. ... . 22
obligation of the directors of corporations, thus:
These principles have been applied by this Court in previous cases. 23
A director is a fiduciary. ... Their powers are powers in trust. ... He
who is in such fiduciary position cannot serve himself first and his cestuis AN AMENDMENT TO THE CORPORATION BY-LAW WHICH RENDERS A STOCKHOLDER
second. ... He cannot manipulate the affairs of his corporation to their INELIGIBLE TO BE DIRECTOR, IF HE BE ALSO DIRECTOR IN A CORPORATION WHOSE BUSINESS
detriment and in disregard of the standards of common decency. He cannot IS IN COMPETITION WITH THAT OF THE OTHER CORPORATION, HAS BEEN SUSTAINED AS VALID
by the intervention of a corporate entity violate the ancient precept against
serving two masters ... He cannot utilize his inside information and strategic It is a settled state law in the United States, according to Fletcher, that corporations
position for his own preferment. He cannot violate rules of fair play by doing have the power to make by-laws declaring a person employed in the service of a rival
indirectly through the corporation what he could not do so directly. He company to be ineligible for the corporation's Board of Directors. ... (A)n amendment which
cannot violate rules of fair play by doing indirectly though the corporation renders ineligible, or if elected, subjects to removal, a director if he be also a director in a
what he could not do so directly. He cannot use his power for his personal corporation whose business is in competition with or is antagonistic to the other corporation
advantage and to the detriment of the stockholders and creditors no matter is valid." 24 This is based upon the principle that where the director is so employed in the
how absolute in terms that power may be and no matter how meticulous he service of a rival company, he cannot serve both, but must betray one or the other. Such an
is to satisfy technical requirements. For that power is at all times subject to amendment "advances the benefit of the corporation and is good." An exception exists in
the equitable limitation that it may not be exercised for the aggrandizement, New Jersey, where the Supreme Court held that the Corporation Law in New Jersey
preference or advantage of the fiduciary to the exclusion or detriment of the prescribed the only qualification, and therefore the corporation was not empowered to add
cestuis. additional qualifications. 25 This is the exact opposite of the situation in the Philippines
because as stated heretofore, section 21 of the Corporation Law expressly provides that a
And in Cross v. West Virginia Cent, & P. R. R. Co., 21 it was said: corporation may make by-laws for the qualifications of directors. Thus, it has been held that
an officer of a corporation cannot engage in a business in direct competition with that of the
... A person cannot serve two hostile and adverse master, without corporation where he is a director by utilizing information he has received as such officer,
detriment to one of them. A judge cannot be impartial if personally under "the established law that a director or officer of a corporation may not enter into a
interested in the cause. No more can a director. Human nature is too weak - competing enterprise which cripples or injures the business of the corporation of which he is
for this. Take whatever statute provision you please giving power to an officer or director. 26
stockholders to choose directors, and in none will you find any express
prohibition against a discretion to select directors having the company's It is also well established that corporate officers "are not permitted to use their
position of trust and confidence to further their private interests." 27 In a case where directors

26
MATABABE FILES
Corporation Law

of a corporation cancelled a contract of the corporation for exclusive sale of a foreign firm's corporation as a legitimate object of by-law protection. With respect to
products, and after establishing a rival business, the directors entered into a new contract attorneys or persons associated with a firm which is attorney for another
themselves with the foreign firm for exclusive sale of its products, the court held that equity bank, in addition to the direct conflict or potential conflict of interest, there
would regard the new contract as an offshoot of the old contract and, therefore, for the is also the danger of inadvertent leakage of confidential information through
benefit of the corporation, as a "faultless fiduciary may not reap the fruits of his misconduct casual office discussions or accessibility of files. Defendant's directors
to the exclusion of his principal. 28 determined that its welfare was best protected if this opportunity for
conflicting loyalties and potential misuse and leakage of confidential
The doctrine of "corporate opportunity" 29 is precisely a recognition by the courts information was foreclosed.
that the fiduciary standards could not be upheld where the fiduciary was acting for two
entities with competing interests. This doctrine rests fundamentally on the unfairness, in In McKee the Court further listed qualificational by-laws upheld by the courts, as follows:
particular circumstances, of an officer or director taking advantage of an opportunity for his
own personal profit when the interest of the corporation justly calls for protection. 30 (1) A director shall not be directly or indirectly interested as a stockholder in
any other firm, company, or association which competes with the subject
It is not denied that a member of the Board of Directors of the San Miguel corporation.
Corporation has access to sensitive and highly confidential information, such as: (a) marketing
strategies and pricing structure; (b) budget for expansion and diversification; (c) research and (2) A director shall not be the immediate member of the family of any
development; and (d) sources of funding, availability of personnel, proposals of mergers or stockholder in any other firm, company, or association which competes with
tie-ups with other firms. the subject corporation,

It is obviously to prevent the creation of an opportunity for an officer or director of (3) A director shall not be an officer, agent, employee, attorney, or trustee in
San Miguel Corporation, who is also the officer or owner of a competing corporation, from any other firm, company, or association which compete with the subject
taking advantage of the information which he acquires as director to promote his individual or corporation.
corporate interests to the prejudice of San Miguel Corporation and its stockholders, that the
questioned amendment of the by-laws was made. Certainly, where two corporations are (4) A director shall be of good moral character as an essential qualification to
competitive in a substantial sense, it would seem improbable, if not impossible, for the holding office.
director, if he were to discharge effectively his duty, to satisfy his loyalty to both corporations
and place the performance of his corporation duties above his personal concerns. (5) No person who is an attorney against the corporation in a law suit is
eligible for service on the board. (At p. 7.)
Thus, in McKee & Co. v. First National Bank of San Diego, supra the court sustained as
valid and reasonable an amendment to the by-laws of a bank, requiring that its directors These are not based on theorical abstractions but on human experience that a
should not be directors, officers, employees, agents, nominees or attorneys of any other person cannot serve two hostile masters without detriment to one of them.
banking corporation, affiliate or subsidiary thereof. Chief Judge Parker, in McKee, explained
the reasons of the court, thus: The offer and assurance of petitioner that to avoid any possibility of his taking unfair
advantage of his position as director of San Miguel Corporation, he would absent himself
... A bank director has access to a great deal of information concerning the from meetings at which confidential matters would be discussed, would not detract from the
business and plans of a bank which would likely be injurious to the bank if validity and reasonableness of the by-laws here involved. Apart from the impractical results
known to another bank, and it was reasonable and prudent to enlarge this that would ensue from such arrangement, it would be inconsistent with petitioner's primary
minimum disqualification to include any director, officer, employee, agent, motive in running for board membership which is to protect his investments in San Miguel
nominee, or attorney of any other bank in California. The Ashkins case, supra, Corporation. More important, such a proposed norm of conduct would be against all
specifically recognizes protection against rivals and others who might accepted principles underlying a director's duty of fidelity to the corporation, for the policy of
acquire information which might be used against the interests of the the law is to encourage and enforce responsible corporate management. As explained by

27
MATABABE FILES
Corporation Law

Oleck: 31 "The law win not tolerate the passive attitude of directors ... without active and spreading false rumors or making use of any other artifice to restrain free
conscientious participation in the managerial functions of the company. As directors, it is their competition in the market.
duty to control and supervise the day to day business activities of the company or to
promulgate definite policies and rules of guidance with a vigilant eye toward seeing to it that 3. Any person who, being a manufacturer, producer, or processor of any
these policies are carried out. It is only then that directors may be said to have fulfilled their merchandise or object of commerce or an importer of any merchandise or
duty of fealty to the corporation." object of commerce from any foreign country, either as principal or agent,
wholesale or retailer, shall combine, conspire or agree in any manner with
Sound principles of corporate management counsel against sharing sensitive any person likewise engaged in the manufacture, production, processing,
information with a director whose fiduciary duty of loyalty may well require that he disclose assembling or importation of such merchandise or object of commerce or
this information to a competitive arrival. These dangers are enhanced considerably where the with any other persons not so similarly engaged for the purpose of making
common director such as the petitioner is a controlling stockholder of two of the competing transactions prejudicial to lawful commerce, or of increasing the market
corporations. It would seem manifest that in such situations, the director has an economic price in any part of the Philippines, or any such merchandise or object of
incentive to appropriate for the benefit of his own corporation the corporate plans and commerce manufactured, produced, processed, assembled in or imported
policies of the corporation where he sits as director. into the Philippines, or of any article in the manufacture of which such
manufactured, produced, processed, or imported merchandise or object of
Indeed, access by a competitor to confidential information regarding marketing commerce is used.
strategies and pricing policies of San Miguel Corporation would subject the latter to a
competitive disadvantage and unjustly enrich the competitor, for advance knowledge by the There are other legislation in this jurisdiction, which prohibit monopolies and
competitor of the strategies for the development of existing or new markets of existing or combinations in restraint of trade. 33
new products could enable said competitor to utilize such knowledge to his advantage. 32
There is another important consideration in determining whether or not the Basically, these anti-trust laws or laws against monopolies or combinations in
amended by-laws are reasonable. The Constitution and the law prohibit combinations in restraint of trade are aimed at raising levels of competition by improving the consumers'
restraint of trade or unfair competition. Thus, section 2 of Article XIV of the Constitution effectiveness as the final arbiter in free markets. These laws are designed to preserve free and
provides: "The State shall regulate or prohibit private monopolies when the public interest so unfettered competition as the rule of trade. "It rests on the premise that the unrestrained
requires. No combinations in restraint of trade or unfair competition shall be snowed." interaction of competitive forces will yield the best allocation of our economic resources, the
lowest prices and the highest quality ... ." 34 they operate to forestall concentration of
Article 186 of the Revised Penal Code also provides: economic power. 35 The law against monopolies and combinations in restraint of trade is
aimed at contracts and combinations that, by reason of the inherent nature of the
Art. 186. Monopolies and combinations in restraint of trade. The penalty of contemplated acts, prejudice the public interest by unduly restraining competition or unduly
prision correccional in its minimum period or a fine ranging from two obstructing the course of trade. 36
hundred to six thousand pesos, or both, shall be imposed upon:
The terms "monopoly", "combination in restraint of trade" and "unfair competition"
1. Any person who shall enter into any contract or agreement or shall take appear to have a well defined meaning in other jurisdictions. A "monopoly" embraces any
part in any conspiracy or combination in the form of a trust or otherwise, in combination the tendency of which is to prevent competition in the broad and general sense,
restraint of trade or commerce or to prevent by artificial means free or to control prices to the detriment of the public. 37 In short, it is the concentration of
competition in the market. business in the hands of a few. The material consideration in determining its existence is not
that prices are raised and competition actually excluded, but that power exists to raise prices
2. Any person who shag monopolize any merchandise or object of trade or or exclude competition when desired. 38Further, it must be considered that the Idea of
commerce, or shall combine with any other person or persons to monopoly is now understood to include a condition produced by the mere act of individuals.
monopolize said merchandise or object in order to alter the price thereof by Its dominant thought is the notion of exclusiveness or unity, or the suppression of

28
MATABABE FILES
Corporation Law

competition by the qualification of interest or management, or it may be thru agreement and Shared information on cost accounting may lead to price fixing. Certainly, shared
concert of action. It is, in brief, unified tactics with regard to prices. 39 information on production, orders, shipments, capacity and inventories may lead to control of
production for the purpose of controlling prices.
From the foregoing definitions, it is apparent that the contentions of petitioner are
not in accord with reality. The election of petitioner to the Board of respondent Corporation Obviously, if a competitor has access to the pricing policy and cost conditions of the
can bring about an illegal situation. This is because an express agreement is not necessary for products of San Miguel Corporation, the essence of competition in a free market for the
the existence of a combination or conspiracy in restraint of trade. 40 It is enough that a purpose of serving the lowest priced goods to the consuming public would be frustrated, The
concert of action is contemplated and that the defendants conformed to the competitor could so manipulate the prices of his products or vary its marketing strategies by
arrangements, 41 and what is to be considered is what the parties actually did and not the region or by brand in order to get the most out of the consumers. Where the two competing
words they used. For instance, the Clayton Act prohibits a person from serving at the same firms control a substantial segment of the market this could lead to collusion and combination
time as a director in any two or more corporations, if such corporations are, by virtue of their in restraint of trade. Reason and experience point to the inevitable conclusion that the
business and location of operation, competitors so that the elimination of competition inherent tendency of interlocking directorates between companies that are related to each
between them would constitute violation of any provision of the anti-trust laws. 42 There is other as competitors is to blunt the edge of rivalry between the corporations, to seek out
here a statutory recognition of the anti-competitive dangers which may arise when an ways of compromising opposing interests, and thus eliminate competition. As respondent
individual simultaneously acts as a director of two or more competing corporations. A SMC aptly observes, knowledge by CFC-Robina of SMC's costs in various industries and
common director of two or more competing corporations would have access to confidential regions in the country win enable the former to practice price discrimination. CFC-Robina can
sales, pricing and marketing information and would be in a position to coordinate policies or segment the entire consuming population by geographical areas or income groups and
to aid one corporation at the expense of another, thereby stifling competition. This situation change varying prices in order to maximize profits from every market segment. CFC-Robina
has been aptly explained by Travers, thus: could determine the most profitable volume at which it could produce for every product line
in which it competes with SMC. Access to SMC pricing policy by CFC-Robina would in effect
The argument for prohibiting competing corporations from sharing destroy free competition and deprive the consuming public of opportunity to buy goods of
even one director is that the interlock permits the coordination of policies the highest possible quality at the lowest prices.
between nominally independent firms to an extent that competition between
them may be completely eliminated. Indeed, if a director, for example, is to Finally, considering that both Robina and SMC are, to a certain extent, engaged in
be faithful to both corporations, some accommodation must result. Suppose agriculture, then the election of petitioner to the Board of SMC may constitute a violation of
X is a director of both Corporation A and Corporation B. X could hardly vote the prohibition contained in section 13(5) of the Corporation Law. Said section provides in
for a policy by A that would injure B without violating his duty of loyalty to B part that "any stockholder of more than one corporation organized for the purpose of
at the same time he could hardly abstain from voting without depriving A of engaging in agriculture may hold his stock in such corporations solely for investment and not
his best judgment. If the firms really do compete in the sense of vying for for the purpose of bringing about or attempting to bring about a combination to exercise
economic advantage at the expense of the other there can hardly be any control of incorporations ... ."
reason for an interlock between competitors other than the suppression of
competition. 43 (Emphasis supplied.) Neither are We persuaded by the claim that the by-law was Intended to prevent the
candidacy of petitioner for election to the Board. If the by-law were to be applied in the case
According to the Report of the House Judiciary Committee of the U. S. Congress on of one stockholder but waived in the case of another, then it could be reasonably claimed that
section 9 of the Clayton Act, it was established that: "By means of the interlocking the by-law was being applied in a discriminatory manner. However, the by law, by its terms,
directorates one man or group of men have been able to dominate and control a great applies to all stockholders. The equal protection clause of the Constitution requires only that
number of corporations ... to the detriment of the small ones dependent upon them and to the by-law operate equally upon all persons of a class. Besides, before petitioner can be
the injury of the public. 44 declared ineligible to run for director, there must be hearing and evidence must be submitted
to bring his case within the ambit of the disqualification. Sound principles of public policy and
management, therefore, support the view that a by-law which disqualifies a competition from
election to the Board of Directors of another corporation is valid and reasonable.

29
MATABABE FILES
Corporation Law

Whether or not respondent SEC gravely abused its discretion in denying petitioner's request for
In the absence of any legal prohibition or overriding public policy, wide latitude may an examination of the records of San Miguel International Inc., a fully owned subsidiary of San
be accorded to the corporation in adopting measures to protect legitimate corporation Miguel Corporation
interests. Thus, "where the reasonableness of a by-law is a mere matter of judgment, and
upon which reasonable minds must necessarily differ, a court would not be warranted in Respondent San Miguel Corporation stated in its memorandum that petitioner's
substituting its judgment instead of the judgment of those who are authorized to make by- claim that he was denied inspection rights as stockholder of SMC "was made in the teeth of
laws and who have expressed their authority. 45 undisputed facts that, over a specific period, petitioner had been furnished numerous
documents and information," to wit: (1) a complete list of stockholders and their
Although it is asserted that the amended by-laws confer on the present Board stockholdings; (2) a complete list of proxies given by the stockholders for use at the annual
powers to perpetua themselves in power such fears appear to be misplaced. This power, but stockholders' meeting of May 18, 1975; (3) a copy of the minutes of the stockholders' meeting
is very nature, is subject to certain well established limitations. One of these is inherent in the of March 18,1976; (4) a breakdown of SMC's P186.6 million investment in associated
very convert and definition of the terms "competition" and "competitor". "Competition" companies and other companies as of December 31, 1975; (5) a listing of the salaries,
implies a struggle for advantage between two or more forces, each possessing, in allowances, bonuses and other compensation or remunerations received by the directors and
substantially similar if not Identical degree, certain characteristics essential to the business corporate officers of SMC; (6) a copy of the US $100 million Euro-Dollar Loan Agreement of
sought. It means an independent endeavor of two or more persons to obtain the business SMC; and (7) copies of the minutes of all meetings of the Board of Directors from January
patronage of a third by offering more advantageous terms as an inducement to secure 1975 to May 1976, with deletions of sensitive data, which deletions were not objected to by
trade. 46 The test must be whether the business does in fact compete, not whether it is petitioner.
capable of an indirect and highly unsubstantial duplication of an isolated or non- Further, it was averred that upon request, petitioner was informed in writing on September
characteristics activity. 47 It is, therefore, obvious that not every person or entity engaged in 18, 1976; (1) that SMC's foreign investments are handled by San Miguel International, Inc.,
business of the same kind is a competitor. Such factors as quantum and place of business, incorporated in Bermuda and wholly owned by SMC; this was SMC's first venture abroad,
Identity of products and area of competition should be taken into consideration. It is, having started in 1948 with an initial outlay of ?500,000.00, augmented by a loan of Hongkong
therefore, necessary to show that petitioner's business covers a substantial portion of the $6 million from a foreign bank under the personal guaranty of SMC's former President, the
same markets for similar products to the extent of not less than 10% of respondent late Col. Andres Soriano; (2) that as of December 31, 1975, the estimated value of SMI would
corporation's market for competing products. While We here sustain the validity of the amount to almost P400 million (3) that the total cash dividends received by SMC from SMI
amended by-laws, it does not follow as a necessary consequence that petitioner is ipso since 1953 has amount to US $ 9.4 million; and (4) that from 1972-1975, SMI did not declare
facto disqualified. Consonant with the requirement of due process, there must be due hearing cash or stock dividends, all earnings having been used in line with a program for the setting
at which the petitioner must be given the fullest opportunity to show that he is not covered up of breweries by SMI
by the disqualification. As trustees of the corporation and of the stockholders, it is the
responsibility of directors to act with fairness to the stockholders. 48 Pursuant to this These averments are supported by the affidavit of the Corporate Secretary, enclosing
obligation and to remove any suspicion that this power may be utilized by the incumbent photocopies of the afore-mentioned documents. 51
members of the Board to perpetuate themselves in power, any decision of the Board to
disqualify a candidate for the Board of Directors should be reviewed by the Securities behind Pursuant to the second paragraph of section 51 of the Corporation Law, "(t)he record
Exchange Commission en banc and its decision shall be final unless reversed by this Court on of all business transactions of the corporation and minutes of any meeting shall be open to
certiorari. 49 Indeed, it is a settled principle that where the action of a Board of Directors is an the inspection of any director, member or stockholder of the corporation at reasonable
abuse of discretion, or forbidden by statute, or is against public policy, or is ultra vires, or is a hours."
fraud upon minority stockholders or creditors, or will result in waste, dissipation or
misapplication of the corporation assets, a court of equity has the power to grant appropriate The stockholder's right of inspection of the corporation's books and records is based
relief. 50 upon their ownership of the assets and property of the corporation. It is, therefore, an
incident of ownership of the corporate property, whether this ownership or interest be
III termed an equitable ownership, a beneficial ownership, or a ownership. 52 This right is
predicated upon the necessity of self-protection. It is generally held by majority of the courts

30
MATABABE FILES
Corporation Law

that where the right is granted by statute to the stockholder, it is given to him as such and records in another jurisdiction, and is not legally subject to the control of the parent company,
must be exercised by him with respect to his interest as a stockholder and for some purpose although it owned a vast majority of the stock of the subsidiary. 63 Likewise, inspection of the
germane thereto or in the interest of the corporation. 53 In other words, the inspection has to books of an allied corporation by stockholder of the parent company which owns all the stock
be germane to the petitioner's interest as a stockholder, and has to be proper and lawful in of the subsidiary has been refused on the ground that the stockholder was not within the
character and not inimical to the interest of the corporation. 54 In Grey v. Insular Lumber, 55 this class of "persons having an interest." 64
Court held that "the right to examine the books of the corporation must be exercised in good
faith, for specific and honest purpose, and not to gratify curiosity, or for specific and honest In the Nash case, 65 The Supreme Court of New York held that the contractual right of
purpose, and not to gratify curiosity, or for speculative or vexatious purposes. The weight of former stockholders to inspect books and records of the corporation included the right to
judicial opinion appears to be, that on application for mandamus to enforce the right, it is inspect corporation's subsidiaries' books and records which were in corporation's possession
proper for the court to inquire into and consider the stockholder's good faith and his purpose and control in its office in New York."
and motives in seeking inspection. 56 Thus, it was held that "the right given by statute is not
absolute and may be refused when the information is not sought in good faith or is used to In the Bailey case, 66 stockholders of a corporation were held entitled to inspect the
the detriment of the corporation." 57 But the "impropriety of purpose such as will defeat records of a controlled subsidiary corporation which used the same offices and had Identical
enforcement must be set up the corporation defensively if the Court is to take cognizance of officers and directors.
it as a qualification. In other words, the specific provisions take from the stockholder the
burden of showing propriety of purpose and place upon the corporation the burden of In his "Urgent Motion for Production and Inspection of Documents" before
showing impropriety of purpose or motive. 58 It appears to be the general rule that respondent SEC, petitioner contended that respondent corporation "had been attempting to
stockholders are entitled to full information as to the management of the corporation and the suppress information for the stockholders" and that petitioner, "as stockholder of
manner of expenditure of its funds, and to inspection to obtain such information, especially respondent corporation, is entitled to copies of some documents which for some reason or
where it appears that the company is being mismanaged or that it is being managed for the another, respondent corporation is very reluctant in revealing to the petitioner
personal benefit of officers or directors or certain of the stockholders to the exclusion of notwithstanding the fact that no harm would be caused thereby to the corporation." 67 There
others." 59 is no question that stockholders are entitled to inspect the books and records of a
corporation in order to investigate the conduct of the management, determine the financial
While the right of a stockholder to examine the books and records of a corporation condition of the corporation, and generally take an account of the stewardship of the officers
for a lawful purpose is a matter of law, the right of such stockholder to examine the books and directors. 68
and records of a wholly-owned subsidiary of the corporation in which he is a stockholder is a
different thing. In the case at bar, considering that the foreign subsidiary is wholly owned by
respondent San Miguel Corporation and, therefore, under its control, it would be more in
Some state courts recognize the right under certain conditions, while others do not. accord with equity, good faith and fair dealing to construe the statutory right of petitioner as
Thus, it has been held that where a corporation owns approximately no property except the stockholder to inspect the books and records of the corporation as extending to books and
shares of stock of subsidiary corporations which are merely agents or instrumentalities of the records of such wholly subsidiary which are in respondent corporation's possession and
holding company, the legal fiction of distinct corporate entities may be disregarded and the control.
books, papers and documents of all the corporations may be required to be produced for
examination, 60 and that a writ of mandamus, may be granted, as the records of the subsidiary IV
were, to all incontents and purposes, the records of the parent even though subsidiary was Whether or not respondent SEC gravely abused its discretion in allowing the stockholders of
not named as a party. 61 mandamus was likewise held proper to inspect both the subsidiary's respondent corporation to ratify the investment of corporate funds in a foreign corporation
and the parent corporation's books upon proof of sufficient control or dominion by the
parent showing the relation of principal or agent or something similar thereto. 62 Petitioner reiterates his contention in SEC Case No. 1423 that respondent corporation
invested corporate funds in SMI without prior authority of the stockholders, thus violating
On the other hand, mandamus at the suit of a stockholder was refused where the section 17-1/2 of the Corporation Law, and alleges that respondent SEC should have
subsidiary corporation is a separate and distinct corporation domiciled and with its books and

31
MATABABE FILES
Corporation Law

investigated the charge, being a statutory offense, instead of allowing ratification of the accomplish the purpose of its incorporation, the vote of approval of the
investment by the stockholders. stockholders is necessary. In any case, the purchase of such shares or
Respondent SEC's position is that submission of the investment to the stockholders for securities must be subject to the limitations established by the Corporations
ratification is a sound corporate practice and should not be thwarted but encouraged. law; namely, (a) that no agricultural or mining corporation shall be restricted
to own not more than 15% of the voting stock of nay agricultural or mining
Section 17-1/2 of the Corporation Law allows a corporation to "invest its funds in any corporation; and (c) that such holdings shall be solely for investment and not
other corporation or business or for any purpose other than the main purpose for which it for the purpose of bringing about a monopoly in any line of commerce of
was organized" provided that its Board of Directors has been so authorized by the affirmative combination in restraint of trade." The Philippine Corporation Law by
vote of stockholders holding shares entitling them to exercise at least two-thirds of the Sulpicio S. Guevara, 1967 Ed., p. 89) (Emphasis supplied.)
voting power. If the investment is made in pursuance of the corporate purpose, it does not 40. Power to invest corporate funds. A private corporation has the power
need the approval of the stockholders. It is only when the purchase of shares is done solely to invest its corporate funds "in any other corporation or business, or for any
for investment and not to accomplish the purpose of its incorporation that the vote of purpose other than the main purpose for which it was organized, provide
approval of the stockholders holding shares entitling them to exercise at least two-thirds of that 'its board of directors has been so authorized in a resolution by the
the voting power is necessary. 69 affirmative vote of stockholders holding shares in the corporation entitling
them to exercise at least two-thirds of the voting power on such a propose
As stated by respondent corporation, the purchase of beer manufacturing facilities at a stockholders' meeting called for that purpose,' and provided further,
by SMC was an investment in the same business stated as its main purpose in its Articles of that no agricultural or mining corporation shall in anywise be interested in
Incorporation, which is to manufacture and market beer. It appears that the original any other agricultural or mining corporation. When the investment is
investment was made in 1947-1948, when SMC, then San Miguel Brewery, Inc., purchased a necessary to accomplish its purpose or purposes as stated in its articles of
beer brewery in Hongkong (Hongkong Brewery & Distillery, Ltd.) for the manufacture and incorporation the approval of the stockholders is not necessary."" (Id., p. 108)
marketing of San Miguel beer thereat. Restructuring of the investment was made in 1970-1971 (Emphasis ours.) (pp. 258-259).
thru the organization of SMI in Bermuda as a tax free reorganization.
Assuming arguendo that the Board of Directors of SMC had no authority to make the
Under these circumstances, the ruling in De la Rama v. Manao Sugar Central Co., Inc., assailed investment, there is no question that a corporation, like an individual, may ratify and
supra, appears relevant. In said case, one of the issues was the legality of an investment made thereby render binding upon it the originally unauthorized acts of its officers or other
by Manao Sugar Central Co., Inc., without prior resolution approved by the affirmative vote of agents. 70 This is true because the questioned investment is neither contrary to law, morals,
2/3 of the stockholders' voting power, in the Philippine Fiber Processing Co., Inc., a company public order or public policy. It is a corporate transaction or contract which is within the
engaged in the manufacture of sugar bags. The lower court said that "there is more logic in corporate powers, but which is defective from a supported failure to observe in its execution
the stand that if the investment is made in a corporation whose business is important to the the. requirement of the law that the investment must be authorized by the affirmative vote of
investing corporation and would aid it in its purpose, to require authority of the stockholders the stockholders holding two-thirds of the voting power. This requirement is for the benefit
would be to unduly curtail the power of the Board of Directors." This Court affirmed the ruling of the stockholders. The stockholders for whose benefit the requirement was enacted may,
of the court a quo on the matter and, quoting Prof. Sulpicio S. Guevara, said: therefore, ratify the investment and its ratification by said stockholders obliterates any defect
which it may have had at the outset. "Mere ultra vires acts", said this Court in Pirovano, 71 "or
"j. Power to acquire or dispose of shares or securities. A private those which are not illegal and void ab initio, but are not merely within the scope of the
corporation, in order to accomplish is purpose as stated in its articles of articles of incorporation, are merely voidable and may become binding and enforceable when
incorporation, and subject to the limitations imposed by the Corporation ratified by the stockholders.
Law, has the power to acquire, hold, mortgage, pledge or dispose of shares,
bonds, securities, and other evidence of indebtedness of any domestic or Besides, the investment was for the purchase of beer manufacturing and marketing
foreign corporation. Such an act, if done in pursuance of the corporate facilities which is apparently relevant to the corporate purpose. The mere fact that
purpose, does not need the approval of stockholders; but when the purchase respondent corporation submitted the assailed investment to the stockholders for ratification
of shares of another corporation is done solely for investment and not to at the annual meeting of May 10, 1977 cannot be construed as an admission that respondent

32
MATABABE FILES
Corporation Law

corporation had committed an ultra vires act, considering the common practice of In resume, subject to the qualifications aforestated judgment is hereby rendered
corporations of periodically submitting for the gratification of their stockholders the acts of GRANTING the petition by allowing petitioner to examine the books and records of San
their directors, officers and managers. Miguel International, Inc. as specified in the petition. The petition, insofar as it assails the
validity of the amended by- laws and the ratification of the foreign investment of respondent
WHEREFORE, judgment is hereby rendered as follows: corporation, for lack of necessary votes, is hereby DISMISSED. No costs.

The Court voted unanimously to grant the petition insofar as it prays that petitioner
be allowed to examine the books and records of San Miguel International, Inc., as specified by
him.

On the matter of the validity of the amended by-laws of respondent San Miguel
Corporation, six (6) Justices, namely, Justices Barredo, Makasiar, Antonio, Santos, Abad
Santos and De Castro, voted to sustain the validity per se of the amended by-laws in question
and to dismiss the petition without prejudice to the question of the actual disqualification of
petitioner John Gokongwei, Jr. to run and if elected to sit as director of respondent San
Miguel Corporation being decided, after a new and proper hearing by the Board of Directors
of said corporation, whose decision shall be appealable to the respondent Securities and
Exchange Commission deliberating and acting en banc and ultimately to this Court. Unless
disqualified in the manner herein provided, the prohibition in the afore-mentioned amended
by-laws shall not apply to petitioner.

The afore-mentioned six (6) Justices, together with Justice Fernando, voted to
declare the issue on the validity of the foreign investment of respondent corporation as moot.

Chief Justice Fred Ruiz Castro reserved his vote on the validity of the amended by-
laws, pending hearing by this Court on the applicability of section 13(5) of the Corporation
Law to petitioner.

Justice Fernando reserved his vote on the validity of subject amendment to the by-
laws but otherwise concurs in the result.

Four (4) Justices, namely, Justices Teehankee, Concepcion, Jr., Fernandez and
Guerrero filed a separate opinion, wherein they voted against the validity of the questioned
amended bylaws and that this question should properly be resolved first by the SEC as the
agency of primary jurisdiction. They concur in the result that petitioner may be allowed to run
for and sit as director of respondent SMC in the scheduled May 6, 1979 election and
subsequent elections until disqualified after proper hearing by the respondent's Board of
Directors and petitioner's disqualification shall have been sustained by respondent SEC en
banc and ultimately by final judgment of this Court.

33
MATABABE FILES
Corporation Law

Republic of the Philippines least two-thirds (2/3) of the voting power in the corporation. The same requirement is found
SUPREME COURT in Section 40 of the present Corporation Code.
Manila
FIRST DIVISION Same; Deed of Assignment; Civil Law; Donation; Liberality as a consideration in the
deed of assignment of the respondent PAMBUSCO in favor of its former corporate officer for
G.R. No. 91478 February 7, 1991 services rendered is not just an ordinary deed of assignment but a donation.Respondent
court, in upholding the questioned deed of assignment, which appears to be without any
ROSITA PEA petitioner, consideration at all, held that the consideration thereof is the liberality of the respondent
vs. PAMBUSCO in favor of its former corporate officer, respondent Enriquez, for services
THE COURT OF APPEALS, SPOUSES RISING T. YAP and CATALINA YAP, PAMPANGA BUS CO., rendered. Assuming this to be so, then as correctly argued by petitioner, it is not just an
INC., JESUS DOMINGO, JOAQUIN BRIONES, SALVADOR BERNARDEZ, MARCELINO ENRIQUEZ ordinary, deed of assignment, but is in fact a donation. Under Article 725 of the Civil Code, in
and EDGARDO A. ZABAT, respondents. order to be valid, such a donation must be made in a public document and the acceptance
must be made in the same or in a separate instrument. In the latter case, the donor shall be
Corporation Law; By-laws; Quorum; Three (3) out of five (5) members of the board of notified of the acceptance in an authentic form and such step must be noted in both
directors present in the special meeting of respondent PAMBUSCO do not constitute a quorum instruments. Non-compliance with this requirement renders the donation null and void. Since
to validly transact business. Section 4 of its amended by-laws requires at least four (4) undeniably the deed of assignment dated March 8, 1975 in question, shows that there was no
members present to constitute a quorum in a special meeting of its board of directors.The acceptance of the donation in the same and in a separate document, the said deed of
by-laws of a corporation are its own private laws which substantially have the same effect as assignment is thus void ab initio and of no force and effect.
the laws of the corporation. They are in effect, written, into the charter. In this sense they
become part of the fundamental law of the corporation with which the corporation and its PETITION for certiorari to review the decision and resolution of the Court of Appeals.
directors and officers must comply. Apparently, only three (3) out of five (5) members of the
board of directors of respondent PAMBUSCO convened on November 19, 1974 by virtue of a The facts are stated in the opinion of the Court.
prior notice of a special meeting. There was no quorum to validly transact business since,
under Section 4 of the amended by-laws hereinabove reproduced, at least four (4) members Cesar L. Villanueva for petitioner.
must be present to constitute a quorum in a special meeting of the board of directors of Martin N. Roque for private respondents.
respondent PAMBUSCO.
GANCAYCO, J.:
Same; Board of Directors; Only persons who own at least one (1) share in their own
right may qualify to be directors of a corporation.As a matter of fact, the three (3) alleged The validity of the redemption of a foreclosed real property is the center of this
directors who attended the special meeting on November 19,1974 were not listed as directors controversy.
of respondent PAMBUSCO in the latest general information sheet of respondent PAMBUSCO
filed with the SEC dated 18 March 1951. Similarly, the latest list of stockholders of respondent The facts as found by the respondent court are not disputed.
PAMBUSCO on file with the SEC does not show that the said alleged directors were among
the stockholders of respondent PAMBUSCO. Under Section 30 of the then applicable A reading of the records shows that [Pampanga Bus Co.] PAMBUSCO,
Corporation Law, only persons who own at least one (1) share in their own right may qualify original owners of the lots in question under TCT Nos. 4314, 4315 and 4316,
to be directors of a corporation. Further, under Section 28 1/2 ofthe said law, the sale or mortgaged the same to the Development Bank of the Philippines (DBP) on January 3,
disposition of all and/or substantially all properties of the corporation requires, in addition to 1962 in consideration of the amount of P935,000.00. This mortgage was foreclosed.
a proper board resolution, the affirmative votes of the stockholders holding at least two- In the foreclosure sale under Act No. 3135 held on October 25, 1974, the said
thirds (2/3) of the voting power in the corporation in a meeting duly called for that purpose. properties were awarded to Rosita Pea as highest bidder. A certificate of sale was
No doubt, the questioned resolution was not confirmed at a subsequent stockholders issued in her favor by the Senior Deputy Sheriff of Pampanga, Edgardo A. Zabat,
meeting duly called for the purpose by the affirmative votes of the stockholders holding at

34
MATABABE FILES
Corporation Law

upon payment of the sum of P128,000.00 to the Office of the Provincial Sheriff (Exh. On Sept. 10, 1975, the CFI Branch III, Pampanga in the aforementioned Civil
23). The certificate of sale was registered on October 29, 1974 (Exh. G). Case No. 4310, entitled Dante Gutierrez, et al. vs. PAMBUSCO, et al., ordered the
Register of Deeds of Pampanga . . . to desist from registering or noting in his registry
On November 19, 1974, the board of directors of PAMBUSCO, through three of property . . . any of the following documents under contract, until further orders:
(3) out of its five (5) directors, resolved to assign its right of redemption over the
aforesaid lots and authorized one of its members, Atty. Joaquin Briones "to execute (a) Deed of Assignment dated March 18, 1975 executed by the defendant
and sign a Deed of Assignment for and in behalf of PAMBUSCO in favor of any Pampanga Bus Company in virtue of a resolution of its Board of Directors in favor of
interested party . . ." (Exh. 24). Consequently, on March 18, 1975, Briones executed a defendant Marcelino Enriquez;
Deed of Assignment of PAMBUSCO's redemption right over the subject lots in favor
of Marcelino Enriquez (Exh. 25). The latter then redeemed the said properties and a (b) A Certificate of Redemption issued by defendant Deputy Sheriff Edgardo
certificate of redemption dated August 15, 1975 was issued in his favor by Sheriff Zabat in favor of defendant Marcelino Enriquez dated August 15, 1975;
Zabat upon payment of the sum of one hundred forty thousand, four hundred
seventy four pesos P140,474.00) to the Office of the Provincial Sheriff of Pampanga (c) Deed of Sale dated August 16, 1975 executed by defendant Marcelino
(Exh. 26). Enriquez in favor of defendant Rising Yap. (Original Record, p. 244)

A day after the aforesaid certificate was issued, Enriquez executed a deed of On November 17, 1975, the Land Registration Commission opined under LRC
absolute sale of the subject properties in favor of plaintiffs-appellants, the spouses Resolution No. 1029 that "the levy on attachment in favor of Capitol Allied Trading
Rising T. Yap and Catalina Lugue, for the sum of P140,000.00 (Exh. F). (represented by Dante Gutierrez) should be carried over on the new title that would
be issued in the name of Rising Yap in the event that he is able to present the
On August 18, 1975, a levy on attachment in favor of Capitol Allied Trading owner's duplicates of the certificates of title herein involved" (Exh. G).
was entered as an additional encumbrance on TCT Nos. 4314, 4315 and 4316 and a
Notice of a pending consulta was also annotated on the same titles concerning the Meanwhile, defendant Pea, through counsel, wrote the Sheriff asking for
Allied Trading case entitled Dante Gutierrez, et al. vs. PAMBUSCO (Civil Case No. 4310) the execution of a deed of final sale in her favor on the ground that "the one (1) year
in which the registrability of the aforesaid lots in the name of the spouses Yap was period of redemption has long elapsed without any valid redemption having been
sought to be resolved (Exh. 20-F). The certificate of sale issued by the Sheriff in favor exercised;" hence she "will now refuse to receive the redemption money . . . (Exh.
of defendant Pea, the resolution of the PAMBUSCO's board of directors assigning 28).
its redemption rights to any interested party, the deed of assignment PAMBUSCO
executed in favor of Marcelino B. Enriquez, the certificate of redemption issued by On Dec. 30, 1977, plaintiff Yap wrote defendant Pea asking payment of back
the Sheriff in favor of Enriquez as well as the deed of absolute sale of the subject lots rentals in the amount of P42,750.00 "for the use and occupancy of the land and
executed by Enriquez in favor of the plaintiffs-appellants were all annotated on the house located at Sta. Lucia, San Fernando, Pampanga," and informing her of an
same certificates of title likewise on August 18, 1975. Also, on the same date, the increase in monthly rental to P2,000; otherwise, to vacate the premises or face an
Office of the Provincial Sheriff of San Fernando, Pampanga informed defendant- eviction cum collection suit (Exh. D).
appellee by registered mail "that the properties under TCT Nos. 4314, 4315 and 4316 . .
. . were all redeemed by Mr. Marcelino B. Enriquez on August 15,1975 . . . ;" and that In the meantime, the subject lots, formerly under TCT Nos. 4314, 4315 and
she may now get her money at the Sheriffs Office (Exh. J and J-1). 4316 were registered on June 16, 1978 in the name of the spouses Yap under TCT Nos.
On September 8, 1975, Pea wrote the Sheriff notifying him that the redemption was 148983-R, 148984-R and 148985-R, with an annotation of a levy on attachment in
not valid as it was made under a void deed of assignment. She then requested the favor of Capitol Allied Trading. The LRC Resolution No. 1029 allowing the conditioned
recall of the said redemption and a restraint on any registration or transaction registration of the subject lots in the name of the spouses Yap was also annotated on
regarding the lots in question (Exh. 27). TCT No. 4315 on June 16, 1978 and the notice of a pending consulta noted thereon on
August 18, 1975 was cancelled on the same date.

35
MATABABE FILES
Corporation Law

No Trial on the merits was held concerning Civil Case No. 4310. In an order of Deeds to be issued in their names despite an order from the then CFI, Br. III,
dated February 17, 1983, the case was dismissed without prejudice. Pampanga in Civil Case No. 4310, entitled Dante Gutierrez, et al. vs. Pampanga Bus
Company, Inc., et al., to desist from registering or noting in his registry of property . . .
Despite the foregoing, defendant-appellee Pea remained in possession of any of the above-mentioned documents under contest, until further orders. (Record,
the lots in question hence, the spouses Yap were prompted to file the instant case. 1 p. 11).

The antecedents of the present petition are as follows: For its part, defendant Washington Distillery stated that it has never
occupied the subject lots hence they should not have been impleaded in the
Plaintiffs-appellants, the spouses Rising T. Yap and Catalina Lugue, are the complaint.
registered owners of the lots in question under Transfer Certificate of Title (TCT) Nos.
148983-R, 148984-R, 148985-R. In the complaint filed on December 15, 1978, The defendants, therefore, prayed that the complaint be dismissed; that the
appellants sought to recover possession over the subject lands from defendants deed of assignment executed in favor of Marcelino Enriquez, the certificate of
Rosita Pea and Washington Distillery on the ground that being registered owners, redemption issued by the Provincial Sheriff also in favor of Marcelino Enriquez, and
they have to enforce their right to possession against defendants who have been the deed of sale of these parcels of land executed by Marcelino Enriquez in favor of
allegedly in unlawful possession thereof since October 1974 "when the previous the plaintiffs herein be all declared null and void; and further, that TCT Nos. 148983-R,
owners assigned (their) right to collect rentals . . . in favor of plaintiffs" (Record, p. 5). 148984-R and 148985-R, covering these parcels issued in the plaintiffs name be
The amount claimed as damages is pegged on the total amount of unpaid rentals cancelled and, in lieu thereof, corresponding certificates of title over these same
from October 1974 (as taken from the allegations in the complaint) up to December parcels be issued in the name of defendant Rosita Pea.
1978 at a monthly rate of P1,500.00 'and the further sum of P2,000.00 a month from
January 1979 until the defendants finally vacate the . . . premises in question with Thereafter, the defendants with prior leave of court filed a third-party
interest at the legal rate (Record, p. 61). complaint third-party defendants PAMBUSCO, Jesus Domingo, Joaquin Briones,
Salvador Bernardez (as members of the Board of Directors of PAMBUSCO),
In their answer, defendants Rosita Pea and Washington Distillery denied Marcelino Enriquez, and Deputy Sheriff Edgardo Zabat of Pampanga. All these third-
the material allegations of the complaint and by way of an affirmative and special party defendants, how ever, were declared as in default for failure to file their
defense asserted that Pea is now the legitimate owner of the subject lands for answer, except Edgardo Zabat who did file his answer but failed to appear at the pre-
having purchased the same in a foreclosure proceeding instituted by the DBP . . . trial.
against PAMBUSCO . . . and no valid redemption having been effected within the
period provided by law. It was contended that plaintiffs could not have acquired After trial, a decision was rendered by the court in favor of the defendants-
ownership over the subject properties under a deed of absolute sale executed in appellees, to wit:
their favor by one Marcelino B. Enriquez who likewise could not have become [the]
owner of the properties in question by redeeming the same on August 18, 1975 (Exh. WHEREFORE, and in view of all the foregoing, judgment is hereby
26) under an alleged[ly] void deed of assignment executed in his favor on March 18, rendered dismissing the complaint filed by the plaintiffs against the
1975 by the original owners of the land in question, the PAMBUSCO. The defense was defendants and declaring as null and void the following:
that since the deed of assignment executed by PAMBUSCO in favor of Enriquez was
void ab initio for being an ultra vires act of its board of directors and, for being (a) The resolution of the Board of Directors of PAMBUSCO approved on
without any valuable consideration, it could not have had any legal effect; hence, all November 19, 1974 assigning the PAMBUSCO's right of redemption
the acts which flowed from it and all the rights and obligations which derived from concerning the parcels involved herein
the aforesaid void deed are likewise void and without any legal effect.
(b) The deed of assignment dated March 18, 1975 executed in favor of
Further, it was alleged in the same Answer that plaintiffs are buyers in bad Marcelino Enriquez pursuant to the resolution referred to in the preceding
faith because they have caused the titles of the subject properties with the Register paragraph;

36
MATABABE FILES
Corporation Law

THE RESPONDENT COURT OF APPEALS ERRED IN HOLDING THAT THE TRIAL


(c) The certificate of redemption dated August 15, 1975 issued by Deputy COURT HAD NO JURISDICTION TO RULE ON THE VALIDITY OF THE QUESTIONED
Sheriff Edgardo Zabat in favor of Marcelino Enriquez concerning these RESOLUTION AND TRANSFERS.
parcels;
(d) The deed of absolute sale dated August 15, 1975 executed by Marcelino Second Assignment of Error
Enriquez in favor of the plaintiffs concerning the same parcels and
THE RESPONDENT COURT OF APPEALS ERRED IN HOLDING THAT
(e) TCT Nos. 148983-R, 148984-R and 148985-R of the Register of Deeds of PETITIONER HAS NO LEGAL STANDING TO ASSAIL THE VALIDITY OF THE
Pampanga in the name of the plaintiffs also covering these parcels. QUESTIONED RESOLUTION AND THE SERIES OF SUCCEEDING TRANSACTIONS
LEADING TO THE REGISTRATION OF THE SUBJECT PROPERTIES IN FAVOR OF THE
Third-party defendant Edgardo Zabat, in his capacity as Deputy RESPONDENTS YAP.
Sheriff of Pampanga is directed to execute in favor of defendant Rosita Pea
the corresponding certificate of final sale involving the parcels bought by her Third Assignment of Error
in the auction sale of October 25, 1974 for which a certificate of sale had
been issued to her. THE RESPONDENT COURT OF APPEALS ERRED IN HOLDING THAT THE
RESOLUTION OF RESPONDENT PAMBUSCO, ADOPTED ON 19 NOVEMBER 1974,
Finally, the third-party defendants herein except Deputy Sheriff ASSIGNING ITS RIGHT OF REDEMPTION IS NOT VOID OR AT THE VERY LEAST
Edgardo Zabat are hereby ordered to pay the defendants/third party LEGALLY DEFECTIVE.
plaintiffs, jointly and severally, the amount of P10,000.00 as attorney's fees
plus costs.2 Fourth Assignment of Error

Thus, an appeal from said judgment of the trial court was interposed by private THE RESPONDENT COURT OF APPEALS ERRED IN HOLDING THAT THE DEED
respondents to the Court of Appeals wherein in due course a decision was rendered on June OF ASSIGNMENT, DATED 8 MARCH 1975, IN FAVOR OF RESPONDENT ENRIQUEZ IS
20, 1989, the dispositive part of which reads as follows: NOT VOID OR AT THE VERY LEAST VOIDABLE OR RESCISSIBLE.

WHEREFORE, premises considered, the judgment of the trial court on appeal Fifth Assignment of Error
is REVERSED. Defendant-appellee Pea is hereby ordered to vacate the lands in
question and pay the plaintiffs-appellants the accrued rentals from October, 1974 in THE RESPONDENT COURT OF APPEALS ERRED IN NOT HOLDING THAT THE
the amount of P1,500.00 per month up to December, 1978 and the amount of QUESTIONED DEED OF ASSIGNMENT, DATED 8 MARCH 1975, WAS VOID AB
P2,000.00 per month thereafter, until appellee finally vacate (sic) the premises with INITIO FOR FAILING TO COMPLY WITH THE FORMALITIES MANDATORILY REQUIRED
interest at the legal rate. UNDER THE LAW FOR DONATIONS.
SO ORDERED.3
Sixth Assignment of Error
A motion for reconsideration filed by the appellee was denied in a resolution dated
December 27, 1989. THE RESPONDENT COURT OF APPEALS ERRED IN HOLDING THAT
RESPONDENTS YAP ARE PURCHASERS IN GOOD FAITH AND IN FURTHER HOLDING
Hence, this petition for review on certiorari of said decision and resolution of the THAT IT WAS TOO LATE FOR PETITIONER TO INTERPOSE THE ISSUE THAT
appellate court predicated on the following assigned errors: RESPONDENTS YAP WERE PURCHASERS IN BAD FAITH.

First Assignment of Error Seventh Assignment of Error

37
MATABABE FILES
Corporation Law

THE RESPONDENT COURT OF APPEALS ERRED IN REVERSING THE DECISION


OF THE TRIAL COURT. 4 We note however, in reading the complaint that the plaintiff is seeking the
declaration of the nullity of the deed of sale, not as a party in the deed, or because
The petition is impressed with merit. she is obliged principally or subsidiarily under the deed, but because she has an
interest that is affected by the deed. This Court has held that a person who is not a
First, the preliminary issues. party obliged principally or subsidiarily in a contract may exercise an action for nullity of
the contract if he is prejudiced in his rights with respect to one of the contracting
The respondent court ruled that the trial court has no jurisdiction to annul the board parties, and can show the detriment which would positively result to him from the
resolution as the matter falls within the jurisdiction of the Securities and Exchange contract in which he had no intervention, Indeed, in the case now before Us, the
Commission (SEC) and that petitioner did not have the proper standing to have the same complaint alleges facts which show that plaintiff suffered detriment as a result of the
declared null and void. deed of sale entered into by and between defendant PHHC and defendant Melisenda
L. Santos. We believe that the plaintiff should be given a chance to present evidence
In Philex Mining Corporation vs. Reyes, 5 this Court held that it is the fact of to establish that she suffered detriment and that she is entitled to relief. (Emphasis
relationship between the parties that determines the proper and exclusive jurisdiction of the supplied.)
SEC to hear and decide intra-corporate disputes; that unless the controversy has arisen
between and among stockholders of the corporation, or between the stockholders and the There can be no question in this case that the questioned resolution and series of
officers of the corporation, then the case is not within the jurisdiction of the SEC. Where the transactions resulting in the registration of the properties in the name of respondent Yap
issue involves a party who is neither a stockholder or officer of the corporation, the same is spouses adversely affected the rights of petitioner to the said properties. Consequently,
not within the jurisdiction of the SEC. petitioner has the legal standing to question the validity of said resolution and transactions.

In Union Glass & Container Corporation vs. Securities and Exchange Commission, 6 this As to the question of validity of the board resolution of respondent PAMBUSCO
Court defined the relationships which are covered within "intra-corporate disputes" under adopted on November 19, 1974, Section 4, Article III of the amended by-laws of respondent
Presidential Decree No. 902-A, as amended, as follows: PAMBUSCO, provides as follows:

Otherwise stated, in order that the SEC can take cognizance of a case, the Sec. 4. Notices of regular and special meetings of the Board of Directors shall
controversy must pertain to any of the following relationships (a) between the be mailed to each Director not less than five days before any such meeting, and
corporation, partnership or association and the public; (b) between the corporation, notices of special meeting shall state the purpose or purposes thereof Notices of
partnership or association and its stockholders, partners, members, or officers; (c) regular meetings shall be sent by the Secretary and notices of special meetings by
between the corporation, partnership or association and the state in so far as its the President or Directors issuing the call. No failure or irregularity of notice of
franchise, permit or license to operate is concerned; and (d) among the stockholders, meeting shall invalidate any regular meeting or proceeding thereat; Provided a
partners or associates themselves. quorum of the Board is present, nor of any special meeting; Provided at least four
Directors are present. (Emphasis supplied.) 8
In this case, neither petitioner nor respondents Yap spouses are stockholders or
officers of PAMBUSCO. Consequently, the issue of the validity of the series of transactions The trial court in finding the resolution void held as follows:
resulting in the subject properties being registered in the names of respondents Yap may be
resolved only by the regular courts. On the other hand, this Court finds merit in the position taken by the
defendants that the questioned resolution should be declared invalid it having been
Respondent court held that petitioner being a stranger to the questioned resolution approved in a meeting attended by only 3 of the 5 members of the Board of Directors
and series of succeeding transactions has no legal standing to question their validity. of PAMBUSCO which attendance is short of the number required by the by-laws of
the corporation.
In Teves vs. People's Homesite and Housing Corporation, 7 this Court held:

38
MATABABE FILES
Corporation Law

xxx xxx xxx subject special meeting is itself declared void, it does not follow that the acts of the
board therein are ipso facto void and without any legal effect. Without the
In the meeting of November 19, 1974 when the questioned resolution was declaration of nullity of the subject board proceedings, its validity should be
approved, the three members of the Board of Directors of PAMBUSCO who were maintained and the acts borne out of it should be presumed valid. Considering that
present were Jesus Domingo, Joaquin Briones, and Salvador Bernardez The the subject special board meeting has not been declared void in a proper proceeding,
remaining 2 others, namely: Judge Pio Marcos and Alfredo Mamuyac were both nor even in the trial by the court below, there is no reason why the acts of the board
absent therefrom. in the said special meeting should be treated as void AB. initio. . . . 10

As it becomes clear that the resolution approved on November 19, 1974 is


null and void it having been approved by only 3 of the members of the Board of The Court disagrees.
Directors who were the only ones present at the said meeting, the deed of
assignment subsequently executed in favor of Marcelino Enriquez pursuant to this The by-laws of a corporation are its own private laws which substantially have the
resolution also becomes null and void. . . . 9 same effect as the laws of the corporation. They are in effect, written, into the charter. In this
sense they become part of the fundamental law of the corporation with which the
However, the respondent court overturning said legal conclusions of the trial court made the corporation and its directors and officers must comply. 11
following disquisition:
Apparently, only three (3) out of five (5) members of the board of directors of
It should be noted that the provision in Section 4, Article III of PAMBUSCO's respondent PAMBUSCO convened on November 19, 1974 by virtue of a prior notice of a
amended by-laws would apply only in case of a failure to notify the members of the special meeting. There was no quorum to validly transact business since, under Section 4 of
board of directors on the holding of a special meeting, . . . . the amended by-laws hereinabove reproduced, at least four (4) members must be present to
constitute a quorum in a special meeting of the board of directors of respondent PAMBUSCO.
In the instant case, however, there was no proof whatsoever, either by way
of documentary or testimonial evidence, that there was such a failure or irregularity Under Section 25 of the Corporation Code of the Philippines, the articles of
of notice as to make the aforecited provision apply. There was not even such an incorporation or by-laws of the corporation may fix a greater number than the majority of the
allegation in the Answer that should have necessitated a proof thereof. The fact number of board members to constitute the quorum necessary for the valid transaction of
alone that only three (3) out of five (5) members of the board of directors attended business. Any number less than the number provided in the articles or by-laws therein cannot
the subject special meeting, was not enough to declare the aforesaid proceeding constitute a quorum and any act therein would not bind the corporation; all that the
void ab initio, much less the board resolution borne out of it, when there was no attending directors could do is to adjourn. 12
proof of irregularity nor failure of notice and when the defense made in the Answer
did not touch upon the said failure of attendance. Therefore, the judgment declaring Moreover, the records show that respondent PAMBUSCO ceased to operate as of
the nullity of the subject board resolution must be set aside for lack of proof. November 15, 1949 as evidenced by a letter of the SEC to said corporation dated April 17,
1980. 13 Being a dormant corporation for several years, it was highly irregular, if not
Moreover, there is no categorical declaration in the by-laws that a failure to anomalous, for a group of three (3) individuals representing themselves to be the directors of
comply with the attendance requirement in a special meeting should make all the respondent PAMBUSCO to pass a resolution disposing of the only remaining asset of the
acts of the board therein null and void ab initio. A cursory reading of the subject corporation in favor of a former corporate officer.
provision, as aforequoted, would show that its framers only intended to make
voidable a board meeting held without the necessary compliance with the As a matter of fact, the three (3) alleged directors who attended the special meeting
attendance requirement in the by-laws. Just the use of the word "invalidate" already on November 19, 1974 were not listed as directors of respondent PAMBUSCO in the latest
denotes a legal imputation of validity to the questioned board meeting absent its general information sheet of respondent PAMBUSCO filed with the SEC dated 18 March
invalidation in the proceedings prescribed by the corporation's by-laws and/or the 1951. 14 Similarly, the latest list of stockholders of respondent PAMBUSCO on file with the SEC
general incorporation law. More significantly, it should be noted that even if the

39
MATABABE FILES
Corporation Law

does not show that the said alleged directors were among the stockholders of respondent SO ORDERED.
PAMBUSCO. 15
Narvasa, Cruz, Grio-Aquino and Medialdea, JJ., concur.
Under Section 30 of the then applicable Corporation Law, only persons who own at
least one (1) share in their own right may qualify to be directors of a corporation. Further,
under Section 28 1/2 of the said law, the sale or disposition of an and/or substantially all
properties of the corporation requires, in addition to a proper board resolution, the
affirmative votes of the stockholders holding at least two-thirds (2/3) of the voting power in
the corporation in a meeting duly called for that purpose. No doubt, the questioned
resolution was not confirmed at a subsequent stockholders meeting duly called for the
purpose by the affirmative votes of the stockholders holding at least two-thirds (2/3) of the
voting power in the corporation. The same requirement is found in Section 40 of the present
Corporation Code.

It is also undisputed that at the time of the passage of the questioned resolution,
respondent PAMBUSCO was insolvent and its only remaining asset was its right of
redemption over the subject properties. Since the disposition of said redemption right of
respondent PAMBUSCO by virtue of the questioned resolution was not approved by the
required number of stockholders under the law, the said resolution, as well as the subsequent
assignment executed on March 8, 1975 assigning to respondent Enriquez the said right of
redemption, should be struck down as null and void.
Respondent court, in upholding the questioned deed of assignment, which appears to be
without any consideration at all, held that the consideration thereof is the liberality of the
respondent PAMBUSCO in favor of its former corporate officer, respondent Enriquez, for
services rendered. Assuming this to be so, then as correctly argued by petitioner, it is not just
an ordinary deed of assignment, but is in fact a donation. Under Article 725 of the Civil Code,
in order to be valid, such a donation must be made in a public document and the acceptance
must be made in the same or in a separate instrument. In the latter case, the donor shall be
notified of the acceptance in an authentic form and such step must be noted in both
instruments. 16

Non-compliance with this requirement renders the donation null and


void. Since undeniably the deed of assignment dated March 8, 1975 in question, 18 shows
17

that there was no acceptance of the donation in the same and in a separate document, the
said deed of assignment is thus void ab initio and of no force and effect.

WHEREFORE, the petition is GRANTED. The questioned decision of the respondent


Court of Appeals dated June 20, 1989 and its resolution dated December 27, 1989 are hereby
REVERSED AND SET ASIDE and another judgment is hereby rendered AFFIRMING in toto the
decision of the trial court.

40
MATABABE FILES
Corporation Law

Republic of the Philippines


SUPREME COURT Same; Same; Same; Practice, no matter how long continued, cannot give rise to any
Manila vested right if it is contrary to law.Nor can petitioner claim a vested right to sit in the board
SECOND DIVISION on the basis of practice. Practice, no matter how long continued, cannot give rise to any
vested right if it is contrary to law. Even less tenable is petitioners claim that its right is
G.R. No. 108905 October 23, 1997 coterminus with the existence of the association.

GRACE CHRISTIAN HIGH SCHOOL, petitioner, PETITION for review on certiorari of a decision of the Court of Appeals.
vs.
THE COURT OF APPEALS, GRACE VILLAGE ASSOCIATION, INC., ALEJANDRO G. BELTRAN, and The facts are stated in the opinion of the Court.
ERNESTO L. GO, respondents.
MENDOZA, J.:
Corporation Law; Board of Directors; The board of directors of corporations must be
elected from among the stockholders or mem-bers.These provisions of the former and The question for decision in this case is the right of petitioner's representative to sit
present corporation law leave no room for doubt as to their meaning: the board of directors in the board of directors of respondent Grace Village Association, Inc. as a permanent
of corporations must be elected from among the stockholders or members. There may be member thereof. For fifteen years from 1975 until 1989 petitioner's representative had
corporations in which there are unelected members in the board but it is clear that in the been recognized as a "permanent director" of the association. But on February 13, 1990,
examples cited by petitioner the unelected members sit as ex officio members, i.e., by virtue petitioner received notice from the association's committee on election that the latter was
of and for as long as they hold a particular office. But in the case of petitioner, there is no "reexamining" (actually, reconsidering) the right of petitioner's representative to continue as
reason at all for its representative to be given a seat in the board. Nor does petitioner claim a an unelected member of the board. As the board denied petitioner's request to be allowed
right to such seat by virtue of an office held. In fact it was not given such seat in the representation without election, petitioner brought an action for mandamus in the Home
beginning. It was only in 1975 that a proposed amendment to the by-laws sought to give it Insurance and Guaranty Corporation. Its action was dismissed by the hearing officer whose
one. decision was subsequently affirmed by the appeals board. Petitioner appealed to the Court of
Appeals, which in turn upheld the decision of the HIGC's appeals board. Hence this petition
Same; Same; By-Laws; No provision of the by-laws can be adopted if it is contrary to for review based on the following contentions:
law.Since the provision in question is contrary to law, the fact that for fifteen years it has
not been questioned or challenged but, on the contrary, appears to have been implemented 1. The Petitioner herein has already acquired a vested right to a permanent
by the members of the association cannot forestall a later challenge to its validity. Neither can seat in the Board of Directors of Grace Village Association;
it attain validity through acquiescence because, if it is contrary to law, it is beyond the power
of the members of the association to waive its invalidity. For that matter the members of the 2. The amended By-laws of the Association drafted and promulgated by a
association may have formally adopted the provision in question, but their action would be of Committee on December 20, 1975 is valid and binding; and
no avail because no provision of the by-laws can be adopted if it is contrary to law. 3. The Practice of tolerating the automatic inclusion of petitioner as a
permanent member of the Board of Directors of the Association without the
Same; Same; Same; Tolerance cannot be considered a ratification.It is probable benefit of election is allowed under the law. 1
that, in allowing petitioners representative to sit on the board, the members of the
association were not aware that this was contrary to law. It should be noted that they did not Briefly stated, the facts are as follows:
actually implement the provision in question except perhaps insofar as it increased the
number of directors from 11 to 15, but certainly not the allowance of petitioners Petitioner Grace Christian High School is an educational institution offering
representative as an unelected member of the board of directors. It is more accurate to say preparatory, kindergarten and secondary courses at the Grace Village in Quezon City. Private
that the members merely tolerated petitioners representative and tolerance cannot be respondent Grace Village Association, Inc., on the other hand, is an organization of lot and/or
considered ratification. building owners, lessees and residents at Grace Village, while private respondents Alejandro

41
MATABABE FILES
Corporation Law

G. Beltran and Ernesto L. Go were its president and chairman of the committee on election, Petitioner requested the chairman of the election committee to change the notice of
respectively, in 1990, when this suit was brought. election by following the procedure in previous elections, claiming that the notice issued for
As adopted in 1968, the by-laws of the association provided in Article IV, as follows: the 1990 elections ran "counter to the practice in previous years" and was "in violation of the
The annual meeting of the members of the Association shall be held on the by-laws (of 1975)" and "unlawfully deprive[d] Grace Christian High School of its vested right
first Sunday of January in each calendar year at the principal office of the [to] a permanent seat in the board." 5
Association at 2:00 P.M. where they shall elect by plurality vote and by secret
balloting, the Board of Directors, composed of eleven (11) members to serve As the association denied its request, the school brought suit for mandamus in the
for one (1) year until their successors are duly elected and have qualified. 2 Home Insurance and Guaranty Corporation to compel the board of directors of the
association to recognize its right to a permanent seat in the board. Petitioner based its claim
It appears, that on December 20, 1975, a committee of the board of directors on the following portion of the proposed amendment which, it contended, had become part
prepared a draft of an amendment to the by-laws, reading as follows: 3 of the by-laws of the association as Article VI, paragraph 2, thereof:

VI. ANNUAL MEETING The Charter and Associate Members shall elect the Directors of the
Association. The candidates receiving the first fourteen (14) highest number
The Annual Meeting of the members of the Association shall be held on of votes shall be declared and proclaimed elected until their successors are
the second Thursday of January of each year. Each Charter or elected and qualified. GRACE CHRISTIAN HIGH SCHOOL representative is a
Associate Member of the Association is entitled to vote. He shall be entitled permanent Director of the ASSOCIATION.
to as many votes as he has acquired thru his monthly membership fees
only computed on a ratio of TEN (P10.00) PESOS for one vote. It appears that the opinion of the Securities and Exchange Commission on the validity
of this provision was sought by the association and that in reply to the query, the SEC
The Charter and Associate Members shall elect the Directors of the rendered an opinion to the effect that the practice of allowing unelected members in the
Association. The candidates receiving the first fourteen (14) highest number board was contrary to the existing by-laws of the association and to 92 of the Corporation
of votes shall be declared and proclaimed elected until their successors are Code (B.P. Blg. 68).
elected and qualified. GRACE CHRISTIAN HIGH SCHOOL representative is a
permanent Director of the ASSOCIATION. Private respondent association cited the SEC opinion in its answer. Additionally, the
association contended that the basis of the petition for mandamus was merely "a proposed
This draft was never presented to the general membership for approval. by-laws which has not yet been approved by competent authority nor registered with the SEC
Nevertheless, from 1975, after it was presumably submitted to the board, up to 1990, or HIGC." It argued that "the by-laws which was registered with the SEC on January 16, 1969
petitioner was given a permanent seat in the board of directors of the association. On should be the prevailing by-laws of the association and not the proposed amended by-laws." 6
February 13, 1990, the association's committee on election in a letter informed James Tan,
principal of the school, that "it was the sentiment that all directors should be elected by In reply, petitioner maintained that the "amended by-laws is valid and binding" and
members of the association" because "to make a person or entity a permanent Director that the association was estopped from questioning the by-laws. 7
would deprive the right of voters to vote for fifteen (15) members of the Board," and "it is
undemocratic for a person or entity to hold office in perpetuity." 4For this reason, Tan was A preliminary conference was held on March 29, 1990 but nothing substantial was
told that "the proposal to make the Grace Christian High School representative as a agreed upon. The parties merely agreed that the board of directors of the association should
permanent director of the association, although previously tolerated in the past elections meet on April 17, 1990 and April 24, 1990 for the purpose of discussing the amendment of the
should be reexamined." Following this advice, notices were sent to the members of the by-laws and a possible amicable settlement of the case. A meeting was held on April 17, 1990,
association that the provision on election of directors of the 1968 by-laws of the association but the parties failed to reach an agreement. Instead, the board adopted a resolution
would be observed. declaring the 1975 provision null and void for lack of approval by members of the association
and the 1968 by-laws to be effective.

42
MATABABE FILES
Corporation Law

On June 20, 1990, the hearing officer of the HIGC rendered a decision dismissing Petitioner appealed to the Court of Appeals but petitioner again lost as the appellate
petitioner's action. The hearing officer held that the amended by-laws, upon which petitioner court on February 9, 1993, affirmed the decision of the HIGC. The Court of Appeals held that
based its claim, "[was] merely a proposed by-laws which, although implemented in the past, there was no valid amendment of the association's by-laws because of failure to comply with
had not yet been ratified by the members of the association nor approved by competent the requirement of its existing by-laws, prescribing the affirmative vote of the majority of the
authority"; that, on the contrary, in the meeting held on April 17, 1990, the directors of the members of the association at a regular or special meeting called for the adoption of
association declared "the proposed by-law dated December 20, 1975 prepared by the amendment to the by-laws. Article XIX of the by-laws provides: 10
committee on by-laws . . . null and void" and the by-laws of December 17, 1968 as the
"prevailing by-laws under which the association is to operate until such time that the The members of the Association by an affirmative vote of the
proposed amendments to the by-laws are approved and ratified by a majority of the members majority at any regular or special meeting called for the purpose, may alter,
of the association and duly filed and approved by the pertinent government agency." The amend, change or adopt any new by-laws.
hearing officer rejected petitioner's contention that it had acquired a vested right to a
permanent seat in the board of directors. He held that past practice in election of directors This provision of the by-laws actually implements 22 of the Corporation Law (Act No.
could not give rise to a vested right and that departure from such practice was justified 1459) which provides:
because it deprived members of association of their right to elect or to be voted in office, not
to say that "allowing the automatic inclusion of a member representative of petitioner as 22. The owners of a majority of the subscribed capital stock, or a
permanent director [was] contrary to law and the registered by-laws of respondent majority of the members if there be no capital stock, may, at a regular or
association." 8 special meeting duly called for the purpose, amend or repeal any by-law or
adopt new by-laws. The owners of two-thirds of the subscribed capital stock,
The appeals board of the HIGC affirmed the decision of the hearing officer in its or two-thirds of the members if there be no capital stock, may delegate to
resolution dated September 13, 1990. It cited the opinion of the SEC based on 92 of the the board of directors the power to amend or repeal any by-law or to adopt
Corporation Code which reads: new by-laws: Provided, however, That any power delegated to the board of
directors to amend or repeal any by-law or adopt new by-laws shall be
92. Election and term of trustees. Unless otherwise provided in the considered as revoked whenever a majority of the stockholders or of the
articles of incorporation or the by-laws, the board of trustees of non-stock members of the corporation shall so vote at a regular or special
corporations, which may be more than fifteen (15) in number as may be fixed meeting. And provided, further, That the Director of the Bureau of
in their articles of incorporation or by-laws, shall, as soon as organized, so Commerce and Industry shall not hereafter file an amendment to the by-laws
classify themselves that the term of office of one-third (1/3) of the number of any bank, banking institution or building and loan association, unless
shall expire every year; and subsequent elections of trustees comprising one- accompanied by certificate of the Bank Commissioner to the effect that such
third (1/3) of the board of trustees shall be held annually and trustees so amendments are in accordance with law.
elected shall have a term of three (3) years. Trustees thereafter elected to fill
vacancies occurring before the expiration of a particular term shall hold The proposed amendment to the by-laws was never approved by the majority of the
office only for the unexpired period. members of the association as required by these provisions of the law and by-laws. But
petitioner contends that the members of the committee which prepared the proposed
The HIGC appeals board denied claims that the school "[was] being deprived amendment were duly authorized to do so and that because the members of the association
of its right to be a member of the Board of Directors of respondent association," thereafter implemented the provision for fifteen years, the proposed amendment for all
because the fact was that "it may nominate as many representatives to the intents and purposes should be considered to have been ratified by them. Petitioner
Association's Board as it may deem appropriate." It said that "what is merely being contends: 11
upheld is the act of the incumbent directors of the Board of correcting a long
standing practice which is not anchored upon any legal basis." 9 Considering, therefore, that the "agents" or committee were duly
authorized to draft the amended by-laws and the acts done by the "agents"
were in accordance with such authority, the acts of the "agents" from the

43
MATABABE FILES
Corporation Law

very beginning were lawful and binding on the homeowners (the


principals) per se without need of any ratification or adoption. The more has xxx xxx xxx
the amended by-laws become binding on the homeowners when the
homeowners followed and implemented the provisions of the amended by- If fact, the truth is that this is allowed and is being practiced by
laws. This is not merely tantamount to tacit ratification of the acts done by some corporations duly organized and existing under the laws of the
duly authorized "agents" but express approval and confirmation of what the Philippines.
"agents" did pursuant to the authority granted to them.
One example is the Plus XII Catholic Center, Inc. Under the by-laws
Corollarily, petitioner claims that it has acquired a vested right to a permanent seat in of this corporation, that whoever is the Archbishop of Manila is considered a
the board. Says petitioner: member of the board of trustees without benefit of election. And not only
that. He also automatically sits as the Chairman of the Board of Trustees,
The right of the petitioner to an automatic membership in the board again without need of any election.
of the Association was granted by the members of the Association
themselves and this grant has been implemented by members of the board Another concrete example is the Cardinal Santos Memorial Hospital,
themselves all through the years. Outside the present membership of the Inc. It is also provided in the by-laws of this corporation that whoever is the
board, not a single member of the Association has registered any desire to Archbishop of Manila is considered a member of the board of trustees year
remove the right of herein petitioner to an automatic membership in the after year without benefit of any election and he also sits automatically as
board. If there is anybody who has the right to take away such right of the the Chairman of the Board of Trustees.
petitioner, it would be the individual members of the Association through a
referendum and not the present board some of the members of which are It is actually 28 and 29 of the Corporation Law not 92 of the present law or 29
motivated by personal interest. of the former one which require members of the boards of directors of corporations to be
elected. These provisions read:
Petitioner disputes the ruling that the provision in question, giving
petitioner's representative a permanent seat in the board of the association, is 28. Unless otherwise provided in this Act, the corporate powers of all
contrary to law. Petitioner claims that that is not so because there is really no corporations formed under this Act shall be exercised, all business conducted
provision of law prohibiting unelected members of boards of directors of and all property of such corporations controlled and held by a board of not less
corporations. Referring to 92 of the present Corporation Code, petitioner says: than five nor more than eleven directors to be elected from among the holders
of stock or, where there is no stock, from the members of the
It is clear that the above provision of the Corporation Code only corporation: Provided, however, That in corporations, other than banks, in
provides for the manner of election of the members of the board of trustees which the United States has or may have a vested interest, pursuant to the
of non-stock corporations which may be more than fifteen in number and powers granted or delegated by the Trading with the Enemy Act, as
which manner of election is even subject to what is provided in the articles amended, and similar Acts of Congress of the United States relating to the
of incorporation or by-laws of the association thus showing that the above same subject, or by Executive Order No. 9095 of the President of the United
provisions [are] not even mandatory. States, as heretofore or hereafter amended, or both, the directors need not
be elected from among the holders of the stock, or, where there is no stock
Even a careful perusal of the above provision of the Corporation from the members of the corporation. (emphasis added)
Code would not show that it prohibits a non-stock corporation or association
from granting one of its members a permanent seat in its board of directors 29. At the meeting for the adoption of the original by-laws, or at such
or trustees. If there is no such legal prohibition then it is allowable provided subsequent meeting as may be then determined, directors shall be elected
it is so provided in the Articles of Incorporation or in the by-laws as in the to hold their offices for one year and until their successors are elected and
instant case. qualified. Thereafter the directors of the corporation shall be elected annually

44
MATABABE FILES
Corporation Law

by the stockholders if it be a stock corporation or by the members if it be a of the members of the association to waive its invalidity. For that matter the members of the
nonstock corporation, and if no provision is made in the by-laws for the time of association may have formally adopted the provision in question, but their action would be of
election the same shall be held on the first Tuesday after the first Monday in no avail because no provision of the by-laws can be adopted if it is contrary to law. 13
January. Unless otherwise provided in the by-laws, two weeks' notice of the
election of directors must be given by publication in some newspaper of It is probable that, in allowing petitioner's representative to sit on the board, the
general circulation devoted to the publication of general news at the place members of the association were not aware that this was contrary to law. It should be noted
where the principal office of the corporation is established or located, and that they did not actually implement the provision in question except perhaps insofar as it
by written notice deposited in the post-office, postage pre-paid, addressed increased the number of directors from 11 to 15, but certainly not the allowance of petitioner's
to each stockholder, or, if there be no stockholders, then to each member, representative as an unelected member of the board of directors. It is more accurate to say
at his last known place of residence. If there be no newspaper published at that the members merely tolerated petitioner's representative and tolerance cannot be
the place where the principal office of the corporation is established or considered ratification.
located, a notice of the election of directors shall be posted for a period of
three weeks immediately preceding the election in at least three public Nor can petitioner claim a vested right to sit in the board on the basis of "practice."
places, in the place where the principal office of the corporation is Practice, no matter how long continued, cannot give rise to any vested right if it is contrary to
established or located. (Emphasis added) law. Even less tenable is petitioner's claim that its right is "coterminus with the existence of
the association." 14
The present Corporation Code (B.P. Blg. 68), which took effect on May 1,
1980, 12 similarly provides: Finally, petitioner questions the authority of the SEC to render an opinion on the
validity of the provision in question. It contends that jurisdiction over this case is exclusively
23. The Board of Directors or Trustees. Unless otherwise provided in this vested in the HIGC.
Code, the corporate powers of all corporations formed under this Code shall
be exercised, all business conducted and all property of such corporations But this case was not decided by the SEC but by the HIGC. The HIGC merely cited as
controlled and held by the board of directors or trustees to be elected from authority for its ruling the opinion of the SEC chairman. The HIGC could have cited any other
among the holders of stocks, or where there is no stock, from among the authority for the view that under the law members of the board of directors of a corporation
members of the corporation, who shall hold office for one (1) year and until must be elected and it would be none the worse for doing so.
their successors are elected and qualified. (Emphasis added)
WHEREFORE, the decision of the Court of Appeals is AFFIRMED.
These provisions of the former and present corporation law leave no room for doubt
as to their meaning: the board of directors of corporations must be elected from among the SO ORDERED.
stockholders or members. There may be corporations in which there are unelected members
in the board but it is clear that in the examples cited by petitioner the unelected members sit Puno and Torres, Jr., JJ., concur.
as ex officio members, i.e., by virtue of and for as long as they hold a particular office. But in
the case of petitioner, there is no reason at all for its representative to be given a seat in the
board. Nor does petitioner claim a right to such seat by virtue of an office held. In fact it was
not given such seat in the beginning. It was only in 1975 that a proposed amendment to the
by-laws sought to give it one.

Since the provision in question is contrary to law, the fact that for fifteen years it has
not been questioned or challenged but, on the contrary, appears to have been implemented
by the members of the association cannot forestall a later challenge to its validity. Neither can
it attain validity through acquiescence because, if it is contrary to law, it is beyond the power

45
MATABABE FILES
Corporation Law

Republic of the Philippines petitioner and private respondent corporation. VGCCI claims a prior right over the subject
SUPREME COURT share anchored mainly on Sec. 3, Art. VIII of its by-laws which provides that after a member
Manila shall have been posted as delinquent, the Board may order his/her/its share sold to satisfy the
FIRST DIVISION claims of the Club . . . It is pursuant to this provision that VGCCI also sold the subject share at
public auction, of which it was the highest bidder. VGCCI caps its argument by asserting that
G.R. No. 117604 March 26, 1997 its corporate by-laws should prevail. The bone of contention, thus, is the proper
interpretation and application of VGCCIs aforequoted bylaws, a subject which irrefutably calls
CHINA BANKING CORPORATION, petitioner, for the special competence of the SEC.
vs.
COURT OF APPEALS, and VALLEY GOLF and COUNTRY CLUB, INC., respondents. Same; Same; Same; Estoppel; The plaintiff who files a complaint with one court which
has no jurisdiction over it is not estopped from filing the same complaint later with the
Securities and Exchange Commission; Actions; Jurisdiction; The better policy in competent court.In Zamora v. Court of Appeals, this Court, through Mr. Justice Isagani A.
determining which body has jurisdiction over a case would be to consider not only the status of Cruz, declared that: It follows that as a rule the filing of a complaint with one court which has
relationship of the parties but also the nature of the question that is the subject of their no jurisdiction over it does not prevent the plaintiff from filing the same complaint later with
controversy.The basic issue we must first hurdle is which body has jurisdiction over the the competent court. The plaintiff is not estopped from doing so simply because it made a
controversy, the regular courts or the SEC. P.D. No. 902-A conferred upon the SEC the mistake before in the choice of the proper forum. . . .
following pertinent powers: * * * The aforecited law was expounded upon in Viray v. CA and
in the recent cases of Mainland Construction Co., Inc. v. Movilla and Bernardo v. CA, thus: . . . . Appeals; Procedural Rules; Remand of Cases; The remand of the case or of an issue to
The better policy in determining which body has jurisdiction over a case would be to consider the lower court for further reception of evidence is not necessary where the Supreme Court is
not only the status or relationship of the parties but also the nature of the question that is the in position to resolve the dispute based on the records before it and particularly where the
subject of their controversy. ends of justice would not be subserved by the remand thereof.Applicable to this case is the
principle succinctly enunciated in the case of Heirs of Crisanta Y. Gabriel-Almoradie v. Court of
Same; Same; Same; Corporation Law; The purchase of a share or membership Appeals, citing Escudero v. Dulay and The Roman Catholic Archbishop of Manila v. Court of
certificate at public auction by a party (and the issuance to it of the corresponding Certificate Appeals: In the interest of the public and for the expeditious administration of justice the
of Sale) transfers ownership of the same to the latter and thus entitle it to have the said share issue on infringement shall be resolved by the court considering that this case has dragged on
registered in its name as a member.As to the first query, there is no question that the for years and has gone from one forum to another. It is a rule of procedure for the Supreme
purchase of the subject share or membership certificate at public auction by petitioner (and Court to strive to settle the entire controversy in a single proceeding leaving no root or
the issuance to it of the corresponding Certificate of Sale) transferred ownership of the same branch to bear the seeds of future litigation. No useful purpose will be served if a case or the
to the latter and thus entitled petitioner to have the said share registered in its name as a determination of an issue in a case is remanded to the trial court only to have its decision
member of VGCCI. It is readily observed that VGCCI did not assail the transfer directly and has raised again to the Court of Appeals and from there to the Supreme Court. We have laid down
in fact, in its letter of 27 September 1974, expressly recognized the pledge agreement the rule that the remand of the case or of an issue to the lower court for further reception of
executed by the original owner, Calapatia, in favor of petitioner and has even noted said evidence is not necessary where the Court is in position to resolve the dispute based on the
agreement in its corporate books. In addition, Calapatia, the original owner of the subject records before it and particularly where the ends of justice would not be subserved by the
share, has not contested the said transfer. By virtue of the afore-mentioned sale, petitioner remand thereof. Moreover, the Supreme Court is clothed with ample authority to review
became a bona fide stockholder of VGCCI and, therefore, the conflict that arose between matters, even those not raised on appeal if it finds that their consideration is necessary in
petitioner and VGCCI aptly exemplifies an intra-corporate controversy between a corporation arriving at a just disposition of the case.
and its stockholder under Sec. 5(b) of P.D. 902-A.
Loans; Pledge; The contracting parties to a pledge agreement may stipulate that the
Same; Same; Same; Same; By-Laws; The proper interpretation and application of a said pledge will also stand as security for any future advancements (or renewals thereof) that
corporations by-laws is a subject which irrefutably calls for the special competence of the the pledgor may procure from the pledgee.VGCCI assails the validity of the pledge
SEC.An important consideration, moreover, is the nature of the controversy between agreement executed by Calapatia in petitioners favor. It contends that the same was null and

46
MATABABE FILES
Corporation Law

void for lack of consideration because the pledge agreement was entered into on 21 August the issuance of Membership Certificate No. 1219. What Calapatia owed the corporation were
1974 but the loan or promissory note which it secured was obtained by Calapatia much later merely the monthly dues.
or only on 3 August 1983. VGCCIs contention is unmeritorious. A careful perusal of the pledge
agreement will readily reveal that the contracting parties explicitly stipulated therein that the Hence, the aforequoted provision does not apply.
said pledge will also stand as security for any future advancements (or renewals thereof) that
Calapatia (the pledgor) may procure from petitioner. PETITION for review on certiorari of a decision of the Court of Appeals.

Corporation Law; By-Laws; In order to be bound, a third party must have acquired The facts are stated in the opinion of the Court
knowledge of the pertinent by-laws at the time the transaction or agreement between said
third person and the shareholder was entered into.In order to be bound, the third party KAPUNAN, J.:
must have acquired knowledge of the pertinent bylaws at the time the transaction or
agreement between said third party and the shareholder was entered into, in this case, at the Through a petition for review on certiorari under Rule 45 of the Revised Rules of
time the pledge agreement was executed. VGCCI could have easily informed petitioner of its Court, petitioner China Banking Corporation seeks the reversal of the decision of the Court of
by-laws when it sent notice formally recognizing petitioner as pledgee of one of its shares Appeals dated 15 August 1994 nullifying the Securities and Exchange Commission's order and
registered in Calapatias name. Petitioners belated notice of said by-laws at the time of resolution dated 4 June 1993 and 7 December 1993, respectively, for lack of jurisdiction.
foreclosure will not suffice. Similarly impugned is the Court of Appeals' resolution dated 4 September 1994 which denied
petitioner's motion for reconsideration.
Same; Words and Phrases; A membership share is quite different in character from a
pawn ticket.Similarly, VGCCIs contention that petitioner is duty-bound to know its by-laws The case unfolds thus:
because of Art. 2099 of the Civil Code which stipulates that the creditor must take care of the
thing pledged with the diligence of a good father of a family, fails to convince. The case of On 21 August 1974, Galicano Calapatia, Jr. (Calapatia, for brevity) a stockholder of
Cruz & Serrano v. Chua A. H. Lee, is clearly not applicable: In applying this provision to the private respondent Valley Golf & Country Club, Inc. (VGCCI, for brevity), pledged his Stock
situation before us it must be borne in mind that the ordinary pawn ticket is a document by Certificate No. 1219 to petitioner China Banking Corporation (CBC, for brevity). 1
virtue of which the property in the thing pledged passes from hand to hand by mere delivery
of the ticket; and the contract of the pledge is, therefore, absolvable to bearer. It results that On 16 September 1974, petitioner wrote VGCCI requesting that the aforementioned
one who takes a pawn ticket in pledge acquires domination over the pledge; and it is the pledge agreement be recorded in its books. 2
holder who must renew the pledge, if it is to be kept alive. It is quite obvious from the In a letter dated 27 September 1974, VGCCI replied that the deed of pledge executed by
aforequoted case that a membership share is quite different in character from a pawn ticket Calapatia in petitioner's favor was duly noted in its corporate books. 3
and to reiterate, petitioner was never informed of Calapatias unpaid accounts and the
restrictive provisions in VGCCIs by-laws. On 3 August 1983, Calapatia obtained a loan of P20,000.00 from petitioner, payment
of which was secured by the aforestated pledge agreement still existing between Calapatia
Same; Same; The term unpaid claim in Sec. 63 of the Corporation Code refers to and petitioner. 4
any unpaid claim arising from unpaid subscription, and not to any indebtedness which a
subscriber or stockholder may owe the corporation arising from any other transaction, such Due to Calapatia's failure to pay his obligation, petitioner, on 12 April 1985, filed a
as monthly dues.Finally, Sec. 63 of the Corporation Code which provides that no shares of petition for extrajudicial foreclosure before Notary Public Antonio T. de Vera of Manila,
stock against which the corporation holds any unpaid claim shall be transferable in the books requesting the latter to conduct a public auction sale of the pledged stock. 5
of the corporation cannot be utilized by VGCCI. The term unpaid claim refers to any
unpaid claim arising from unpaid subscription, and not to any indebtedness which a On 14 May 1985, petitioner informed VGCCI of the above-mentioned foreclosure
subscriber or stockholder may owe the corporation arising from any other transaction. In proceedings and requested that the pledged stock be transferred to its (petitioner's) name
the case at bar, the subscription for the share in question has been fully paid as evidenced by and the same be recorded in the corporate books. However, on 15 July 1985, VGCCI wrote

47
MATABABE FILES
Corporation Law

petitioner expressing its inability to accede to petitioner's request in view of Calapatia's valid reason not to transfer the share in the name of the petitioner in the books of (VGCCI)
unsettled accounts with the club. 6 until liquidation of
delinquency." 15 Consequently, the case was dismissed. 16
Despite the foregoing, Notary Public de Vera held a public auction on 17 September
1985 and petitioner emerged as the highest bidder at P20,000.00 for the pledged stock. On 14 April 1992, Hearing Officer Perea denied petitioner's motion for
Consequently, petitioner was issued the corresponding certificate of sale. 7 reconsideration. 17

On 21 November 1985, VGCCI sent Calapatia a notice demanding full payment of his Petitioner appealed to the SEC en banc and on 4 June 1993, the Commission issued an
overdue account in the amount of P18,783.24. 8 Said notice was followed by a demand letter order reversing the decision of its hearing officer. It declared thus:
dated 12 December 1985 for the same amount 9and another notice dated 22 November 1986
for P23,483.24. 10 The Commission en banc believes that appellant-petitioner has a prior right
over the pledged share and because of pledgor's failure to pay the principal
On 4 December 1986, VGCCI caused to be published in the newspaper Daily Express a debt upon maturity, appellant-petitioner can proceed with the foreclosure of
notice of auction sale of a number of its stock certificates, to be held on 10 December 1986 at the pledged share.
10:00 a.m. Included therein was Calapatia's own share of stock (Stock Certificate No. 1219).
WHEREFORE, premises considered, the Orders of January 3, 1992 and April
Through a letter dated 15 December 1986, VGCCI informed Calapatia of the 14, 1992 are hereby SET ASIDE. The auction sale conducted by appellee-
termination of his membership due to the sale of his share of stock in the 10 December 1986 respondent Club on December 10, 1986 is declared NULL and VOID. Finally,
auction. 11 appellee-respondent Club is ordered to issue another membership certificate
On 5 May 1989, petitioner advised VGCCI that it is the new owner of Calapatia's Stock in the name of appellant-petitioner bank.
Certificate No. 1219 by virtue of being the highest bidder in the 17 September 1985 auction and
requested that a new certificate of stock be issued in its name. 12 SO ORDERED. 18

On 2 March 1990, VGCCI replied that "for reason of delinquency" Calapatia's stock VGCCI sought reconsideration of the abovecited order. However, the SEC denied the
was sold at the public auction held on 10 December 1986 for P25,000.00. 13 same in its resolution dated 7 December 1993. 19

On 9 March 1990, petitioner protested the sale by VGCCI of the subject share of stock The sudden turn of events sent VGCCI to seek redress from the Court of Appeals. On
and thereafter filed a case with the Regional Trial Court of Makati for the nullification of the 10 15 August 1994, the Court of Appeals rendered its decision nullifying and setting aside the
December 1986 auction and for the issuance of a new stock certificate in its name. 14 orders of the SEC and its hearing officer on ground of lack of jurisdiction over the subject
matter and, consequently, dismissed petitioner's original complaint. The Court of Appeals
On 18 June 1990, the Regional Trial Court of Makati dismissed the complaint for lack declared that the controversy between CBC and VGCCI is not intra-corporate. It ruled as
of jurisdiction over the subject matter on the theory that it involves an intra-corporate dispute follows:
and on 27 August 1990 denied petitioner's motion for reconsideration.
In order that the respondent Commission can take cognizance of a case, the
On 20 September 1990, petitioner filed a complaint with the Securities and Exchange controversy must pertain to any of the following relationships: (a) between
Commission (SEC) for the nullification of the sale of Calapatia's stock by VGCCI; the the corporation, partnership or association and the public; (b) between the
cancellation of any new stock certificate issued pursuant thereto; for the issuance of a new corporation, partnership or association and its stockholders, partners,
certificate in petitioner's name; and for damages, attorney's fees and costs of litigation. members, or officers; (c) between the corporation, partnership or
association and the state in so far as its franchise, permit or license to
On 3 January 1992, SEC Hearing Officer Manuel P. Perea rendered a decision in favor operate is concerned, and (d) among the stockholders, partners or
of VGCCI, stating in the main that "(c)onsidering that the said share is delinquent, (VGCCI) had associates themselves (Union Glass and Container Corporation vs. SEC,

48
MATABABE FILES
Corporation Law

November 28, 1983, 126 SCRA 31). The establishment of any of the
relationship mentioned will not necessarily always confer jurisdiction over 2. IT FAILED TO AFFIRM THE DECISION OF THE SECURITIES AND EXCHANGE
the dispute on the Securities and Exchange Commission to the exclusion of COMMISSION EN BANC DATED JUNE 04, 1993 DESPITE PREPONDERANT
the regular courts. The statement made in Philex Mining Corp. vs. Reyes, 118 EVIDENCE SHOWING THAT PETITIONER IS THE LAWFUL OWNER OF
SCRA 602, that the rule admits of no exceptions or distinctions is not that MEMBERSHIP CERTIFICATE NO. 1219 FOR ONE SHARE OF RESPONDENT
absolute. The better policy in determining which body has jurisdiction over a VALLEY GOLF.
case would be to consider not only the status or relationship of the parties
but also the nature of the question that is the subject of their controversy The petition is granted.
(Viray vs. Court of Appeals, November 9, 1990, 191 SCRA 308, 322-323).
The basic issue we must first hurdle is which body has jurisdiction over the
Indeed, the controversy between petitioner and respondent bank controversy, the regular courts or the SEC.
which involves ownership of the stock that used to belong to Calapatia, Jr. is
not within the competence of respondent Commission to decide. It is not P. D. No. 902-A conferred upon the SEC the following pertinent powers:
any of those mentioned in the aforecited case.
Sec. 3. The Commission shall have absolute jurisdiction, supervision and
WHEREFORE, the decision dated June 4, 1993, and order dated control over all corporations, partnerships or associations, who are the
December 7, 1993 of respondent Securities and Exchange Commission grantees of primary franchises and/or a license or permit issued by the
(Annexes Y and BB, petition) and of its hearing officer dated January 3, 1992 government to operate in the Philippines, and in the exercise of its authority,
and April 14, 1992 (Annexes S and W, petition) are all nullified and set aside it shall have the power to enlist the aid and support of and to deputize any
for lack of jurisdiction over the subject matter of the case. Accordingly, the and all enforcement agencies of the government, civil or military as well as
complaint of respondent China Banking Corporation (Annex Q, petition) is any private institution, corporation, firm, association or person.
DISMISSED. No pronouncement as to costs in this instance.
xxx xxx xxx
SO ORDERED. 20
Sec. 5. In addition to the regulatory and adjudicative functions of
Petitioner moved for reconsideration but the same was denied by the Court of the Securities and Exchange Commission over corporations, partnerships
Appeals in its resolution dated 5 October 1994. 21 and other forms of associations registered with it as expressly granted under
existing laws and decrees, it shall have original and exclusive jurisdiction to
Hence, this petition wherein the following issues were raised: hear and decide cases involving:

II a) Devices or schemes employed by or any acts of the


ISSUES board of directors, business associates, its officers or
partners, amounting to fraud and misrepresentation which
WHETHER OR NOT RESPONDENT COURT OF APPEALS (Former Eighth may be detrimental to the interest of the public and/or of
Division) GRAVELY ERRED WHEN: the stockholders, partners, members of associations or
organizations registered with the Commission.
1. IT NULLIFIED AND SET ASIDE THE DECISION DATED JUNE 04, 1993 AND
ORDER DATED DECEMBER 07, 1993 OF THE SECURITIES AND EXCHANGE b) Controversies arising out of intra-corporate or
COMMISSION EN BANC, AND WHEN IT DISMISSED THE COMPLAINT OF partnership relations, between and among stockholders,
PETITIONER AGAINST RESPONDENT VALLEY GOLF ALL FOR LACK OF members, or associates; between any or all of them and the
JURISDICTION OVER THE SUBJECT MATTER OF THE CASE; corporation, partnership or association of which they are

49
MATABABE FILES
Corporation Law

stockholders, members or associates, respectively; and By virtue of the afore-mentioned sale, petitioner became a bona fide stockholder of
between such corporation, partnership or association and VGCCI and, therefore, the conflict that arose between petitioner and VGCCI aptly exemplies
the State insofar as it concerns their individual franchise or an intra-corporate controversy between a corporation and its stockholder under Sec. 5(b) of
right to exist as such entity; P.D. 902-A.

c) Controversies in the election or appointment of An important consideration, moreover, is the nature of the controversy between
directors, trustees, officers, or managers of such petitioner and private respondent corporation. VGCCI claims a prior right over the subject
corporations, partnerships or associations. share anchored mainly on Sec. 3, Art VIII of its by-laws which provides that "after a member
shall have been posted as delinquent, the Board may order his/her/its share sold to satisfy the
d) Petitions of corporations, partnerships or associations to claims of the Club. . ." 26 It is pursuant to this provision that VGCCI also sold the subject share
be declared in the state of suspension of payments in cases at public auction, of which it was the highest bidder. VGCCI caps its argument by asserting
where the corporation, partnership or association that its corporate by-laws should prevail. The bone of contention, thus, is the proper
possesses property to cover all of its debts but foresees the interpretation and application of VGCCI's aforequoted by-laws, a subject which irrefutably
impossibility of meeting them when they respectively fall calls for the special competence of the SEC.
due or in cases where the corporation, partnership or
association has no sufficient assets to cover its liabilities, We reiterate herein the sound policy enunciated by the Court in Abejo v. De la Cruz 27:
but is under the Management Committee created pursuant
to this Decree. 6. In the fifties, the Court taking cognizance of the move to vest jurisdiction
in administrative commissions and boards the power to resolve specialized
The aforecited law was expounded upon in Viray v. CA 22 and in the recent cases disputes in the field of labor (as in corporations, public transportation and
of Mainland Construction Co., Inc. v. Movilla 23 and Bernardo v. CA, 24 thus: public utilities) ruled that Congress in requiring the Industrial Court's
intervention in the resolution of labor-management controversies likely to
. . . .The better policy in determining which body has jurisdiction over a case cause strikes or lockouts meant such jurisdiction to be exclusive, although it
would be to consider not only the status or relationship of the parties but did not so expressly state in the law. The Court held that under the "sense-
also the nature of the question that is the subject of their controversy. making and expeditious doctrine of primary jurisdiction . . . the courts cannot
or will not determine a controversy involving a question which is within the
Applying the foregoing principles in the case at bar, to ascertain which tribunal has jurisdiction of an administrative tribunal, where the question demands the
jurisdiction we have to determine therefore whether or not petitioner is a stockholder of exercise of sound administrative discretion requiring the special knowledge,
VGCCI and whether or not the nature of the controversy between petitioner and private experience, and services of the administrative tribunal to determine technical
respondent corporation is intra-corporate. and intricate matters of fact, and a uniformity of ruling is essential to comply
with the purposes of the regulatory statute administered.
As to the first query, there is no question that the purchase of the subject share or In this era of clogged court dockets, the need for specialized administrative
membership certificate at public auction by petitioner (and the issuance to it of the boards or commissions with the special knowledge, experience and
corresponding Certificate of Sale) transferred ownership of the same to the latter and thus capability to hear and determine promptly disputes on technical matters or
entitled petitioner to have the said share registered in its name as a member of VGCCI. It is essentially factual matters, subject to judicial review in case of grave abuse
readily observed that VGCCI did not assail the transfer directly and has in fact, in its letter of 27 of discretion, has become well nigh indispensable. Thus, in 1984, the Court
September 1974, expressly recognized the pledge agreement executed by the original owner, noted that "between the power lodged in an administrative body and a
Calapatia, in favor of petitioner and has even noted said agreement in its corporate court, the unmistakable trend has been to refer it to the former.
books. 25 In addition, Calapatia, the original owner of the subject share, has not contested the 'Increasingly, this Court has been committed to the view that unless the law
said transfer. speaks clearly and unequivocably, the choice should fall on [an
administrative agency.]'" The Court in the earlier case of Ebon v. De Guzman,

50
MATABABE FILES
Corporation Law

noted that the lawmaking authority, in restoring to the labor arbiters and the raised questions on the merits of the case. In turn, in its responsive pleadings, private
NLRC their jurisdiction to award all kinds of damages in labor cases, as respondent duly answered and countered all the issues raised by petitioner.
against the previous P.D. amendment splitting their jurisdiction with the
regular courts, "evidently, . . . had second thoughts about depriving the Applicable to this case is the principle succinctly enunciated in the case of Heirs of
Labor Arbiters and the NLRC of the jurisdiction to award damages in labor Crisanta Y. Gabriel-Almoradie v. Court of Appeals, 29 citing Escudero v. Dulay 30 and The Roman
cases because that setup would mean duplicity of suits, splitting the cause of Catholic Archbishop of Manila v. Court of Appeals. 31
action and possible conflicting findings and conclusions by two tribunals on
one and the same claim." In the interest of the public and for the expeditious administration
of justice the issue on infringement shall be resolved by the court
In this case, the need for the SEC's technical expertise cannot be over-emphasized considering that this case has dragged on for years and has gone from one
involving as it does the meticulous analysis and correct interpretation of a corporation's by- forum to another.
laws as well as the applicable provisions of the Corporation Code in order to determine the
validity of VGCCI's claims. The SEC, therefore, took proper cognizance of the instant case. It is a rule of procedure for the Supreme Court to strive to settle the
entire controversy in a single proceeding leaving no root or branch to bear
VGCCI further contends that petitioner is estopped from denying its earlier position, the seeds of future litigation. No useful purpose will be served if a case or
in the first complaint it filed with the RTC of Makati (Civil Case No. 90-1112) that there is no the determination of an issue in a case is remanded to the trial court only to
intra-corporate relations between itself and VGCCI. have its decision raised again to the Court of Appeals and from there to the
Supreme Court.
VGCCI's contention lacks merit.
We have laid down the rule that the remand of the case or of an
In Zamora v. Court of Appeals, 28 this Court, through Mr. Justice Isagani A. Cruz, issue to the lower court for further reception of evidence is not necessary
declared that: where the Court is in position to resolve the dispute based on the records
before it and particularly where the ends of justice would not be subserved
It follows that as a rule the filing of a complaint with one court which has no by the remand thereof. Moreover, the Supreme Court is clothed with ample
jurisdiction over it does not prevent the plaintiff from filing the same authority to review matters, even those not raised on appeal if it finds that
complaint later with the competent court. The plaintiff is not estopped from their consideration is necessary in arriving at a just disposition of the case.
doing so simply because it made a mistake before in the choice of the proper
forum. . . . In the recent case of China Banking Corp., et al. v. Court of Appeals, et al., 32 this Court,
through Mr. Justice Ricardo J. Francisco, ruled in this wise:
We remind VGCCI that in the same proceedings before the RTC of Makati, it
categorically stated (in its motion to dismiss) that the case between itself and petitioner is At the outset, the Court's attention is drawn to the fact that since
intra-corporate and insisted that it is the SEC and not the regular courts which has jurisdiction. the filing of this suit before the trial court, none of the substantial issues
This is precisely the reason why the said court dismissed petitioner's complaint and led to have been resolved. To avoid and gloss over the issues raised by the parties,
petitioner's recourse to the SEC. as what the trial court and respondent Court of Appeals did, would unduly
prolong this litigation involving a rather simple case of foreclosure of
Having resolved the issue on jurisdiction, instead of remanding the whole case to the mortgage. Undoubtedly, this will run counter to the avowed purpose of the
Court of Appeals, this Court likewise deems it procedurally sound to proceed and rule on its rules, i.e., to assist the parties in obtaining just, speedy and inexpensive
merits in the same proceedings. determination of every action or proceeding. The Court, therefore, feels that
the central issues of the case, albeit unresolved by the courts below, should
It must be underscored that petitioner did not confine the instant petition for review now be settled specially as they involved pure questions of law.
on certiorari on the issue of jurisdiction. In its assignment of errors, petitioner specifically Furthermore, the pleadings of the respective parties on file have amply

51
MATABABE FILES
Corporation Law

ventilated their various positions and arguments on the matter necessitating VGCCI likewise insists that due to Calapatia's failure to settle his delinquent accounts, it had
prompt adjudication. the right to sell the share in question in accordance with the express provision found in its by-
laws.
In the case at bar, since we already have the records of the case (from the
proceedings before the SEC) sufficient to enable us to render a sound judgment and since Private respondent's insistence comes to naught. It is significant to note that VGCCI
only questions of law were raised (the proper jurisdiction for Supreme Court review), we can, began sending notices of delinquency to Calapatia after it was informed by petitioner
therefore, unerringly take cognizance of and rule on the merits of the case. (through its letter dated 14 May 1985) of the foreclosure proceedings initiated against
Calapatia's pledged share, although Calapatia has been delinquent in paying his monthly dues
The procedural niceties settled, we proceed to the merits. to the club since 1975. Stranger still, petitioner, whom VGCCI had officially recognized as the
VGCCI assails the validity of the pledge agreement executed by Calapatia in petitioner's favor. pledgee of Calapatia's share, was neither informed nor furnished copies of these letters of
It contends that the same was null and void for lack of consideration because the pledge overdue accounts until VGCCI itself sold the pledged share at another public auction. By doing
agreement was entered into on 21 August so, VGCCI completely disregarded petitioner's rights as pledgee. It even failed to give
33
1974 but the loan or promissory note which it secured was obtained by Calapatia much later petitioner notice of said auction sale. Such actuations of VGCCI thus belie its claim of good
or only on 3 August 1983. 34 faith.
VGCCI's contention is unmeritorious.
In defending its actions, VGCCI likewise maintains that petitioner is bound by its by-
A careful perusal of the pledge agreement will readily reveal that the contracting laws. It argues in this wise:
parties explicitly stipulated therein that the said pledge will also stand as security for any
future advancements (or renewals thereof) that Calapatia (the pledgor) may procure from The general rule really is that third persons are not bound by the by-
petitioner: laws of a corporation since they are not privy thereto (Fleischer v. Botica
Nolasco, 47 Phil. 584). The exception to this is when third persons have
xxx xxx xxx actual or constructive knowledge of the same. In the case at bar, petitioner
had actual knowledge of the by-laws of private respondent when petitioner
This pledge is given as security for the prompt payment when due foreclosed the pledge made by Calapatia and when petitioner purchased the
of all loans, overdrafts, promissory notes, drafts, bills or exchange, share foreclosed on September 17, 1985. This is proven by the fact that prior
discounts, and all other obligations of every kind which have heretofore thereto, i.e., on May 14, 1985 petitioner even quoted a portion of private
been contracted, or which may hereafter be contracted, by the PLEDGOR(S) respondent's by-laws which is material to the issue herein in a letter it wrote
and/or DEBTOR(S) or any one of them, in favor of the PLEDGEE, including to private respondent. Because of this actual knowledge of such by-laws
discounts of Chinese drafts, bills of exchange, promissory notes, etc., then the same bound the petitioner as of the time when petitioner
without any further endorsement by the PLEDGOR(S) and/or Debtor(s) up to purchased the share. Since the by-laws was already binding upon petitioner
the sum of TWENTY THOUSAND (P20,000.00) PESOS, together with the when the latter purchased the share of Calapatia on September 17, 1985 then
accrued interest thereon, as hereinafter provided, plus the costs, losses, the petitioner purchased the said share subject to the right of the private
damages and expenses (including attorney's fees) which PLEDGEE may incur respondent to sell the said share for reasons of delinquency and the right of
in connection with the collection thereof. 35 (Emphasis ours.) private respondent to have a first lien on said shares as these rights are
provided for in the by-laws very very clearly. 36
The validity of the pledge agreement between petitioner and Calapatia cannot thus
be held suspect by VGCCI. As candidly explained by petitioner, the promissory note of 3 VGCCI misunderstood the import of our ruling in Fleischer v. Botica Nolasco Co.: 37
August 1983 in the amount of P20,000.00 was but a renewal of the first promissory note
covered by the same pledge agreement. And moreover, the by-law now in question cannot have any effect on
the appellee. He had no knowledge of such by-law when the shares were
assigned to him. He obtained them in good faith and for a valuable

52
MATABABE FILES
Corporation Law

consideration. He was not a privy to the contract created by said by-law individuals composing it and having the direction, management and control
between the shareholder Manuel Gonzales and the Botica Nolasco, Inc. Said of its affairs, in whole or in part, in the management and control of its affairs
by-law cannot operate to defeat his rights as a purchaser. and activities. (9 Fletcher 4166, 1982 Ed.)

An unauthorized by-law forbidding a shareholder to sell his shares The purpose of a by-law is to regulate the conduct and define the
without first offering them to the corporation for a period of thirty days is duties of the members towards the corporation and among themselves.
not binding upon an assignee of the stock as a personal contract, although They are self-imposed and, although adopted pursuant to statutory
his assignor knew of the by-law and took part in its adoption. (10 Cyc., 579; authority, have no status as public law. (Ibid.)
Ireland vs. Globe Milling Co., 21 R.I., 9.)
Therefore, it is the generally accepted rule that third persons are
When no restriction is placed by public law on the transfer of not bound by by-laws, except when they have knowledge of the provisions
corporate stock, a purchaser is not affected by any contractual restriction of either actually or constructively. In the case of Fleisher v. Botica Nolasco, 47
which he had no notice. (Brinkerhoff-Farris Trust & Savings Co. vs. Home Phil. 584, the Supreme Court held that the by-law restricting the transfer of
Lumber Co., 118 Mo., 447.) shares cannot have any effect on the transferee of the shares in question as
he "had no knowledge of such by-law when the shares were assigned to
The assignment of shares of stock in a corporation by one who has him. He obtained them in good faith and for a valuable consideration. He was
assented to an unauthorized by-law has only the effect of a contract by, and not a privy to the contract created by the by-law between the shareholder . .
enforceable against, the assignor; the assignee is not bound by such by-law . and the Botica Nolasco, Inc. Said by-law cannot operate to defeat his right as
by virtue of the assignment alone. (Ireland vs. Globe Milling Co., 21 R.I., 9.) a purchaser. (Emphasis supplied.)

A by-law of a corporation which provides that transfers of stock By analogy of the above-cited case, the Commission en banc is of the
shall not be valid unless approved by the board of directors, while it may be opinion that said case is applicable to the present controversy. Appellant-
enforced as a reasonable regulation for the protection of the corporation petitioner bank as a third party can not be bound by appellee-respondent's
against worthless stockholders, cannot be made available to defeat the by-laws. It must be recalled that when appellee-respondent communicated
rights of third persons. (Farmers' and Merchants' Bank of Lineville vs. to appellant-petitioner bank that the pledge agreement was duly noted in
Wasson, 48 Iowa, 336.) (Emphasis ours.) the club's books there was no mention of the shareholder-pledgor's unpaid
accounts. The transcript of stenographic notes of the June 25, 1991 Hearing
In order to be bound, the third party must have acquired knowledge of the pertinent reveals that the pledgor became delinquent only in 1975. Thus, appellant-
by-laws at the time the transaction or agreement between said third party and the petitioner was in good faith when the pledge agreement was contracted.
shareholder was entered into, in this case, at the time the pledge agreement was executed.
VGCCI could have easily informed petitioner of its by-laws when it sent notice formally The Commission en banc also believes that for the exception to the
recognizing petitioner as pledgee of one of its shares registered in Calapatia's name. general accepted rule that third persons are not bound by by-laws to be
Petitioner's belated notice of said by-laws at the time of foreclosure will not suffice. The ruling applicable and binding upon the pledgee, knowledge of the provisions of the
of the SEC en banc is particularly instructive: VGCI By-laws must be acquired at the time the pledge agreement was
contracted. Knowledge of said provisions, either actual or constructive, at
By-laws signifies the rules and regulations or private laws enacted the time of foreclosure will not affect pledgee's right over the pledged
by the corporation to regulate, govern and control its own actions, affairs share. Art. 2087 of the Civil Code provides that it is also of the essence of
and concerns and its stockholders or members and directors and officers these contracts that when the principal obligation becomes due, the things
with relation thereto and among themselves in their relation to it. In other in which the pledge or mortgage consists maybe alienated for the payment
words, by-laws are the relatively permanent and continuing rules of action to the creditor.
adopted by the corporation for its own government and that of the

53
MATABABE FILES
Corporation Law

In a letter dated March 10, 1976 addressed to Valley Golf Club, Inc., In applying this provision to the situation before us it must be borne
the Commission issued an opinion to the effect that: in mind that the ordinary pawn ticket is a document by virtue of which the
property in the thing pledged passes from hand to hand by mere delivery of
According to the weight of authority, the pledgee's right is the ticket; and the contract of the pledge is, therefore, absolvable to bearer.
entitled to full protection without surrender of the It results that one who takes a pawn ticket in pledge acquires domination
certificate, their cancellation, and the issuance to him of over the pledge; and it is the holder who must renew the pledge, if it is to be
new ones, and when done, the pledgee will be fully kept alive.
protected against a subsequent purchaser who would be
charged with constructive notice that the certificate is It is quite obvious from the aforequoted case that a membership share is
covered by the pledge. (12-A Fletcher 502) quite different in character from a pawn ticket and to reiterate, petitioner was never
informed of Calapatia's unpaid accounts and the restrictive provisions in VGCCI's by-
The pledgee is entitled to retain possession of the stock laws.
until the pledgor pays or tenders to him the amount due on
the debt secured. In other words, the pledgee has the right Finally, Sec. 63 of the Corporation Code which provides that "no shares of stock
to resort to its collateral for the payment of the debts. (Ibid, against which the corporation holds any unpaid claim shall be transferable in the books of the
502) corporation" cannot be utilized by VGCCI. The term "unpaid claim" refers to "any unpaid claim
arising from unpaid subscription, and not to any indebtedness which a subscriber or
To cancel the pledged certificate outright and the issuance stockholder may owe the corporation arising from any other transaction." 40 In the case at
of new certificate to a third person who purchased the bar, the subscription for the share in question has been fully paid as evidenced by the
same certificate covered by the pledge, will certainly defeat issuance of Membership Certificate No. 1219. 41 What Calapatia owed the corporation were
the right of the pledgee to resort to its collateral for the merely the monthly dues. Hence, the aforequoted provision does not apply.
payment of the debt. The pledgor or his representative or
registered stockholders has no right to require a return of WHEREFORE, premises considered, the assailed decision of the Court of Appeals is REVERSED
the pledged stock until the debt for which it was given as and the order of the SEC en banc dated 4 June 1993 is hereby AFFIRMED.
security is paid and satisfied, regardless of the length of
time which have elapsed since debt was created. (12-A SO ORDERED.
Fletcher 409)
Padilla, Bellosillo, Vitug and Hermosisima, Jr., JJ., concur.
A bona fide pledgee takes free from any latent or secret equities or
liens in favor either of the corporation or of third persons, if he has no notice
thereof, but not otherwise. He also takes it free of liens or claims that may
subsequently arise in favor of the corporation if it has notice of the pledge,
although no demand for a transfer of the stock to the pledgee on the
corporate books has been made. (12-A Fletcher 5634, 1982 ed., citing Snyder
v. Eagle Fruit Co., 75 F2d739) 38

Similarly, VGCCI's contention that petitioner is duty-bound to know its by-laws


because of Art. 2099 of the Civil Code which stipulates that the creditor must take care of the
thing pledged with the diligence of a good father of a family, fails to convince. The case
of Cruz & Serrano v. Chua A. H. Lee, 39 is clearly not applicable:

54
MATABABE FILES
Corporation Law

Republic of the Philippines APPEAL from a judgment of the Court of First Instance of Occidental Negros.
SUPREME COURT
Manila The facts are stated in the opinion of the Court
EN BANC
Taada, Teehankee and Carreon for plaintiffs-appellants.
G.R. No. L-15092 May 18, 1962 Hilado and Hilado for defendant-appellee.

ALFREDO MONTELIBANO, ET AL., plaintiffs-appellants, REYES, J.B.L., J.:


vs.
BACOLOD-MURCIA MILLING CO., INC., defendant-appellee. Appeal on points of law from a judgment of the Court of First Instance of Occidental
Negros, in its Civil Case No. 2603, dismissing plaintiff's complaint that sought to compel the
Sugar Centrals; Milling Contracts; Concessions given by central to planters, if defendant Milling Company to increase plaintiff's share in the sugar produced from their
retracted, will constitute fraud; Case at Bar.Since there is no rational explanation for the cane, from 60% to 62.33%, starting from the 1951-1952 crop year.
company's assenting to the further concessions asked by the planters before the contracts
were signed, except as further inducement for the planters to agree to the extension of the It is undisputed that plaintiffs-appellants, Alfredo Montelibano, Alejandro
contract period, to allow the company now to retract such concessions would be to sanction Montelibano, and the Limited co-partnership Gonzaga and Company, had been and are sugar
a fraud upon the planters who relied on such additional stipulations. planters adhered to the defendant-appellee's sugar central mill under identical milling
contracts. Originally executed in 1919, said contracts were stipulated to be in force for 30
Contracts; Novation; Modification before a bargain not novation in law.There can years starting with the 1920-21 crop, and provided that the resulting product should be divided
be no novation unless two distinct and successive binding contracts take place, with the later in the ratio of 45% for the mill and 55% for the planters. Sometime in 1936, it was proposed to
one designed to replace the preceding convention. Modifications introduced before a bargain execute amended milling contracts, increasing the planters' share to 60% of the manufactured
becomes obligatory can in no sense constitute novation in law. sugar and resulting molasses, besides other concessions, but extending the operation of the
milling contract from the original 30 years to 45 years. To this effect, a printed Amended
Same; Assent and concurrence of parties necessary to perfect a contract; Setting down Milling Contract form was drawn up. On August 20, 1936, the Board of Directors of the
of terms not important except in certain cases.Except in the cases of statutory forms or appellee Bacolod-Murcia Milling Co., Inc., adopted a resolution (Acts No. 11, Acuerdo No. 1)
solemn agreements, it is the assent and concurrence of the parties, and not the setting down granting further concessions to the planters over and above those contained in the printed
of its terms, that constitutes a binding contract. Amended Milling Contract. The bone of contention is paragraph 9 of this resolution, that
reads as follows:
Corporations; Exercise of charter powers; Test to be applied."It is a question,
therefore, in each case, of the logical relation of the act to the corporate purpose expressed ACTA No. 11
in the charter. If that act is one which is lawful in itself, and not otherwise prohibited, is done
for the purpose of serving corporate ends, and is reasonably tributary to the promotion of SESSION DE LA JUNTA DIRECTIVA
those ends, in a substantial, and not in a remote and fanciful, sense, it may fairly be AGOSTO 20, 1936
considered within charter powers. The test to be applied is whether the act in question is in
direct and immediate furtherance of the corporation's business, fairly incident to the express Acuerdo No. 1. Previa mocion debidamente secundada, la Junta en
powers and reasonably necessary to their exercise. If so, the corporation has the power to do consideracion a una peticion de los plantadores hecha por un comite
it; otherwise, not." (Fletcher Cyc. Corp., Vol. 6, Rev. Ed. 1950, pp. 266-268) nombrado por los mismos, acuerda enmendar el contrato de molienda
enmendado medientelas siguentes:
Same; Same; Question on probable losses or decrease in profits not reviewable by
courts.Whether or not a valid and binding resolution passed by the board of directors, will xxx xxx xxx
cause losses or decrease the profits of the corporation, may not be reviewed by the courts.

55
MATABABE FILES
Corporation Law

9.a Que si durante la vigencia de este contrato de Molienda Enmendado, for the concessions embodied in the resolution of August 20, 1936. That the resolution
lascentrales azucareras, de Negros Occidental, cuya produccion anual de formed an integral part of the amended milling contract, signed on September 10, and not a
azucar centrifugado sea mas de una tercera parte de la produccion total de separate bargain, is further shown by the fact that a copy of the resolution was simply
todas lascentrales azucareras de Negros Occidental, concedieren a sus attached to the printed contract without special negotiations or agreement between the
plantadores mejores condiciones que la estipuladas en el presente contrato, parties.
entonces esas mejores condiciones se concederan y por el presente se It follows from the foregoing that the terms embodied in the resolution of August 20, 1936
entenderan concedidas a los platadores que hayan otorgado este Contrato were supported by the same causa or consideration underlying the main amended milling
de Molienda Enmendado. contract; i.e., the promises and obligations undertaken thereunder by the planters, and,
particularly, the extension of its operative period for an additional 15 years over and beyond
Appellants signed and executed the printed Amended Milling Contract on September the 30 years stipulated in the original contract. Hence, the conclusion of the court below that
10, 1936, but a copy of the resolution of August 10, 1936, signed by the Central's General the resolution constituted gratuitous concessions not supported by any consideration is
Manager, was not attached to the printed contract until April 17, 1937; with the notation legally untenable.

Las enmiendas arriba transcritas forman parte del contrato de molienda All disquisition concerning donations and the lack of power of the directors of the
enmendado, otorgado por y la Bacolod-Murcia Milling Co., Inc. respondent sugar milling company to make a gift to the planters would be relevant if the
resolution in question had embodied a separate agreement after the appellants had already
In 1953, the appellants initiated the present action, contending that three Negros bound themselves to the terms of the printed milling contract. But this was not the case.
sugar centrals (La Carlota, Binalbagan-Isabela and San Carlos), with a total annual production When the resolution was adopted and the additional concessions were made by the
exceeding one-third of the production of all the sugar central mills in the province, had company, the appellants were not yet obligated by the terms of the printed contract, since
already granted increased participation (of 62.5%) to their planters, and that under paragraph they admittedly did not sign it until twenty-one days later, on September 10, 1936. Before that
9 of the resolution of August 20, 1936, heretofore quoted, the appellee had become obligated date, the printed form was no more than a proposal that either party could modify at its
to grant similar concessions to the plaintiffs (appellants herein). The appellee Bacolod-Murcia pleasure, and the appellee actually modified it by adopting the resolution in question. So that
Milling Co., inc., resisted the claim, and defended by urging that the stipulations contained in by September 10, 1936 defendant corporation already understood that the printed terms
the resolution were made without consideration; that the resolution in question was, were not controlling, save as modified by its resolution of August 20, 1936; and we are
therefore, null and void ab initio, being in effect a donation that was ultra vires and beyond satisfied that such was also the understanding of appellants herein, and that the minds of the
the powers of the corporate directors to adopt. parties met upon that basis. Otherwise there would have been no consent or "meeting of the
minds", and no binding contract at all. But the conduct of the parties indicates that they
After trial, the court below rendered judgment upholding the stand of the defendant assumed, and they do not now deny, that the signing of the contract on September 10, 1936,
Milling company, and dismissed the complaint. Thereupon, plaintiffs duly appealed to this did give rise to a binding agreement. That agreement had to exist on the basis of the printed
Court. terms as modified by the resolution of August 20, 1936, or not at all. Since there is no rational
explanation for the company's assenting to the further concessions asked by the planters
We agree with appellants that the appealed decisions can not stand. It must be before the contracts were signed, except as further inducement for the planters to agree to
remembered that the controverted resolution was adopted by appellee corporation as a the extension of the contract period, to allow the company now to retract such concessions
supplement to, or further amendment of, the proposed milling contract, and that it was would be to sanction a fraud upon the planters who relied on such additional stipulations.
approved on August 20, 1936, twenty-one days prior to the signing by appellants on
September 10, of the Amended Milling Contract itself; so that when the Milling Contract was The same considerations apply to the "void innovation" theory of appellees. There
executed, the concessions granted by the disputed resolution had been already incorporated can be no novation unless two distinct and successive binding contracts take place, with the
into its terms. No reason appears of record why, in the face of such concessions, the later designed to replace the preceding convention. Modifications introduced before a
appellants should reject them or consider them as separate and apart from the main bargain becomes obligatory can in no sense constitute novation in law.
amended milling contract, specially taking into account that appellant Alfredo Montelibano
was, at the time, the President of the Planters Association (Exhibit 4, p. 11) that had agitated

56
MATABABE FILES
Corporation Law

Stress is placed on the fact that the text of the Resolution of August 20, 1936 was not reasonable exercise and performance of such duty. Whether the business of a
attached to the printed contract until April 17, 1937. But, except in the case of statutory forms corporation should be operated at a loss during depression, or close down at a
or solemn agreements (and it is not claimed that this is one), it is the assent and concurrence smaller loss, is a purely business and economic problem to be determined by the
(the "meeting of the minds") of the parties, and not the setting down of its terms, that directors of the corporation and not by the court. It is a well-known rule of law that
constitutes a binding contract. And the fact that the addendum is only signed by the General questions of policy or of management are left solely to the honest decision of
Manager of the milling company emphasizes that the addition was made solely in order that officers and directors of a corporation, and the court is without authority to
the memorial of the terms of the agreement should be full and complete. substitute its judgment of the board of directors; the board is the business manager
Much is made of the circumstance that the report submitted by the Board of Directors of the of the corporation, and so long as it acts in good faith its orders are not reviewable
appellee company in November 19, 1936 (Exhibit 4) only made mention of 90%, the planters by the courts. (Fletcher on Corporations, Vol. 2, p. 390).
having agreed to the 60-40 sharing of the sugar set forth in the printed "amended milling
contracts", and did not make any reference at all to the terms of the resolution of August 20, And it appearing undisputed in this appeal that sugar centrals of La Carlota, Hawaiian
1936. But a reading of this report shows that it was not intended to inventory all the details of Philippines, San Carlos and Binalbagan (which produce over one-third of the entire annual
the amended contract; numerous provisions of the printed terms are alao glossed over. The sugar production in Occidental Negros) have granted progressively increasing participations
Directors of the appellee Milling Company had no reason at the time to call attention to the to their adhered planter at an average rate of
provisions of the resolution in question, since it contained mostly modifications in detail of
the printed terms, and the only major change was paragraph 9 heretofore quoted; but when
62.333% for the 1951-52 crop year;
the report was made, that paragraph was not yet in effect, since it was conditioned on other
centrals granting better concessions to their planters, and that did not happen until after 64.2% for 1952-53;
1950. There was no reason in 1936 to emphasize a concession that was not yet, and might
never be, in effective operation. 64.3% for 1953-54;

There can be no doubt that the directors of the appellee company had authority to 64.5% for 1954-55; and
modify the proposed terms of the Amended Milling Contract for the purpose of making its 63.5% for 1955-56,
terms more acceptable to the other contracting parties. The rule is that
the appellee Bacolod-Murcia Milling Company is, under the terms of its Resolution of August
It is a question, therefore, in each case of the logical relation of the act to the 20, 1936, duty bound to grant similar increases to plaintiffs-appellants herein.
corporate purpose expressed in the charter. If that act is one which is lawful in itself,
and not otherwise prohibited, is done for the purpose of serving corporate ends, and WHEREFORE, the decision under appeal is reversed and set aside; and judgment is
is reasonably tributary to the promotion of those ends, in a substantial, and not in a decreed sentencing the defendant-appellee to pay plaintiffs-appellants the differential or
remote and fanciful sense, it may fairly be considered within charter powers. The test increase of participation in the milled sugar in accordance with paragraph 9 of the appellee
to be applied is whether the act in question is in direct and immediate furtherance of Resolution of August 20, 1936, over and in addition to the 60% expressed in the printed
the corporation's business, fairly incident to the express powers and reasonably Amended Milling Contract, or the value thereof when due, as follows:
necessary to their exercise. If so, the corporation has the power to do it; otherwise,
not. (Fletcher Cyc. Corp., Vol. 6, Rev. Ed. 1950, pp. 266-268) 0,333% to appellants Montelibano for the 1951-1952 crop year, said appellants having
received an additional 2% corresponding to said year in October, 1953;
As the resolution in question was passed in good faith by the board of directors, it is
valid and binding, and whether or not it will cause losses or decrease the profits of the 2.333% to appellant Gonzaga & Co., for the 1951-1952 crop year; and to all appellants
central, the court has no authority to review them. thereafter

They hold such office charged with the duty to act for the corporation 4.2% for the 1952-1953 crop year;
according to their best judgment, and in so doing they cannot be controlled in the 4.3% for the 1953-1954 crop year;

57
MATABABE FILES
Corporation Law

4.5% for the 1954-1955 crop year;


3.5% for the 1955-1956 crop year;

with interest at the legal rate on the value of such differential during the time they were
withheld; and the right is reserved to plaintiffs-appellants to sue for such additional increases
as they may be entitled to for the crop years subsequent to those herein adjudged.

Costs against appellee, Bacolod-Murcia Milling Co.

Padilla, Bautista Angelo, Labrador, Concepcion, Barrera, Paredes and Dizon, JJ., concur.

58
MATABABE FILES
Corporation Law

Republic of the Philippines and Antonio de las Alas were personally liable for the checks issued as corporate officers and
SUPREME COURT authorized signatories of the check. Personal liability of a corporate director, trustee or
Manila officer along (although not necessarily) with the corporation may so validly attach, as a rule,
FIRST DIVISION only when: 1. He assents (a) to a patently unlawful act of the corporation, or (b) for bad faith
or gross negligence in directing its affairs, or (c) for conflict of interest, resulting in damages
G.R. No. 109491 February 28, 2001 to the corporation, its stockholders or other persons; 2. He consents to the issuance of
watered down stocks or who, having knowledge thereof, does not forthwith file with the
ATRIUM MANAGEMENT CORPORATION, petitioner, corporate secretary his written objection thereto; 3. He agrees to hold himself personally
vs. and solidarily liable with the corporation; or 4. He is made, by a specific provision of law, to
COURT OF APPEALS, E.T. HENRY AND CO., LOURDES VICTORIA M. DE LEON, RAFAEL DE personally answer for his corporate action.
LEON, JR., AND HI-CEMENT CORPORATION, respondents.
Same; Same; Checks; A treasurer of a corporation whose negligence in signing a
---------------------------------------- confirmation letter for rediscounting of crossed checks, knowing fully well that the checks
were strictly endorsed for deposit only to the payees account and not to be further
G.R. No. 121794 February 28, 2001 negotiated, resulting in damage to the corporation may be personally liable therefor.In the
LOURDES M. DE LEON, petitioner, case at bar, Lourdes M. de Leon and Antonio de las Alas as treasurer and Chairman of
vs. HiCement were authorized to issue the checks. However, Ms. de Leon was negligent when
COURT OF APPEALS, ATRIUM MANAGEMENT CORPORATION, AND HI-CEMENT she signed the confirmation letter requested by Mr. Yap of Atrium and Mr. Henry of E.T.
CORPORATION, respondents. Henry for the rediscounting of the crossed checks issued in favor of E.T. Henry. She was
aware that the checks were strictly endorsed for deposit only to the payees account and not
Corporation Law; Ultra Vires Acts; Checks; The act of issuing checks for the purpose of to be further negotiated. What is more, the confirmation letter contained a clause that was
securing a loan to finance the activities of the corporation is well within the ambit of a valid not true, that is, that the checks issued to E.T. Henry were in payment of Hydro oil bought by
corporate act, hence, not an ultra vires act.Hi-Cement, however, maintains that the checks Hi-Cement from E.T. Henry. Her negligence resulted in damage to the corporation. Hence,
were not issued for consideration and that Lourdes and E.T. Henry engaged in a kiting Ms. de Leon may be held personally liable therefor.
operation to raise funds for E.T. Henry, who admittedly was in need of financial assistance.
The Court finds that there was no sufficient evidence to show that such is the case. Lourdes Negotiable Instrument Law; Checks; Words and Phrases; Holder in Due Course,
M. de Leon is the treasurer of the corporation and is authorized to sign checks for the Explained.The next issue is whether or not petitioner Atrium was a holder of the checks in
corporation. At the time of the issuance of the checks, there were sufficient funds in the bank due course. The Negotiable Instruments Law, Section 52 defines a holder in due course, thus:
to cover payment of the amount of P2 million pesos. It is, however, our view that there is A holder in due course is a holder who has taken the instrument under the following
basis to rule that the act of issuing the checks was well within the ambit of a valid corporate conditions: (a) That it is complete and regular upon its face; (b) That he became the holder of
act, for it was for securing a loan to finance the activities of the corporation, hence, not an it before it was overdue, and without notice that it had been previously dishonored, if such
ultra vires act. was the fact; (c) That he took it in good faith and for value; (d) That at the time it was
negotiated to him he had no notice of any infirmity in the instrument or defect in the title of
Same; Same; Words and Phrases; Ultra Vires Acts, Explained.An ultra vires act is the person negotiating it.
one committed outside the object for which a corporation is created as defined by the law of
its organization and therefore beyond the power conferred upon it by law. The term ultra Same; Same; A person to whom a crossed check was endorsed by the payee of said
vires is distinguished from an illegal act for the former is merely voidable which may be check could not be considered a holder in due course.In the instant case, the checks were
enforced by performance, ratification, or estoppel, while the latter is void and cannot be crossed checks and specifically indorsed for deposit to payees account only. From the
validated. beginning, Atrium was aware of the fact that the checks were all for deposit only to payees
Same; Same; Instances when personal liability of corporate directors, trustees or account, meaning E.T. Henry. Clearly, then, Atrium could not be considered a holder in due
officers may validly attach.The next question to determine is whether Lourdes M. de Leon course.

59
MATABABE FILES
Corporation Law

Corporation from liability and dismissing the complaint as against it. The appellate court ruled
Same; Same; A holder not in due course may still recover on the instrument.It does that: (1) Lourdes M. de Leon was not authorized to issue the subject checks in favor of E.T.
not follow as a legal proposition that simply because petitioner Atrium was not a holder in Henry, Inc.; (2) The issuance of the subject checks by Lourdes M. de Leon and the late Antonio
due course for having taken the instruments in question with notice that the same was for de las Alas constituted ultra vires acts; and (3) The subject checks were not issued for valuable
deposit only to the account of payee E.T. Henry that it was altogether precluded from consideration.5
recovering on the instrument. The Negotiable Instruments Law does not provide that a holder
not in due course can not recover on the instrument. At the trial, Atrium presented as its witness Carlos C. Syquia who testified that in
February 1981, Enrique Tan of E.T. Henry approached Atrium for financial assistance, offering
Same; Same; The disadvantage of a holder not in due course is that the negotiable to discount four RCBC checks in the total amount of P2 million, issued by Hi-Cement in favor
instrument is subject to defenses as if it were non-negotiable, such as absence or failure of of E.T. Henry. Atrium agreed to discount the checks, provided it be allowed to confirm with Hi-
consideration.The disadvantage of Atrium in not being a holder in due course is that the Cement the fact that the checks represented payment for petroleum products which E.T.
negotiable instrument is subject to defenses as if it were non-negotiable. One such defense is Henry delivered to Hi-Cement. Carlos C. Syquia identified two letters, dated February 6, 1981
absence or failure of consideration. and February 9, 1981 issued by Hi-Cement through Lourdes M. de Leon, as treasurer,
PETITION for review on certiorari of a decision of the Court of Appeals. confirming the issuance of the four checks in favor of E.T. Henry in payment for petroleum
products.6
The facts are stated in the opinion of the Court. Atrium Management Corporation vs.
Court of Appeals, 353 SCRA 23, G.R. No. 109491, G.R. No. 121794 February 28, 2001 Respondent Hi-Cement presented as witness Ms. Erlinda Yap who testified that she
was once a secretary to the treasurer of Hi-Cement, Lourdes M. de Leon, and as such she was
PARDO, J.: familiar with the four RCBC checks as the postdated checks issued by Hi-Cement to E.T. Henry
upon instructions of Ms. de Leon. She testified that E.T. Henry offered to give Hi-Cement a
What is before the Court are separate appeals from the decision of the Court of loan which the subject checks would secure as collateral.7
Appeals,1 ruling that Hi-Cement Corporation is not liable for four checks amounting to P2
million issued to E.T. Henry and Co. and discounted to Atrium Management Corporation. On July 20, 1989, the Regional Trial Court, Manila, Branch 09 rendered a decision, the
dispositive portion of which reads:
On January 3, 1983, Atrium Management Corporation filed with the Regional Trial
Court, Manila an action for collection of the proceeds of four postdated checks in the total "WHEREFORE, in view of the foregoing considerations, and plaintiff having
amount of P2 million. Hi-Cement Corporation through its corporate signatories, petitioner proved its cause of action by preponderance of evidence, judgment is hereby
Lourdes M. de Leon,2 treasurer, and the late Antonio de las Alas, Chairman, issued checks in rendered ordering all the defendants except defendant Antonio de las Alas to pay
favor of E.T. Henry and Co. Inc., as payee. E.T. Henry and Co., Inc., in turn, endorsed the four plaintiff jointly and severally the amount of TWO MILLION (P2,000,000.00) PESOS
checks to petitioner Atrium Management Corporation for valuable consideration. Upon with the legal rate of interest from the filling of the complaint until fully paid, plus the
presentment for payment, the drawee bank dishonored all four checks for the common sum of TWENTY THOUSAND (P20,000.00) PESOS as and for attorney's fees and the
reason "payment stopped". Atrium, thus, instituted this action after its demand for payment cost of suit."
of the value of the checks was denied.3
All other claims are, for lack of merit dismissed.
After due proceedings, on July 20, 1989, the trial court rendered a decision ordering SO ORDERED."8
Lourdes M. de Leon, her husband Rafael de Leon, E.T. Henry and Co., Inc. and Hi-Cement
Corporation to pay petitioner Atrium, jointly and severally, the amount of P2 million In due time, both Lourdes M. de Leon and Hi-Cement appealed to the Court of
corresponding to the value of the four checks, plus interest and attorney's fees. 4 Appeals.9

On appeal to the Court of Appeals, on March 17, 1993, the Court of Appeals Lourdes M. de Leon submitted that the trial court erred in ruling that she was
promulgated its decision modifying the decision of the trial court, absolving Hi-Cement solidarilly liable with Hi-Cement for the amount of the check. Also, that the trial court erred in

60
MATABABE FILES
Corporation Law

ruling that Atrium was an ordinary holder, not a holder in due course of the rediscounted In G. R. No. 121794 (de Leon, petitioner):
checks.10
1. Whether the Court of Appeals erred in holding petitioner personally liable for the
Hi-Cement on its part submitted that the trial court erred in ruling that even if Hi- Hi-Cement checks issued to E.T. Henry;
Cement did not authorize the issuance of the checks, it could still be held liable for the checks.
And assuming that the checks were issued with its authorization, the same was without any 2. Whether the Court of Appeals erred in ruling that Atrium is a holder in due course;
consideration, which is a defense against a holder in due course and that the liability shall be
borne alone by E.T. Henry.11 3. Whether the Court of Appeals erred in ruling that petitioner Lourdes M. de Leon as
signatory of the checks was personally liable for the value of the checks, which were
On March 17, 1993, the Court of Appeals promulgated its decision modifying the declared to be issued without consideration;
ruling of the trial court, the dispositive portion of which reads:
4. Whether the Court of Appeals erred in ordering petitioner to pay Hi-Cement
"Judgement is hereby rendered: attorney's fees and costs.15

(1) dismissing the plaintiff's complaint as against defendants Hi-Cement Corporation We affirm the decision of the Court of Appeals.
and Antonio De las Alas;
We first resolve the issue of whether the issuance of the checks was an ultra vires act.
(2) ordering the defendants E.T. Henry and Co., Inc. and Lourdes M. de Leon, jointly The record reveals that Hi-Cement Corporation issued the four (4) checks to extend financial
and severally to pay the plaintiff the sum of TWO MILLION PESOS (P2,000,000.00) assistance to E.T. Henry, not as payment of the balance of the P30 million pesos cost of hydro
with interest at the legal rate from the filling of the complaint until fully paid, plus oil delivered by E.T. Henry to Hi-Cement. Why else would petitioner de Leon ask for
P20,000.00 for attorney's fees. counterpart checks from E.T. Henry if the checks were in payment for hydro oil delivered
by E.T. Henry to Hi-Cement?
(3) Ordering the plaintiff and defendants E.T. Henry and Co., Inc. and Lourdes M. de
Leon, jointly and severally to pay defendant Hi-Cement Corporation, the sum of Hi-Cement, however, maintains that the checks were not issued for consideration
P20,000.00 as and for attorney's fees. and that Lourdes and E.T. Henry engaged in a "kiting operation" to raise funds for E.T. Henry,
With cost in this instance against the appellee Atrium Management Corporation and who admittedly was in need of financial assistance. The Court finds that there was no
appellant Lourdes Victoria M. de Leon. sufficient evidence to show that such is the case. Lourdes M. de Leon is the treasurer of the
corporation and is authorized to sign checks for the corporation. At the time of the issuance
So ordered."12 of the checks, there were sufficient funds in the bank to cover payment of the amount of P2
million pesos.
Hence, the recourse to this Court.13
It is, however, our view that there is basis to rule that the act of issuing the checks
The issues raised are the following: was well within the ambit of a valid corporate act, for it was for securing a loan to finance the
activities of the corporation, hence, not an ultra vires act.
In G. R. No. 109491 (Atrium, petitioner):
"An ultra vires act is one committed outside the object for which a corporation is
1. Whether the issuance of the questioned checks was an ultra vires act; created as defined by the law of its organization and therefore beyond the power conferred
2. Whether Atrium was not a holder in due course and for value; and upon it by law"16 The term "ultra vires" is "distinguished from an illegal act for the former is
3. Whether the Court of Appeals erred in dismissing the case against Hi-Cement and merely voidable which may be enforced by performance, ratification, or estoppel, while the
ordering it to pay P20,000.00 as attorney's fees.14 latter is void and cannot be validated."17

61
MATABABE FILES
Corporation Law

The next question to determine is whether Lourdes M. de Leon and Antonio de las (d) That at the time it was negotiated to him he had no notice of any
Alas were personally liable for the checks issued as corporate officers and authorized infirmity in the instrument or defect in the title of the person negotiating it."
signatories of the check.
In the instant case, the checks were crossed checks and specifically indorsed for
"Personal liability of a corporate director, trustee or officer along (although not deposit to payee's account only. From the beginning, Atrium was aware of the fact that the
necessarily) with the corporation may so validly attach, as a rule, only when: checks were all for deposit only to payee's account, meaning E.T. Henry. Clearly, then, Atrium
could not be considered a holder in due course.
"1. He assents (a) to a patently unlawful act of the corporation, or (b) for bad faith or
gross negligence in directing its affairs, or (c) for conflict of interest, resulting in However, it does not follow as a legal proposition that simply because petitioner
damages to the corporation, its stockholders or other persons; Atrium was not a holder in due course for having taken the instruments in question with
notice that the same was for deposit only to the account of payee E.T. Henry that it was
"2. He consents to the issuance of watered down stocks or who, having knowledge altogether precluded from recovering on the instrument. The Negotiable Instruments Law
thereof, does not forthwith file with the corporate secretary his written objection does not provide that a holder not in due course can not recover on the instrument. 19
thereto;
"3. He agrees to hold himself personally and solidarily liable with the corporation; or The disadvantage of Atrium in not being a holder in due course is that the negotiable
instrument is subject to defenses as if it were non-negotiable.20 One such defense is absence
"4. He is made, by a specific provision of law, to personally answer for his corporate or failure of consideration.21
action."18
We need not rule on the other issues raised, as they merely follow as a consequence
In the case at bar, Lourdes M. de Leon and Antonio de las Alas as treasurer and of the foregoing resolutions.
Chairman of Hi-Cement were authorized to issue the checks. However, Ms. de Leon was
negligent when she signed the confirmation letter requested by Mr. Yap of Atrium and Mr. WHEREFORE, the petitions are hereby DENIED. The decision and resolution of the
Henry of E.T. Henry for the rediscounting of the crossed checks issued in favor of E.T. Henry. Court of Appeals in CA-G. R. CV No. 26686, are hereby AFFIRMED in toto. No costs.
She was aware that the checks were strictly endorsed for deposit only to the payee's account Republic of the Philippines
and not to be further negotiated. What is more, the confirmation letter contained a clause SUPREME COURT
that was not true, that is, "that the checks issued to E.T. Henry were in payment of Hydro oil Manila
bought by Hi-Cement from E.T. Henry". Her negligence resulted in damage to the corporation. THIRD DIVISION
Hence, Ms. de Leon may be held personally liable therefor.
The next issue is whether or not petitioner Atrium was a holder of the checks in due G.R. No. 111448 January 16, 2002
course. The Negotiable Instruments Law, Section 52 defines a holder in due course, thus:
AF REALTY & DEVELOPMENT, INC. and ZENAIDA R. RANULLO, petitioners,
"A holder in due course is a holder who has taken the instrument under the following vs.
conditions: DIESELMAN FREIGHT SERVICES, CO., MANUEL C. CRUZ, JR. and MIDAS DEVELOPMENT
CORPORATION, respondents.
(a) That it is complete and regular upon its face;
Corporation Law; Agency; Contracts or acts of a corporation must be made either by
(b) That he became the holder of it before it was overdue, and without the board of directors or by a corporate agent duly authorized by the board; Absent such valid
notice that it had been previously dishonored, if such was the fact; delegation/authorization the rule is that the declarations of an individual director relating to
the affairs of the corporation, but not in the course of, or connected with, the performance of
(c) That he took it in good faith and for value; authorized duties of such director, are held not binding on the corporation.Section 23 of the
Corporation Code expressly provides that the corporate powers of all corporations shall be

62
MATABABE FILES
Corporation Law

exercised by the board of directors. Just as a natural person may authorize another to do
certain acts in his behalf, so may the board of directors of a corporation validly delegate some On June 29, 1988, AF Realty confirmed its intention to buy the lot. Hence, Ranullo
of its functions to individual officers or agents appointed by it. Thus, contracts or acts of a asked Polintan for the board resolution of Dieselman authorizing the sale of the property.
corporation must be made either by the board of directors or by a corporate agent duly However, Polintan could only give Ranullo the original copy of TCT No. 39849, the tax
authorized by the board. Absent such valid delegation/authorization, the rule is that the declaration and tax receipt for the lot, and a photocopy of the Articles of Incorporation of
declarations of an individual director relating to the affairs of the corporation, but not in the Dieselman.7
course of, or connected with, the performance of authorized duties of such director, are held
not binding on the corporation. On August 2, 1988, Manuel F. Cruz, Sr., president of Dieselman, acknowledged receipt
of the said P300,000.00 as "earnest money" but required AF Realty to finalize the sale
PETITION for review on certiorari of a decision of the Court of Appeals. at P4,000.00 per square meter.8 AF Realty replied that it has paid an initial down payment of
P300,000.00 and is willing to pay the balance.9
The facts are stated in the opinion of the Court.
However, on August 13, 1988, Mr. Cruz, Sr. terminated the offer and demanded from
SANDOVAL-GUTIERREZ, J.: AF Realty the return of the title of the lot earlier delivered by Polintan.10

Petition for review on certiorari assailing the Decision dated December 10, 1992 and Claiming that there was a perfected contract of sale between them, AF Realty filed
the Resolution (Amending Decision) dated August 5, 1993 of the Court of Appeals in CA-G.R. with the Regional Trial Court, Branch 160, Pasig City a complaint for specific performance (Civil
CV No. 30133. Case No. 56278) against Dieselman and Cruz, Jr.. The complaint prays that Dieselman be
ordered to execute and deliver a final deed of sale in favor of AF Realty. 11 In its amended
Dieselman Freight Service Co. (Dieselman for brevity) is a domestic corporation and a complaint,12 AF Realty asked for payment of P1,500,000.00 as compensatory damages;
registered owner of a parcel of commercial lot consisting of 2,094 square meters, located at P400,000.00 as attorney's fees; and P500,000.00 as exemplary damages.
104 E. Rodriguez Avenue, Barrio Ugong, Pasig City, Metro Manila. The property is covered by
Transfer Certificate of Title No. 39849 issued by the Registry of Deeds of the Province of In its answer, Dieselman alleged that there was no meeting of the minds between the
Rizal.1 parties in the sale of the property and that it did not authorize any person to enter into such
transaction on its behalf.
On May 10, 1988, Manuel C. Cruz, Jr., a member of the board of directors of
Dieselman, issued a letter denominated as "Authority To Sell Real Estate" 2 to Cristeta N. Meanwhile, on July 30, 1988, Dieselman and Midas Development Corporation (Midas)
Polintan, a real estate broker of the CNP Real Estate Brokerage. Cruz, Jr. authorized Polintan executed a Deed of Absolute Sale13 of the same property. The agreed price was P2,800.00 per
"to look for a buyer/buyers and negotiate the sale" of the lot at P3,000.00 per square meter, square meter. Midas delivered to Dieselman P500,000.00 as down payment and deposited
or a total of P6,282,000.00. Cruz, Jr. has no written authority from Dieselman to sell the lot. the balance of P5,300,000.00 in escrow account with the PCIBank.

In turn, Cristeta Polintan, through a letter3 dated May 19, 1988, authorized Felicisima Constrained to protect its interest in the property, Midas filed on April 3, 1989 a
("Mimi") Noble4 to sell the same lot. Motion for Leave to Intervene in Civil Case No. 56278. Midas alleged that it has purchased the
property and took possession thereof, hence Dieselman cannot be compelled to sell and
Felicisima Noble then offered for sale the property to AF Realty & Development, Inc. convey it to AF Realty. The trial court granted Midas' motion.
(AF Realty) at P2,500.00 per square meter.5 Zenaida Ranullo, board member and vice-
president of AF Realty, accepted the offer and issued a check in the amount of P300,000.00 After trial, the lower court rendered the challenged Decision holding that the acts of
payable to the order of Dieselman. Polintan received the check and signed an Cruz, Jr. bound Dieselman in the sale of the lot to AF Realty. 14 Consequently, the perfected
"Acknowledgement Receipt"6 indicating that the amount of P300,000.00 represents the contract of sale between Dieselman and AF Realty bars Midas' intervention. The trial court
partial payment of the property but refundable within two weeks should AF Realty also held that Midas acted in bad faith when it initially paid Dieselman P500,000.00 even
disapprove Ranullo's action on the matter. without seeing the latter's title to the property. Moreover, the notarial report of the sale was

63
MATABABE FILES
Corporation Law

not submitted to the Clerk of Court of the Quezon City RTC and the balance of P5,300,000.00 "SO ORDERED."17
purportedly deposited in escrow by Midas with a bank was not established.
AF Realty now comes to this Court via the instant petition alleging that the Court of
The dispositive portion of the trial court's Decision reads: Appeals committed errors of law.

"WHEREFORE, foregoing considered, judgment is hereby rendered ordering The focal issue for consideration by this Court is who between petitioner AF Realty
defendant to execute and deliver to plaintiffs the final deed of sale of the property and respondent Midas has a right over the subject lot.
covered by the Transfer Certificate of Title No. 39849 of the Registry of Deed of Rizal,
Metro Manila District II, including the improvements thereon, and ordering The Court of Appeals, in reversing the judgment of the trial court, made the following
defendants to pay plaintiffs attorney's fees in the amount of P50,000.00 and to pay ratiocination:
the costs.
"The counterclaim of defendants is necessarily dismissed. "From the foregoing scenario, the fact that the board of directors of Dieselman never
"The counterclaim and/or the complaint in intervention are likewise dismissed authorized, verbally and in writing, Cruz, Jr. to sell the property in question or to look
for buyers and negotiate the sale of the subject property is undeniable.
"SO ORDERED."15
"While Cristeta Polintan was actually authorized by Cruz, Jr. to look for buyers and
Dissatisfied, all the parties appealed to the Court of Appeals. negotiate the sale of the subject property, it should be noted that Cruz, Jr. could not
confer on Polintan any authority which he himself did not have. Nemo dat quod non
AF Realty alleged that the trial court erred in not holding Dieselman liable for moral, habet. In the same manner, Felicisima Noble could not have possessed authority
compensatory and exemplary damages, and in dismissing its counterclaim against Midas. broader in scope, being a mere extension of Polintan's purported authority, for it is a
legal truism in our jurisdiction that a spring cannot rise higher than its source.
Upon the other hand, Dieselman and Midas claimed that the trial court erred in Succinctly stated, the alleged sale of the subject property was effected through
finding that a contract of sale between Dieselman and AF Realty was perfected. Midas further persons who were absolutely without any authority whatsoever from Dieselman.
averred that there was no bad faith on its part when it purchased the lot from Dieselman.
"The argument that Dieselman ratified the contract by accepting the P300,000.00 as
In its Decision dated December 10, 1992, the Court of Appeals reversed the judgment partial payment of the purchase price of the subject property is equally untenable.
of the trial court holding that since Cruz, Jr. was not authorized in writing by Dieselman to sell The sale of land through an agent without any written authority is void.
the subject property to AF Realty, the sale was not perfected; and that the Deed of Absolute
Sale between Dieselman and Midas is valid, there being no bad faith on the part of the latter. xxx xxx xxx
The Court of Appeals then declared Dieselman and Cruz, Jr. jointly and severally liable to AF
Realty for P100,000.00 as moral damages; P100,000.00 as exemplary damages; and "On the contrary, anent the sale of the subject property by Dieselman to intervenor
P100,000.00 as attorney's fees.16 Midas, the records bear out that Midas purchased the same from Dieselman on 30
July 1988. The notice of lis pendens was subsequently annotated on the title of the
On August 5, 1993, the Court of Appeals, upon motions for reconsideration filed by property by plaintiffs on 15 August 1988. However, this subsequent annotation of the
the parties, promulgated an Amending Decision, the dispositive portion of which reads: notice of lis pendens certainly operated prospectively and did not retroact to make
the previous sale of the property to Midas a conveyance in bad faith. A subsequently
"WHEREFORE, The Decision promulgated on October 10, 1992, is hereby AMENDED in registered notice of lis pendens surely is not proof of bad faith. It must therefore be
the sense that only defendant Mr. Manuel Cruz, Jr. should be made liable to pay the borne in mind that the 30 July 1988 deed of sale between Midas and Dieselman is a
plaintiffs the damages and attorney's fees awarded therein, plus the amount of document duly certified by notary public under his hand and seal. x x x. Such a deed
P300,000.00 unless, in the case of the said P300,000.00, the same is still deposited of sale being public document acknowledged before a notary public is admissible as
with the Court which should be restituted to plaintiffs. to the date and fact of its execution without further proof of its due execution and

64
MATABABE FILES
Corporation Law

delivery (Bael vs. Intermediate Appellate Court, 169 SCRA617; Joson vs. Baltazar, 194 acting in their stead, are subject to thesame rules, liabilities, and incapacities as are
SCRA 114) and to prove the defects and lack of consent in the execution thereof, the agents of individuals and private persons." (Emphasis supplied)
evidence must be strong and not merely preponderant x x x." 18
Pertinently, Article 1874 of the same Code provides:
We agree with the Court of Appeals.
"ART. 1874. When a sale of piece of land or any interest therein is through an agent,
Section 23 of the Corporation Code expressly provides that the corporate powers of the authority of the latter shall be in writing; otherwise, the sale shall be void."
all corporations shall be exercised by the board of directors. Just as a natural person may (Emphasis supplied)
authorize another to do certain acts in his behalf, so may the board of directors of a
corporation validly delegate some of its functions to individual officers or agents appointed Considering that respondent Cruz, Jr., Cristeta Polintan and Felicisima Ranullo were
by it.19 Thus, contracts or acts of a corporation must be made either by the board of directors not authorized by respondent Dieselman to sell its lot, the supposed contract is void. Being a
or by a corporate agent duly authorized by the board. 20 Absent such valid void contract, it is not susceptible of ratification by clear mandate of Article 1409 of the Civil
delegation/authorization, the rule is that the declarations of an individual director relating to Code, thus:
the affairs of the corporation, but not in the course of, or connected with, the performance of
authorized duties of such director, are held not binding on the corporation.21 "ART. 1409. The following contracts are inexistent and void from the very beginning:

In the instant case, it is undisputed that respondent Cruz, Jr. has no written authority xxx
from the board of directors of respondent Dieselman to sell or to negotiate the sale of the lot,
much less to appoint other persons for the same purpose. Respondent Cruz, Jr.'s lack of such (7) Those expressly prohibited or declared void by law.
authority precludes him from conferring any authority to Polintan involving the subject realty.
Necessarily, neither could Polintan authorize Felicisima Noble. Clearly, the collective acts of "These contracts cannot be ratified. Neither can the right to set up the defense of
respondent Cruz, Jr., Polintan and Noble cannot bind Dieselman in the purported contract of illegality be waived." (Emphasis supplied)
sale.
Upon the other hand, the validity of the sale of the subject lot to respondent Midas is
Petitioner AF Realty maintains that the sale of land by an unauthorized agent may be unquestionable. As aptly noted by the Court of Appeals,24 the sale was authorized by a board
ratified where, as here, there is acceptance of the benefits involved. In this case the receipt by resolution of respondent Dieselman dated May 27, 1988.
respondent Cruz, Jr. from AF Realty of the P300,000.00 as partial payment of the lot
effectively binds respondent Dieselman.22 The Court of Appeals awarded attorney's fees and moral and exemplary damages in
favor of petitioner AF Realty and against respondent Cruz, Jr.. The award was made by reason
We are not persuaded. of a breach of contract imputable to respondent Cruz, Jr. for having acted in bad faith. We are
no persuaded. It bears stressing that petitioner Zenaida Ranullo, board member and vice-
Involved in this case is a sale of land through an agent. Thus, the law on agency under president of petitioner AF Realty who accepted the offer to sell the property, admitted in her
the Civil Code takes precedence. This is well stressed in Yao Ka Sin Trading vs. Court of testimony25that a board resolution from respondent Dieselman authorizing the sale is
Appeals:23 necessary to bind the latter in the transaction; and that respondent Cruz, Jr. has no such
"Since a corporation, such as the private respondent, can act only through its officers written authority. In fact, despite demand, such written authority was not presented to
and agents, all acts within the powers of said corporation may be performed by her.26 This notwithstanding, petitioner Ranullo tendered a partial payment for the
agents of its selection; and, except so far as limitations or restrictions may be unauthorized transaction. Clearly, respondent Cruz, Jr. should not be held liable for damages
imposed by special charter, by-law, or statutory provisions, the same general and attorney's fees.
principles of law which govern the relation of agency for a natural person govern
the officer or agent of a corporation, of whatever status or rank, in respect to WHEREFORE, the assailed Decision and Resolution of the Court of Appeals are
his power to act for the corporation; and agents when once appointed, or members hereby AFFIRMED with MODIFICATION in the sense that the award of damages and

65
MATABABE FILES
Corporation Law

attorney's fees is deleted. Respondent Dieselman is ordered to return to petitioner AF Realty


its partial payment of P300,000.00. Costs against petitioners.

SO ORDERED.

66
MATABABE FILES
Corporation Law

Republic of the Philippines contrary to law, morals, or public order, or contravene some rules of public policy or public
SUPREME COURT duty, and are, like similar transactions between individuals, void. They can not serve as basis
Manila of a court action, nor acquire validity by performance, ratification, or estoppel. On the other
EN BANC hand, ultra vires acts or those which are not illegal and void ab initio but are merely within the
scope of the article of incorporation, are merely voidable and may become binding and
G.R. No. L-5377 December 29, 1954 enforceable when ratified by the stockholders.

MARIA CLARA PIROVANA ET AL., plaintiffs-appellees, 6. ID.; ID.; "ULTRA VIRES" ACTS; RATIFICATION BY STOCKHOLDERS OF "ULTRA VIRES"
vs. ACTS CURES INFIRMITY.The ratification by the stockholders of an ultra vires act which is not
THE DE LA RAMA STEAMSHIP CO., defendant-appellant. illegal cures the infirmity of the corporate act and makes it perfectly valid and enforceable,
specially so if it is not merely executory but executed and consummated and no creditors are
1. CORPORATIONS; DONATIONS; DONATION GlVEN "OUT OF GRATITUDE FOR SERVICES prejudiced thereby.
RENDERED" Is REMUNERATIVE.A donation given by the corporation to the minor children of
its late president because he "was to a large extent responsible for the rapid and very 7. ATTORNEY'S FEES, WHEN MAY BE AWARDED AS DAMAGES.When the
successful development and expansion of the activities of this company" is remunerative in defendant's act or omission has compelled the plaintiff to litigate with third persons or to
nature in contemplation of law. incur expenses to protect his interest, attorney's fees may be awarded as damages (Article
2208, paragraph 2, of the new Civil Code).
2. ID.; ID.; PERFECTED DONATION CAN ONLY BE RESCINDED ON LEGAL GROUNDS. APPEAL from a judgment of the Court of First Instance of Rizal. Tan, J.
Where the donation made by the corporation has not only been granted in several resolutions
duly adopted by its board of directors but also it has been formally ratified by its stockholders, The facts are stated in the opinion of the Court.
with the concurrence of its only creditor, and accepted by the donee, the donation -has
reached the stage of perfection which is valid and binding upon the corporation and as such Del Rosario and Garcia for appellant.
cannot be rescinded unless there exist legal grounds for doing so. Vicente J. Francisco for appellees.

3. ID.; ID.; DONATION DISTINGUISHED FROM GRATUITY.While a donation may BAUTISTA ANGELO, J.:
technically be different from a gratuity, in substance they are the same. They are even similar
to a pension. Thus, it was said that "A pension is a gratuity only when it is granted for services This is an appeal from a decision of the Court of First Instance of Rizal declaring the
previously rendered, and which at the time they were rendered gave rise to no legal donation made by the defendant in favor of the minor children of the late Enrico Pirovano of
obligation." (Words and Phrases, Permanent Edition, p. 675; O'Dea vs. Ck, 169 Pac., 306, 176 the proceeds of the insurance policies taken on his life valid and binding, and ordering said
Cal., 659.) defendant to pay to said minor children the sum of P583,813.59, with interest thereon at the
rate of per cent from the date of filing of the complaint, plus an additional amount equivalent
4. ID.; POWERS OF A CORPORATION; ACTS PERFORMED WITHIN THE POWERS to 20 per cent of said sum of P538,813.59 as damages by way of attorney's fees and the costs
GRANTED ARE NOT "ULTRA VIRES".Where the corporation was given broad and almost of action.
unlimited powers to carry out the purposes for which it was organized among them, to aid in
any other manner any person in the affairs and prosperity of whom it has a lawful interest, a Plaintiffs herein are the minor children of the late Enrico Pirovano represented by
donation made to the heirs of its late president in recognition of the valuable services their mother and judicial guardian Estefania R. Pirovano. They seek to enforce certain
rendered by the latter which had immensely contributed to its growth, comes within this resolutions adopted by the Board of Directors and stockholders of the defendant company
broad grant of power and can not be considered an ultra vires act. giving to said minor children of the proceeds of the insurance policies taken on the life of their
deceased father Enrico Pirovano with the company as beneficiary. Defendant's main defense
5. ID.; ID.; "ULTRA VIRES" ILLEGAL ACTS DISTINGUISHED; EFFECT OF RATIFICATION BY is: that said resolutions and the contract executed pursuant thereto are ultra vires, and, if
STOCKHOLDERS.Illegal acts of a corporation contemplate the doing of an act which is valid, the obligation to pay the amount given is not yet due and demandable.

67
MATABABE FILES
Corporation Law

Enrico Pirovano became the president of the defendant company and under his
The trial court resolved all the issues raised by the parties in favor of the plaintiffs management the company grew and progressed until it became a multi-million corporation by
and, after considering the evidence, both oral and documentary, arrived at the following the time Pirovano was executed by the Japanese during the occupation. On May 13, 1941, the
conclusions: capital stock of the corporation was increased to P2,000,000, after which a 100 per cent stock
dividend was declared. Subsequently, or before the outbreak of the war , new stock dividends
First. That the contract executed between the plaintiffs and the defendant is a of 200 per cent and 33 1/3 per cent were again declared. On December 4, 1941, the capital
renumerative donation. stock was once more increased to P5,000,000. Under Pirovano's management, the assets of
the company grew and increased from an original paid up capital of around P240,000 to
Second. That said contract or donation is not ultra vires, but an act executed within P15,538,024.37 by September 30, 1941 (Exhibit HH).
the powers of the defendant corporation in accordance with its articles of
incorporation and by laws, sanctioned and approved by its Board of Directors and In the meantime, Don Esteban de la Rama, who practically owned and controlled the
stockholders; and subsequently ratified by other subsequent acts of the defendant stock of the defendant corporation, distributed his shareholding among his five daughters,
company. namely, Leonor, Estefania, Lourdes, Lolita and Conchita and his wife Natividad Aguilar so that,
at that time, or on July 10, 1946, the stockholding of the corporation stood as follows: Esteban
Third. That the said donation is in accordance with the trend of modern and more de la Rama, 869 shares, Leonor de la Rama, 3,375 shares, Estefania de la Rama, 3,368 shares,
enlightened legislation in its treatment of questions between labor and capital. Lourdes de la Rama, 3,368 shares, Lolita de la Rama, 3,368 shares, Conchita de la Rama, 3,376
shares, and Natividad Aguilar, 2,136 shares. The other stockholders , namely, Eliseo Hervas,
Fourth. That the condition mentioned in the donation is null and void because it Tomas Concepcion, Antonio Juanco, and Jose Aguilar, who were merely employees of Don
depends on the provisions of Article 1115 of the old Civil Code. Esteban, were given 40 shares each, while Pio Pedrosa, Marcial P. Lichauco and Rafael Roces,
one share each, because they merely represented the National Development Company. This
Fifth. That if the condition is valid, its non-fulfillment is due to the desistance of the Company was given representation in the Board Of Directors of the corporation because at
defendant company from obeying and doing the wishes and mandates of the that time the latter had an outstanding bonded indebtedness to the National Development
majority of the stockholders. Company.

Sixth. That the non-payment of the debt in favor of the National Development This bonded indebtedness was incurred on February 26, 1940 and was in the amount
Company is not due to the lack of funds, nor to lack of authority, but the desire of the of P7,500.00. The bond held by the National Development Company was redeemable within a
President of the corporation to preserve and continue the Government participation period of 20 years from March 1, 1940,. bearing interest at the rate of 5 per cent per annum.
in the company. To secure said bonded indebtedness, all the assets of the De la Rama Steamship Co., Inc., and
properties of Don Esteban de la Rama, as well as those of the Hijos de I. de la Rama and Co.,
Seventh. That due demands were made by the plaintiffs and their attorneys and Inc., a sister corporation owned by Don Esteban and his family, were mortgaged to the
these demands were rejected for no justifiable or legal grounds. National Development Company (Annexes A, B, C, D of Exhibit 3, Deed of Trust). Payments
made by the corporation under the management of Pirovano reduced this bonded
The important facts which need to be considered for purposes of this appeal may be indebtedness to P3,260,855.77.
briefly stated as follows: Defendant is a corporation duly organized in accordance with law
with an authorized capital of P500,000, divided into 5,000 shares, with a par value of P100 Upon arrangement made with the National Development Company, the outstanding
each share. The stockholders were: Esteban de la Rama, 1,800 shares, Leonor de la Rama, 100 bonded indebtedness was converted into non-voting preferred shares of stock of the De la
shares, Estefania de la Rama, 100 shares, and Eliseo Hervas, Tomas Concepcion, Antonio G. Rama company under the express condition that they would bear affixed cumulative dividend
Juanco, and Gaudencio Volasote with 5 shares each. Leonor and Estefania are daughters of of 6 per cent per annum and would be redeemable within 15 years (Exhibits 5 and 7). This
Don Esteban, while the rest his employees. Estefania de la Rama was married to the late conversion was carried out on September 23, 1949, when the National Development Company
Enrico Pirovano and to them four children were born who are the plaintiffs in this case. executed a "Deed of Termination of Trust and Release of Mortgage" in favor of the De la
Rama company (Exhibit 6.) The immediate effect of this conversion was the released from

68
MATABABE FILES
Corporation Law

incumbrance of all the properties Of Don Esteban and of the Hijos de I. de la Rama and Co.,
Inc., which was apparently favorable to the interests of the De la Rama company, but, on the Whereas, the said Enrico Pirovano was largely responsible for the rapid and
other hand, it resulted in the inconvenience that, as holder of the preferred stock, the very successful development of the activities of thus company;
National Development Company, was given to the right to 40 per cent of the membership of
the Board of Directors of the De la Rama company, which meant an increase in the Whereas, early in 1941 this company insured the life of said Enrico Pirovano in
representation of the National Development Company from 2 to 4 of the 9 members of said various Philippine and American Life Insurance companies for the total sum of
Board of Directors. P1,000,000;

The first resolution granting to the Pirovano children the proceeds of the insurance Whereas, the said Enrico Pirovano is survived by his widow, Estefania
policies taken on his life by the defendant company was adopted by the Board of Directors at Pirovano and four minor children, to wit: Esteban, Maria Carla, Enrico and John
a meeting held on July 10, 1946, (Exhibit B). This grant was called in the resolution as "Special Albert, all surnamed Pirovano;
Payment to Minor Heirs of the late Enrico Pirovano". Because of its direct hearing on the
issues involved in this case, said resolution is hereunder reproduced in toto: Whereas, said Enrico Pirovano left practically nothing to his heirs and it is but
fit proper that this company which owes so much to the deceased should make some
SPECIAL PAYMENT TO MINORS HEIRS OF THE LATE ENRICO PIROVANO provision for his children;
Whereas, this company paid premium on Mr. Pirovano's life insurance
The President stated that the principal purpose for which the meeting had policies for a period of only 4 years so that it will receive from the insurance
been called was to discuss the advisability of making some form of compensation to companies sums of money greatly in excess of the premiums paid by this company.
the minor heirs of the late Enrico Pirovano, former President and General Manager of
the Company. As every member of the Board knows, said the President, the late Be it resolved, That out of the proceeds to be collected from the life
Enrico Pirovano who was largely responsible for the very successful development of insurance policies on the life of the late Enrico Pirovano, the sum of P400,000 be set
the activities of the Company prior to war was killed by the Japanese in Manila aside for equal division among the 4 minor children of the deceased, to wit: Esteban,
sometime in 1944 leaving as his only heirs four minor children, Maria Carla, Esteban, Maria Carla, Enrico and John Albert, all surnamed Pirovano, which sum of money shall
Enrico and John Albert. Early in 1941, explained the President, the Company had be convertible into shares of stock of the De la Rama Steamship Company, at par
insured the life of Mr. Pirovano for a million pesos. Following the occupation of the and, for that purpose, that the present registered stockholders of the corporation be
Philippines by Japanese forces the Company was unable to pay the premiums on requested to waive their preemptive right to 4,000 shares of the unissued stock of
those policies issued by Filipino companies and these policies had lapsed. But with the company in order to enable each of the 4 minor heirs of the deceased, to wit:
regards to the York Office of the De la Rama Steamship Co., Inc. had kept up Esteban, Maria Carla, Enrico and John Albert, all surnamed Pirovano, to obtain 1,000
payment of the premiums from year to year. The payments made on account of shares at par;
these premiums, however, are very small compared to the amount which the Resolved, further, that in view of the fact that under the provisions of the indenture
Company will now receive as a result of Mr. Pirovano's death. The President with the National Development Company, it is necessary that action herein proposed
proposed therefore that out of the proceeds of these policies the sum of P400,000 to be confirmed by the Board of Directors of that company, the Secretary is hereby
be set aside for the minor children of the deceased, said sum of money to be instructed to send a copy of this resolution to the proper officers of the National
convertible into 4,000 shares of the stock of the Company, at par, or 1,000 shares for Development Company for appropriate action. (Exhibit B)
each child. This proposal, explained the President as being made by him upon
suggestion of President Roxas, but, he added, that he himself was very much in favor The above resolution, which was adopted on July 10, 1946, was submitted to the
of it also. On motion of Miss Leonor de la Rama duly seconded by Mrs. Lourdes de la stockholders of the De la Rama company at a meeting properly convened, and on that same
Rama de Osmea, the following resolution was, thereupon, unanimously approved: date, July 10, 1946, the same was duly approved.

Whereas, the late Enrico Pirovano, President and General Manager of the De It appears that, although Don Esteban and the Members of his family were agreeable
la Rama Steamship Company, died in Manila sometime in November, 1944: to giving to the Pirovano children the amount of P400,000 out of the proceeds of the

69
MATABABE FILES
Corporation Law

insurance policies taken on the life of Enrico Pirovano, they did not realize that when they Esteban, Maria Carla, Enrico and John Albert, all surnamed Pirovano, subject to the
provided in the above referred two resolutions that said Amount should be paid in the form terms and conditions herein after provided;
of shares of stock, they would be actually giving to the Pirovano children more than what
they intended to give. This came about when Lourdes de la Rama, wife of Sergio Osmea, Jr., That the proceeds of said insurance policies shall be retained by the
showed to the latter copies of said resolutions and asked him to explain their import and Company in the nature of a loan drawing interest at the rate of 5 per cent annum
meaning, and it was value then that Osmea explained that because the value then of the from the date of receipt of payment by the Company from the various insurance
shares of stock was actually 3.6 times their par value, the donation their value, the donation, companies above-mentioned until the time the time the same amounts are paid to
although purporting to be only P400,00, would actually amount to a total of P1,440,000. He the minor heirs of Enrico Pirovano previously mentioned;
further explained that if the Pirovano children would given shares of stock in lieu of the
amount to be donated, the voting strength of the five daughters of Don Esteban in the That all amounts received from the above-mentioned policies shall be
company would be adversely affected in the sense that Mrs. Pirovano would be adversely divided equally among the minors heirs of said Enrico Pirovano;
affected in the sense that Mrs. Pirovano would have a voting power twice as much as that of
her sisters. This caused Lourdes de la Rama to write to the secretary of the corporation, Atty. That the company shall proceed to pay the proceeds of said insurance
Marcial Lichauco, asking him to cancel the waiver she supposedly gave of her pre-emptive policies plus interests that may have accrued to each of the heirs of the said Enrico
rights. Osmea elaborated on this matter at the annual meeting of the stockholders held on Pirovano or their duly appointed representatives after the Company shall have first
December 12, 1946 but at said meeting it was decided to leave the matter in abeyance settled in full the balance of its present remaining bonded indebtedness in the sum of
pending further action on the part of the members of the De la Rama family. the approximately P5,000,000.
The above resolution was carried out by the company and Mrs. Estefania R. Pirovano,
Osmea, in the meantime, took up the matter with Don Esteban and, as the latter acting as guardian of her children, by executing a Memorandum Agreement on
consequence, the latter, on December 30, 1946, addressed to Marcial Lichauco a letter January 10, 1947 and June 17, 1947, respectively, stating therein that the De la Rama Steamship
stating, among other things, that "in view of the total lack of understanding by me and my Co., Inc., shall enter in its books as a loan the proceeds of the life insurance policies taken on
daughters of the two Resolutions abovementioned, namely, Directors' and Stockholders' the life of Pirovano totalling S321,500, which loan would earn interest at the rate of 5 per cent
dated July 10, 1946, as finally resolved by the majority of the Stockholders and Directors per annum. Mrs. Pirovano, in executing the agreement, acted with the express authority
present yesterday, that you consider the abovementioned resolutions nullified." (Exhibit CC). granted to her by the court in an order dated March 26, 1947.

On January 6, 1947, the Board of Directors of the De la Rama company, as a On June 24, 1947, the Board of Directors approved a resolution providing therein that
consequence of the change of attitude of Don Esteban, adopted a resolution changing the instead of the interest on the loan being payable, together with the principal, only after the
form of the donation to the Pirovano children from a donation of 4,000 shares of stock as company shall have first settled in full its bonded indebtedness, said interest may be paid to
originally planned into a renunciation in favor of the children of all the company's "right, title, the Pirovano children "whenever the company is in a position to met said obligation" (Exhibit
and interest as beneficiary in and to the proceeds of the abovementioned life insurance D), and on February 26, 1948, Mrs. Pirovano executed a public document in which she
policies", subject to the express condition that said proceeds should be retained by the formally accepted the donation (Exhibit H). The Dela Rama company took "official notice" of
company as a loan drawing interest at the rate of 5 per cent per annum and payable to the this formal acceptance at a meeting held by its Board of Directors on February 26, 1948.
Pirovano children after the company "shall have first settled in full the balance of its present
remaining bonded indebtedness in the sum of approximately P5,000,000" (Exhibit C). This In connection with the above negotiations, the Board of Directors took up at its
resolution was concurred in by the representatives of the National Development Company. meeting on July 25, 1949, the proposition of Mrs. Pirovano to buy the house at New Rochelle,
The pertinent portion of the resolution reads as follows: New York, owned by the Demwood Realty, a subsidiary of the De la Rama company at its
original costs of $75,000, which would be paid from the funds held in trust belonging to her
Be resolved, that out of gratitude to the late Enrico Pirovano this Company minor children. After a brief discussion relative to the matter, the proposition was approved
renounce as it hereby renounces, all of his right, title, and interest as beneficiary in in a resolution adopted on the same date.
and to the proceeds of the abovementioned life insurance policies in favor of The formal transfer was made in an agreement signed on September 5, 1949 by Mrs.
Pirovano, as guardian of her children, and by the De la Rama company, represented by its new

70
MATABABE FILES
Corporation Law

General Manager, Sergio Osmea, Jr. The transfer of this property was approved by the court Be it resolved, as it is hereby resolved, that in view of the failure of
in its order of September 20, 1949. compliance with the above conditions to which the above donation was made
subject, and in view of the opinion of the Securities and Exchange Commissioner, the
On September 13, 1949, or two years and 3 months after the donation had been stockholders revoke, rescind and annul, as they do thereby revoke, rescind and
approved in the various resolutions herein above mentioned, the stockholders of the De la annul, its ratification and approval on September 13, 1949 of the aforementioned
Rama company formally ratified the donation (Exhibit E), with certain clarifying modifications, resolution of the Board of Directors of January 6, 1947, as amended on June 24, 1947.
including the resolution approving the transfer of the Demwood property to the Pirovano (Exhibit T)
children. The clarifying modifications are quoted hereunder:
In view of the resolution declaring that the corporation failed to comply with the
1. That the payment of the above-mentioned donation shall not be affected until such condition set for the effectivity of the donation and revoking at the same time the approval
time as the Company shall have first duly liquidated its present bonded indebtedness given to it by the corporation, and considering that the corporation can no longer set aside
in the amount of P3,260,855.77 with The National Development Company, or fully said donation because it had no longer set aside said donation because it had long been
redeemed the preferred shares of stock in the amount which shall be issued to the perfected and consummated, the minor children of the late Enrico Pirovano, represented by
National Development Company in lieu thereof; their mother and guardian, Estefania R. de Pirovano, demanded the payment of the credit due
them as of December 31, 1951, amounting to P564,980.89, and this payment having been
2. That any and all taxes, legal fees, and expenses in any way connected with the refused, they instituted the present action in the Court of First Instance of Rizal wherein they
above transaction shall be chargeable and deducted from the proceeds of the life prayed that the be granted an alternative relief of the following tenor: (1) sentencing
insurance policies mentioned in the resolutions of the Board of Directors. (Exhibit E) defendant to pay to the plaintiff the sum of P564,980.89 as of December 31, 1951, with the
corresponding interest thereon; (2) as an alternative relief, sentencing defendant to pay to
Sometime in March 1950, the President of the corporation, Sergio Osmea, Jr., the plaintiffs the interests on said sum of P564,980.89 at the rate of 5 per cent per annum,
addressed an inquiry to the Securities and Exchange Commission asking for opinion regarding and the sum of P564,980.89 after the redemption of the preferred shares of the corporation
the validity of the donation of the proceeds of the insurance policies to the Pirovano children. held by the National Development Company; and (3) in any event, sentencing defendant to
On June 20, 1950 that office rendered its opinion that the donation was void because the pay the plaintiffs damages in the amount of not less than 20 per cent of the sum that may be
corporation could not dispose of its assets by gift and therefore the corporation acted adjudged to the plaintiffs, and the costs of action.
beyond the scope of its corporate powers. This opinion was submitted to the Board of
Directors at its meting on July 12, 1950, on which occasion the president recommend that The only issues which in the opinion of the court need to be determined in order to
other legal ways be studied whereby the donation could be carried out. On September 14, reach a decision in this appeal are: (1) Is the grant of the proceeds of the insurance policies
1950, another meeting was held to discuss the propriety of the donation. At this meeting the taken on the life of the late Enrico Pirovano as embodied in the resolution of the Board of
president expressed the view that, since the corporation was not authorized by its charter to Directors of defendant corporation adopted on January 6, 1947 and June 24, 1947 a
make the donation to the Pirovano children and the majority of the stockholders was in favor remunerative donation as found by the lower court?; (2) IN the affirmative case, has that
of making provision for said children, the manner he believed this could be done would be to donation been perfected before its rescission or nullification by the stockholders of the
declare a cash dividend in favor of the stockholders in the exact amount of the insurance corporation on March 8, 1951?; (3) Can defendant corporation give by way of donation the
proceeds and thereafter have the stockholders make the donation to the children in their proceeds of said insurance policies to the minor children of the late Enrico Pirovano under the
individual capacity. Notwithstanding this proposal of the president, the board took no action law or its articles of corporation, or is that donation an ultra vires act?; and (4) has the
on the matter, and on March 8, 1951, at a stockholders' meeting convened on that date the defendant corporation, by the acts it performed subsequent to the granting of the donation,
majority of the stockholders' voted to revoke the resolution approving the donation to the deliberately prevented the fulfillment of the condition precedent to the payment of said
Pirovano children. donation such that it can be said it has forfeited its right to demand its fulfillment and has
made the donation entirely due and demandable?
The pertinent portion of the resolution reads as follows:
We will discuss these issues separately.

71
MATABABE FILES
Corporation Law

1. To determine the nature of the grant made by the defendant corporation to the Whereas, this company paid premiums on Mr. Pirovano's life insurance policies for a
minor children of the late Enrico Pirovano, we do not need to go far nor dig into the period of only 4 years so that it will receive from the insurance companies sums of
voluminous record that lies at the bottom of this case. We do not even need to inquire into money greatly in excess of the premiums paid by the company,
the interest which has allegedly been shown by President Roxas in the welfare of the children
of his good friend Enrico Pirovano. Whether President Roxas has taken the initiative in the Again, in the resolution approved by the Board of Directors on January 6, 1947, we
move to give something to said children which later culminated in the donation now in also find the following expressive statements which are but a reiteration of those already
dispute, is of no moment for the fact is that, from the mass of evidence on hand, such a expressed in the original resolution:
donation has been given the full indorsement and encouraging support by Don Esteban de la
Rama who was practically the owner of the corporation. We only need to fall back to Whereas, the late Enrico Pirovano, President and General Manager of the De
accomplish this purpose on the several resolutions of the Board of Directors of the la Rama Steamship Co., Inc., died in Manila sometime during the latter part of the
corporations containing said grant for they clearly state the reasons and purposes why the year 1944;
donation has been given.
Whereas, the said Enrico Pirovano was to a large extent responsible for the
Before we proceed further, it is convenient to state here in passing that, before the rapid and very successful development and expansion of the activities of this
Board of Directors had approved its resolution of January 6, 1947, as later amended by company;
another resolution adopted on June 24, 1947, the corporation had already decided to give to
the minor children of the late Enrico Pirovano the sum of P400,000 out of the proceeds of the Whereas, early in 1941, the life of the said Enrico Pirovano was insured in
insurance policies taken on his life in the form of shares, and that when this form was various life companies, to wit:
considered objectionable because its result and effect would be to give to said children a
much greater amount considering the value then of the stock of the corporation, the Board of Whereas, the said Enrico Pirovano is survived by 4 minor children, to wit:
Directors decided to amend the donation in the form and under the terms stated in the Esteban, Maria Carla, Enrico and John Albert, all surnamed Pirovano; and
aforesaid resolutions. Thus, in the original resolution approved by the Board of Directors on Whereas, the said Enrico Pirovano left practically nothing to his heirs and it is
July 10, 1946, wherein the reasons for granting the donation to the minor children of the late but fit and proper that this Company which owes so much to the deceased should
Enrico Pirovano were clearly, we find out the following revealing statements: make some provision for his children;

Whereas, the late Enrico Pirovano President and General Manager of the De Be it resolved, that out of gratitude to the late Enrico Pirovano this Company
la Rama Steamship Company, died in Manila sometime in November, 1944; renounce as it hereby renounces, . . . .

Whereas, the said Enrico Pirovano was largely responsible for the rapid and From the above it clearly appears that the corporation thought of giving the
very successful development of the activities of this company; donation to the children of the late Enrico Pirovano because he "was to a large extent
Whereas, early in 1941 this company insured the life of said Enrico Pirovano in various responsible for the rapid and very successful development and expansion of the activities of
Philippine and American Life Insurance companies for the total sum of P1,000,000; this company"; and also because he "left practically nothing to his heirs and it is but fit and
proper that this company which owes so much to the deceased should make some provision
Whereas, the said Enrico Pirovano is survived by his widow, Estefania to his children", and so, the donation was given "out of gratitude to the late Enrico Pirovano."
Pirovano and 4 minor children, to wit: Esteban, Maria Carla, Enrico and John Albert, We do not need to stretch our imagination to see that a grant or donation given under these
all surnamed Pirovano; circumstances is remunerative in nature in contemplation of law.

Whereas, the said Enrico Pirovano left practically nothing to his heirs and it is That which is made to a person in consideration of his merits or for services
but fit and proper that this company which owes so much to the deceased should rendered to the donor, provided they do not constitute recoverable debts, or that in
make some provisions for his children; which a burden less than the value of the thing given is imposed upon the donee, is
also a donation." (Art. 619, old Civil Code.)

72
MATABABE FILES
Corporation Law

Pirovano executed a public document on February 26, 1948 making similar acceptance of the
In donations made to a person for services rendered to the donor, the donation. And this acceptance was officially recorded by the corporation when on the same
donor's will is moved by acts which directly benefit him. The motivating cause is date its Board of Directors approved a resolution taking "official notice" of said acceptance.
gratitude, acknowledgment of a favor, a desire to compensate. A donation made to
one who saved the donor's life, or a lawyer who renounced his fees for services (d) On July 25, 1949, the Board of Directors approved the proposal of Mrs. Pirovano
rendered to the donor, would fall under this class of donations. These donations are to buy the house at New Rochelle, New York, owned by a subsidiary of the corporation at the
called remunerative donations . (Sinco and Capistrano, The Civil Code, Vol. 1, p. 676; costs of S75,000 which would be paid from the sum held in trust belonging to her minor
Manresa, 5th ed., pp. 72-73.) children. And this agreement was actually carried out in a document signed by the general
manager of the corporation and by Mrs. Pirovano, who acted on the matter with the express
2. The next question to be determined is whether the donation has been perfected authority of the court.
such that the corporation can no longer rescind it even if it wanted to. The answer to this
question cannot but be in the affirmative considering that the same has not only been (e) And on September 30, 1949, or two years and 3 months after the donation had
granted in several resolutions duly adopted by the Board of Directors of the defendant been executed, the stockholders of the defendant corporation formally ratified and gave
corporation, and in all these corporate acts the concurrence of the representatives of the approval to the donation as embodied in the resolutions above referred to, subject to certain
National Development Company, the only creditor whose interest may be affected by the modifications which did not materially affect the nature of the donation.
donation, has been expressly given. The corporation has even gone further. It actually
transferred the ownership of the credit subject of donation to the Pirovano children with the There can be no doubt from the foregoing relation of facts the donation was a
express understanding that the money would be retained by the corporation subject to the corporate act carried out by the corporation not only with the sanction of its Board of
condition that the latter would pay interest thereon at the rate of 5 per cent per annum Directors but also of its stockholders. It is evident that the donation has reached the stage of
payable whenever said corporation may be in a financial position to do so. Thus, the following perfection which is valid and binding upon the corporation and as such cannot be rescinded
acts of the corporation as reflected from the evidence bear this out: unless there is exists legal grounds for doing so. In this case, we see none. The two reasons
given for the rescission of said donation in the resolution of the corporation adopted on
(a) The donation was embodied in a resolution duly approved by the Board of March 8, 1951, to wit: that the corporation failed to comply with the conditions to which the
Directors on January 6, 19437. In this resolution, the representatives of the National above donation was made subject, and that in the opinion of the Securities and Exchange
Development Company, have given their concurrence. This is the only creditor which can be Commission said donation is ultra vires, are not, in our opinion, valid and legal as to justify the
considered as being adversely affected by the donation. The resolution of June 24, 1947 did rescission of a perfected donation. These reasons, as we will discuss in the latter part of this
not modify the substance of the former resolution for it merely provided that instead of the decision, cannot be invoked by the corporation to rescind or set at naught the donation, and
interest on the loan being payable, together with the principal, only after the corporation had the only way by which this can be done is to show that the donee has been in default, or that
first settled in full its bonded indebtedness, said interest would be paid "whenever the the donation has not been validly executed, or is illegal or ultra vires, and such is not the case
company is in a position to meet said obligation." as we will see hereafter. We therefore declare that the resolution approved by the
stockholders of the defendant corporation on March 8, 1951 did not and cannot have the
(b) The resolution of January 6, 1947 was actually carried out when the company and effect of nullifying the donation in question.
Mrs. Estefania R. Pirovano, executed a memorandum agreement stating therein hat the
proceeds of the insurance policies would be entered in the books of the corporation as a loan 3. The third question to be determined is: Can defendant corporation give by way of
which would bear an interest at the rate of 5 per cent per annum, and said agreement was donation the proceeds of said insurance policies to the minor children of the late Enrico
signed by Mrs. Pirovano as judicial guardian of her children after she had been expressly Pirovano under the law or its articles of corporation, or is that donation an ultra vires act? To
authorized by the court to accept the donation in behalf of her children. answer this question it is important for us to examine the articles of incorporation of the De la
Rama company to see this question it is important for us to examine the articles of
(c) While the donation can be considered as duly executed by the execution of the incorporation of the De la Rama company to see if the act or donation is outside of their
document stated in the preceding paragraph, and by the entry in the books of the scope. Paragraph second of said articles provides:
corporation of the donation as a loan, a further record of said execution was made when Mrs.

73
MATABABE FILES
Corporation Law

Second. The purposes for which said corporation is formed are: which this corporation has a lawful interest, and to do such acts and things as may be
necessary to protect, preserve, improve, or enhance the value of any such obligation
(a) To purchase, charter, hire, build, or otherwise acquire steam or other ships or or interest; and, in general, to do such other acts in connection with the purposes for
vessels, together with equipments and furniture therefor, and to employ the same in which this corporation has been formed which is calculated to promote the interest
conveyance and carriage of goods, wares and merchandise of every description, and of the corporation or to enhance the value of its property and to exercise all the
of passengers upon the high seas. rights, powers and privileges which are now or may hereafter be conferred by the
(b) To sell, let, charter, or otherwise dispose of the said vessels or other property of laws of the Philippines upon corporations formed under the Philippine Corporation
the company. Act; to execute from time to time general or special powers of attorney to persons,
firms, associations or corporations either in the Philippines, in the United States, or in
(c) To carry on the business of carriers by water. any other country and to revoke the same as and when the Directors may determine
and to do any and or all of the things hereinafter set forth and to the same extent as
(d) To carry on the business of shipowners in all of its branches. natural persons might or could do.

(e) To purchase or take on lease, lands, wharves, stores, lighters, barges and other After a careful perusal of the provisions above quoted we find that the corporation
things which the company may deem necessary or advisable to be purchased or was given broad and almost unlimited powers to carry out the purposes for which it was
leased for the necessary and proper purposes of the business of the company, and organized among them, (1) "To invest and deal with the moneys of the company not
from time to time to sell the dispose of the same. immediately required, in such manner as from time to time may be determined" and, (2) "to
aid in any other manner any person, association, or corporation of which any obligation or in
(f) To promote any company or companies for the purposes of acquiring all or any of which any interest is held by this corporation or in the affairs or prosperity of which this
the property or liabilities of this company, or both, or for any other purpose which corporation has a lawful interest." The world deal is broad enough to include any manner of
may seem directly or indirectly calculated to benefit the company. disposition, and refers to moneys not immediately required by the corporation, and such
disposition may be made in such manner as from time to time may be determined by the
(g) To invest and deal with the moneys of the company and immediately required, in corporations. The donation in question undoubtedly comes within the scope of this broad
such manner as from time to time may be determined. power for it is a fact appearing in the evidence that the insurance proceeds were not
immediately required when they were given away. In fact, the evidence shows that the
(h) To borrow, or raise, or secure the payment of money in such manner as the corporation declared a 100 per cent cash dividend, or P2,000,000, and later on another 30 per
company shall think fit. cent cash dividend. This is clear proof of the solvency of the corporation. It may be that, as
insinuated, Don Esteban wanted to make use of the insurance money to rehabilitate the
(i) Generally, to do all such other thing and to transact all business as may be directly central owned by a sister corporation, known as Hijos de I. de la Rama and Co., Inc., situated
or indirectly incidental or conducive to the attainment of the above object, or any of in Bago, Negros Occidental, but this, far from reflecting against the solvency of the De la
them respectively. Rama company, only shows that the funds were not needed by the corporation.
Under the second broad power we have the above stated, that is, to aid in any other
(j) Without in any particular limiting or restricting any of the objects and powers of manner any person in the affairs and prosperity of whom the corporation has a lawful interest,
the corporation, it is hereby expressly declared and provided that the corporation the record of this case is replete with instances which clearly show that the corporation knew
shall have power to issue bonds and provided that the corporation shall have power well its scope and meaning so much so that, with the exception of the instant case, no one
to issue bonds and other obligations, to mortgage or pledge any stocks, bonds or has lifted a finger to dispute their validity. Thus, under this broad grant of power, this
other obligations or any property which may be required by said corporations; to corporation paid to the heirs of one Florentino Nonato, an engineer of one of the ships of the
secure any bonds, guarantees or other obligations by it issued or incurred; to lend company who died in Japan, a gratuity of P7,000, equivalent to one month salary for each
money or credit to and to aid in any other manner any person, association, or year of service. It also gave to Ramon Pons, a captain of one of its ships , a retirement gratuity
corporation of which any obligation or in which any interest is held by this corporation equivalent to one month salary for every year of service, the same to be based upon his
or in the affairs or prosperity of which this corporation or in the affairs or prosperity of highest salary. And it contributed P2,000 to the fund raised by the Associated Steamship Lines

74
MATABABE FILES
Corporation Law

for the widow of the late Francis Gispert, secretary of said Association, of which the De la Code. In essence they are the same. Such being the case, it may be said that this donation is
Rama Steamship Co., Inc., was a member along with about 30 other steamship companies. In gratuity in a large sense for it was given for valuable services rendered an ultra vires act in the
this instance, Gispert was not even an employee of the corporation. And invoking this vast light of the following authorities:
power, the corporation even went to the extent of contributing P100,000 to the Liberal Party
campaign funds, apparently in the hope that by conserving its cordial relations with that party Indeed, some cases seem to hold that the giving of a pure gratuity to
it might continue to retain the patronage of the administration. All these acts executed directors is ultra vires of corporation, so that it could not be legalized even if the
before and after the donation in question have never been questioned and were willingly and approval of the shareholders; but this position has no sound reason to support it,
actually carried out. and is opposed to the weight of authority (Suffaker vs. Kierger's Assignee, 53 S.W.
Rep. 288; !07 Ky. 200; 46 L.R.A. 384).
We don't see much distinction between these acts of generosity or benevolence
extended to some employees of the corporation, and even to some in whom the corporation But although business corporations cannot contribute to charity or
was merely interested because of certain moral or political considerations, and the donation benevolence, yet they are not required always to insist on the full extent of their
which the corporation has seen fit to give to the children of the late Enrico Pirovano from the legal rights. They are not forbidden for the recognizing moral obligation of which
point of view of the power of the corporation as expressed in its articles of incorporation. strict law takes no cognizance. They are not prohibited from establishing a reputation
And if the former had been sanctioned and had been considered valid and intra vires, we see for board, liberal, equitable dealing which may stand them in good stead in
no plausible reasons why the latter should now be deemed ultra vires. It may perhaps be competition with less fair rivals. Thus, an incorporated fire insurance company which
argued that the donation given to the children of the late Enrico Pirovano is so large and policies except losses from explosions may nevertheless pay a loss from that cause
disproportionate that it can hardly be considered a pension of gratuity that can be placed on a when other companies are accustomed to do so, such liberal dealing being deemed
par with the instances above mentioned, but this argument overlooks one consideration: the conducive to the prosperity of the corporation." (Modern Law of Corporations,
gratuity here given was not merely motivated by pure liberality or act of generosity, but by a Machen, Vol. 1, p. 81).
deep sense of recognition of the valuable services rendered by the late Enrico Pirovano which
had immensely contributed to the growth of the corporation to the extent that from its So, a bank may grant a five years pension to the family at one of its officers.
humble capitalization it blossomed into a multi-million corporation that it is today. In other In all cases in this sorts, the amount of the gratuity rests entirely within the discretion
words of the very resolutions granting the donation or gratuity, said donation was given not of the company, unless indeed it be all together out of the reason and fitness. But
only because the company was so indebted to him that it saw fit and proper to make where the company has ceased to be going concerned, this power to make gifts or
provisions for his children, but it did so out of a sense of gratitude. Another factor that we present it at the end. (Modern Law of Corporations, Machen, Vol. 1, p. 82.).
should bear in mind is that Enrico Pirovano was not only a high official of the company but
was at the same time a member of the De la Rama family, and the recipient of the donation Payment of Gratitude out of Capital. There seems on principle no reason to doubt
are the grandchildren of Don Esteban de la Rama. This we, may say, is the motivating root that gifts or gratuities wherever they are lawful may be paid out of capital as well as
cause behind the grant of this bounty. out of profits. (Modern Law of corporations, Machen, Vol. 1 p. 83.).
It may be contended that a donation is different from a gratuity. While technically this may be
so in substance they are the same. They are even similar to a pension. Thus, it was granted for Whether desirable to supplement implied powers of this kind by express provisions.
services previously rendered, and which at the time they were rendered gave rise to no legal Enough has been said to show that the implied powers of a corporation to give
obligation. " (Words and Phrases, Permanent Edition, p. 675; O'Dea vs. Cook,, 169 Pac., 306, gratuities to its servants and officers, as well as to strangers, are ample, so that there
176 Cal., 659.) Or stated in another way, a "Gratuity is mere bounty given by the Government is therefore no need to supplement them by express provisions." (modern Law of
in consideration or recognition or meritorious services and springs from the appreciation an d Corporations, Machen, Vol. 1, p. 83.) 1
graciousness of the Government", (Ilagan vs. Ilaya, G.R. No. 33507, Dec. 20 1930) or "A
gratuity is something given freely, or without recompense, a gift, something voluntarily given Granting arguendo that the donation given by Pirovano children is outside the scope
in return for a favor or services; a bounty; a tip." Wood Mercantile Co. vs. Cole, 209 S.W. 2d. of the powers of the defendant corporation, or the scope of the powers that it may exercise
290; Mendoza vs. Dizon, 77 Phil., 533, 43 Off. Gaz. p. 4633. We do not see much difference under the law, or it is an ultra vires act, still it may said that the same can not be invalidated, or
between this definition of gratuity and a remunerative donation contemplated in the Civil declared legally ineffective for the reason alone, it appearing that the donation represents not

75
MATABABE FILES
Corporation Law

only the act of the Board of Directors but of the stockholders themselves as shown by the of the corporation or the other party to the transaction to raise the objection,
fact that the same has been expressly ratified in a resolution duly approved by the latter. By particularly where the benefits are retained
this ratification, the infirmity of the corporate act, it may has been obliterated thereby making
the cat perfectly valid and enforceable. This is specially so if the donation is not merely As appears in paragraphs 960-964 supra, the general rule is that a
executory but executed and consummated and no creditors are prejudice, or if there are corporation must act in the manner and with the formalities, if any, prescribed by its
creditors affected, the latter has expressly given their confirmity. character or by the general law. However, a corporation transaction or contract
which is within the corporation powers, which is neither wrong in itself nor against
In making this pronouncement, advertence should made of the nature of the ultra public policy, but which is defective from a failure to observe in its execution a
vires act that is in question. A little digression needs be made on this matter to show the requirement of law enacted for the benefit or protection of a certain class, is
different legal effect that may result consequent upon the performance of a particular ultra voidable and is valid until avoided, not void until validated; the parties for whose
vires act on the part of the corporation. may authorities may be cited interpreting or defining, benefit the requirement was enacted may ratify it or be estoppel to assert its
extent, and scope of an ultra vires act, but all of them are uniform and unanimous that the invalidity, and third persons acting in good faith are not usually affected by an
same may be either an act performed merely outside the scope of the powers granted to it by irregularity on the part of the corporation in the exercise of its granted powers. (19
it articles of incorporation, or one which is contrary to law or violative of any principle which C.J.S., 423-24.)
will void any contract whether done individually or collectively. In other words, a distinction
should be made between corporate acts or contracts which are illegal and those which are It is true that there are authorities which told that ultra vires acts, or those performed
merely ultra vires. The former contemplates the doing of an act which is contrary to law, beyond the powers conferred upon the corporation either by law or by its articles of
morals, or public policy or public duty, and are, like similar transactions between the incorporation, are not only voidable, but wholly void and of no legal effect, and that such acts
individuals void. They cannot serve as basis of a court action, nor require validity ultra cannot be validated by ratification or be the basis of any action in court; but such ruling does
vires acts on the other hand, or those which are not illegal and void ab initio, but are merely not constitute the weight of authority, the reason being that they fail to make the important
within are not illegal and void ab initio, but are not merely within the scope of the articles of distinction we have above adverted to. Because rule has been rejected by most of the state
incorporation, are merely voidable and may become binding and enforceable when ratified by courts and even by the modern treaties or corporations (7 Flethcer, Cyc. Corps., 563-564). And
the stockholders. now it can be said that the majority of the cases hold that acts which are merely ultra vires, or
acts which are not illegal, may be ratified by the stockholders of a corporation (Brooklyn
Strictly speaking, an ultra vires act is one outside the scope of the power Heights R. Co. vs. Brooklyn City R. Co., 135 N.Y. Supp. 1001).
conferred by the legislature, and although the term has been used indiscriminately, it
is properly distinguishable from acts which are illegal, in excess or abuse of power, or Strictly speaking, an act of a corporation outside of its character powers is
executed in an unauthorized manner, or acts within corporate powers but outside just as such ultra vires where all the stockholders consent thereto as in a case where
the authority of particular officers or agents (19 C. J. S. 419). none of the stockholders expressly or cannot be ratified so as to make it valid, even
though all the stockholders consent thereto; but inasmuch as the stockholders in
Corporate transactions which are illegal because prohibited by statute or reality constitute the corporation, it should , it would seem, be estopped to
against public policy are ordinarily void and unenforceable regardless of the part allege ultra vires, and it is generally so held where there are no creditors, or the
performance, ratification, or estoppel; but general prohibitions against exceeding creditors are not injured thereby, and where the rights of the state or the public are not
corporate powers and prohibitions intended to protect a particular class or specifying involved, unless the act is not only ultra vires but in addition illegal and void. of course,
the consequences of violation may not preclude enforcement of the transaction and such consent of all the stockholders cannot adversely affect creditors of the
an action may be had for the part unaffected by the illegality or for equitable corporation nor preclude a proper attack by the state because of such ultra vires act.
restitution. (19 C.J.S. 421.) (7 Fletcher Corp., Sec. 3432, p. 585)
Generally, a transaction within corporate powers but executed in an irregular or
unauthorized manner is voidable only, and may become enforceable by reason of Since it is not contended that the donation under consideration is illegal, or contrary
ratification or express or implied assent by the stockholders or by reason of estoppel to any of the express provision of the articles of incorporation, nor prejudicial to the creditors
of the defendant corporation, we cannot but logically conclude, on the strength of the

76
MATABABE FILES
Corporation Law

authorities we have quoted above, that said donation, even if ultravires in the supposition we due to a deliberate desistance of the corporation and not to lack of funds to redeem the
have adverted to, is not void, and if voidable its infirmity has been cured by ratification and preferred shares of the National Development Company. The conclusions arrived at by the
subsequent acts of the defendant corporation. The defendant corporation, therefore, is now trial court on this point are as follows:
prevented or estopped from contesting the validity of the donation. This is specially so in this
case when the very directors who conceived the idea of granting said donation are practically Fourth. that the condition mentioned in the donation is null and void because it
the stockholders themselves, with few nominal exception. This applies to the new depends on the exclusive will of the donor, in accordance with the provisions of
stockholder Jose Cojuangco who acquired his interest after the donation has been made Article 1115 of the Old Civil Code.
because of the rule that a "purchaser of shares of stock cannot avoid ultra vires acts of the
corporation authorized by its vendor, except those done after the purchase" (7 Fletcher, Cyc. Fifth. That if the condition is valid, its non-fulfillment is due to the desistance of the
Corps. section 3456, p. 603; Pascual vs. Del Saz Orozco, 19 Phil., 82.) Indeed, how can the defendant company from obeying and doing the wishes and mandate of the majority
stockholders now pretend to revoke the donation which has been partly consummated? How of the stockholders.
can the corporation now set at naught the transfer made to Mrs. Pirovano of the property in
New York, U.S.A., the price of which was paid by her but of the proceeds of the insurance Sixth. That the non-payment of the debt in favor of the National Development
policies given as donation. To allow the corporation to undo what it has done would only be Company is due to the lack of funds, nor to lack of authority, but to the desire of the
most unfair but would contravene the well-settled doctrine that the defense of ultra President of the corporation to preserve and continue the Government participation
vires cannot be set up or availed of in completed transactions (7 Fletcher, Cyc. Corps. Section in the company.
3497, p. 652; 19 C.J.S., 431).
To this views of the trial court, we fail to agree. There are many factors we can
4. We now come to the fourth and last question that the defendant corporation, by consider why the failure to immediately redeem the preferred shares issued to the National
the acts it has performed subsequent to the granting of the donation, deliberately prevented Development Company as desired by the minor children of the late Enrico Pirovano cannot or
the fulfillment of the condition precedent to the payment of said donation such that it can be should not be attributed to a mere desire on the part of the corporation to delay the
said it has forfeited entirely due and demandable. redemption, or to prejudice the interest of the minors, but rather to protect the interest of
the corporation itself. One of them is the text of the very resolution approved by the National
It should be recalled that the original resolution of the Board of Directors adopted on Development Company on February 18, 1949 which prescribed the terms and conditions
July 10, 1946 which provided for the donation of P400,000 out of the proceeds which the De under which it expressed its conformity to the conversion of the bonded indebtedness into
la Rama company would collect on the insurance policies taken on the life of the late Enrico preferred shares of stock. The text of the resolution above mentioned reads:
Pirovano was, as already stated above, amended on January 6, 1947 to include, among the
conditions therein provided, that the corporation shall proceed to pay said amount, as well as Resolved: That the outstanding bonded indebtedness of the Dela Rama Steamship
the interest due thereon, after it shall have settled in full balance of its bonded indebtedness Co., Inc., in the approximate amount of P3,260,855.77 be converted into non-voting
in the sum of P5,000,000. It should be recalled that on September 13, 1949, or more than 2 preferred shares of stock of said company, said shares to bear a fixed dividend of 6
years after the last amendment referred too above, the stockholders adopted another percent per annum which shall be cumulative and redeemable within 15 years. Said
resolution whereby they formally ratified said donation but subject to the following shares shall be preferred as to assets in the event of liquidation or dissolution of said
clarifications: (1) that the amount of the donation shall not be effected until such time as the company but shall be non-participating.
company shall have first duly liquidated its present bonded indebtedness in the amount of
P3,260,855.77 to the National Development Company, or shall have first fully redeemed the It is plain from the text of the above resolution that the defendant corporation had 15
preferred shares of stock in the amount to be issued to said company in lieu thereof, and (2) years from February 18, 1949, or until 1964, within which to effect the redemption of the
that any and all taxes, legal fees, and expenses connected with the transaction shall be preferred shares issued to the National Development Company. This condition cannot but be
chargeable from the proceeds of said insurance policies. binding and obligatory upon the donees, if they desire to maintain the validity of the
The trial court, in considering these conditions in the light of the acts subsequently performed donation, for it is not only the basis upon which the stockholders of the defendant
by the corporation in connection with the proceeds of the insurance policies, considered said corporation expressed their willingness to ratify the donation, but it is also by way which its
conditions null and void, or at most not written because in its pinion their non-fulfillment was creditor, the National Development Company, would want it to be. If the defendant

77
MATABABE FILES
Corporation Law

corporation is given 15 years within which to redeem the preferred shares, and that period entered in the books of the corporation as loans and, therefore, they were invested for their
would expire in 1964, one cannot blame the corporation for availing itself of this period if in its own benefit. As General Manager Osmea said, "we were first confronted with the problem
opinion it would redound to its best interest. It cannot therefore be said that the fulfillment of of the withdrawals of the family which had to be repaid back to the National Development
the condition for the payment of the donation is one that wholly depends on the exclusive Company and one of the most practical solutions to that was to declare dividends and reduce
will of the donor, as the lower court has concluded, simply because it failed to meet the the amounts of their withdrawals", which then totalled about P3,000,000.
redemption of said shares in her manner desired by the donees. While it may be admitted that
because of the disposition of the assets of the corporation upon the suggestion of its general All things considered, we are of the opinion that the finding of the lower court that
manager more than enough funds had been raised to effect the immediate redemption of the the failure of the defendant corporation to comply with the condition of the donation is
above shares, it is not correct to say that the management has completely failed in its duty to merely due to its desistance from obeying the mandate of the majority of the stockholders
pay its obligations for, according to the evidence, a substantial portion of the indebtedness and not to lack of funds, or to lack of authority, has no foundation in law or in fact, and,
has been paid and only a balance of about P1,805,169.98 was outstanding when the therefore, its conclusion that because of such desistance that condition should be deemed as
stockholders of the corporation decided to revoke or cancel the donation. (Exhibit P.) fulfilled and the payment of the donation due and demandable, is not justified. In this respect,
the decision of the lower court should be reversed.
But there are other good reasons why all the available funds have not been actually
applied to the redemption of the preferred shares, one of them being the "desire of the Having reached the foregoing conclusion, we deem it unnecessary to discuss the
president of the corporation to preserve and continue the government participation in the other issues raised by the parties in their briefs.
company" which even the lower court found it to be meritorious, which is one way by which it
could continue receiving the patronage and protection of the government. Another reason is The lower court adjudicated to plaintiff an additional amount equivalent to 20 per
that the redemption of the shares does not depend on the will of the corporation alone but to cent of the amount claimed as damages by way of attorney's fees, and in our opinion, this
a great extent on the will of a third party, the National Development Company. In fact, as the award can be justified under Article 2208, paragraph 2, of the new Civil Code, which provides:
evidence shows, this Company had pledged these shares to the Philippine National Bank and "When the defendant's act or omission has compelled the plaintiff to litigate with third
the Rehabilitation Finance Corporation as a security to obtain certain loans to finance the persons or to incur expenses to protect his interest", attorney's fees nay be awarded as
purchase of certain ships to be built for the use of the company under management contract damages. However, the majority believes that this award should be reduced to 10 per cent.
entered into between the corporation and the National Development Company, and this was
what prevented the corporation from carrying out its offer to pay the sum P1,956,513.07 on Wherefore, the decision appealed from should be modified as follows: (a) that the
April 5, 1951. Had this offer been accepted, or favorably acted upon by the National donation made in favor of the children of the late Enrico Pirovano of the proceeds of the
Development Company, the indebtedness would have been practically liquidated, leaving insurance policies taken on his life is valid and binding on the defendant corporation, (b) that
outstanding only one certificate worth P217,390.45. Of course, the corporation could have said donation, which amounts to a total of P583,813.59, including interest, as it appears in the
insisted in redeeming the shares if it wanted to even to the extent of taking a court action if books of the corporation as of August 31, 1951, plus interest thereon at the rate of 5 per cent
necessary to force its creditor to relinquish the shares that may be necessary to accomplish per annum from the filing of the complaint, should be paid to the plaintiffs after the
the redemption, but such would be a drastic step which would have not been advisable defendant corporation shall have fully redeemed the preferred shares issued to the National
considering the policy right along maintained by the corporation to preserve its cordial and Development Company under the terms and conditions stated in the resolutions of the Board
smooth relation with the government. At any rate, whether such attitude be considered as a of Directors of January 6, 1947 and June 24, 1947, as amended by the resolution of the
mere excuse to justify the delay in effecting the redemption of the shares, or a mere desire on stockholders adopted on September 13,1949; and (c) defendant shall pay to plaintiffs an
the part of the corporation to retain in its possession more funds available to attend to other additional amount equivalent to 10 per cent of said amount of P583,813.59 as damages by way
pressing need as demanded by the interest of the corporation, we fail to see in such an of attorney's fees, and to pay the costs of action.
attitude an improper motive to circumvent the early realization of the desire of the minors to
obtain the immediate payment of the donation which was made dependent upon the Paras, C. J., Pablo Bengzon, Padilla, Montemayor, Jugo, Concepcion, and Reyes, J. B. L., concur.
redemption of said shares there being no clear evidence that may justify such design. Anyway, Reyes, A., concurs in the result.
a great portion of the funds went to the stockholders themselves by way of dividends to
offset, so it appears, the huge advances that the corporation had made to them which were

78
MATABABE FILES
Corporation Law

Republic of the Philippines Co., which have been issued to the Benguet Consolidated Mining Co., and to secure to the
SUPREME COURT Balatoc Mining Co., the restoration of a large sum of money alleged to have been unlawfully
Manila collected by the Benguet Consolidated Mining Co., with legal interest, after deduction
EN BANC therefrom of the amount expended by the latter company under a contract between the two
companies, bearing date of March 9, 1927. The complaint was afterwards amended so as to
G.R. No. L-37331 March 18, 1933 include a prayer for the annulment of this contract. Shortly prior to the institution of this
lawsuit, the Benguet Consolidated Mining Co., transferred to H. E. Renz, as trustee, the
FRED M. HARDEN, J.D. HIGHSMITH, and JOHN C. HART, in their own behalf and in that all certificate for 600,000 shares of the Balatoc Mining Co. which constitute the principal subject
other stockholders of the Balatoc Mining Company, etc., plaintiffs-appellants, matter of the action. This was done apparently to facilitate the splitting up to the shares in
vs. the course of the sale or distribution. To prevent this the plaintiffs, upon filing their original
BENGUET CONSOLIDATED MINING COMPANY, BALATOC MINING COMPANY, H. E. RENZ, complaint, procured a preliminary injunction restraining the defendants, their agents and
JOHN W. JAUSSERMANN, and A. W. BEAM, defendants-appellees. servants, from selling, assigning or transferring the 600,000 shares of the Balatoc Mining Co.,
or any part thereof, and from removing said shares from the Philippine Islands. This explains
CORPORATIONS; MlNING CORPORATION; PROHIBITION AGAINST OWNING INTEREST the connection of Renz with the case. The other individual defendants are made merely as
IN OTHER MINING CORPORATION; RIGHT OF ACTION.Inasmuch as the Corporation Law officials of the Benguet Consolidated Mining Co. Upon hearing the cause the trial court
contains, in section 190 (A), provisions fully penalizing the violation of subsection 5 of section dismissed the complaint and dissolved the preliminary injunction, with costs against the
13 of Act No. 1459,which prohibits the acquisition by one mining corporation of any interest plaintiffs. From this judgment the plaintiffs appealed.
in another,and inasmuch as these provisions have been enacted in the exercise of the
general police powers of the Government, it results that, where one mining corporation The facts which have given rise this lawsuit are simple, as the financial interests
acquires a prohibited interest in another such corporation, the shareholders of the latter involve are immense. Briefly told these facts are as follows: The Benguet Consolidated Mining
cannot maintain an action to annul the contract by which such interest was acquired. The Co. was organized in June, 1903, as a sociedad anonima in conformity with the provisions of
remedy must be sought in a criminal proceeding or quo warranto action, under section 190 Spanish law; while the Balatoc Mining Co. was organized in December 1925, as a corporation,
(A), instituted by the Government. Until thus assailed in a direct proceeding the contract by in conformity with the provisions of the Corporation Law (Act No. 1459). Both entities were
which the interest was acquired will be treated as valid, as between the parties. organized for the purpose of engaging in the mining of gold in the Philippine Islands, and their
respective properties are located only a few miles apart in the subprovince of Benguet. The
APPEAL from a judgment of the Court of First Instance of Manila. Goddard, J. capital stock of the Balatoc Mining Co. consists of one million shares of the par value of one
The facts are stated in the opinion of the Court peso (P1) each.

Gibbs and McDonough and Roman Ozaeta for appellants. When the Balatoc Mining Co. was first organized the properties acquired by it were
DeWitt, Perkins and Brady for appellees. largely undeveloped; and the original stockholders were unable to supply the means needed
Ross, Lawrence and Selph for appellee Balatoc Mining Company. for profitable operation. For this reason, the board of directors of the corporation ordered a
suspension of all work, effective July 31, 1926. In November of the same year a general
STREET, J.: meeting of the company's stockholders appointed a committee for the purpose of interesting
outside capital in the mine. Under the authority of this resolution the committee approached
This action was originally instituted in the Court of First Instance of the City of Manila A. W. Beam, then president and general manager of the Benguet Company, to secure the
by F. M. Harden, acting in his own behalf and that of all other stockholders of the Balatoc capital necessary to the development of the Balatoc property. As a result of the negotiations
Mining Co. who might join in the action and contribute to the expense of the suit. With the thus begun, a contract, formally authorized by the management of both companies, was
plaintiff Harden two others, J. D. Highsmith and John C. Hart, subsequently associated executed on March 9, 1927, the principal features of which were that the Benguet Company
themselves. The defendants are the Benguet Consolidated Mining Co., the Balatoc Mining Co., was to proceed with the development and construct a milling plant for the Balatoc mine, of a
H. E. Renz, John W. Haussermann, and A. W. Beam. The principal purpose of the original capacity of 100 tons of ore per day, and with an extraction of at least 85 per cent of the gold
action was to annul a certificate covering 600,000 shares of the stock of the Balatoc Mining content. The Benguet Company also agreed to erect an appropriate power plant, with the

79
MATABABE FILES
Corporation Law

aerial tramlines and such other surface buildings as might be needed to operate the mine. In creation of corporations in the Philippine Islands. This rather elaborate piece of legislation is
return for this it was agreed that the Benguet Company should receive from the treasurer of embodied in what is called our Corporation Law (Act No. 1459 of the Philippine Commission).
the Balatoc Company shares of a par value of P600,000, in payment for the first P600,000 be The evident purpose of the commission was to introduce the American corporation into the
thus advanced to it by the Benguet Company. Philippine Islands as the standard commercial entity and to hasten the day when the sociedad
anonima of the Spanish law would be obsolete. That statute is a sort of codification of
The performance of this contract was speedily begun, and by May 31, 1929, the American corporate law.
Benguet Company had spent upon the development the sum of P1,417,952.15. In
compensation for this work a certificate for six hundred thousand shares of the stock of the For the purposes general description only, it may be stated that the sociedad
Balatoc Company has been delivered to the Benguet Company, and the excess value of the anonima is something very much like the English joint stock company, with features
work in the amount of P817,952.15 has been returned to the Benguet Company in cash. resembling those of both the partnership is shown in the fact that sociedad, the generic
Meanwhile dividends of the Balatoc Company have been enriching its stockholders, and at the component of its name in Spanish, is the same word that is used in that language to designate
time of the filing of the complaint the value of its shares had increased in the market from a other forms of partnership, and in its organization it is constructed along the same general
nominal valuation to more than eleven pesos per share. While the Benguet Company was lines as the ordinary partnership. It is therefore not surprising that for purposes of loose
pouring its million and a half into the Balatoc property, the arrangements made between the translation the expression sociedad anonima has not infrequently the other hand, the affinity
two companies appear to have been viewed by the plaintiff Harden with complacency, he of this entity to the American corporation has not escaped notice, and the
being the owner of many thousands of the shares of the Balatoc Company. But as soon as the expression sociedad anonima is now generally translated by the word corporation. But when
success of the development had become apparent, he began this litigation in which he has the word corporation is used in the sense of sociedad anonima and close discrimination is
been joined by two others of the eighty shareholders of the Balatoc Company. necessary, it should be associated with the Spanish expression sociedad anonima either in a
parenthesis or connected by the word "or". This latter device was adopted in sections 75 and
Briefly, the legal point upon which the action is planted is that it is unlawful for the 191 of the Corporation Law.
Benguet Company to hold any interest in a mining corporation and that the contract by which
the interest here in question was acquired must be annulled, with the consequent obliteration In drafting the Corporation Law the Philippine Commission inserted bodily, in
of the certificate issued to the Benguet Company and the corresponding enrichment of the subsection (5) of section 13 of that Act (No. 1459) the words which we have already quoted
shareholders of the Balatoc Company. from section 75 of the Act of Congress of July 1, 1902 (Philippine Bill); and it is of course
obvious that whatever meaning originally attached to this provision in the Act of Congress,
When the Philippine Islands passed to the sovereignty of the United States, in the the same significance should be attached to it in section 13 of our Corporation Law.
attention of the Philippine Commission was early drawn to the fact that there is no entity in
Spanish law exactly corresponding to the notion of the corporation in English and American As it was the intention of our lawmakers to stimulate the introduction of the
law; and in the Philippine Bill, approved July 1, 1902, the Congress of the United States American Corporation into Philippine law in the place of the sociedad anonima, it was
inserted certain provisions, under the head of Franchises, which were intended to control the necessary to make certain adjustments resulting from the continued co-existence, for a time,
lawmaking power in the Philippine Islands in the matter of granting of franchises, privileges of the two forms of commercial entities. Accordingly, in section 75 of the Corporation Law, a
and concessions. These provisions are found in section 74 and 75 of the Act. The provisions of provision is found making the sociedad anonima subject to the provisions of the Corporation
section 74 have been superseded by section 28 of the Act of Congress of August 29, 1916, but Law "so far as such provisions may be applicable", and giving to the sociedades
in section 75 there is a provision referring to mining corporations, which still remains the law, anonimas previously created in the Islands the option to continue business as such or to
as amended. This provisions, in its original form, reads as follows: "... it shall be unlawful for reform and organize under the provisions of the Corporation Law. Again, in section 191 of the
any member of a corporation engaged in agriculture or mining and for any corporation Corporation Law, the Code of Commerce is repealed in so far as it relates to sociedades
organized for any purpose except irrigation to be in any wise interested in any other anonimas. The purpose of the commission in repealing this part of the Code of Commerce was
corporation engaged in agriculture or in mining." to compel commercial entities thereafter organized to incorporate under the Corporation
Law, unless they should prefer to adopt some form or other of the partnership. To this
Under the guidance of this and certain other provisions thus enacted by Congress, provision was added another to the effect that existing sociedades anonimas, which elected to
the Philippine Commission entered upon the enactment of a general law authorizing the continue their business as such, instead of reforming and reorganizing under the Corporation

80
MATABABE FILES
Corporation Law

Law, should continue to be governed by the laws that were in force prior to the passage of instituted by the Attorney-General or by any provincial fiscal by order of said
this Act "in relation to their organization and method of transacting business and to the rights Attorney-General: . . . .
of members thereof as between themselves, but their relations to the public and public
officials shall be governed by the provisions of this Act." Upon a survey of the facts sketched above it is obvious that there are two
fundamental questions involved in this controversy. The first is whether the plaintiffs can
As already observed, the provision above quoted from section 75 of the Act Congress maintain an action based upon the violation of law supposedly committed by the Benguet
of July 1, 1902 (Philippine Bill), generally prohibiting corporations engaged in mining and Company in this case. The second is whether, assuming the first question to be answered in
members of such from being interested in any other corporation engaged in mining, was the affirmative, the Benguet Company, which was organized as a sociedad anonima, is a
amended by section 7 of Act No. 3518 of the Philippine Legislature, approved by Congress corporation within the meaning of the language used by the Congress of the United States,
March 1, 1929. The change in the law effected by this amendment was in the direction of and later by the Philippine Legislature, prohibiting a mining corporation from becoming
liberalization. Thus, the inhibition contained in the original provision against members of a interested in another mining corporation. It is obvious that, if the first question be answered
corporation engaged in agriculture or mining from being interested in other corporations in the negative, it will be unnecessary to consider the second question in this lawsuit.
engaged in agriculture or in mining was so modified as merely to prohibit any such member
from holding more than fifteen per centum of the outstanding capital stock of another such Upon the first point it is at once obvious that the provision referred to was adopted
corporation. Moreover, the explicit prohibition against the holding by any corporation (except by the lawmakers with a sole view to the public policy that should control in the granting of
for irrigation) of an interest in any other corporation engaged in agriculture or in mining was mining rights. Furthermore, the penalties imposed in what is now section 190 (A) of the
so modified as to limit the restriction to corporations organized for the purpose of engaging Corporation Law for the violation of the prohibition in question are of such nature that they
in agriculture or in mining. can be enforced only by a criminal prosecution or by an action of quo warranto. But these
proceedings can be maintained only by the Attorney-General in representation of the
As originally drawn, our Corporation Law (Act No. 1459) did not contain any Government.
appropriate clause directly penalizing the act of a corporation, a member of a corporation , in
acquiring an interest contrary to paragraph (5) of section 13 of the Act. The Philippine What room then is left for the private action which the plaintiffs seek to assert in this
Legislature undertook to remedy this situation in section 3 of Act No. 2792 of the Philippine case? The defendant Benguet Company has committed no civil wrong against the plaintiffs,
Legislature, approved on February 18, 1919, but this provision was declared invalid by this and if a public wrong has been committed, the directors of the Balatoc Company, and the
court in Government of the Philippine Islands vs. El Hogar Filipino (50 Phil., 399), for lack of an plaintiff Harden himself, were the active inducers of the commission of that wrong. The
adequate title to the Act. Subsequently the Legislature reenacted substantially the same contract, supposing it to have been unlawful in fact, has been performed on both sides, by
penal provision in section 21 of Act No. 3518, under a title sufficiently broad to comprehend the building of the Balatoc plant by the Benguet Company and the delivery to the latter of the
the subject matter. This part of Act No. 3518 became effective upon approval by the certificate of 600,000 shares of the Balatoc Company. There is no possibility of really undoing
Governor-General, on December 3, 1928, and it was therefore in full force when the contract what has been done. Nobody would suggest the demolition of the mill. The Balatoc Company
now in question was made. is secure in the possession of that improvement, and talk about putting the parties in status
quo ante by restoring the consideration with interest, while the Balatoc Company remains in
This provision was inserted as a new section in the Corporation Law, forming section possession of what it obtained by the use of that money, does not quite meet the case. Also,
1990 (A) of said Act as it now stands. Omitting the proviso, which seems not to be pertinent to mulct the Benguet Company in many millions of dollars in favor of individuals who have not
to the present controversy, said provision reads as follows: the slightest equitable right to that money in a proposition to which no court can give a ready
assent.
SEC. 190 (A). Penalties. The violation of any of the provisions of this Act and its
amendments not otherwise penalized therein, shall be punished by a fine of not The most plausible presentation of the case of the plaintiffs proceeds on the
more than five thousand pesos and by imprisonment for not more than five years, in assumption that only one of the contracting parties has been guilty of a misdemeanor,
the discretion of the court. If the violation is committed by a corporation, the same namely, the Benguet Company, and that the other party, the Balatoc Company, is wholly
shall, upon such violation being proved, be dissolved by quo warranto proceedings innocent to participation in that wrong. The plaintiffs would then have us apply the second
paragraph of article 1305 of the Civil Code which declares that an innocent party to an illegal

81
MATABABE FILES
Corporation Law

contract may recover anything he may have given, while he is not bound to fulfill any promise
he may have made. But, supposing that the first hurdle can be safely vaulted, the general Having shown that the plaintiffs in this case have no right of action against the
remedy supplied in article 1305 of the Civil Code cannot be invoked where an adequate special Benguet Company for the infraction of law supposed to have been committed, we forego cny
remedy is supplied in a special law. It has been so held by this court in Go Chioco vs. discussion of the further question whether a sociedad anonima created under Spanish law,
Martinez (45 Phil., 256, 280), where we refused to apply that article to a case of nullity arising such as the Benguet Company, is a corporation within the meaning of the prohibitory
upon a usurious loan. The reason given for the decision on this point was that the Usury Act, provision already so many times mentioned. That important question should, in our opinion,
as amended, contains all the provisions necessary for the effectuation of its purposes, with be left until it is raised in an action brought by the Government.
the result that the remedy given in article 1305 of the Civil Code is unnecessary. Much more is
that idea applicable to the situation now before us, where the special provisions give ample The judgment which is the subject of his appeal will therefore be affirmed, and it is so
remedies for the enforcement of the law by action in the name of the Government, and ordered, with costs against the appellants.
where no civil wrong has been done to the party here seeking redress.
Avancea, C.J., Villamor, Ostrand, Villa-Real, Abad Santos, Hull, Vickers, Imperial and
The view of the case presented above rest upon considerations arising upon our own Butte, JJ., concur.
statutes; and it would seem to be unnecessary to ransack the American decisions for
analogies pertinent to the case. We may observe, however, that the situation involved is not
unlike that which has frequently arisen in the United States under provisions of the National
Bank Act prohibiting banks organized under that law from holding real property. It has been
uniformly held that a trust deed or mortgaged conveying property of this kind to a bank, by
way of security, is valid until the transaction is assailed in a direct proceeding instituted by the
Government against the bank, and the illegality of such tenure supplies no basis for an action
by the former private owner, or his creditor, to annul the conveyance. (National Bank vs.
Matthews, 98 U. S., 621; Kerfoot vs. Farmers & M. Bank, 218 U. S., 281.) Other analogies point
in the same direction. (South & Ala. R. Ginniss vs. B. & M. Consol. etc. Mining Co., 29 Mont.,
428; Holmes & Griggs Mfg. Co. vs. Holmes & Wessell Metal Co., 127 N. Y., 252; Oelbermann vs.
N. Y. & N. R. Co., 77 Hun., 332.)

Most suggestive perhaps of all the cases in Compaia Azucarera de Carolina vs.
Registrar (19 Porto Rico, 143), for the reason that this case arose under a provision of the
Foraker Act, a law analogous to our Philippine Bill. It appears that the registrar had refused to
register two deeds in favor of the Compaia Azucarera on the ground that the land thereby
conveyed was in excess of the area permitted by law to the company. The Porto Rican court
reversed the ruling of the registrar and ordered the registration of the deeds, saying:

Thus it may be seen that a corporation limited by the law or by its charter has until
the State acts every power and capacity that any other individual capable of
acquiring lands, possesses. The corporation may exercise every act of ownership over
such lands; it may sue in ejectment or unlawful detainer and it may demand specific
performance. It has an absolute title against all the world except the State after a
proper proceeding is begun in a court of law. ... The Attorney General is the exclusive
officer in whom is confided the right to initiate proceedings for escheat or attack the
right of a corporation to hold land.

82
MATABABE FILES
Corporation Law

Republic of the Philippines


SUPREME COURT On May 17, 1948, the Acoje Mining Company, Inc. wrote the Director of Posts
Manila requesting the opening of a post, telegraph and money order offices at its mining camp at
EN BANC Sta. Cruz, Zambales, to service its employees and their families that were living in said camp.
Acting on the request, the Director of Posts wrote in reply stating that if aside from free
G.R. No. L-18062 February 28, 1963 quarters the company would provide for all essential equipment and assign a responsible
employee to perform the duties of a postmaster without compensation from his office until
REPUBLIC OF THE PHILIPPINES, plaintiff-appellee, such time as funds therefor may be available he would agree to put up the offices requested.
vs. The company in turn replied signifying its willingness to comply with all the requirements
ACOJE MINING COMPANY, INC., defendant-appellant. outlined in the letter of the Director of Posts requesting at the same time that it be furnished
with the necessary forms for the early establishment of a post office branch.
Corporations; Ultra vires act defined; When corporate acts may be performed outside
the scope of powers expressly con-ferred.While as a rule an ultra vires act is one committed On April 11, 1949, the Director of Posts again wrote a letter to the company stating
out-side the object for which a corporation is created as defined by the law of its organization among other things that "In cases where a post office will be opened under circumstances
and therefore beyond the powers conferred upon it by law (19 C.J.S., Section 965, p. 419), similar to the present, it is the policy of this office to have the company assume direct
there are however certain corporate acts that may be performed out-side of the scope of the responsibility for whatever pecuniary loss may be suffered by the Bureau of Posts by reason
powers expressly conferred if they are necessary to promote the interest or welfare of the of any act of dishonesty, carelessness or negligence on the part of the employee of the
corporation, such as the establishment, in the case at bar, of a local post office in a mining company who is assigned to take charge of the post office," thereby suggesting that a
camp which is far removed from the postal facilities or means of communications accorded resolution be adopted by the board of directors of the company expressing conformity to the
to- people living in a city or municipality. above condition relative to the responsibility to be assumed buy it in the event a post office
branch is opened as requested. On September 2, 1949, the company informed the Director of
Same; Same; Ultra vires act distinguished from illegal act; Enforcement of ultra vires Posts of the passage by its board of directors of a resolution of the following tenor: "That the
act on the ground of estoppel.An illegal act is void and cannot be validated, while an ultra requirement of the Bureau of Posts that the Company should accept full responsibility for all
vires act is merely voidable and can be enforced by performance, ratifi-cation or estoppel, or cash received by the Postmaster be complied with, and that a copy of this resolution be
on equitable grounds. In the present case the validity of the resolution of the Board of forwarded to the Bureau of Posts." The letter further states that the company feels that that
Directors of the corporation accepting full responsibility in connection with funds to be resolution fulfills the last condition imposed by the Director of Posts and that, therefore, it
received by its postmaster, should be upheld on the ground of estoppel. would request that an inspector be sent to the camp for the purpose of acquainting the
postmaster with the details of the operation of the branch office.
Same; Assumption of responsibility; Responsibility in pres-ent case that of principal
and not that of guarantor.That the responsibility of the defendant corporation is not just The post office branch was opened at the camp on October 13, 1949 with one Hilario
that of a guarantor but of a principal is clear from the resolution of its Board of Directors in M. Sanchez as postmaster. He is an employee of the company. On May 11, 1954, the
which the corporation assumed full responsibility for all cash received by the Postmaster. postmaster went on a three-day leave but never returned.

APPEAL from a decision of the Court of First Instance of Manila. Bocar, J. The company immediately informed the officials of the Manila Post Office and the
provincial auditor of Zambales of Sanchez' disappearance with the result that the accounts of
The facts are stated in the opinion of the Court. the postmaster were checked and a shortage was found in the amount of P13,867.24.
The several demands made upon the company for the payment of the shortage in
Office of the Solicitor General for plaintiff-appellee. line with the liability it has assumed having failed, the government commenced the present
action on September 10, 1954 before the Court of First Instance of Manila seeking to recover
Jalandoni & Jamir for defendant-appellant. the amount of Pl3,867.24. The company in its answer denied liability for said amount
BAUTISTA ANGELO, J.: contending that the resolution of the board of directors wherein it assumed responsibility for

83
MATABABE FILES
Corporation Law

the act of the postmaster is ultra vires, and in any event its liability under said resolution is
only that of a guarantor who answers only after the exhaustion of the properties of the The claim that the resolution adopted by the board of directors of appellant company
principal, aside from the fact that the loss claimed by the plaintiff is not supported by the is an ultra vires act cannot also be entertained it appearing that the same covers a subject
office record. which concerns the benefit, convenience and welfare of its employees and their families.
While as a rule an ultra vires act is one committed outside the object for which a corporation is
Wherefore, the parties respectfully pray that the foregoing stipulation of facts be created as defined by the law of its organization and therefore beyond the powers conferred
admitted and approved by this Honorable Court, without prejudice to the parties adducing upon it by law (19 C.J.S., Section 965, p. 419), there are however certain corporate acts that
other evidence to prove their case not covered by this stipulation of facts. may be performed outside of the scope of the powers expressly conferred if they are
necessary to promote the interest or welfare of the corporation. Thus, it has been held that
After trial, the court a quo found that, of the amount claimed by plaintiff totalling "although not expressly authorized to do so a corporation may become a surety where the
P13,867.24, only the sum of P9,515.25 was supported by the evidence, and so it rendered particular transaction is reasonably necessary or proper to the conduct of its business," 1 and
judgment for the plaintiff only for the amount last mentioned. The court rejected the here it is undisputed that the establishment of the local post office is a reasonable and proper
contention that the resolution adopted by the company is ultra vires and that the obligation it adjunct to the conduct of the business of appellant company. Indeed, such post office is a
has assumed is merely that of a guarantor. vital improvement in the living condition of its employees and laborers who came to settle in
Defendant took the present appeal. its mining camp which is far removed from the postal facilities or means of communication
accorded to people living in a city or municipality..
The contention that the resolution adopted by the company dated August 31, 1949 is
ultra vires in the sense that it has no authority to act on a matter which may render the Even assuming arguendo that the resolution in question constitutes an ultra vires act,
company liable as a guarantor has no factual or legal basis. In the first place, it should be the same however is not void for it was approved not in contravention of law, customs, public
noted that the opening of a post office branch at the mining camp of appellant corporation order or public policy. The term ultra vires should be distinguished from an illegal act for the
was undertaken because of a request submitted by it to promote the convenience and former is merely voidable which may be enforced by performance, ratification, or estoppel,
benefit of its employees. The idea did not come from the government, and the Director of while the latter is void and cannot be validated. 2 It being merely voidable, an ultra vires act
Posts was prevailed upon to agree to the request only after studying the necessity for its can be enforced or validated if there are equitable grounds for taking such action. Here it is
establishment and after imposing upon the company certain requirements intended to fair that the resolution be upheld at least on the ground of estoppel. On this point, the
safeguard and protect the interest of the government. Thus, after the company had signified authorities are overwhelming:
its willingness to comply with the requirement of the government that it furnish free quarters
and all the essential equipment that may be necessary for the operation of the office The weight of authority in the state courts is to the effect that a transaction
including the assignment of an employee who will perform the duties of a postmaster, the which is merely ultra vires and not malum in se or malum prohibitum, is, if performed
Director of Posts agreed to the opening of the post office stating that "In cases where a post by one party, not void as between the parties to all intents and purposes, and that an
office will be opened under circumstances similar to the present, it is the policy of this office action may be brought directly on the transaction and relief had according to its
to have the company assume direct responsibility for whatever pecuniary loss may be terms. (19 C.J.S., Section 976, p. 432, citing Nettles v. Rhett, C.C.A.S.C., 94 F. 2d,
suffered by the Bureau of Posts by reason of any act of dishonesty, carelessness or negligence reversing, D.C., 20 F. Supp. 48)
on the part of the employee of the company who is assigned to take charge of the post
office," and accepting this condition, the company, thru its board of directors, adopted This rule is based on the consideration that as between private corporations,
forthwith a resolution of the following tenor: "That the requirement of the Bureau of Posts one party cannot receive the benefits which are embraced in total performance of a
that the company should accept full responsibility for all cash received by the Postmaster, be contract made with it by another party and then set up the invalidity of the
complied with, and that a copy of this resolution be forwarded to the Bureau of Posts." On transaction as a defense." (London & Lancashire Indemnity Co. of America v.
the basis of the foregoing facts, it is evident that the company cannot now be heard to Fairbanks Steam Shovel Co., 147 N.E. 329, 332, 112 Ohio St. 136.)
complain that it is not liable for the irregularity committed by its employee upon the technical
plea that the resolution approved by its board of directors is ultra vires. The least that can be
said is that it cannot now go back on its plighted word on the ground of estoppel.

84
MATABABE FILES
Corporation Law

The defense of ultra vires rests on violation of trust or duty toward


stockholders, and should not be entertained where its allowance will do greater
wrong to innocent parties dealing with corporation..
The acceptance of benefits arising from the performance by the other party may give
rise to an estoppel precluding repudiation of the transaction. (19 C.J.S., Section 976,
p. 433.)

The current of modern authorities favors the rule that where the ultra vires
transaction has been executed by the other party and the corporation has received the
benefit of it, the law interposes an estoppel, and will not permit the validity of the
transaction or contract to be questioned, and this is especially true where there is
nothing in the circumstances to put the other party to the transaction on notice that
the corporation has exceeded its powers in entering into it and has in so doing
overstepped the line of corporate privileges. (19 C.J.S., Section 977, pp. 435-
437, citing Williams v. Peoples Building & Loan Ass'n, 97 S.W. 2d 930, 193 Ark. 118;
Hays v. Galion Gas Light Co., 29 Ohio St. 330)

Neither can we entertain the claim of appellant that its liability is only that of a
guarantor. On this point, we agree with the following comment of the court a quo: "A mere
reading of the resolution of the Board of Directors dated August 31, 1949, upon which the
plaintiff based its claim would show that the responsibility of the defendant company is not
just that of a guarantor. Notice that the phraseology and the terms employed are so clear and
sweeping and that the defendant assumed 'full responsibility for all cash received by the
Postmaster.' Here the responsibility of the defendant is not just that of a guarantor. It is
clearly that of a principal."

WHEREFORE, the decision appealed from is affirmed. No costs.

Bengzon, C.J., Padilla, Labrador, Concepcion, Reyes, J.B.L., Barrera, Paredes, Dizon, Regala and
Makalintal, JJ. concur.

85
MATABABE FILES
Corporation Law

Republic of the Philippines Same; It is not the quantity of similar acts which establishes apparent authority, but
SUPREME COURT the vesting of a corporate officer with the power to bind the corporation.Petitioners
Manila argument is not persuasive. Apparent authority is derived not merely from practice. Its
FIRST DIVISION existence may be ascertained through (1) the general manner in which the corporation holds
out an officer or agent as having the power to act or, in other words, the apparent authority
G.R. No. 117847 October 7, 1998 to act in general, with which it clothes him; or (2) the acquiescence in his acts of a particular
nature, with actual or constructive knowledge thereof, whether within or beyond the scope
PEOPLE'S AIRCARGO AND WAREHOUSING CO. INC., petitioner, of his ordinary powers. It requires presentation of evidence of similar act(s) executed either in
vs. its favor or in favor of other parties. It is not the quantity of similar acts which establishes
COURT OF APPEALS and STEFANI SAO, respondents. apparent authority, but the vesting of a corporate officer with the power to bind the
corporation.
Actions; Appeals; Pleadings and Practice; The Rules of Court specify reversible
errorsnot grave abuse of discretionas grounds for a petition for review under Rule Same; Estoppel; It is familiar doctrine that if a corporation knowingly permits one of
45.The Court will overlook the lapse of petitioner in alleging grave abuse of discretion as its its officers, or any other agent, to act within the scope of an apparent authority, it holds him
ground for seeking a reversal of the assailed Decision. Although the Rules of Court specify out to the public as possessing the power to do those acts, and thus, the corporation will, as
reversible errors as grounds for a petition for review under Rule 45, the Court will lay aside against anyone who has in good faith dealt with it through such agent, be estopped from
for the nonce this procedural lapse and consider the allegations of grave abuse as denying the agents authority.Private respondent should not be faulted for believing that
statements of reversible errors of law. Punsalans conformity to the contract in dispute was also binding on petitioner. It is familiar
doctrine that if a corporation knowingly permits one of its officers, or any other agent, to act
Corporation Law; In the absence of authority from the board of directors, no person, within the scope of an apparent authority, it holds him out to the public as possessing the
not even its officers, can validly bind a corporation.The general rule is that, in the absence of power to do those acts; and thus, the corporation will, as against anyone who has in good
authority from the board of directors, no person, not even its officers, can validly bind a faith dealt with it through such agent, be estopped from denying the agents authority.
corporation. A corporation is a juridical person, separate and distinct from its stockholders
and members, having x x x powers, attributes and properties expressly authorized by law or Same; Even if a certain contract is outside the usual powers of the president, the
incident to its existence. Being a juridical entity, a corporation may act through its board of corporations ratification of the same and acceptance of benefits make it binding.Private
directors, which exercises almost all corporate powers, lays down all corporate business respondent prepared an operations manual and conducted a seminar for the employees of
policies and is responsible for the efficiency of management, as provided in Section 23 of the petitioner in accordance with their contract. Petitioner accepted the operations manual,
Corporation Code of the Philippines. submitted it to the Bureau of Customs and allowed the seminar for its employees. As a result
of its aforementioned actions, petitioner was given by the Bureau of Customs a license to
Same; The authority of certain individuals to bind the corporation is generally derived operate a bonded warehouse. Granting arguendo then that the Second Contract was outside
from law, corporate bylaws or authorization from the board, either expressly or impliedly by the usual powers of the president, petitioners ratification of said contract and acceptance of
habit, custom or acquiescence in the general course of business.Under Sec. 23, Corporation benefits have made it binding, nonetheless. The enforceability of contracts under Article
Code, the power and the responsibility to decide whether the corporation should enter into a 1403(2) is ratified by the acceptance of benefits under them under Article 1405.
contract that will bind the corporation is lodged in the board, subject to the articles of
incorporation, bylaws, or relevant provisions of law. However, just as a natural person may Same; In the absence of a charter or bylaw provision to the contrary, the president of
authorize another to do certain acts for and on his behalf, the board of directors may validly a corporation is presumed to have the authority to act within the domain of the general
delegate some of its functions and powers to officers, committees or agents. The authority of objectives of its business and within the scope of his or her usual duties.Inasmuch as a
such individuals to bind the corporation is generally derived from law, corporate bylaws or corporate president is often given general supervision and control over corporate operations,
authorization from the board, either expressly or impliedly by habit, custom or acquiescence the strict rule that said officer has no inherent power to act for the corporation is slowly
in the general course of business. giving way to the realization that such officer has certain limited powers in the transaction of
the usual and ordinary business of the corporation. In the absence of a charter or bylaw

86
MATABABE FILES
Corporation Law

provision to the contrary, the president is presumed to have the authority to act within the
domain of the general objectives of its business and within the scope of his or her usual WHEREFORE, in light of all the foregoing, judgment is hereby rendered, ordering
duties. [petitioner] to pay [private respondent] the amount of sixty thousand (P60,000.00) pesos
representing payment of [private respondents] services in preparing the manual of
Contracts; Evidence; The issue of whether a contract is simulated or real is factual in operations and in the conduct of a seminar for [petitioner]. The Counterclaim is hereby
nature.The issue of whether the contract is simulated or real is factual in nature, and the dismissed.
Court eschews factual examination in a petition for review under Rule 45 of the Rules of
Court. This rule, however, admits of exceptions, one of which is a conflict between the factual Aggrieved by what he considered a minuscule award of P60,000, private respondent
findings of the lower and of the appellate courts as in the case at bar. appealed to the Court of Appeals[4] (CA) which, in its Decision promulgated February 28, 1994,
granted his prayer for P400,000, as follows:[5]
Same; Words and Phrases; A contract is simulated if the parties do not intend to be WHEREFORE, PREMISES CONSIDERED, the appealed judgment is hereby MODIFIED in that
bound at all (absolutely simulated), or if the parties conceal their true agreement (relatively [petitioner] is ordered to pay [private respondent] the amount of four hundred thousand
simulated).A fictitious and simulated agreement lacks consent which is essential to a valid pesos (P400,000.00) representing payment of [private respondents] services in preparing the
and enforceable contract. A contract is simulated if the parties do not intend to be bound at manual of operations and in the conduct of a seminar for [petitioner].
all (absolutely simulated), or if the parties conceal their true agreement (relatively simulated).
As no new ground was raised by petitioner, reconsideration of the above-mentioned
Same; Interpretation of Contracts; Contemporaneous and subsequent acts are also Decision was denied in the Resolution promulgated on October 28, 1994.
principal factors in the determination of the will of the contracting parties. The Facts
Contemporaneous and subsequent acts are also principal factors in the determination of the Petitioner is a domestic corporation, which was organized in the middle of 1986 to
will of the contracting parties. The circumstances outlined above do not establish any operate a customs bonded warehouse at the old Manila International Airport in Pasay City. [6]
intention to simulate the contract in dispute. On the contrary, the legal presumption is always
on the validity of contracts. A corporation, by accepting benefits of a transaction entered into To obtain a license for the corporation from the Bureau of Customs, Antonio Punsalan
without authority, has ratified the agreement and is, therefore, bound by it. Jr., the corporation president, solicited a proposal from private respondent for the
preparation of a feasibility study.[7] Private respondent submitted a letter-proposal dated
PETITION for review on certiorari of a decision of the Court of Appeals. October 17, 1986 (First Contract hereafter) to Punsalan, which is reproduced hereunder:[8]

The facts are stated in the opinion of the Court. Dear Mr. Punsalan:
PANGANIBAN, J.:
With reference to your request for professional engineering consultancy services for
Contracts entered into by a corporate president without express prior board your proposed MIA Warehousing Project may we offer the following outputs and the
approval bind the corporation, when such officers apparent authority is established and when corresponding rate and terms of agreement:
these contracts are ratified by the corporation. ====================================
The Case Project Feasibility Study consisting of
Market Study
This principle is stressed by the Court in rejecting the Petition for Review of the February Technical Study
28, 1994 Decision and the October 28, 1994 Resolution of the Court of Appeals in CA-GR CV Financial Feasibility Study
No. 30670. Preparation of pertinent documentation requirements for the
application
In a collection case[1] filed by Stefani Sao against Peoples Aircargo and Warehousing Co., =====================================================
Inc., the Regional Trial Court (RTC) of Pasay City, Branch 110, rendered a Decision [2] dated The above services will be provided for a fee of [p]esos 350,000.00 payable
October 26, 1990, the dispositive portion of which reads: [3] according to the following schedule:

87
MATABABE FILES
Corporation Law

===================================================== President
Fifty percent (50%) .upon confirmation of the agreement
Twenty-five percent (25%)..15 days after the confirmation of the agreement CONFORME & RECEIVED from PAIRCARGO, the
Twenty-five percent (25%)..upon submission of the specified outputs amount of ONE HUNDRED THOUSAND PESOS
The outputs will be completed and submitted within 30 days upon confirmation of (P100,000.00), this 17th day of October,
the agreement and receipt by us of the first fifty percent payment. 1986 as 1st installment payment of the
--------------------------------------------------------------------------------------------- service agreement dated October 13, 1986.
Thank you.
(S)STEFANI C. SAO
Yours truly, CONFORME: (T)STEFANI C. SAO

(S)STEFANI C. SAO (S)ANTONIO C. PUNSALAN, JR. Accordingly, private respondent prepared a feasibility study for petitioner which
(T)STEFANI C. SAO (T)ANTONIO C. PUNSALAN, JR. eventually paid him the balance of the contract price, although not according to the schedule
agreed upon.[11]
Consultant for President, PAIRCARGO
Industrial Engineering On December 4, 1986, upon Punsalans request, private respondent sent petitioner
another letter-proposal (Second Contract hereafter), which reads:
Initially, Cheng Yong, the majority stockholder of petitioner, objected to private Peoples Air Cargo & Warehousing Co., Inc. Old MIA Compound, Metro Manila
respondents offer, as another company priced a similar proposal at only P15,000.[9] However,
Punsalan preferred private respondents services because of the latters membership in the Attention: Mr. ANTONIO PUN[S]ALAN, JR.
task force, which was supervising the transition of the Bureau of Customs from the Marcos President
government to the Aquino administration.[10]
On October 17, 1986, petitioner, through Punsalan, sent private respondent a letter, Dear Mr. Pun[s]alan:
confirming their agreement as follows: This is to formalize our proposal for consultancy services to your company the scope of
which is defined in the attached service description.
Dear Mr. Sao:
The total service you have decided to avail xxx would be available upon signing of the
With regard to the services offered by your company in your letter dated 13 October 1986, for conforme below and would come [in] the amount of FOUR HUNDRED THOUSAND
the preparation of the necessary study and documentations to support our Application for PESOS (P400,000.00) payable at the schedule defined as follows (with the balance
Authority to Operate a public Customs Bonded Warehouse located at the old MIA Compound covered by post-dated cheques):
in Pasay City, please be informed that our company is willing to hire your services and will pay
the amount of THREE HUNDRED FIFTY THOUSAND PESOS (P350,000.00) as follows: Downpayment upon signing conforme . . . P80,000.00
15 January 1987 . . . . . . . . . . . . . 53,333.00
P100,000.00 - upon signing of the agreement; 150,000.00 - on or before October 31, 1986, 30 January 1987 . . . . . . . . . . . . . 53,333.00
with the favorable Recommendation of the CBW on our application. 15 February 1987 . . . . . . . . . . . . . 53,333.00
100,000.00 - upon receipt of the study in final form. 28 February 1987 . . . . . . . . . . . . . 53,333.00
15 March1987 . . . . . . . . . . . . . 53,333.00
Very truly yours, 30 March 1987 . . . . . . . . . . . . . 53,333.00

(S)ANTONIO C. PUNSALAN With this package, you are assured of the highest service quality as our performance
(T)ANTONIO C. PUNSALAN record shows we always deliver no less.

88
MATABABE FILES
Corporation Law

then Commissioner Miriam Defensor-Santiago on March 7, 1988.[15] Meanwhile, Punsalan sold


Thank you very much. his shares in petitioner-corporation and resigned as its president in 1987.[16]

Yours truly, On February 9, 1988, private respondent filed a collection suit against petitioner. He
(S)STEFANI C. SAO alleged that he had prepared an operations manual for petitioner, conducted a seminar-
(T)STEFANI C. SAO workshop for its employees and delivered to it a computer program; but that, despite
Industrial Engineering Consultant demand, petitioner refused to pay him for his services.

CONFORME: Petitioner, in its answer, denied that private respondent had prepared an operations
(S)ANTONIO C. PUNSALAN JR. manual and a computer program or conducted a seminar-workshop for its employees. It
(T)PAIRCARGO CO. INC. further alleged that the letter-agreement was signed by Punsalan without authority, in
collusion with [private respondent] in order to unlawfully get some money from [petitioner],
During the trial, the lower court observed that the Second Contract bore, at the lower and despite his knowledge that a group of employees of the company had been
right portion of the letter, the following notations in pencil: commissioned by the board of directors to prepare an operations manual. [17]

1. Operations Manual The trial court declared the Second Contract unenforceable or simulated. However, since
2. Seminar/workshop for your employees private respondent had actually prepared the operations manual and conducted a training
seminar for petitioner and its employees, the trial court awarded P60,000 to the former, on
P400,000 - package deal the ground that no one should be unjustly enriched at the expense of another (Article 2142,
50% upon completion of seminar/workshop Civil Code). The trial court determined the amount in light of the evidence presented by
50% upon approval by the Commissioner defendant on the usual charges made by a leading consultancy firm on similar services.[18]
The Ruling of the Court of Appeals
The Manual has already been approved by the Commissioner but payment has not yet
been made." To Respondent Court, the pivotal issue of private respondents appeal was the
enforceability of the Second Contract. It noted that petitioner did not appeal the Decision of
The lower left corner of the letter also contained the following notations: the trial court, implying that it had agreed to pay the P60,000 award. If the contract was valid
and enforceable, then petitioner should be held liable for the full amount stated therein,
1st letter - 4 Dec. 1986 not P60,000 as held by the lower court.
2nd letter - 15 June 1987 with Hinanakit.
Rejecting the finding of the trial court that the December 4, 1986 contract was simulated
On January 10, 1987, Andy Villaceren, vice president of petitioner, received the or unenforceable, the CA ruled in favor of its validity and enforceability. According to the
operations manual prepared by private respondent. [12] Petitioner submitted said operations Court of Appeals, the evidence on record shows that the president of petitioner-corporation
manual to the Bureau of Customs in connection with the formers application to operate a had entered into the First Contract, which was similar to the Second Contract. Thus, petitioner
bonded warehouse; thereafter, in May 1987, the Bureau issued to it a license to operate, had clothed its president with apparent authority to enter into the disputed agreement. As it
enabling it to become one of the three public customs bonded warehouses at the had also become the practice of the petitioner-corporation to allow its president to negotiate
international airport.[13] Private respondent also conducted, in the third week of January 1987 and execute contracts necessary to secure its license as a customs bonded warehouse
in the warehouse of petitioner, a three-day training seminar for the latters employees.[14] without prior board approval, the board itself, by its acts and through acquiescence,
practically laid aside the normal requirement of prior express approval. The Second Contract
On March 25, 1987, private respondent joined the Bureau of Customs as special assistant was declared valid and binding on the petitioner, which was held liable to private respondent
to then Commissioner Alex Padilla, a position he held until he became technical assistant to in the full amount of P400,000.

89
MATABABE FILES
Corporation Law

Disagreeing with the CA, petitioner lodged this petition before us.[19] person, separate and distinct from its stockholders and members, having xxx powers,
attributes and properties expressly authorized by law or incident to its existence. [22]
The Issues
Being a juridical entity, a corporation may act through its board of directors, which
Instead of alleging reversible errors, petitioner imputes grave abuse of discretion to the exercises almost all corporate powers, lays down all corporate business policies and is
Court of Appeals, viz.:[20] responsible for the efficiency of management,[23]as provided in Section 23 of the Corporation
Code of the Philippines:
I. xxx [I]n ruling that the subject letter-agreement for services was binding on the corporation
simply because it was entered into by its president[;] SEC. 23. The Board of Directors or Trustees. -- Unless otherwise provided in this Code, the
corporate powers of all corporations formed under this Code shall be exercised, all business
II. xxx [I]n ruling that the subject letter-agreement for services was binding on the conducted and all property of such corporations controlled and held by the board of directors
corporation notwithstanding the lack of any board authority since it was the purported or trustees x x x.
practice to allow the president to enter into contracts of said nature (citing one previous
instance of a similar contract)[;] and Under this provision, the power and the responsibility to decide whether the corporation
should enter into a contract that will bind the corporation is lodged in the board, subject to
III. xxx [I]n ruling that the subject letter-agreement for services was a valid contract and not the articles of incorporation, bylaws, or relevant provisions of law. [24] However, just as a
merely simulated." natural person may authorize another to do certain acts for and on his behalf, the board of
directors may validly delegate some of its functions and powers to officers, committees or
The Court will overlook the lapse of petitioner in alleging grave abuse of discretion as its agents. The authority of such individuals to bind the corporation is generally derived from law,
ground for seeking a reversal of the assailed Decision. Although the Rules of Court specify corporate bylaws or authorization from the board, either expressly or impliedly by habit,
reversible errors as grounds for a petition for review under Rule 45, the Court will lay aside for custom or acquiescence in the general course of business, viz.: [25]
the nonce this procedural lapse and consider the allegations of grave abuse as statements of
reversible errors of law. A corporate officer or agent may represent and bind the corporation in transactions
with third persons to the extent that [the] authority to do so has been conferred upon him,
Petitioner does not contest its liability; it merely disputes the amount of such and this includes powers which have been intentionally conferred, and also such powers as, in
accountability. Hence, the resolution of this petition rests on the sole issue of the the usual course of the particular business, are incidental to, or may be implied from, the
enforceability and validity of the Second Contract, more specifically: (1) whether the president powers intentionally conferred, powers added by custom and usage, as usually pertaining to
of the petitioner-corporation had apparent authority to bind petitioner to the Second the particular officer or agent, and such apparent powers as the corporation has caused
Contract; and (2) whether the said contract was valid and not merely simulated. persons dealing with the officer or agent to believe that it has conferred.

The Courts Ruling Accordingly, the appellate court ruled in this case that the authority to act for and to
bind a corporation may be presumed from acts of recognition in other instances, wherein the
The petition is not meritorious. power was in fact exercised without any objection from its board or shareholders. Petitioner
had previously allowed its president to enter into the First Contract with private respondent
First Issue: Apparent Authority of a Corporate President without a board resolution expressly authorizing him; thus, it had clothed its president with
apparent authority to execute the subject contract.
Petitioner argues that the disputed contract is unenforceable, because Punsalan, its
president, was not authorized by its board of directors to enter into said contract. Petitioner rebuts, arguing that a single isolated agreement prior to the subject contract
does not constitute corporate practice, which Webster defines as frequent or customary
The general rule is that, in the absence of authority from the board of directors, no action. It cites Board of Liquidators v. Kalaw,[26] in which the practice of NACOCO allowing its
person, not even its officers, can validly bind a corporation. [21] A corporation is a juridical general manager to negotiate and execute contract in its copra trading activities for and on

90
MATABABE FILES
Corporation Law

its behalf, without prior board approval, was inferred from sixty contracts not one, as in the seminar for its employees. As a result of its aforementioned actions, petitioner was given by
present case -- previously entered into by the corporation without such board resolution. the Bureau of Customs a license to operate a bonded warehouse. Granting arguendo then
that the Second Contract was outside the usual powers of the president, petitioners
Petitioners argument is not persuasive. Apparent authority is derived not merely from ratification of said contract and acceptance of benefits have made it binding,
practice. Its existence may be ascertained through (1) the general manner in which the nonetheless. The enforceability of contracts under Article 1403(2) is ratified by the acceptance
corporation holds out an officer or agent as having the power to act or, in other words, the of benefits under them under Article 1405.
apparent authority to act in general, with which it clothes him; or (2) the acquiescence in his
acts of a particular nature, with actual or constructive knowledge thereof, whether within or Inasmuch as a corporate president is often given general supervision and control over
beyond the scope of his ordinary powers.[27] It requires presentation of evidence of similar corporate operations, the strict rule that said officer has no inherent power to act for the
act(s) executed either in its favor or in favor of other parties.[28] It is not the quantity of similar corporation is slowly giving way to the realization that such officer has certain limited powers
acts which establishes apparent authority, but the vesting of a corporate officer with the in the transaction of the usual and ordinary business of the corporation.[31] In the absence of a
power to bind the corporation. charter or bylaw provision to the contrary, the president is presumed to have the authority to
act within the domain of the general objectives of its business and within the scope of his or
In the case at bar, petitioner, through its president Antonio Punsalan Jr., entered into the her usual duties.[32]
First Contract without first securing board approval. Despite such lack of board approval,
petitioner did not object to or repudiate said contract, thus clothing its president with the Hence, it has been held in other jurisdictions that the president of a corporation
power to bind the corporation. The grant of apparent authority to Punsalan is evident in the possesses the power to enter into a contract for the corporation, when the conduct on the
testimony of Yong -- senior vice president, treasurer and major stockholder of part of both the president and the corporation [shows] that he had been in the habit of acting
petitioner. Testifying on the First Contract, he said:[29] in similar matters on behalf of the company and that the company had authorized him so to
act and had recognized, approved and ratified his former and similar actions. [33]Furthermore,
A: Mr. [Punsalan] told me that he prefer[s] Mr. Sao because Mr. Sao is very influential with a party dealing with the president of a corporation is entitled to assume that he has the
the Collector of Customs[s]. Because the Collector of Custom[s] will be the one to authority to enter, on behalf of the corporation, into contracts that are within the scope of
approve our project study and I objected to that, sir. And I said it [was an exorbitant] the powers of said corporation and that do not violate any statute or rule on public policy.[34]
price. And Mr. Punsalan he is the [p]resident, so he [gets] his way.
Second Issue: Alleged Simulation of the First Contract
Q: And so did the company eventually pay this P350,000.00 to Mr. Sao?
As an alternative position, petitioner seeks to pare down its liabilities by limiting its
A: Yes, sir. exposure from P400,000 to only P60,000, the amount awarded by the RTC. Petitioner
capitalizes on the badges of fraud cited by the trial court in declaring said contract either
The First Contract was consummated, implemented and paid without a hitch. simulated or unenforceable, viz.:

Hence, private respondent should not be faulted for believing that Punsalans conformity xxx The October 1986 transaction with [private respondent] involved P350,000. The
to the contract in dispute was also binding on petitioner. It is familiar doctrine that if a same was embodied in a letter which bore therein not only the conformity of
corporation knowingly permits one of its officers, or any other agent, to act within the scope [petitioners] then President Punsalan but also drew a letter-confirmation from the latter
of an apparent authority, it holds him out to the public as possessing the power to do those for, indeed, he was clothed with authority to enter into the contract after the same was
acts; and thus, the corporation will, as against anyone who has in good faith dealt with it brought to the attention and consideration of [petitioner]. Not only that, a [down
through such agent, be estopped from denying the agents authority. [30] payment] was made. In the alleged agreement of December 4, 1986 subject of the
present case, the amount is even bigger-P400,000.00. Yet, the alleged letter-agreement
Furthermore, private respondent prepared an operations manual and conducted a drew no letter of confirmation. And no [down payment] and postdated checks were
seminar for the employees of petitioner in accordance with their contract. Petitioner given. Until the filing of the present case in February 1988, no written demand for
accepted the operations manual, submitted it to the Bureau of Customs and allowed the payment was sent to [petitioner]. [Private respondents] claim that he sent one in

91
MATABABE FILES
Corporation Law

writing, and one was sent by his counsel who manifested that [h]e was looking for a down, partial or full payment), even after completion of private respondents obligations,
copy in [his] files fails in light of his failure to present any such copy. These and the imports only a defect in the performance of the contract on the part of petitioner. Second, the
following considerations, to wit: delay in the filing of action was not fatal to private respondents cause. Despite the lapse of
one year after private respondent completed his services or eight months after the alleged
1) Despite the fact that no [down payment] and/or postdated checks [partial payments] (as last demand forpayment in June 1987, the action was still filed within the allowable period,
purportedly stipulated in the alleged contract) [was given, private respondent] went ahead considering that an action based on a written contract prescribes only after ten years from
with the services[;] the time the right of action accrues.[37] Third, a misspelling in the contract does not establish
vitiation of consent, cause or object of the contract. Fourth, a confirmation letter is not an
2) [There was a delay in the filing of the present suit, more than a year after [private essential element of a contract; neither is it necessary to perfect one. Fifth, private
respondent] allegedly completed his services or eight months after the alleged last verbal respondents failure to implead the corporate president does not establish collusion between
demand for payment made on Punsalan in June 1987; them. Petitioner could have easily filed a third-party claim against Punsalan if it believed that it
had recourse against the latter.Lastly, the mere fact that the contract price was six times the
3) Does not Punsalans writing allegedly in June 1987 on the alleged letter-agreement of your alleged going rate does not invalidate it.[38] In short, these badges do not establish simulation
employees[,] when it should have been our employees, as he was then still connected with of said contract.
[petitioner], indicate that the letter-agreement was signed by Punsalan when he was no
longer connected with [petitioner] or, as claimed by [petitioner], that Punsalan signed it A fictitious and simulated agreement lacks consent which is essential to a valid and
without [petitioners] authority and must have been done in collusion with plaintiff in order to enforceable contract.[39] A contract is simulated if the parties do not intend to be bound at all
unlawfully get some money from [petitioner]? (absolutely simulated),[40] or if the parties conceal their true agreement (relatively
simulated).[41] In the case at bar, petitioner received from private respondent a letter-offer
4) If, as [private respondent] claims, the letter was returned by Punsalan after affixing containing the terms of the former, including a stipulation of the consideration for the latters
thereon his conformity, how come xxx when Punsalan allegedly visited [private respondent] services. Punsalans conformity, as well as the receipt and use of the operations manual,
in his office at the Bureau of Customs, in June 1987, Punsalan brought (again?) the letter (with shows petitioners consent to or, at the very least, ratification of the contract. To repeat,
the pencil [notation] at the left bottom portion allegedly already written)? petitioner even submitted the manual to the Bureau of Customs and allowed private
respondent to conduct the seminar for its employees. Private respondent heard no objection
5) How come xxx [private respondent] did not even keep a copy of the alleged service from the petitioner, until he claimed payment for the services he had rendered.
contract allegedly attached to the letter-agreement?
Contemporaneous and subsequent acts are also principal factors in the determination of
6) Was not the letter-agreement a mere draft, it bearing the corrections made by Punsalan of the will of the contracting parties.[42] The circumstances outlined above do not establish any
his name (the letter n is inserted before the last letter o in Antonio) and of the spelling of his intention to simulate the contract in dispute. On the contrary, the legal presumption is always
family name (Punsalan, not Punzalan)? on the validity of contracts. A corporation, by accepting benefits of a transaction entered into
without authority, has ratified the agreement and is, therefore, bound by it. [43]
7) Why was not Punsalan impleaded in the case?
WHEREFORE, the petition is hereby DENIED and the assailed Decision AFFIRMED. Costs
The issue of whether the contract is simulated or real is factual in nature, and the Court against petitioner.
eschews factual examination in a petition for review under Rule 45 of the Rules of
Court.[35] This rule, however, admits of exceptions, one of which is a conflict between the SO ORDERED.
factual findings of the lower and of the appellate courts[36] as in the case at bar.
Davide, Jr. (Chairman), Bellosillo, Vitug, and Quisumbing, JJ., concur.
After judicious deliberation, the Court agrees with the appellate court that the alleged
badges of fraud mentioned earlier have not affected in any manner the perfection of the
Second Contract or proved the alleged simulation thereof. First, the lack of payment (whether

92
MATABABE FILES
Corporation Law

Republic of the Philippines money, which may then be available for wage increments. It is not a case of a corporation
SUPREME COURT distributing dividends in favor of its stockholders, in which case, such dividends would be the
Manila absolute property of the stockholders and hence, out of reach by creditors of the
EN BANC corporation. Here, the petitioner was acting as stockholder itself, and in that case, the right to
a share in such dividends, by way of salary increases, may not be denied its employees.
G.R. No. L-48237 June 30, 1987
Same; Unfair Labor Practice; Reduction of capital to evade employees demand for
MADRIGAL & COMPANY, INC., petitioner, salary adjustments, mass lay-off of employees under the guise of retrenchment policy
vs. constitute unfair labor practice.Accordingly, this court is convinced that the petitioners
HON. RONALDO B. ZAMORA, PRESIDENTIAL ASSISTANT FOR LEGAL AFFAIRS, THE HON. capital reduction efforts were, to begin with, a subterfuge, a deception as it were, to
SECRETARY OF LABOR, and MADRIGAL CENTRAL OFFICE EMPLOYEES UNION, respondents. camouflage the fact that it had been making profits, and consequently, to justify the mass lay
off in its employee ranks, especially of union members. They were nothing but a premature
G.R. No. L-49023 June 30, 1987 and plain distribution of corporate assets to obviate a just sharing to labor of the vast profits
obtained by its joint efforts with capital through the years. Surely, we can neither
MADRIGAL & COMPANY, INC., petitioner, countenance nor condone this. It is an unfair labor practice. As we observed in Peoples Bank
vs. and Trust Company v. Peoples Bank and Trust Co. Employees Union: xxx xxx xxx As has been
HON. MINISTER OF LABOR and MADRIGAL CENTRAL OFFICE EMPLOYEES held by this Court in Insular Lumber Company vs. CA, et al., L-23875, August 29, 1969, 29 SCRA
UNION, respondents. 371, retrenchment can only be availed of if the company is losing or meeting financial reverses
in its operation, which certainly is not the case at bar. Undisputed is the fact, that the Bank at
Evidence; Findings of administrative agencies are accorded not only respect but even no time incurred losses. As a matter of fact. the net earnings of the Bank would be in the
finality.As a general rule, the findings of administrative agencies are accorded not only average of P2,000,000.00 a year from 1960 to 1969 and, during this period of nine (9) years,
respect but even finality. This is especially true with respect to the Department of Labor, the Bank continuously declared dividends to its stockholders. Thus the mass layoff or
which performs not only a statutory function but carries out a Constitutional mandate as well. dismissal of the 65 employees under the guise of retrenchment policy of the Bank is a lame
Our jurisdiction, as a rule, is confined to cases of grave abuse of discretion. But for certiorari excuse and a veritable smoke-screen of its scheme to bust the Union and thus unduly disturb
to lie, there must be such arbitrary and whimsical exercise of power, or that discretion was the employment tenure of the employees concerned, which act is certainly an unfair labor
exercised despotically. practice.

Labor Law; Dividends received by the company are corporate earnings arising from PETITIONS for certiorari and prohibition to review the decision of the Secretary of Labor.
corporate investment.The petitioner would, however, have us believe that it in fact
sustained losses. Whatever profits it earned, so it claims were in the nature of dividends The facts are stated in the opinion of the Court.
declared on its shareholdings in other companies in the earning of which the employees had
no participation whatsoever. Cash dividends, according to it, are the absolute property of SARMIENTO, J.:
the stockholders and cannot be made available for disposition if only to meet the employees
economic demands. There is no merit in this contention. We agree with the National Labor These are two petitions for certiorari and prohibition filed by the petitioner, the
Relations Commission that [t]he dividends received by the company are corporate earnings Madrigal & Co., Inc. The facts are undisputed.
arising from corporate investment. Indeed, as found by the Commission, the petitioner had The petitioner was engaged, among several other corporate objectives, in the
entered such earnings in its financial statements as profits, which it would not have done if management of Rizal Cement Co., Inc.1 Admittedly, the petitioner and Rizal Cement Co., Inc.
they were not in fact profits. Moreover, it is incorrect to say that such profitsin the form of are sister companies.2 Both are owned by the same or practically the same stockholders.3 On
dividendsare beyond the reach of the petitioners creditors since the petitioner had December 28, 1973, the respondent, the Madrigal Central Office Employees Union, sought for
received them as compensation for its management services in favor of the companies it the renewal of its collective bargaining agreement with the petitioner, which was due to
managed as a shareholder thereof. As such shareholder. the dividends paid to it were its own expire on February 28, 1974.4 Specifically, it proposed a wage increase of P200.00 a month, an

93
MATABABE FILES
Corporation Law

allowance of P100.00 a month, and other economic benefits.5 The petitioner, however, the dismissals "to be contrary to law" 23 and ordered the petitioner to reinstate some 40
requested for a deferment in the negotiations. employees, 37 of them with backwages. 24 The petitioner then moved for reconsideration,
On July 29, 1974, by an alleged resolution of its stockholders, the petitioner reduced its capital which the Acting Labor Secretary, Amado Inciong, denied. 25
stock from 765,000 shares to 267,366 shares.6 This was effected through the distribution of
the marketable securities owned by the petitioner to its stockholders in exchange for their Thereafter, the petitioner filed an appeal to the Office of the President. The
shares in an equivalent amount in the corporation.7 respondent, the Presidential Assistant on Legal Affairs, affirmed with modification the Labor
Department's decision, thus:
On August 22, 1975, by yet another alleged stockholders' action, the petitioner
reduced its authorized capitalization from 267,366 shares to 110,085 shares, again, through xxx xxx xxx
the same scheme.8
1. Eliseo Dizon, Eugenio Evangelista and Benjamin Victorio are excluded from the
After the petitioner's failure to sit down with the respondent union, the latter, on order of reinstatement.
August 28, 1974, commenced Case No. LR-5415 with the National Labor Relations Commission
on a complaint for unfair labor practice.9 In due time, the petitioner filed its position 2. Rogelio Meneses and Roberto Taladro who appear to have voluntarily retired and
paper, 10 alleging operational losses. Pending the resolution of Case No. LR-5415, the paid their retirement pay, their cases are left to the judgment of the Secretary of
petitioner, in a letter dated November 17, 1975, 11 informed the Secretary of Labor that Rizal Labor who is in a better position to assess appellant's allegation as to their
Cement Co., Inc., "from which it derives income" 12 "as the General Manager or Agent" 13 had retirement.
"ceased operating temporarily." 14 "In addition, "because of the desire of the stockholders to
phase out the operations of the Madrigal & Co., Inc. due to lack of business incentives and 3. The rest are hereby reinstated with six (6) months backwages, except Aleli
prospects, and in order to prevent further losses," 15it had to reduce its capital stock on two Contreras, Teresita Eusebio and Norma Parlade who are to be reinstated without
occasions "As the situation, therefore, now stands, the Madrigal & Co., Inc. is without backwages.
substantial income to speak of, necessitating a reorganization, by way of retrenchment, of its
employees and operations." 16 The petitioner then requested that it "be allowed to effect said SO ORDERED. 26
reorganization gradually considering all the circumstances, by phasing out in at least three (3)
stages, or in a manner the Company deems just, equitable and convenient to all concerned, xxx xxx xxx
about which your good office will be apprised accordingly." 17 The letter, however, was not
verified and neither was it accompanied by the proper supporting papers. For this reason, the On May 15, 1978, the petitioner came to this court. (G.R. No. 48237.)
Department of Labor took no action on the petitioner's request. Meanwhile, on May 25, 1977, the National Labor Relations Commission rendered a decision
affirming the labor arbiter's judgment in Case No. LR-5415. 27 The petitioner appealed to the
On January 19, 1976, the labor arbiter rendered a decision 18 granting, among other Secretary of Labor. On June 9, 1978, the Secretary of Labor dismissed the appeal. 28 Following
things, a general wage increase of P200.00 a month beginning March 1, 1974 plus a monthly these successive reversals, the petitioner came anew to this court. (G.R. No. 49023.)
living allowance of P100.00 monthly in favor of the petitioner's employees. The arbiter
specifically found that the petitioner "had been making substantial profits in its By our resolution dated October 9, 1978, we consolidated G.R. No. 48237 with G.R.
operation" 19 since 1972 through 1975. The petitioner appealed. No. 49023. 29 We likewise issued temporary restraining orders. 30

On January 29, 1976, the petitioner applied for clearance to terminate the services of In G.R. No. 48237, the petitioner argues, that.
a number of employees pursuant supposedly to its retrenchment program. On February 3,
1976, the petitioner applied for clearance to terminate 18 employees more. 20 On the same xxx xxx xxx
date, the respondent union went to the Regional Office (No. IV) of the Department of Labor
(NLRC Case No. R04-2-1432-76) to complain of illegal lockout against the petitioner. 21Acting I. SAID RESPONDENTS ERRED IN HOLDING THAT THERE WAS NO VALID COMPLIANCE WITH
on this complaint, the Secretary of 22 Labor, in a decision dated December 14, 1976, 22 found THE CLEARANCE REQUIREMENT.

94
MATABABE FILES
Corporation Law

certiorari to lie, there must be such arbitrary and whimsical exercise of power, or that
II. SAID RESPONDENTS ERRED IN NOT HOLDING THAT THERE IS NO LOCKOUT HERE IN LEGAL discretion was exercised despotically. 36
CONTEMPLATION, MUCH LESS FOR UNION-BUSTING PURPOSES.
In no way can the questioned decisions be seen as arbitrary. The decisions
III. RESPONDENT PRESIDENTIAL ASSISTANT ERRED IN ORDERING THE REINSTATEMENT OF themselves show why.
THE REST OF AFFECTED MEMBERS OF RESPONDENT UNION WITH SIX (6) MONTHS
BACKWAGES, EXCEPT ALELI CONTRERAS, TERESITA EUSEBIO AND NORMA PARLADE WHO Anent Case No. R04-2-1432-76 (G.R. No. 48237), we are satisfied with the correctness
ARE TO BE REINSTATED WITHOUT BACKWAGES. of the respondent Presidential Assistant for Legal Affairs' findings.

IV. RESPONDENT PRESIDENTIAL ASSISTANT ERRED IN LEAVING TO THE JUDGMENT OF We quote:


RESPONDENT SECRETARY THE CASES OF ROGELIO MENESES AND ROBERTO TALADRO WHO
HAD VOLUNTARILY RETIRED AND PAID THEIR RETIREMENT PAY.31 xxx xxx xxx

xxx xxx xxx In urging reversal of the appealed decision, appellant contends that (1) its
letter dated November 17, 1975, constitute "substantial compliance with the
while in G.R. No. 49023, it submits that: clearance requirement to terminate;" and (2) individual appellees' dismissal had no
relation to any union activities, but was the result of an honest-to-goodness
xxx xxx xxx retrenchment policy occasioned by loss of income due to cessation of operation.

1. RESPONDENT MINISTER ERRED IN AFFIRMING THE DECISION EN BANC OF THE We find the first contention to be without merit. Aside from the fact that the
NATIONAL LABOR RELATIONS COMMISSION DESPITE CLEAR INDICATIONS IN THE RECORD controversial letter was unverified, with not even a single document submitted in
THAT THE AWARD WAS PREMATURE IN THE ABSENCE OF A DEADLOCK IN NEGOTIATION support thereof, the same failed to specify the individual employees to be affected
AND THE FAILURE ON THE PART OF THE LABOR ARBITER TO RESOLVE THE MAIN IF NOT by the intended retrenchment. Not only this, but the letter is so vague and indefinite
ONLY ISSUE OF REFUSAL TO BARGAIN, THEREBY DEPRIVING PETITIONER OF ITS RIGHT TO regarding the manner of effecting appellant's retrenchment plan as to provide the
DUE PROCESS. Secretary of (sic) a reasonable basis on which to determine whether the request for
2. ASSUMING ARGUENDO THAT THERE WAS A DEADLOCK IN NEGOTIATION, retrenchment was valid or otherwise, and whether the mechanics in giving effect
RESPONDENT MINISTER ERRED NEVERTHELESS IN NOT FINDING THAT THE ECONOMIC thereto was just or unjust to the employees concerned. In fact, to be clearly implied
BENEFITS GRANTED IN THE FORM OF SALARY INCREASES ARE UNFAIR AND VIOLATIVE OF from the letter is that the implementary measures needed to give effect to the
THE MANDATORY GUIDELINES PRESCRIBED UNDER PRESIDENTIAL DECREE NO. 525 AND intended retrenchment are yet to be thought of or concretized in the indefinite
IGNORING THE UNDISPUTED FACT THAT PETITIONER HAD VIRTUALLY CEASED OPERATIONS future, measures about which the office of the Secretary "will be apprised
AFTER HAVING TWICE DECREASED ITS CAPITAL STOCKS AND, THEREFORE, NOT FINANCIALLY accordingly." All these, and more, as correctly found by the Acting Secretary, cannot
CAPABLE TO ABSORB SUCH AWARD OF BENEFITS. 32 but show that the letter is insufficient in form and substance to constitute a valid
xxx xxx xxx compliance with the clearance requirement. That being so, it matters little whether
or not complainant union or any of its members failed to interpose any opposition
There is no merit in these two (2) petitions. thereto.

As a general rule, the findings of administrative agencies are accorded not only It cannot be over-emphasized that the purpose in requiring a prior clearance
respect but even finality. 33 This is especially true with respect to the Department of Labor, by the Secretary of Labor, in cases of shutdown or dismissal of employees, is to
which performs not only a statutory function but carries out a Constitutional mandate as afford said official ample opportunity to examine and determine the reasonableness
well. 34 Our jurisdiction, as a rule, is confined to cases of grave abuse of discretion. 35 But for of the request. This is made imperative in order to give meaning and substance to the
constitutional mandate that the State must "afford protection to labor," and

95
MATABABE FILES
Corporation Law

guarantee their "security of tenure." Indeed, the rules require that the application for surgery in Oct., 1975." As to Aleli Contreras, there is no showing that the Secretary of
clearance be filed ten (10) days before the intended shutdown or dismissal, serving a Labor or appellant ever acted on the clearance. If we were to follow the logic of
copy thereof to the employees affected in order that the latter may register their appellant, Contreras should not have been included in the application for clearance
own individual objections against the grant of the clearance. But how could this filed on Feb. 3, 1976. The fact that she was included shows that up to that time, she
requirement of notice to the employees have been complied with, when, as observed was still considered as a regular employee. It was for these reasons, coupled with the
by the Acting Secretary in his modificatory decision dated June 30, 1977 "the latter of length of service that these employees have rendered appellant, that the Acting
November 17, 1975 does not even state definitely the employees involved" upon Secretary ordered their reinstatement but without backwages. 37
whom service could be made.
With respect to appellant's second contention, we agree with the Acting Secretary's xxx xxx xxx
findings that individual appellee's dismissal was an offshoot of the union's demand
for a renegotiation of the then validly existing collective bargaining Agreement. With respect Lo Case No. LR-5415 (G.R. No. 49023), we are likewise content with the findings
of the National Labor Relations Commission. Thus:
xxx xxx xxx
xxx xxx xxx
The pattern of appellant's acts after the decision of the Labor Arbiter in Case
No. LR-5415 has convinced us that its sole objective was to render moot and Appellant now points that the only issue certified to compulsory arbitration
academic the desire of the union to exercise its right to bargain collectively with is "refusal to bargain" and it is, therefore, premature to dictate the terms of the CBA
management, especially so when it is considered in the light of the fact that under on the assumption that there was already a deadlock in negotiation. Appellant
the said decision the demand by the union for wage increase and allowances was further contends that, assuming there was deadlock in negotiation, the economic
granted. What renders appellant's motive suspect was its haste in terminating the benefits granted are unreasonable and violative of the guideline prescribed by P.D.
services of individual appellees, without waiting the outcome of its appeal in Case 525.
No. LR-5415. The amount involved by its offer to pay double separation could very
well have been used to pay the salaries of those employees whose services were On the other hand, it is the union's stance that its economic demands are
sought to be terminated, until the resolution of its appeal with the NLRC, since justified by, the persistent increase in the cost of living and the substantial earnings
anyway, if its planned retrenchment is found to be justifiable and done in good faith, of the company from 1971 to 1975.
its only liability is to answer for the separation pay provided by law. By and large,
therefore, we agree with the Acting Secretary that, under the circumstances It bears to stress that although the union's petition was precipitated by the
obtaining in this case, "respondent's action [was] a systematic and deliberate company's refusal to bargain, there are glaring circumstances pointing out that the
attempt to get rid of complainants because of their union activities. parties also submitted "deadlock" to arbitration. The petition itself is couched in
general terms, praying for arbitration of the union's "dispute" with the respondent
We now come to the individual cases of Aleli Contreras, Teresita Eusebio and concerning proposed changes in the collective bargaining agreement." It is
Norma Parlade. It is appellant's claim that these three (3) should not be reinstated supported with a copy of the proposed changes which just goes to show that the
inasmuch as they have abandoned their work by their continued absences, and union, aside from the issue concerning respondent's refusal to bargain, sought
moreover in the case of Contreras, she failed to oppose the application for clearance determination of the merit of its proposals. On the part of the appellant company, it
filed against her on October 24, 1975. However, appellant's payrolls for December 16- pleaded financial incapacity to absorb the proposed economic benefits during the
31, 1975, January 1-15, 1976 and January 16-31, 1976, show that the three (3) were "on initial stage of the proceedings below. Even the evidence and arguments proferred
leave without pay." As correctly appreciated by the Acting Secretary, these "payrolls below by both parties are relevant to deadlock issue. In the face of these factual
prove, first, that "leave" has been granted to these employees, and, second, that it is environment, it is our view that the Labor Arbiter below did not commit a reversible
a practice in the company to grant "leaves without pay" without loss of employment error in rendering judgment on the proposed CBA changes. At any rate, the minimum
status, to those who have exhausted their authorized leaves." As regards, Norma requirements of due process was satisfied because as heretofore stated, the
Parlade, the records show that she "truly incurred illness and actually underwent

96
MATABABE FILES
Corporation Law

appellant was given Opportunity, and had in fact, presented evidence and argument That the petitioner made quite handsome profits is clear from the records. The labor
in avoidance of the proposed CBA changes. arbiter stated in his decision in the collective agreement case (Case No. LR-5415):

We do not also subscribe to appellant's argument that by reducing its xxx xxx xxx
capital, it is made evident that it is phasing out its operations. On the contrary,
whatever may be the reason behind such reductions, it is indicative of an intention to A clear scrutiny of the financial reports of the respondent [herein petitioner]
keep the company a going concern. So much so that until now almost four (4) years reveals that it had been making substantial profits in the operation.
later, it is still very much in existence and operational as before. In 1972, when it still had 765,000 common shares, of which 305,000 were unissued
and 459,000 outstanding capitalized at P16,830,000.00, the respondent made a net
We now come to the question concerning the equitableness of the profit of P2,403,211.58. Its total assets were P70,821,317.81.
economic benefits granted below. It requires no evidence to show that the
employees concerned deserve some degree of upliftment due to the unabated In 1973, based on the same capitalization, its profit increased to
increase in the cost of living especially in Metro Manila. Of course the company would P2,724,465.33. Its total assets increased to P83,240,473.73.
like us to believe that it is losing and is therefore not financially capable of improving
the present CBA to favor its employees. In support of such assertion, the company In 1974, although its capitalization was reduced from P16,830,000.00 to
points that the profits reflected in its yearly Statement of Income and Expenses are P11,230,459.36, its profits were further increased to P2,922,349.70. Its assets were
dividends from security holdings. We, however, reject as puerile its suggestion to P78,842,175.75.
dissociate the dividends it received from security holdings on the pretext that they
belong exclusively to its stockholders. The dividends received by the company are The reduction in its assets by P4,398,297.98 was due to the fact that its
corporate earnings arising from corporate investment which no doubt are attended capital stock was reduced by the amount of P5,599,540.54.
to by the employees involved in this proceedings. Otherwise. it would not have been In 1975, for the period of only six months, the respondent reported a net profit of
reflected as part of profits in the company's yearly financial statements. In P547,414.72, which when added to the surplus of P5,591.214.19, makes a total surplus
determining the reasonableness of the economic grants below, we have, therefore, of P6,138,628.91 as of June 30, 1975. 39
scrutinized the company's Statement of Income and Expenses from 1972 to 1975 and
after equating the welfare of the employees with the substantial earnings of the xxx xxx xxx
company, we find the award to be predicated on valid justifications.
The petitioner would, however, have us believe that it in fact sustained losses.
The salary increase we herein sanction is also in keeping with the rational Whatever profits it earned, so it claims were in the nature of dividends "declared on its
that made imperative the enactment of the Termination Pay Law since in case the shareholdings in other companies in the earning of which the employees had no participation
respondent company really closes down, the employees will receive higher whatsoever." 40 "Cash dividends," according to it, "are the absolute property of the
separation pay or retirement benefits to tide them over while seeking another stockholders and cannot be made available for disposition if only to meet the employees'
employment. 38 economic demands." 41

What clearly emerges from the recorded facts is that the petitioner, awash with There is no merit in this contention. We agree with the National Labor Relations
profits from its business operations but confronted with the demand of the union for wage Commission that "[t]he dividends received by the company are corporate earnings arising
increases, decided to evade its responsibility towards the employees by a devised capital from corporate investment." 42 Indeed, as found by the Commission, the petitioner had
reduction. While the reduction in capital stock created an apparent need for retrenchment, it entered such earnings in its financial statements as profits, which it would not have done if
was, by all indications, just a mask for the purge of union members, who, by then, had they were not in fact profits. 43
agitated for wage increases. In the face of the petitioner company's piling profits, the
unionists had the right to demand for such salary adjustments. Moreover, it is incorrect to say that such profits in the form of dividends are
beyond the reach of the petitioner's creditors since the petitioner had received them as

97
MATABABE FILES
Corporation Law

compensation for its management services in favor of the companies it managed as a the estate is up for partition without the petitioner's business closing shop and inevitably,
shareholder thereof. As such shareholder, the dividends paid to it were its own money, which without the petitioner laying off its employees.
may then be available for wage increments. It is not a case of a corporation distributing
dividends in favor of its stockholders, in which case, such dividends would be the absolute As regards the question whether or not the petitioner's letter dated November 17,
property of the stockholders and hence, out of reach by creditors of the corporation. Here, 1975 47 was in substantial compliance with legal clearance requirements, suffice it to state that
the petitioner was acting as stockholder itself, and in that case, the right to a share in such apart from the Secretary of Labor's valid observation that the same "did not constitute a
dividends, by way of salary increases, may not be denied its employees. sufficient clearance as contemplated by law, " 48 the factual circumstances show that the
letter in question was itself a part of the "systematic and deliberate attempt to get rid of [the
Accordingly, this court is convinced that the petitioner's capital reduction efforts union members] because of their union activities." 49 Hence, whether or not the said letter
were, to begin with, a subterfuge, a deception as it were, to camouflage the fact that it had complied with the legal formalities is beside the point since under the circumstances,
been making profits, and consequently, to justify the mass layoff in its employee ranks, retrenchment was, in all events, unjustified. Parenthetically, the clearance required under
especially of union members. They were nothing but a premature and plain distribution of Presidential Decree No. 850 has been done away with by Batas Blg. 130, approved on August
corporate assets to obviate a just sharing to labor of the vast profits obtained by its joint 21, 1981.
efforts with capital through the years. Surely, we can neither countenance nor condone this.
It is an unfair labor practice. During the pendency of these petitions, the petitioner submitted manifestations to
the effect that certain employees have accepted retirement benefits pursuant to its
As we observed in People's Bank and Trust Company v. People's Bank and Trust Co. retrenchment scheme. 50 This is a matter of defense that should be raised before the National
Employees Union: 44 Labor Relations Commission.

xxx xxx xxx To do away with the protracted process of determining the earnings acquired by the
employees as a result of ad interim employment, and to erase any doubt as to the amount of
As has been held by this Court in Insular Lumber Company vs. CA, et al., L- backwages due them, this court, in line with the precedent set in Mercury Drug Co., Inc. v.
23875, August 29, 1969, 29 SCRA 371, retrenchment can only be availed of if the Court of Industrial Relations, 51 affirmed in a long line of decisions that came later, 52 hereby
company is losing or meeting financial reverses in its operation, which certainly is not fixes the amount of backwages at three (3) years pay reckoned at the increased rates decreed
the case at bar. Undisputed is the fact, that the Bank "at no time incurred losses. " As by the labor arbiter in Case No. LR-5415 without deduction or qualification.
a matter of fact, "the net earnings of the Bank would be in the average of
P2,000,000.00 a year from 1960 to 1969 and, during this period of nine (9) years, the WHEREFORE, the petitions are hereby DISMISSED. Subject to the modification as to
Bank continuously declared dividends to its stockholders." Thus the mass lay-off or the amount of backwages hereby awarded, the challenged decisions are AFFIRMED. The
dismissal of the 65 employees under the guise of retrenchment policy of the Bank is a temporary restraining orders are LIFTED. With costs against the petitioner.
lame excuse and a veritable smoke-screen of its scheme to bust the Union and thus
unduly disturb the employment tenure of the employees concerned, which act is This decision is IMMEDIATELY EXECUTORY.
certainly an unfair labor practice. 45
SO ORDERED.
Yet, at the same tune, the petitioner would claim that "the phasing out of its
operations which brought about the retrenchment of the affected employees was mainly
dictated be the necessity of its stockholders in their capacity as heirs of the late Don Vicente
Madrigal to partition the estate left by him." 46 It must be noted, however, that the labor
cases were tried on the theory of losses the petitioner was supposed to have incurred to
justify retrenchment. The petitioner cannot change its theory in the Supreme Court.
Moreover, there is nothing in the records that will substantiate this claim. But what is more
important is the fact that it is not impossible to partition the Madrigal estate assuming that

98
MATABABE FILES
Corporation Law

Republic of the Philippines identity of cause of action, the judgment is conclusive in the s econd case, only as to those
SUPREME COURT matters actually and directly controverted and determined, and not as to matters merely
Manila involved therein. This is what is termed conclusiveness of judgment.
FIRST DIVISION
Same; Same; Same; Same; Same; Intervention; A party-in-intervention cannot be
G.R. No. 117897 May 14, 1997 considered a principal in a prior case for purposes of applying the principle of res judicata since
the contrary goes against the true import of the action of intervention as a m ere subsidiary
ISLAMIC DIRECTORATE OF THE PHILIPPINES, MANUEL F. PEREA and SECURITIES & proceeding without an independent life apart from the principal action as well as the intrinsic
EXCHANGE COMMISSION, petitioners, character of the intervenor as a mere subordinate party in the main case whose right may be
vs. said to be only in aid of the right of the original party.Neither of these concepts of res
COURT OF APPEALS and IGLESIA NI CRISTO, respondents. judicata find relevant application in the case at bench. While there may be identity of subject
matter (IDP property) in both cases, there is no identity of parties. The principal parties in G.R.
Actions; Judgments; Res Judicata; Bar by Prior Judgment; Conclusiveness of Judgment; No. 107751 were mortgagee Leticia P. Ligon, as petitioner, and the Iglesia Ni Cristo, as private
Words and Phrases; Section 49, Rule 39 of the Revised Rules of Court lays down the dual respondent. The IDP, as represented by the 1971 Board of Trustees or the Tamano Group, was
aspects of res judicata in actions in personam.Section 49, Rule 39 of the Revised Rules of only made an ancillary party in G.R. No. 107751 as intervenor. It was never originally a principal
Court lays down the dual aspects of res judicata in actions in personam, to wit: Effect of party thereto. it must be noted that intervention is not an independent action, but is merely
judgment.The effect of a judgment or final order rendered by a court or judge of the collateral, accessory, or ancillary to the principal action. It is just an interlocutory proceeding
Philippines, having jurisdiction to pronounce the judgment or order, may be as follows: xxx dependent on or subsidiary to the case between the original parties. Indeed, The IDP-Tamano
xxx xxx (b) In other cases the judgment or order is, with respect to the matter directly Group cannot be considered a principal party in G.R. No. 107751 for purposes of applying the
adjudged or as to any other matter that could have been raised in relation thereto, conclusive principle of res judicata since the contrary goes against the true import of the action of
between the parties and their successors in interest by title subsequent to the intervention as a mere subsidiary proceeding without an independent life apart from the
commencement of the action or special proceeding, litigating for the same thing and under principal action as well as the intrinsic character of the intervenor as a mere subordinate party
the same title and in the same capacity; (c) In any other litigation between the same parties or in the main case whose right may be said to be only in aid of the right of the original party. It is
their succes sors in interest, that only is deemed to have been adjudged in a former judgment only in the present case, actually, where the IDP-Tamano Group became a principal party, as
which appears upon its face to have been so adjudged, or which was actually and necessarily petitioner, with the Iglesia Ni Cristo, as private res pondent. Clearly, there is no identity of
included therein or necessary thereto. Section 49(b) enunciates the first concept of res parties in both cases.
judicata known as bar by prior judgment, whereas , Section 49(c) is referred to as
conclusiveness of judgment. Same; Same; Same; Same; Same; Corporation Law; A juridical person can not be
considered essentially a formal party to a case where it was not duly represented by its
Same; Same; Same; Same; Same; Same; There is bar by former judgment when, legitimate governing board.In this connection, although it is true that Civil Case No. Q-90-
between the first case where the judgment was rendered, and the second case where the 6937, which gave rise to G.R. No. 107751, was entitled, Iglesia Ni Kristo, Plaintiff v. Islamic
judgment is invoked, there is identity of parties, subject matter and cause of action while Directorate of the Philippines, Defendant, the IDP can not be considered essentially a formal
there is conclusiveness of judgment where there is only identity of parties but there is no party thereto for the simple reason that it was not duly represented by a legitimate Board of
identity of cause of action, the judgment being conclusive in the second case only as to those Trustees in that case. As a necessary consequence, Civil Case No. Q-90-6937, a case for
matters actually and directly controverted and determined, and not as to matters merely Specific Performance with Damages, a mere action in personam , did not become final and
involved therein.There is bar by former judgment when, between the first case where the executory insofar as the true IDP is concerned since petitioner corporation, for want of
judgment was rendered, and the s econd case where such judgment is invoked, there is legitimate representation, was effectively deprived of its day in court in said case. Res inter
identity of parties, subject matter and cause of action. When the three identities are present, alios judicatae nullum aliis praejudicium faciunt. Matters adjudged in a cause do not prejudice
the judgment on the merits rendered in the first constitutes an absolute bar to the those who were not parties to it. Elsewise put, no person (natural or juridical) shall be
subsequent action. But where between the first case wherein judgment is rendered and the affected by a proceeding to which he is a stranger.
second case wherein such judgment is invoked, there is only identity of parties but there is no

99
MATABABE FILES
Corporation Law

Same; Same; Same; Same; Same; While it is true that the principle of res judicata is a m embers of the corporation, should be obtained.The Tandang Sora property, it appears
fundamental component of our judicial system, it should be disregarded if its rigid application from the records, constitutes the only property of the IDP. Hence, its sale to a third-party is a
would involve the sacrifice of justice to technicality.In any case, while it is true that the sale or disposition of all the corporate property and assets of I DP falling squarely within the
principle of res judicata is a fundamental component of our judicial system, it should be contemplation of the foregoing section. For the sale to be valid, the m a jorit y vot e of the
disregarded if its rigid application would involve the sacrifice of justice to technicality. legitimate Board of Trustees, concurred in by the vote of at least 2/3 of the bona fide
members of the corporation should have been obtained. These twin requirements were not
Corporation Law; Jurisdiction; Securities and Exchange Commission; The SEC has the met as the Carpizo Group which voted to sell the Tandang Sora property was a fake Board of
unquestionable authority to pass upon the issue as to who among the different contending Trustees , and those whose names and s ignatures were affixed by the Carpizo Group
groups is the legitimate governing board of a corporate body.There can be no question as to together with the sham Board Resolution authorizing the negotiation for the sale were, from
the authority of the SEC to pass upon the issue as to who among the different contending all indications, not bona fide members of the IDP as they were ma de to appear to be.
groups is the legitimate Board of Trustees of the IDP since this is a matter properly falling Apparently, there are only fifteen (15) official members of the petitioner corporation including
within the original and exclusive jurisdiction of the SEC by virtue of Sections 3 and 5(c) of the eight (8) members of the Board of Trustees.
Presidential Decree No. 902-A: Section 3. The Commission shall have absolute jurisdiction,
supervision and control over all corporations, partnerships or associations , who are the Same; Same; Same; Securities and Exchange Commission; Remand of Cases; No end of
grantees of primary franchises and/or a license or permit issued by the government to substantial justice will be served if the Supreme Court reverses the SECs conclusion and
operate in the Philippines x x x x x x. x x x x x x x x x Section 5. In addition to the regulatory remand the case to the regular courts for further litigation over an issue which is already
and adjudicative functions of the Securities and Exchange Commission over corpora ti ons , determinable based on what is in the records.The resolution of the question as to whether
partnerships and other forms of associations registered with it as expressly granted under or not the SEC had jurisdiction to declare the subject sale null and void is rendered moot and
existing laws and decrees, it shall have original and exclusive jurisdiction to hear and decide academic by the inherent nullity of the highly dubious sale due to lack of consent of the I DP,
cases involving: x x x x x x x x x c) Controversies in the selection or appointment of directors, owner of the subject property. No end of substantial justice will be served if we revers e the
trustees, officers, or managers of such corporations, partnerships or associations. x x x. SECs conclusion on the matter, and remand the case to the regular courts for further
litigation over an issue which is already determinable based on what we have in the records.
Same; Contracts; Sales; Where a corporate body never gave its consent, thru a
legitimate governing board, to a deed of absolute sale, the subject sale is void and produces no Land Titles; Under the Torrens System of Registration, the minimum requirement for
effect whatsoever.Premises considered, all acts carried out by the Carpizo Board, one to be a good faith buyer for value is that the vendee at least sees the owners duplicate
particularly the sale of the Tandang Sora property, allegedly in the name of the IDP, have to copy of the title and relies upon the same.Furthermore, the Court observes that the INC
be struck down for having been done without the consent of the IDP thru a legitimate Board bought the questioned property from the Carpizo Group without even seeing the owners
of Trustees. Article 1318 of the New Civil Code lays down the essential requisites of contracts: duplicate copy of the titles covering the property. This is very strange considering that the
There is no contract unless the following requisites concur: (1) Consent of the contracting subject lot is a large piece of real property in Quezon City worth millions , and that under the
parties; (2) Object certain which is the subject matter of the contract; (3) Cause of the Torrens System of Registration, the minimum requirement for one to be a good faith buyer
obligation which is established. All these elements must be present to constitute a valid for value is that the vendee at least sees the owners duplicate copy of the title and relies
contract. For, where even one is absent, the contract is void. As succinctly put by Tolentino, upon the same. The private respondent, presumably knowledgeable on the aforesaid
consent is essential for the existence of a contract, and where it is wanting, the contract is workings of the Torrens System, did not take heed of this and nevertheless went through
non-existent. In this case, the IDP, owner of the subject parcels of land, never gave its with the sale with undue haste. The unexplained eagerness of INC to buy this valuable piece
consent, thru a legitimate Board of Trustees, to the disputed Deed of Absolute Sale executed of land in Quezon City without even being presented with the owners copy of the titles casts
in favor of INC. This is, therefore, a case not only of vitiated consent, but one where consent very serious doubt on the rightfulness of its position as vendee in the transaction.
on the part of one of the supposed contracting parties is totally wanting. Ineluctably, the PETITION for review of a decision of the Court of Appeals.
subject sale is void and produces no effect whatsoever.
The facts are stated in the opinion of the Court. Islamic Directorate of the Phils. vs.
Same; Same; Same; For the sale of the only property of a corporation to be valid, the Court of Appeals, 272 SCRA 454, G.R. No. 117897 May 14, 1997
majority vote of the legitimate board, concurred in by the vote of at least 2/3 of the bona fide

100
MATABABE FILES
Corporation Law

HERMOSISIMA, JR., J.: Thereafter, two Muslim groups sprung, the Carpizo Group, headed by Engineer
Farouk Carpizo, and the Abbas Group, led by Mrs. Zorayda Tamano and Atty. Firdaussi Abbas.
The subject of this petition for review is the Decision of the public respondent Court Both groups claimed to be the legitimate IDP. Significantly, on October 3, 1986, the SEC, in a
of Appeals, 1 dated October 28, 1994, setting aside the portion of the Decision of the suit between these two contending groups, came out with a Decision in SEC Case No. 2687
Securities and Exchange Commission (SEC, for short) in SEC Case No. 4012 which declared null declaring the election of both the Carpizo Group and the Abbas Group as IDP board members
and void the sale of two (2) parcels of land in Quezon City covered by the Deed of Absolute to be null and void.
Sale entered into by and between private respondent Iglesia Ni Cristo (INC, for short) and the
Islamic Directorate of the Philippines, Inc., Carpizo Group, (IDP, for short). The dispositive portion of the SEC Decision reads:

The following facts appear of record. WHEREFORE, judgment is hereby rendered declaring the elections of
both the petitioners 7 and respondents 8 as null and void for being violative of
Petitioner IDP-Tamano Group alleges that sometime in 1971, Islamic leaders of all the Articles of Incorporation of petitioner corporation. With the nullification of
Muslim major tribal groups in the Philippines headed by Dean Cesar Adib Majul organized and the election of the respondents, the approved by-laws which they certified to
incorporated the ISLAMIC DIRECTORATE OF THE PHILIPPINES (IDP), the primary purpose of this Commission as members of the Board of Trustees must necessarily be
which is to establish an Islamic Center in Quezon City for the construction of a "Mosque likewise declared null and void. However, before any election of the members
(prayer place), Madrasah (Arabic School), and other religious infrastructures" so as to of the Board of Trustees could be conducted, there must be an approved by-
facilitate the effective practice of Islamic faith in the area. 2 laws to govern the internal government of the association including the
conduct of election. And since the election of both petitioners and
Towards this end, that is, in the same year, the Libyan government donated money to respondents have been declared null and void, a vacuum is created as to
the IDP to purchase land at Culiat, Tandang Sora, Quezon City, to be used as a Center for the who should adopt the by-laws and certify its adoption. To remedy this
Islamic populace. The land, with an area of 49,652 square meters, was covered by two titles: unfortunate situation that the association has found itself in, the members
Transfer Certificate of Title Nos. RT-26520 (176616) 3 and RT-26521 (170567), 4 both registered of the petitioning corporation are hereby authorized to prepare and adopt
in the name of IDP. their by-laws for submission to the Commission. Once approved, an election
It appears that in 1971, the Board of Trustees of the IDP was composed of the following per of the members of the Board of Trustees shall immediately be called
Article 6 of its Articles of Incorporation: pursuant to the approved by-laws.

Senator Mamintal Tamano 5 SO ORDERED. 9


Congressman Ali Dimaporo
Congressman Salipada Pendatun Neither group, however, took the necessary steps prescribed by the SEC in its
Dean Cesar Adib Majul October 3, 1986 Decision, and, thus, no valid election of the members of the Board of Trustees
Sultan Harun Al-Rashid Lucman of IDP was ever called. Although the Carpizo Group 10 attempted to submit a set of by-laws,
Delegate Ahmad Alonto the SEC found that, aside from Engineer Farouk Carpizo and Atty. Musib Buat, those who
Commissioner Datu Mama Sinsuat prepared and adopted the by-laws were not bona fide members of the IDP, thus rendering the
Mayor Aminkadra Abubakar 6 adoption of the by-laws likewise null and void.

According to the petitioner, in 1972, after the purchase of the land by the Libyan On April 20, 1989, without having been properly elected as new members of the
government in the name of IDP, Martial Law was declared by the late President Ferdinand Board of Trustee of IDP, the Carpizo Group caused to be signed an alleged Board
Marcos. Most of the members of the 1971 Board of Trustees like Senators Mamintal Tamano, Resolution 11 of the IDP, authorizing the sale of the subject two parcels of land to the private
Salipada Pendatun, Ahmad Alonto, and Congressman Al-Rashid Lucman flew to the Middle respondent INC for a consideration of P22,343,400.00, which sale was evidenced by a Deed of
East to escape political persecution. Absolute Sale 12 dated April 20, 1989.

101
MATABABE FILES
Corporation Law

On May 30, 1991, the petitioner 1971 IDP Board of Trustees headed by former Senator
Mamintal Tamano, or the Tamano Group, filed a petition before the SEC, docketed as SEC Judge Celia Lipana-Reyes of Branch 81, Regional Trial Court of Quezon City, denied
Case No. 4012, seeking to declare null and void the Deed of Absolute Sale signed by the petitioner's motion to intervene on the ground of lack of juridical personality of the IDP-
Carpizo Group and the INC since the group of Engineer Carpizo was not the legitimate Board Tamano Group and that the issues being raised by way of intervention are intra-corporate in
of Trustees of the IDP. nature, jurisdiction thereto properly pertaining to the SEC. 15

Meanwhile, private respondent INC, pursuant to the Deed of Absolute Sale executed Apprised of the pendency of SEC Case No. 4012 involving the controverted status of
in its favor, filed an action for Specific Performance with Damages against the vendor, Carpizo the IDP-Carpizo Group but without waiting for the outcome of said case, Judge Reyes, on
Group, before Branch 81 of the Regional Trial Court of Quezon City, docketed as Civil Case No. September 12, 1991, rendered Partial Judgment in Civil Case No. Q-90-6937 ordering the IDP-
Q-90-6937, to compel said group to clear the property of squatters and deliver complete and Carpizo Group to comply with its obligation under the Deed of Sale of clearing the subject lots
full physical possession thereof to INC. Likewise, INC filed a motion in the same case to of squatters and of delivering the actual possession thereof to INC. 16
compel one Mrs. Leticia P. Ligon to produce and surrender to the Register of Deeds of
Quezon City the owner's duplicate copy of TCT Nos. RT-26521 and RT-26520 covering the Thereupon, Judge Reyes in another Order, dated March 2, 1992, pertaining also to
aforementioned two parcels of land, so that the sale in INC's favor may be registered and new Civil Case No. Q-90-6937, treated INC as the rightful owner of the real properties and disposed
titles issued in the name of INC. Mrs. Ligon was alleged to be the mortgagee of the two as follows:
parcels of land executed in her favor by certain Abdulrahman R.T. Linzag and Rowaida Busran-
Sampaco claimed to be in behalf of the Carpizo Group. WHEREFORE, Leticia P. Ligon is hereby ordered to produce and/or
surrender to plaintiff 17 the owner's copy of RT-26521 (170567) and RT-26520
The IDP-Tamano Group, on June 11, 1991, sought to intervene in Civil Case No. Q-90- (176616) in open court for the registration of the Deed of Absolute Sale in
6937 averring, inter alia: the latter's name and the annotation of the mortgage executed in her favor
by herein defendant Islamic Directorate of the Philippines on the new
xxx xxx xxx transfer certificate of title to be issued to plaintiff.

2. That the Intervenor has filed a case before the Securities and Exchange SO ORDERED. 18
Commission (SEC) against Mr. Farouk Carpizo, et. al., who, through false
schemes and machinations, succeeded in executing the Deed of Sale On April 6, 1992, the above Order was amended by Judge Reyes directing Ligon "to
between the IDP and the Iglesia Ni Kristo (plaintiff in the instant case) and deliver the owner's duplicate copies of TCT Nos. RT-26521 (170567) and RT-26520 (176616) to
which Deed of Sale is the subject of the case at bar; the Register of Deeds of Quezon City for the purposes stated in the Order of March 2, 1992." 19

3. That the said case before the SEC is docketed as Case No. 04012, the main Mortgagee Ligon went to the Court of Appeals, thru a petition for certiorari,
issue of which is whether or not the aforesaid Deed of Sale between IDP and docketed as CA-G.R No. SP-27973, assailing the foregoing Orders of Judge Reyes. The
the Iglesia ni Kristo is null and void, hence, Intervenor's legal interest in the appellate court dismissed her petition on October 28, 1992. 20
instant case. A copy of the said case is hereto attached as Annex "A"; Undaunted, Ligon filed a petition for review before the Supreme Court which was docketed
as G.R. No. 107751.
4. That, furthermore, Intervenor herein is the duly constituted body which
can lawfully and legally represent the Islamic Directorate of the Philippines; In the meantime, the SEC, on July 5, 1993, finally came out with a Decision in SEC Case
xxx xxx xxx 13 No. 4012 in this wise:

Private respondent INC opposed the motion arguing, inter alia, that the issue sought 1. Declaring the by-laws submitted by the respondents 21 as unauthorized,
to be litigated by way of intervention is an intra-corporate dispute which falls under the and hence, null and void.
jurisdiction of the SEC. 14

102
MATABABE FILES
Corporation Law

2. Declaring the sale of the two (2) parcels of land in Quezon City covered by would be barred if it were the ease. Quite the contrary, the requisites or res judicata do not
the Deed of Absolute Sale entered into by Iglesia ni Kristo and the Islamic obtain in the case at bench.
Directorate of the Philippines, Inc. 22 null and void; Section 49, Rule 39 of the Revised Rules of Court lays down the dual aspects of res judicata in
actions in personam, to wit:
3. Declaring the election of the Board of Directors, 23 of the corporation from
1986 to 1991 as null and void; Effect of judgment. The effect of a judgment or final order rendered by a
court or judge of the Philippines, having jurisdiction to pronounce the
4. Declaring the acceptance of the respondents, except Farouk Carpizo and judgment or order, may be as follows:
Musnib Buat, as members of the IDP null and void.
No pronouncement as to cost. xxx xxx xxx

SO ORDERED. 24 (b) In other cases the judgment or order is, with respect to the
matter directly adjudged or as to any other matter that could have been
Private respondent INC filed a Motion for Intervention, dated September 7, 1993, in raised in relation thereto, conclusive between the parties and their
SEC Case No. 4012, but the same was denied on account of the fact that the decision of the successors in interest by title subsequent to the commencement of the
case had become final and executory, no appeal having been taken therefrom. 25 action or special proceeding, litigating for the same thing and under the
INC elevated SEC Case No. 4012 to the public respondent Court of Appeals by way of same title and in the same capacity;
a special civil action for certiorari, docketed as CA-G.R SP No. 33295. On October 28, 1994, the
court a quo promulgated a Decision in CA-G.R. SP No. 33295 granting INC's petition. The (c) In any other litigation between the same parties or their
portion of the SEC Decision in SEC Case No. 4012 which declared the sale of the two (2) lots in successors in interest, that only is deemed to have been adjudged in a
question to INC as void was ordered set aside by the Court of Appeals. former judgment which appears upon its face to have been so adjudged, or
which was actually and necessarily included therein or necessary thereto.
Thus, the IDP-Tamano Group brought the instant petition for review, dated
December 21, 1994, submitting that the Court of Appeals gravely erred in: Section 49(b) enunciates the first concept of res judicata known as "bar by prior
judgment," whereas, Section 49(c) is referred to as "conclusiveness of judgment."
1) Not upholding the jurisdiction of the SEC to declare the nullity of the sale;
2) Encouraging multiplicity of suits; and There is "bar by former judgment" when, between the first case where the judgment
3) Not applying the principles of estoppel and laches. 26 was rendered, and the second case where such judgment is invoked, there is identity of
parties, subject matter and cause of action. When the three identities are present, the
While the above petition was pending, however, the Supreme Court rendered judgment on the merits rendered in the first constitutes an absolute bar to the subsequent
judgment in G.R. No. 107751 on the petition filed by Mrs. Leticia P. Ligon. The Decision, dated action. But where between the first case wherein judgment is rendered and the second case
June 1, 1995, denied the Ligon petition and affirmed the October 28, 1992 Decision of the wherein such judgment is invoked, there is only identity of parties but there is no identity of
Court of Appeals in CA-G.R. No. SP-27973 which sustained the Order of Judge Reyes cause of action, the judgment is conclusive in the second case, only as to those matters
compelling mortgagee Ligon to surrender the owner's duplicate copies of TCT Nos. RT-26521 actually and directly controverted and determined, and not as to matters merely involved
(170567) and RT-26520 (176616) to the Register of Deeds of Quezon City so that the Deed of therein. This is what is termed "conclusiveness of judgment." 27
Absolute Sale in INC's favor may be properly registered.
Neither of these concepts of res judicata find relevant application in the case at
Before we rule upon the main issue posited in this petition, we would like to point bench. While there may be identity of subject matter (IDP property) in both cases, there is no
out that our disposition in G.R. No. 107751 entitled, "Ligon v. Court of Appeals," promulgated identity of parties. The principal parties in G.R. No. 107751 were mortgagee Leticia P. Ligon, as
on June 1, 1995, in no wise constitutes res judicata such that the petition under consideration petitioner, and the Iglesia Ni Cristo, as private respondent. The IDP, as represented by the 1971
Board of Trustees or the Tamano Group, was only made an ancillary party in G.R. No. 107751 as

103
MATABABE FILES
Corporation Law

intervenor. 28 It was never originally a principal party thereto. It must be noted that
intervention is not an independent action, but is merely collateral, accessory, or ancillary to In any case, while it is true that the principle of res judicata is a fundamental
the principal action. It is just an interlocutory proceeding dependent on or subsidiary to the component of our judicial system, it should be disregarded if its rigid application would
case between the original involve the sacrifice of justice to technicality. 34
parties. 29 Indeed, the IDP-Tamano Group cannot be considered a principal party in G.R. No.
107751 for purposes of applying the principle of res judicata since the contrary goes against The main question though in this petition is: Did the Court of Appeals commit
the true import of the action of intervention as a mere subsidiary proceeding without an reversible error in setting aside that portion of the SEC's Decision in SEC Case No. 4012 which
independent life apart from the principal action as well as the intrinsic character of the declared the sale of two (2) parcels of land in Quezon City between the IDP-Carpizo Group and
intervenor as a mere subordinate party in the main case whose right may be said to be only in private respondent INC null and void?
aid of the right of the original party. 30 It is only in the present case, actually, where the IDP- We rule in the affirmative.
Tamano Group became a principal party, as petitioner, with the Iglesia Ni Cristo, as private
respondent. Clearly, there is no identity of parties in both cases. There can be no question as to the authority of the SEC to pass upon the issue as to
who among the different contending groups is the legitimate Board of Trustees of the IDP
In this connection, although it is true that Civil Case No. Q-90-6937, which gave rise to since this is a matter properly falling within the original and exclusive jurisdiction of the SEC by
G.R. No. 107751, was entitled, "Iglesia Ni Kristo, Plaintiff v. Islamic Directorate of the Philippines, virtue of Sections 3 and 5(c) of Presidential Decree No. 902-A:
Defendant," 31 the IDP can not be considered essentially a formal party thereto for the simple
reason that it was not duly represented by a legitimate Board of Trustees in that case. As a Sec. 3. The Commission shall have absolute jurisdiction, supervision and control
necessary consequence, Civil Case No. Q-90-6937, a case for Specific Performance with over all corporations, partnership or associations, who are the grantees of
Damages, a mere action in personam, did not become final and executory insofar as the true primary franchises and/or a license or permit issued by the government to
IDP is concerned since petitioner corporation, for want of legitimate representation, was operate in the Philippines . . . .
effectively deprived of its day in court in said case. Res inter alios judicatae nullum allis
praejudicium faciunt. Matters adjudged in a cause do not prejudice those who were not xxx xxx xxx
parties to it. 32 Elsewise put, no person (natural or juridical) shall be affected by a proceeding
to which he is a stranger. 33 Sec. 5. In addition to the regulatory and adjudicative functions of the
Securities and Exchange Commission over corporations, partnerships and
Granting arguendo, that IDP may be considered a principal party in Ligon, res other forms of associations registered with it as expressly granted under
judicata as a "bar by former judgment" will still not set in on the ground that the cause of existing laws and decrees, it shall have original and exclusive jurisdiction to
action in the two cases are different. The cause of action in G.R. No. 107751 is the surrender of hear and decide cases involving:
the owner's duplicate copy of the transfer certificates of title to the rightful possessor
thereof, whereas the cause of action in the present case is the validity of the Carpizo Group- xxx xxx xxx
INC Deed of Absolute Sale.
c) Controversies in the selection or appointment of directors, trustees, officers,
Res Judicata in the form of "conclusiveness of judgment" cannot likewise apply for or managers of such corporations, partnerships or associations. . . . .
the reason that any mention at all in Ligon as to the validity of the disputed Carpizo Board-INC
sale may only be deemed incidental to the resolution of the primary issue posed in said case If the SEC can declare who is the legitimate IDP Board, then by parity of reasoning, it
which is: Who between Ligon and INC has the better right of possession over the owner's can also declare who is not the legitimate IDP Board. This is precisely what the SEC
duplicate copy of the TCTs covering the IDP property? G.R. No. 107751 cannot be considered did in SEC Case No. 4012 when it adjudged the election of the Carpizo Group to the
determinative and conclusive on the matter of the validity of the sale for this particular issue IDP Board of Trustees to be null and
was not the principal thrust of Ligon. To rule otherwise would be to cause grave and void. 35 By this ruling, the SEC in effect made the unequivocal finding that the IDP-
irreparable injustice to IDP which never gave its consent to the sale, thru a legitimate Board of Carpizo Group is a bogus Board of Trustees. Consequently, the Carpizo Group is
Trustees.

104
MATABABE FILES
Corporation Law

bereft of any authority whatsoever to bind IDP in any kind of transaction including the payment of money or other property or consideration, as its board of
the sale or disposition of ID property. directors or trustees may deem expedient, when authorized by the vote of
the stockholders representing at least two-thirds (2/3) of the outstanding
It must be noted that SEC Case No. 4012 is not the first case wherein the SEC had the capital stock; or in case of non-stock corporation, by the vote of at least two-
opportunity to pass upon the status of the Carpizo Group. As far back as October 3, 1986, the thirds (2/3) of the members, in a stockholders' or members' meeting duly called
SEC, in Case No. 2687, 36 in a suit between the Carpizo Group and the Abbas Group, already for the purpose. Written notice of the proposed action and of the time and
declared the election of the Carpizo Group (as well as the Abbas Group) to the IDP Board as place of the meeting shall be addressed to each stockholder or member at
null and void for being violative of the Articles of Incorporation. 37 Nothing thus becomes his place of residence as shown on the books of the corporation and
more settled than that the IDP-Carpizo Group with whom private respondent INC contracted deposited to the addressee in the post office with postage prepaid, or
is a fake Board. served personally: Provided, That any dissenting stockholder may exercise
his appraisal right under the conditions provided in this Code.
Premises considered, all acts carried out by the Carpizo Board, particularly the sale of
the Tandang Sora property, allegedly in the name of the IDP, have to be struck down for A sale or other disposition shall be deemed to cover substantially all
having been done without the consent of the IDP thru a legitimate Board of Trustees. Article the corporate property and assets if thereby the corporation would be
1318 of the New Civil Code lays down the essential requisites of contracts: rendered incapable of continuing the business or accomplishing the purpose
for which it was incorporated.
There is no contract unless the following requisites concur: xxx xxx xxx

(1) Consent of the contracting parties; The Tandang Sora property, it appears from the records, constitutes the only
(2) Object certain which is the subject matter of the contract; property of the IDP. Hence, its sale to a third-party is a sale or disposition of all the corporate
(3) Cause of the obligation which is established. property and assets of IDP falling squarely within the contemplation of the foregoing section.
For the sale to be valid, the majority vote of the legitimate Board of Trustees, concurred in by
All these elements must be present to constitute a valid contract. For, where the vote of at least 2/3 of the bona fide members of the corporation should have been
even one is absent, the contract is void. As succinctly put by Tolentino, consent is obtained. These twin requirements were not met as the Carpizo Group which voted to sell the
essential for the existence of a contract, and where it is wanting, the contract is non- Tandang Sora property was a fake Board of Trustees, and those whose names and signatures
existent. 38 In this case, the IDP, owner of the subject parcels of land, never gave its were affixed by the Carpizo Group together with the sham Board Resolution authorizing the
consent, thru a legitimate Board of Trustees, to the disputed Deed of Absolute Sale negotiation for the sale were, from all indications, not bona fide members of the IDP as they
executed in favor of INC. This is, therefore, a case not only of vitiated consent, but were made to appear to be.
one where consent on the part of one of the supposed contracting parties is totally
wanting. Ineluctably, the subject sale is void and produces no effect whatsoever. Apparently, there are only fifteen (15) official members of the petitioner corporation
including the eight (8) members of the Board of Trustees. 39
The Carpizo Group-INC sale is further deemed null and void ab initio because of the
Carpizo Group's failure to comply with Section 40 of the Corporation Code pertaining to the All told, the disputed Deed of Absolute Sale executed by the fake Carpizo Board and
disposition of all or substantially all assets of the corporation: private respondent INC was intrinsically void ab initio.

Sec. 40. Sale or other disposition of assets. Subject to the provisions of Private respondent INC nevertheless questions the authority of the SEC to nullify the
existing laws on illegal combinations and monopolies, a corporation may, by sale for being made outside of its jurisdiction, the same not being an intra-corporate dispute.
a majority vote of its board of directors or trustees, sell, lease, exchange,
mortgage, pledge or otherwise dispose of all or substantially all of its property The resolution of the question as to whether or not the SEC had jurisdiction to
and assets, including its goodwill, upon terms and conditions and for such declare the subject sale null and void is rendered moot and academic by the inherent nullity of
consideration, which may be money, stocks, bonds or other instruments for the highly dubious sale due to lack of consent of the IDP, owner of the subject property. No

105
MATABABE FILES
Corporation Law

end of substantial justice will be served if we reverse the SEC's conclusion on the matter, and the same was actually received by IDP. Otherwise, INC may run after Engineer Farouk Carpizo
remand the case to the regular courts for further litigation over an issue which is already and his group for the amount of money paid.
determinable based on what we have in the records.
SO ORDERED.
It is unfortunate that private respondent INC opposed the motion for intervention
filed by the 1971 Board of Trustees in Civil Case. No. Q-90-6937, a case for Specific
Performance with Damages between INC and the Carpizo Group on the subject Deed of
Absolute Sale. The legitimate IDP Board could have been granted ample opportunity before
the regional trial court to shed light on the true status of the Carpizo Board and settled the
matter as to the validity of the sale then and there. But INC, wanting to acquire the property
at all costs and threatened by the participation of the legitimate IDP Board in the civil suit,
argued for the denial of the motion averring, inter alia, that the issue sought to be litigated by
the movant is intra-corporate in nature and outside the jurisdiction of the regional trial
court. 40 As a result, the motion for intervention was denied. When the Decision in SEC Case
No. 4012 came out nullifying the sale, INC came forward, this time, quibbling over the issue
that it is the regional trial court, and not the SEC, which has jurisdiction to rule on the validity
of the sale. INC is here trifling with the courts. We cannot put a premium on this clever legal
maneuverings of private respondent which, if countenanced, would result in a failure of
justice.

Furthermore, the Court observes that the INC bought the questioned property from
the Carpizo Group without even seeing the owner's duplicate copy of the titles covering the
property. This is very strange considering that the subject lot is a large piece of real property
in Quezon City worth millions, and that under the Torrens System of Registration, the
minimum requirement for one to be a good faith buyer for value is that the vendee at least
sees the owner's duplicate copy of the title and relies upon the same. 41 The private
respondent, presumably knowledgeable on the aforesaid workings of the Torrens System, did
not take heed of this and nevertheless went through with the sale with undue haste. The
unexplained eagerness of INC to buy this valuable piece of land in Quezon City without even
being presented with the owner's copy of the titles casts very serious doubt on the
rightfulness of its position as vendee in the transaction.

WHEREFORE, the petition is GRANTED. The Decision of the public respondent Court
of Appeals dated October 28, 1994 in CA-G.R. SP No. 33295 is SET ASIDE. The Decision of the
Securities and Exchange Commission dated July 5, 1993 in SEC Case No. 4012 is REINSTATED.
The Register of Deeds of Quezon City is hereby ordered to cancel the registration of the Deed
of Absolute Sale in the name of respondent Iglesia Ni Cristo, if one has already been made. If
new titles have been issued in the name of Iglesia Ni Cristo, the Register of Deeds is hereby
ordered to cancel the same, and issue new ones in the name of petitioner Islamic Directorate
of the Philippines. Petitioner corporation is ordered to return to private respondent whatever
amount has been initially paid by INC as consideration for the property with legal interest, if

106
MATABABE FILES
Corporation Law

Republic of the Philippines organized, provided that its board of directors has been so authorized by the affirmative
SUPREME COURT vote of stockholders holding shares entitling them to exercise at least two-thirds of the
Manila voting power.
EN BANC
APPEAL from a judgment of the Court of First Instance of Manila. Gatmaitan, J.
G.R. No. L-17504 & L-17506 February 28, 1969 The facts are stated in the opinion of the Court. De la Rama vs. Ma-ao Sugar

RAMON DE LA RAMA, FRANCISCO RODRIGUEZ, HORTENCIA SALAS, PAZ SALAS and PATRIA Central Co., Inc., 27 SCRA 247, Nos. L-17504 February 28, 1969
SALAS, heirs of Magdalena Salas, as stockholders on their own behalf and for the benefit of San Juan, Africa and Benedicto for plaintiffs-appellants.
the Ma-ao Sugar Central Co., Inc., and other stockholders thereof who may wish to join in Vicente Hilado and Gianzon, Sison, Yulo and Associates for defendants-appellants.
this action, plaintiffs-appellants,
vs. CAPISTRANO, J.:
MA-AO SUGAR CENTRAL CO., INC., J. AMADO ARANETA, MRS. RAMON S. ARANETA,
ROMUALDO M. ARANETA, and RAMON A. YULO, defendants-appellants. This was a representative or derivative suit commenced on October 20, 1953, in the
Court of First Instance of Manila by four minority stockholders against the Ma-ao Sugar
Corporation Law; Investment of corporate funds in another corporation; When not Central Co., Inc. and J. Amado Araneta and three other directors of the corporation.
violative of Section of the Corporation Law.Plaintiffs-appellants contend that the
investment of corporate funds by defendants-appellants in another corporation constitutes a The complaint comprising the period November, 1946 to October, 1952, stated five
violation of section of the Corporation Law. The Supreme Court held that such an act, if causes of action, to wit: (1) for alleged illegal and ultra-vires acts consisting of self-dealing
done pursuance of the corporate purpose, does not need the approval of the stockholders; irregular loans, and unauthorized investments; (2) for alleged gross mismanagement; (3) for
but when the purchase of shares of another corporation is done solely for investment and not alleged forfeiture of corporate rights warranting dissolution; (4) for alleged damages and
to accomplish the purpose of its incorporation, the vote of approval of the stockholders is attorney's fees; and (5) for receivership.
necessary, and further states that when purpose or purposes as stated in its articles of
incorporation, the approval of the stockholders is not necessary. (Guevara, Philippine Corp. Plaintiffs prayed, in substance, as follows:
Law, 1967 ed., p. 89).
Under the FIRST CAUSE OF ACTION, that the defendant J. Amado Araneta and his
Counterclaims; Dismissal; When justified.The defendants counterclaim on the individual co-defendants be ordered to render an accounting of all transactions made and
allegation that the complaint of plaintiff was premature, improper, malicious and that the carried out by them for defendant corporation, and "to collect, produce and/or pay to the
language is unnecessarily vituperative, abusive and insulting cannot be sustained in the case defendant corporation the outstanding balance of the amounts so diverted and still unpaid to
at bar. On the contrary, the lower court found otherwise, as could be gleaned from the defendant corporation";
decision. With respect to the allegation that the complaint was abusive and insulting, there is Under the SECOND CAUSE OF ACTION, that the individual defendants be held liable and be
no finding that plaintiff had been actuated by bad faith, nor is there anything in the complaint ordered to pay to the defendant corporation "whatever amounts may be recovered by the
essentially libelous especially as the rule is that allegations in pleadings where relevant, are plaintiffs in Civil Case No. 20122, entitled 'Francisco Rodriguez vs. Ma-ao Sugar Central Co.'"; to
privileged even though they may not be clearly proved afterwards. return to the defendant corporation all amounts withdrawn by way of discretionary funds or
backpay, and to account for the difference between the corporation's crop loan accounts
Corporation Law; Investment of corporation for other corporations not similar with its payable and its crop loan accounts receivable;
business; Deemed proper by Section 17 of the Corporation Law.The lower courts order
refraining the appellant corporation from making investment in other companies whose Under the THIRD CAUSE OF ACTION, that the corporation be dissolved and its net
purpose is not connected with the sugar central business should be reversed. This is because assets be distributed to the stockholders; and
section 17 of the Corporation Law allows a corporation to invest its funds in any other
corporation or business, or for any purpose other than the main purpose for which it was

107
MATABABE FILES
Corporation Law

Under the FOURTH CAUSE OF ACTION, that the defendants be ordered "to pay the purpose is not connected with the Sugar Central business; costs of plaintiffs to be
sum of P300,000.00 by way of compensatory, moral and exemplary damages and for borne by the Corporation and J. Amado Araneta.
expenses of litigation, including attorney's fees and costs of the suit."
From this judgment both parties appealed directly to the Supreme Court.
THE FIFTH CAUSE OF ACTION was an application for the provisional remedy of
receivership. Before taking up the errors respectively, assigned by the parties, we should state that
the following findings of the Lower Court on the commission of corporate irregularities by the
In their answer originally filed on December 1, 1953, and amended on February 1, 1955, defendants have not been questioned by the defendants:
defendants denied "the allegations regarding the supposed gross mismanagement,
fraudulent use and diversion of corporate funds, disregard of corporate requirements, abuse 1. Failure to hold stockholders' meetings regularly. No stockholders' meetings were
of trust and violation of fiduciary relationship, etc., supposed to have been discovered by held in 1947, 1950 and 1951;
plaintiffs, all of which are nothing but gratuitous, unwarranted, exaggerated and distorted
conclusions not supported by plain and specific facts and transactions alleged in the 2. Irregularities in the keeping of the books. Untrue entries were made in the books
complaint." which could not simply be considered as innocent errors;
BY WAY OF SPECIAL DEFENSES, the defendants alleged, among other things: (1) that the
complaint "is premature, improper and unjustified"; (2) that plaintiffs did not make an 3. Illegal investments in the Mabuhay Printing, P2,280,00, and the Acoje Mining,
"earnest, not simulated effort" to exhaust first their remedies within the corporation before P7,000.00. The investments were made not in pursuance of the corporate purpose
filing their complaint; (3) that no actual loss had been suffered by the defendant corporation and without the requisite authority of two-thirds of the stockholders;
on account of the transactions questioned by plaintiffs; (4) that the payments by the debtors
of all amounts due to the defendant corporation constituted a full, sufficient and adequate 4. Unauthorized loans to J. Amado Araneta totalling P132,082.00 (which, according to
remedy for the grievances alleged in the complaint and (5) that the dissolution and/or the defendants, had been fully paid), in violation of the by-laws of the corporation
receivership of the defendant corporation would violate and impair the obligation of existing which prohibits any director from borrowing money from the corporation;
contracts of said corporation.
5. Diversion of corporate funds of the Ma-ao Sugar Central Co., Inc. to:
BY WAY OF COUNTERCLAIM, the defendants in substance further alleged, among
others, that the complaint was premature, improper and malicious, and that the language J. Amado Araneta & Co. P243,415.62
used was "unnecessarily vituperative abusive and insulting, particularly against defendant J.
Amado Araneta who appears to be the main target of their hatred." Wherefore, the Luzon Industrial Corp. 585,918.17
defendant sought to recover "compensation for damages, actual, moral, exemplary and
Associated Sugar 463,860.36
corrective, including reasonable attorney's fees."
General Securities 86,743.65
After trial, the Lower Court rendered its Decision (later supplemented by an Order
resolving defendants' Motion for Reconsideration), the dispositive portion of which reads: Bacolod Murcia 501,030.61
Central Azucarera del Danao 97,884.42
IN VIEW WHEREOF, the Court dismisses the petition for dissolution but condemns J.
Amado Araneta to pay unto Ma-ao Sugar Central Co., Inc. the amount of P46,270.00 Talisay-Silay 4,365.90
with 8% interest from the date of the filing of this complaint, plus the costs; the Court
reiterates the preliminary injunction restraining the Ma-ao Sugar Central Co., Inc. The Court found that sums were taken out of the funds of the Ma-ao Sugar Central
management to give any loans or advances to its officers and orders that this Co., Inc. and delivered to these affiliated companies, and vice versa, without the approval of
injunction be as it is hereby made, permanent; and orders it to refrain from making the Ma-ao Board of Directors, in violation of Sec. III, Art. 6-A of the by-laws.
investments in Acoje Mining, Mabuhay Printing, and any other company whose

108
MATABABE FILES
Corporation Law

shares in the corporation entitling them to exercise at least two-thirds of the


The errors assigned in the appeal of the plaintiffs, as appellants, are as follows: voting power on such proposal at the stockholders' meeting called for the
purpose.
I.
the Court is convinced that that law should be understood to mean as the
THE LOWER COURT ERRED IN HOLDING THAT THE INVESTMENT OF CORPORATE authorities state, that it is prohibited to the Corporation to invest in shares of
FUNDS OF THE MA-AO SUGAR CENTRAL CO., INC., IN THE PHILIPPINE FIBER another corporation unless such an investment is authorized by two-thirds of the
PROCESSING CO., INC. WAS NOT A VIOLATION OF SEC. 17- OF THE CORPORATION voting power of the stockholders, if the purpose of the corporation in which
LAW. investment is made is foreign to the purpose of the investing corporation because
surely there is more logic in the stand that if the investment is made in a corporation
II. whose business is important to the investing corporation and would aid it in its
purpose, to require authority of the stockholders would be to unduly curtail the
THE LOWER COURT ERRED IN NOT FINDING THAT THE MA-AO SUGAR CENTRAL CO., Power of the Board of Directors; the only trouble here is that the investment was
INC. WAS INSOLVENT. made without any previous authority of the Board of Directors but was only ratified
afterwards; this of course would have the effect of legalizing the unauthorized act
III. but it is an indication of the manner in which corporate business is transacted by the
Ma-ao Sugar administration, the fact that off and on, there would be passed by the
THE LOWER COURT ERRED IN HOLDING THAT THE DISCRIMINATORY ACTS Board of Directors, resolutions ratifying all acts previously done by the management,
COMMITTED AGAINST PLANTERS DID NOT CONSTITUTE MISMANAGEMENT. e.g. resolutions passed on February 25, 1947, and February 25, 1952, by the Board of
Directors as set forth in the affidavit of Isidro T. Dunca p. 127, etc. Vol. 1. (Decision, pp.
IV. 239-241 of Record on Appeal.)

THE LOWER COURT ERRED IN HOLDING THAT ITS CULPABLE ACTS WERE xxx xxx xxx
INSUFFICIENT FOR THE DISSOLUTION OF THE CORPORATION.
The portions of the Decision of the Lower Court assailed by the plaintiffs as (2) "On the other hand, the Court has noted against plaintiffs that their
appellants are as follows: contention that Ma-ao Sugar is on the verge of bankruptcy has not been clearly
shown; against this are Exh. C to Exh. C-3 perhaps the best proof that insolvency is
(1) ".... Finally, as to the Philippine Fiber, the Court takes it that defendants still far is that this action was filed in 1953 and almost seven years have passed since
admit having invested P655,000.00 in shares of stock of this company but that this then without the company apparently getting worse than it was before; ..."
was ratified by the Board of Directors in Resolutions 60 and 80, Exhibits "R" and "R- (Decision, pp. 243-244, supra.)
2"; more than that, defendants contend that since said company was engaged in the
manufacture of sugar bags it was perfectly legitimate for Ma-ao Sugar either to xxx xxx xxx
manufacture sugar bags or invest in another corporation engaged in said
manufacture, and they quote authorities for the purpose, pp. 28-31, memorandum; (3) "As to the crop loan anomalies in that instead of giving unto the planters
the Court is persuaded to believe that the defendants on this point are correct, the entire amount alloted for that, the Central withheld a certain portion for their
because while Sec. 17-1/2 of the Corporation Law provides that: own use, as can be seen in Appendix A of Exh. C-1, while the theory of plaintiffs is that
since between the amount of P3,791,551.78 the crop loan account payable, and the
No corporation organized under this act shall invest its funds in any amount of P1,708,488.22, the crop loan receivable, there is a difference of
other corporation or business or for any purpose other than the main P2,083,063.56, this would indicate that this latter sum had been used by the Central
purpose for which it was organized unless its board of directors has been so itself for its own purposes; on the other hand, defendants contend that the first
authorized in a resolution by the affirmative vote of stockholders holding amount did not represent the totality of the crop loans obtained from the Bank for

109
MATABABE FILES
Corporation Law

the purpose of relending to the planters, but that it included the Central's own credit THE LOWER COURT ERRED IN ADJUDGING J. AMADO ARANETA TO PAY TO MA-AO
line on its 40% share in the standing crop; and that this irregularity amounts to a SUGAR CENTRAL CO., INC., THE AMOUNT OF P46,270.00, WITH 8% INTEREST FROM
grievance by plaintiffs as planters and not as stockholders, the Court must find that THE DATE OF FILING OF THE COMPLAINT.
as to this count, there is really reason to find that said anomaly is not a clear basis for
the derivative suit, first, because plaintiffs' evidence is not very sufficient to prove II.
clearly the alleged diversion in the face of defendants' defense; there should have THE LOWER COURT ERRED IN NOT ORDERING THE PLAINTIFFS TO PAY THE
been a showing that the Central had no authority to make the diversion; and DEFENDANTS, PARTICULARLY J. AMADO ARANETA, THE DAMAGES PRAYED FOR IN
secondly, if the anomaly existed, there is ground to hold with defendants that it was THE COUNTERCLAIM OF SAID DEFENDANTS.
an anomaly pernicious not to the Central but to the planters; it was not even
pernicious to the stockholders. The portions of the Decision of the Lower Court assailed by the defendants as
appellants are as follows:
Going to the discriminatory acts of J. Amado Araneta, namely, manipulation
of cane allotments, withholding of molasses and alcohol shares, withholding of (1) "As to the alleged juggling of books in that the personal account of J. Amado
trucking allowance, formation of rival planters associations, refusal to deal with Araneta of P46,270.00 was closed on October 31, 1947 by charges transferred to
legitimate planters group, Exh. S; the Court notices that as to the failure to provide loans receivable nor was interest paid on this amount, the Court finds that this is
hauling transportation, this in a way is corroborated by Exh. 7, that part containing related to charge No. 1, namely, the granting of personal loans to J. Amado Araneta;
the decision of the Court of First Instance of Manila, civil 20122, Francisco Rodriguez v. it is really true that according to the books, and as admitted by defendants, J. Amado
Ma-ao Sugar; for the reason, however, that even if these were true, those grievances Araneta secured personal loans; in 1947, the cash advance to him was P132,082.00
were grievances of plaintiffs as planters and not as stockholders just as the (Exh. A); the Court has no doubt that this was against the By-Laws which provided
grievance as to the crop loans already adverted to, this Court will find insufficient that:
merit on this count. (Decision, pp. 230-231, supra.)
The Directors shall not in any case borrow money from the Company. (Sec.
xxx xxx xxx III, Art. 7);

(4) "...; for the Court must admit its limitations and confess that it cannot pretend to the Court therefore finds this count to be duly proved; worse, the Court also
know better than the Board in matters where the Board has not transgressed any finds that as plaintiffs contend, while the books of the Corporation would show that
positive statute or by-law especially where as here, there is the circumstance that the last balance of P46,270.00 was written off as paid, as testified to by Auditor Mr.
presumably, an impartial representative in the Board of Directors, the one from Sanchez, the payment appeared to be nothing more than a transfer of his loan
the Philippine National Bank, against whom apparently plaintiffs have no quarrel, receivable account, stated otherwise, the item was only transferred from the
does not appear to have made any protest against the same; the net result will be to personal account to the loan receivable account, so that again the Court considers
hold that the culpable acts proved are not enough to secure a dissolution; the Court established the juggling of the books; and then again, it is also true that the loans
will only order the correction of abuses, proved as already mentioned; nor will the were secured without any interest and while it is true that in the Directors' meeting
Court grant any more damages one way or the other. (Decision, p. 244, supra.) of 21 October, 1953, it was resolved to collect 8%, the Court does not see how such a
unilateral action of the Board could bind the borrowers. Be it stated that defendants
On the other hand, the errors assigned in the appeal of the defendants as appellants have presented in evidence Exh. 5 photostatic copy of the page in loan receivable
are as follows: and it is sought to be proved that J. Amado Araneta's debt was totally paid on 31
October, 1953; to the Court, in the absence of definite primary proof of actual
I. payment having found out that there had already been a juggling of books, it cannot
just believe that the amount had been paid as noted in the books. (Decision, pp. 233-
235 of Record on Appeal.)

110
MATABABE FILES
Corporation Law

(2) "With respect to the second point in the motion for reconsideration to N.W. 780, 158 N.W. 820, 159 N.W. 564, Ann. Cas. 1918 E 420; Mitchell v. Bank of St.
the effect that the Court did not make any findings of fact on the counterclaim of Paul, 7 Minn. 252).
defendants, although the Court did not say that in so many words, the Court takes it
that its findings of fact on pages 17 to 21 of its decision were enough to justify a The First Assignment of Error in the brief of the plaintiffs as appellants, contending
dismissal of the counterclaim, because the counterclaims were based on the fact that that the investment of corporate funds by the Ma-ao Sugar Co., Inc., in another corporation
the complaint was premature, improper, malicious and that the language is (the Philippine Fiber Processing Co., Inc.) constitutes a violation of Sec. 17- of the
unnecessarily vituperative abusive and insulting; but the Court has not found that the Corporation Law, deserves consideration.
complaint is premature; nor has the Court found that the complaint was malicious;
these findings can be gleaned from the decision with respect to the allegation that Plaintiffs-appellants contend that in 1950 the Ma-ao Sugar Central Co., Inc., through
the complaint was abusive and insulting, the Court does not concur; for it has not its President, J. Amado Araneta,, subscribed for P300,000.00 worth of capital stock of the
seen anything in the evidence that would justify a finding that plaintiffs and been Philippine Fiber Processing Co. Inc., that payments on the subscription were made on
actuated by bad faith, nor is there anything in the complaint essentially libelous; September 20, 1950, for P150,000.00, on April 30, 1951, for P50,000.00, and on March 6, 1952,
especially as the rule is that allegations in pleading where relevant, are privileged for P100,000.00; that at the time the first two payments were made there was no board
even though they may not clearly proved afterwards; so that the Court has not seen resolution authorizing the investment; and that it was only on November 26, 1951, that the
any merit in the counterclaims; and the Court had believed that the decision already President of Ma-ao Sugar Central Co., Inc., was so authorized by the Board of Directors.
carried with it the implication of the dismissal of the counterclaims, but if that is not
enough, the Court makes its position clear on this matter in this order, and clarifies In addition, 355,000 shares of stock of the same Philippine Fiber Processing Co., Inc.,
that it has dismissed the counterclaims of defendant; ..." (Order of September 3, owned by Luzon Industrial, corporation were transferred on May 31, 1952, to the defendant
1960, pp. 248-249, supra.) Ma-ao Sugar Central Co., Inc., with a valuation of P355,000.00 on the basis of P1.00 par value
per share. Again the "investment" was made without prior board resolution, the authorizing
Regarding Assignment of Errors Nos. 2, 3 and 4 contained in the brief of the plaintiffs resolution having been subsequentIy approved only on June 4, 1952.
as appellants, it appears to us that the Lower Court was correct in its appreciation (1) that the
evidence presented did not show that the defendant Ma-ao Sugar Company was insolvent (2) Plaintiffs-appellants also contend that even assuming, arguendo, that the said Board
that the alleged discriminatory acts committed by the defendant Central against the planters Resolutions are valid, the transaction, is still wanting in legality, no resolution having been
were not a proper subject of derivative suit, but, at most, constituted a cause of action of the approved by the affirmative vote of stockholders holding shares in the corporation entitling
individual planters; and (3) that the acts of mismanagement complained of and proved do not them to exercise at least two-thirds of the voting power, as required in Sec. 17- of the
justify a dissolution of the corporation. Corporation Law.

Whether insolvency exists is usually a question of fact, to be determined The legal provision invoked by the plaintiffs, as appellants, Sec. 17- of the
from an inventory of the assets and their value, as well as a consideration of the Corporation Law, provides:
liabilities.... But the mere impairment of capital stock alone does not establish
insolvency there being other evidence as to the corporation being a going concern with No corporation organized under this act shall invest its funds in any other
sufficient assets. Also, the excess of liabilities over assets does not establish insolvency, corporation or business, or for any purpose other than the main purpose for which it
when other assets are available. (Fletcher Cyc. of the Law of Private Corporations, Vol. was organized, unless its board of directors has been so authorized in a resolution by
15A, 1938 Ed pp. 34-37; Emphasis supplied). the affirmative vote of stockholders holding shares in the corporation entitling them
to exercise at least two-thirds of the voting power on such proposal at a
But relief by dissolution will be awarded in such cases only where no other stockholders' meeting called for the purpose ....
adequate remedy is available, and is not available where the rights of the
stockholders can be, or are, protected in some other way. (16 Fletcher Cyc. On the other hand, the defendants, as appellees, invoked Sec. 13, par. 10 of the
Corporations, 1942 Ed., pp. 812-813, citing "Thwing v. McDonald", 134 Minn. 148, 156 Corporation Law, which provides:

111
MATABABE FILES
Corporation Law

SEC. 13. Every corporation has the power: other than the main purpose for which it was organized, provided that 'its board of
directors has been so authorized in a resolution by the affirmative vote of
xxx xxx xxx stockholders holding shares in the corporation entitling them to exercise at least
two-thirds of the voting power on such a proposal at a stockholders' meeting called
(9) To enter into any obligation or contract essential to the proper administration of for that purpose,' and provided further, that no agricultural or mining corporation
its corporate affairs or necessary for the proper transaction of the business or shall in anywise be interested in any other agricultural or mining corporation. When
accomplishment of the purpose for which the corporation was organized; the investment is necessary to accomplish its purpose or purposes as stated in it articles
of incorporation, the approval of the stockholders is not necessary. (Id., p. 108.)
(10) Except as in this section otherwise provided, and in order to accomplish its (Emphasis ours.)
purpose as stated in the articles of incorporation, to acquire, hold, mortgage, pledge
or dispose of shares, bonds, securities and other evidences of indebtedness of any We agree with Professor Guevara.
domestic or foreign corporation.
We therefore agree with the finding of the Lower Court that the investment in
A reading of the two afore-quoted provisions shows that there is need for question does not fall under the purview of Sec. 17- of the Corporation Law.
interpretation of the apparent conflict.
With respect to the defendants' assignment of errors, the second (referring to the
In his work entitled "The Philippine Corporation Law," now in its 5th edition, counterclaim) is clearly without merit. As the Lower Court aptly ruled in its Order of
Professor Sulpicio S. Guevara of the University of the Philippines, College of Law, a well- September 3, 1960 (resolving the defendants' Motion for Reconsideration) the findings of
known authority in commercial law, reconciled these two apparently conflicting legal fact were enough to justify a dismissal of the counterclaim, "because the counterclaims were
provisions, as follows: based on the fact that the complaint was premature, improper, malicious and that the
language is unnecessarily vituperative abusive and insulting; but the Court has not found that
j. Power to acquire or dispose of shares or securities. A private corporation, the complaint is premature; nor has the Court found that the complaint was malicious; these
in order to accomplish its purpose as stated in its articles of incorporation, and findings can be gleaned from the decision; with respect to the allegation that the complaint
subject to the limitations imposed by the Corporation Law, has the power to acquire, was abusive and insulting, the Court does not concur; for it has not seen anything in the
hold, mortgage, pledge or dispose of shares, bonds, securities, and other evidences evidence that would justify a finding that plaintiffs had been actuated by bad faith, nor is
of indebtedness of any domestic or foreign corporation. Such an act, if done in there anything in the complaint essentially libelous especially as the rule is that allegations in
pursuance of the corporate purpose, does not need the approval of the stockholders; pleadings where relevant, are privileged even though they may not be clearly proved
but when the purchase of shares of another corporation is done solely for investment afterwards; ..."
and not to accomplish the purpose of its incorporation, the vote of approval of the
stockholders is necessary. In any case, the purchase of such shares or securities must As regards defendants' first assignment of error, referring to the status of the
be subject to the limitations established by the Corporation Law; namely, (a) that no account of J. Amado Araneta in the amount of P46,270.00, this Court likewise agrees with the
agricultural or mining corporation shall in anywise be interested in any other finding of the Lower Court that Exhibit 5, photostatic copy of the page on loans receivable
agricultural or mining corporation; or (b) that a non-agricultural or non-mining does not constitute definite primary proof of actual payment, particularly in this case where
corporation shall be restricted to own not more than 15% of the voting stock of any there is evidence that the account in question was transferred from one account to another.
agricultural or mining corporation; and (c) that such holdings shall be solely for There is no better substitute for an official receipt and a cancelled check as evidence of
investment and not for the purpose of bringing about a monopoly in any line of payment.
commerce or combination in restraint of trade. (The Philippine Corporation Law by
Sulpicio S. Guevara, 1967 Ed., p. 89.) (Emphasis ours.) In the judgment, the lower court ordered the management of the Ma-ao Sugar
Central Co., Inc. "to refrain from making investments in Acoje Mining, Mabuhay Printing and
40. Power to invest corporate funds. A private corporation has the power any other company whose purpose is not connected with the sugar central business." This
to invest its corporate funds in any other corporation or business, or for any purpose portion of the decision should be reversed because, Sec. 17- of the Corporation Law allows a

112
MATABABE FILES
Corporation Law

corporation to "invest its fund in any other corporation or business, or for any purpose other
than the main purpose for which it was organized," provided that its board of directors has
been so authorized by the affirmative vote of stockholders holding shares entitling them to
exercise at least two-thirds of the voting power.

IN VIEW OF ALL THE FOREGOING, that part of the judgment which orders the Ma-ao
Sugar Central Co., Inc. "to refrain from making investments in Acoje Mining, Mabuhay
Printing, and any other: company whose purpose is not connected with the sugar central
business," is reversed. The other parts of the judgment are, affirmed. No special
pronouncement as to costs.

Concepcion, C.J., Reyes, J.B.L., Dizon, Zaldivar, Castro, Fernando and Barredo, JJ., concur.
Makalintal, Sanchez and Teehankee, JJ., took no part.

113
MATABABE FILES
Corporation Law

Republic of the Philippines Corporation law; Shares of stock; Consideration for which shares of stock may be
SUPREME COURT issued; A share of stock coming from stock dividends declared cannot be issued to one who is
Manila not a stockholder of a corporation.From the provision of Section 16 of the Corporation Law,
EN BANC the consideration for which shares of stock may be issued are: (1) cash; (2) property; and (3)
undistributed profits. Shares of stock are given the special name "stock dividends" only if they
G.R. No. L-21601 December 28, 1968 are issued in lieu of undistributed profits. If shares of stocks are issued in exchange of cash or
property then those shares do not fall under the category of "stock dividends". A corporation
NIELSON & COMPANY, INC., plaintiff-appellant, may legally issue shares of stock in consideration of services rendered to it by a person not a
vs. stockholder, or in payment of its indebtedness. It is the shares of stock ,that are originally
LEPANTO CONSOLIDATED MINING COMPANY, defendant-appellee. issued by the corporation and forming part of the capital that can be exchanged for cash or
services rendered, or property; that is, if the corporation has original shares of stock unsold or
Pleading and practice; Appeal; Change of theory on appeal not allowable.It is the unsubscribed, either coming from the original capitalization or f rom the increased
rule, and the settled doctrine, that a party cannot change his theory on appealthat is, that a capitalization. Those shares of stock may be issued to a person who is not a stockholder, or to
party cannot raise in the appellate court any question of law or of fact that was not raised in a person already a stockholder in exchange for services rendered or for cash or property. But
the court below or which was not within the issue made by the parties in their pleadings (Sec. a share of stock coming from stock dividends declared cannot be issued to one who is not a
19, Rule 49, old Rules of Court; Sec. 18 of Revised Rules of Court; Hautea v. Magallon, L-20345, stockholder of a corporation.
Nov. 28, 1964; Northern Motors, Inc. v. Prince Line, L-13884, Feb. 29, 1960; American Express
Co. v. Natividad, 46 Phil. 207; Agoncillo v. Javier, 38 Phil. 424; Molina v. Somes, 24 Phil. 49). Under Section 16 of the Corporation Law stock dividends can not be issued to a
person who is not a stockholder in payment of services rendered.
Civil Law; Contracts; "Agency" and "lease of service" compared and distinguished.In
both agency and lease of services one of the parties binds himself to render some service to Same; "Stock dividend"; "Dividend"; Concept and nature.A "stock dividend" is any
the other party. Agency, however, is distinguished from lease of work or services in that the dividend payable in shares of stock of the corporation declaring or authorizing such dividend.
basis of agency is representation, while in the lease of work or services the basis is It is, as what the term itself implies, a distribution of the shares of stock of the corporation
employment. The lessor of services does not represent his employer, while the agent among the stockholders as dividends. A stock dividend of a corporation is a dividend paid in
represents his principal. Agency is a preparatory contract, as agency "does not stop with the shares of stock instead of cash, and is properly payable only out of surplus profits (Sec. 16,
agency because the purpose is to enter into other contracts." The most characteristic feature Corporation Law). So, a stock dividend is actually two things: (1) a dividend, and (2) the
of an agency relationship is the agent's power to bring about business relations between his enforced use of the dividend money to purchase additional shares of stock at par. (Words and
principal and third persons. "The agent is destined to execute juridical acts (creation, Phrases, p. 270). When a corporation issues stock dividends, it shows that the corporation's
modification or extinction of relations with .third parties). Lease of services contemplate only accumulated profits have been capitalized instead of distributed to the stockholders or
material (non-juridical) acts." (Reyes & Puno, An Outline of Philippine Civil Law, Vol. V, p. 277). retained as surplus available for distribution, in money or kind, should opportunity offer. Far
from being a realization of profits for the stockholder, it tends rather ,to postpone said
Same; Obligations and contracts; Moratorium law; Republic Act No. 342 not applicable realization, in ,that the fund represented by the new stock has been transferred from surplus
to debts contracted during the war.Republic Act No. 342 does not apply to debts contracted to assets and no longer available for actual distribution (Fisher v. Trinidad, 43 Phil. 973). Thus,
during the war and did not lift the moratorium in relation thereto (Uy v. Kalaw Katigbak, L- it is apparent that stock dividends are issued only to stockholders. This is so because only
1830, Dec. 31, 1949; Sison v. Mirasol, L-4711, Oct. 3, 1952; Compaia Maritima v. Court of stockholders are entitled to dividends. They are the only ones who have a right to a
Appeals, L-14949, May 30, 1960). Said Rep. Act No. 342, however, modified Executive Order proportional share in that part of the surplus which is declared as dividends. A stock dividend
No. 32 as to pre-war debts, making the protection available only to debtors who had war really adds nothing to the interest of the stockholder; the proportional interest of each
damage claims (Sison v. Mirasol, L-4711, Oct. 3, 1952, cited in Abraham, et al. v. Intestate Estate stockholder remains the same (Towne v. Eisner, 62 L. Ed. 372). If a stockholder is deprived of
of Ysmael, et al., L-16741, Jan. 31, 1962). his stock dividendsand this happens if the shares of stock f orming part of the stock
dividends are issued to a nonstockholderthen the proportion of the stockholder's interest
changes radically. Stock dividends are civil fruits of the original investment, and to the owners

114
MATABABE FILES
Corporation Law

of the shares belong the civil fruits (Art. 441, Civil Code). The term "dividend" both in the 5. The court erred in holding that the period of suspension of the contract on account
technical sense and its ordinary acceptation, is that part or portion of the profits of the of the war lasted from February 1942 to June 26, 1948.
enterprise which the corporation, by its governing agents, sets apart for ratable division
among the holders of the capital stock. It means the fund actually set aside, and declared by 6. Assuming arguendo that Nielson is entitled to any relief, the court erred in
the directors of the corporation as a dividend, and duly ordered by the directory, or by the awarding as damages (a) 10% of the cash dividends declared and paid in December,
stockholders, at a corporate meeting, to be divided or distributed among the stockholders 1941; (b) the management fee of P2,500.00 for the month of January, 1942; and (c)
according to their respective interests (7 Thompson on Corporations 134135). the full contract price for the extended period of sixty months, since these damages
were neither demanded nor proved and, in any case, not allowable under the general
MOTION FOR RECONSIDERATION of a Supreme Court decision. law of damages.

The facts are stated in the resolution of the Court. 7. Assuming arguendo that appellant is entitled to any relief, the court erred in
ordering appellee to issue and deliver to appellant shares of stock together with
RESOLUTION fruits thereof.
ZALDIVAR, J.:
8. The court erred in awarding to appellant an undetermined amount of shares of
Lepanto seeks the reconsideration of the decision rendered on December 17, 1966. stock and/or cash, which award cannot be ascertained and executed without further
The motion for reconsideration is based on two sets of grounds the first set consisting of litigation.
four principal grounds, and the second set consisting of five alternative grounds, as follows:
9. The court erred in rendering judgment for attorney's fees.
Principal Grounds:
We are going to dwell on these grounds in the order they are presented.
1. The court erred in overlooking and failing to apply the proper law applicable to the
agency or management contract in question, namely, Article 1733 of the Old Civil 1. In its first principal ground Lepanto claims that its own counsel and this Court had
Code (Article 1920 of the new), by virtue of which said agency was effectively overlooked the real nature of the management contract entered into by and between
revoked and terminated in 1945 when, as stated in paragraph 20 of the complaint, Lepanto and Nielson, and the law that is applicable on said contract. Lepanto now asserts for
"defendant voluntarily ... prevented plaintiff from resuming management and the first time and this is done in a motion for reconsideration - that the management contract
operation of said mining properties." in question is a contract of agency such that it has the right to revoke and terminate the said
contract, as it did terminate the same, under the law of agency, and particularly pursuant to
2. The court erred in holding that paragraph II of the management contract (Exhibit Article 1733 of the Old Civil Code (Article 1920 of the New Civil Code).
C) suspended the period of said contract.
3. The court erred in reversing the ruling of the trial judge, based on well-settled We have taken note that Lepanto is advancing a new theory. We have carefully
jurisprudence of this Supreme Court, that the management agreement was only examined the pleadings filed by Lepanto in the lower court, its memorandum and its brief on
suspended but not extended on account of the war. appeal, and never did it assert the theory that it has the right to terminate the management
contract because that contract is one of agency which it could terminate at will. While it is
4. The court erred in reversing the finding of the trial judge that Nielson's action had true that in its ninth and tenth special affirmative defenses, in its answer in the court below,
prescribed, but considering only the first claim and ignoring the prescriptibility of the Lepanto pleaded that it had the right to terminate the management contract in question, that
other claims. plea of its right to terminate was not based upon the ground that the relation between
Lepanto and Nielson was that of principal and agent but upon the ground that Nielson had
Alternative Grounds: allegedly not complied with certain terms of the management contract. If Lepanto had
thought of considering the management contract as one of agency it could have amended its
answer by stating exactly its position. It could have asserted its theory of agency in its

115
MATABABE FILES
Corporation Law

memorandum for the lower court and in its brief on appeal. This, Lepanto did not do. It is the
rule, and the settled doctrine of this Court, that a party cannot change his theory on appeal In both agency and lease of services one of the parties binds himself to render some
that is, that a party cannot raise in the appellate court any question of law or of fact that was service to the other party. Agency, however, is distinguished from lease of work or services in
not raised in the court below or which was not within the issue made by the parties in their that the basis of agency is representation, while in the lease of work or services the basis is
pleadings (Section 19, Rule 49 of the old Rules of Court, and also Section 18 of the new Rules employment. The lessor of services does not represent his employer, while the agent
of Court; Hautea vs. Magallon, L-20345, November 28, 1964; Northern Motors, Inc. vs. Prince represents his principal. Manresa, in his "Commentarios al Codigo Civil Espaol" (1931, Tomo
Line, L-13884, February 29, 1960; American Express Co. vs. Natividad, 46 Phil. 207; Agoncillo vs. IX, pp. 372-373), points out that the element of representation distinguishes agency from
Javier, 38 Phil. 424 and Molina vs. Somes, 24 Phil 49). lease of services, as follows:

At any rate, even if we allow Lepanto to assert its new theory at this very late stage Nuestro art. 1.709 como el art. 1.984 del Codigo de Napoleon y cuantos textos legales
of the proceedings, this Court cannot sustain the same. citamos en las concordancias, expresan claramente esta idea de la representacion,
"hacer alguna cosa por cuenta o encargo de otra" dice nuestro Codigo; "poder de
Lepanto contends that the management contract in question (Exhibit C) is one of hacer alguna cosa para el mandante o en su nombre" dice el Codigo de Napoleon, y
agency because: (1) Nielson was to manage and operate the mining properties and mill on en tales palabras aparece vivo y luminoso el concepto y la teoria de la representacion,
behalf, and for the account, of Lepanto; and (2) Nielson was authorized to represent Lepanto tan fecunda en ensenanzas, que a su sola luz es como se explican las diferencias que
in entering, on Lepanto's behalf, into contracts for the hiring of laborers, purchase of separan el mandato del arrendamiento de servicios, de los contratos inominados, del
supplies, and the sale and marketing of the ores mined. All these, Lepanto claims, show that consejo y de la gestion de negocios.
Nielson was, by the terms of the contract, destined to execute juridical acts not on its own En efecto, en el arrendamiento de servicios al obligarse para su ejecucion, se trabaja,
behalf but on behalf of Lepanto under the control of the Board of Directors of Lepanto "at all en verdad, para el dueno que remunera la labor, pero ni se le representa ni se obra en
times". Hence Lepanto claims that the contract is one of agency. Lepanto then maintains that su nombre....
an agency is revocable at the will of the principal (Article 1733 of the Old Civil Code),
regardless of any term or period stipulated in the contract, and it was in pursuance of that On the basis of the interpretation of Article 1709 of the old Civil Code, Article 1868 of
right that Lepanto terminated the contract in 1945 when it took over and assumed exclusive the new Civil Code has defined the contract of agency in more explicit terms, as follows:
management of the work previously entrusted to Nielson under the contract. Lepanto finally By the contract of agency a person binds himself to render some service or
maintains that Nielson as an agent is not entitled to damages since the law gives to the to do something in representation or on behalf of another, with the consent or
principal the right to terminate the agency at will. authority of the latter.

Because of Lepanto's new theory We consider it necessary to determine There is another obvious distinction between agency and lease of services. Agency is
the nature of the management contract whether it is a contract of agency or a contract of a preparatory contract, as agency "does not stop with the agency because the purpose is to
lease of services. Incidentally, we have noted that the lower court, in the decision appealed enter into other contracts." The most characteristic feature of an agency relationship is the
from, considered the management contract as a contract of lease of services. agent's power to bring about business relations between his principal and third persons. "The
agent is destined to execute juridical acts (creation, modification or extinction of relations
Article 1709 of the Old Civil Code, defining contract of agency, provides: with third parties). Lease of services contemplate only material (non-juridical) acts." (Reyes
and Puno, "An Outline of Philippine Civil Law," Vol. V, p. 277).
By the contract of agency, one person binds himself to render some service or do
something for the account or at the request of another. In the light of the interpretations we have mentioned in the foregoing paragraphs let
us now determine the nature of the management contract in question. Under the contract,
Article 1544, defining contract of lease of service, provides: Nielson had agreed, for a period of five years, with the right to renew for a like period, to
explore, develop and operate the mining claims of Lepanto, and to mine, or mine and mill,
In a lease of work or services, one of the parties binds himself to make or construct such pay ore as may be found therein and to market the metallic products recovered
something or to render a service to the other for a price certain. therefrom which may prove to be marketable, as well as to render for Lepanto other services

116
MATABABE FILES
Corporation Law

specified in the contract. We gather from the contract that the work undertaken by Nielson old Civil Code. We find, however, a proviso in the management contract which militates
was to take complete charge subject at all times to the general control of the Board of against this stand of Lepanto. Paragraph XI of the contract provides:
Directors of Lepanto, of the exploration and development of the mining claims, of the hiring
of a sufficient and competent staff and of sufficient and capable laborers, of the prospecting Both parties to this agreement fully recognize that the terms of this
and development of the mine, of the erection and operation of the mill, and of the Agreement are made possible only because of the faith or confidence that the
benefication and marketing of the minerals found on the mining properties; and in carrying Officials of each company have in the other; therefore, in order to assure that such
out said obligation Nielson should proceed diligently and in accordance with the best mining confidence and faith shall abide and continue, NIELSON agrees that LEPANTO may
practice. In connection with its work Nielson was to submit reports, maps, plans and cancel this Agreement at any time upon ninety (90) days written notice, in the event
recommendations with respect to the operation and development of the mining properties, that NIELSON for any reason whatsoever, except acts of God, strike and other causes
make recommendations and plans on the erection or enlargement of any existing mill, beyond its control, shall cease to prosecute the operation and development of the
dispatch mining engineers and technicians to the mining properties as from time to time may properties herein described, in good faith and in accordance with approved mining
reasonably be required to investigate and make recommendations without cost or expense to practice.
Lepanto. Nielson was also to "act as purchasing agent of supplies, equipment and other
necessary purchases by Lepanto, provided, however, that no purchase shall be made without It is thus seen, from the above-quoted provision of paragraph XI of the management
the prior approval of Lepanto; and provided further, that no commission shall be claimed or contract, that Lepanto could not terminate the agreement at will. Lepanto could terminate or
retained by Nielson on such purchase"; and "to submit all requisition for supplies, all cancel the agreement by giving notice of termination ninety days in advance only in the event
constricts and arrangement with engineers, and staff and all matters requiring the that Nielson should prosecute in bad faith and not in accordance with approved mining
expenditures of money, present or future, for prior approval by Lepanto; and also to make practice the operation and development of the mining properties of Lepanto. Lepanto could
contracts subject to the prior approve of Lepanto for the sale and marketing of the minerals not terminate the agreement if Nielson should cease to prosecute the operation and
mined from said properties, when said products are in a suitable condition for marketing."1 development of the mining properties by reason of acts of God, strike and other causes
beyond the control of Nielson.
It thus appears that the principal and paramount undertaking of Nielson under the
management contract was the operation and development of the mine and the operation of The phrase "Both parties to this agreement fully recognize that the terms of this
the mill. All the other undertakings mentioned in the contract are necessary or incidental to agreement are made possible only because of the faith and confidence of the officials of each
the principal undertaking these other undertakings being dependent upon the work on the company have in the other" in paragraph XI of the management contract does not qualify the
development of the mine and the operation of the mill. In the performance of this principal relation between Lepanto and Nielson as that of principal and agent based on trust and
undertaking Nielson was not in any way executing juridical acts for Lepanto, destined to confidence, such that the contractual relation may be terminated by the principal at any time
create, modify or extinguish business relations between Lepanto and third persons. In other that the principal loses trust and confidence in the agent. Rather, that phrase simply implies
words, in performing its principal undertaking Nielson was not acting as an agent of Lepanto, the circumstance that brought about the execution of the management contract. Thus, in the
in the sense that the term agent is interpreted under the law of agency, but as one who was annual report for 19362, submitted by Mr. C. A. Dewit, President of Lepanto, to its
performing material acts for an employer, for a compensation. stockholders, under date of March 15, 1937, we read the following:

It is true that the management contract provides that Nielson would also act as To the stockholders
purchasing agent of supplies and enter into contracts regarding the sale of mineral, but the
contract also provides that Nielson could not make any purchase, or sell the minerals, without xxx xxx xxx
the prior approval of Lepanto. It is clear, therefore, that even in these cases Nielson could not
execute juridical acts which would bind Lepanto without first securing the approval of The incorporation of our Company was effected as a result of negotiations
Lepanto. Nielson, then, was to act only as an intermediary, not as an agent. with Messrs. Nielson & Co., Inc., and an offer by these gentlemen to Messrs. C. I.
Cookes and V. L. Lednicky, dated August 11, 1936, reading as follows:
Lepanto contends that the management contract in question being one of agency it
had the right to terminate the contract at will pursuant to the provision of Article 1733 of the

117
MATABABE FILES
Corporation Law

Messrs. Cookes and Lednicky, the proposed corporation, and, in case you accept this proposition, that a
Present detailed operating contract will be entered into, covering the relationships
Re: Mankayan Copper Mines between the parties.

GENTLEMEN: Yours very truly,


(Sgd.) L. R. Nielson
After an examination of your property by our engineers, we have decided to
offer as we hereby offer to underwrite the entire issue of stock of a Pursuant to the provisions of paragraph 2 of this offer, Messrs. Nielson &
corporation to be formed for the purpose of taking over said properties, said Co., took subscriptions for One Million Fifty Thousand Pesos (P1,050,000.00) in
corporation to have an authorized capital of P1,750,000.00, of which shares of our Company and their underwriting and brokerage commission has been
P700,000.00 will be issued in escrow to the claim-owners in exchange for paid. More than fifty per cent of these subscriptions have been paid to the Company
their claims, and the balance of P1,050,000.00 we will sell to the public at par in cash. The claim owners have transferred their claims to the Corporation, but the
or take ourselves. P700,000.00 in stock which they are to receive therefor, is as yet held in escrow.
The arrangement will be under the following conditions:
Immediately upon the formation of the Corporation Messrs. Nielson & Co.,
1. The subscriptions for cash shall be payable 50% at time of subscription and assumed the Management of the property under the control of the Board of
the balance subject to the call of the Board of Directors of the proposed Directors. A modification in the Management Contract was made with the consent of
corporation. all the then stockholders, in virtue of which the compensation of Messrs. Nielson &
Co., was increased to P2,500.00 per month when mill construction began. The formal
2. We shall have an underwriting and brokerage commission of 10% of the Management Contract was not entered into until January 30, 1937.
P1,050,000.00 to be sold for cash to the public, said commission to be
payable from the first payment of 50% on each subscription. xxx xxx xxx

3. We will bear the cost of preparing and mailing any prospectus that may be Manila, March 15, 1937
required, but no such prospectus will be sent out until the text thereof has (Sgd.) C. A. DeWitt President
been first approved by the Board of Directors of the proposed corporation.
We can gather from the foregoing statements in the annual report for 1936, and from
4. That after the organization of the corporation, all operating contract be the provision of paragraph XI of the Management contract, that the employment by Lepanto
entered into between ourselves and said corporation, under the terms which of Nielson to operate and manage its mines was principally in consideration of the know-how
the property will be developed and mined and a mill erected, under our and technical services that Nielson offered Lepanto. The contract thus entered into pursuant
supervision, our compensation to be P2,000.00 per month until the property to the offer made by Nielson and accepted by Lepanto was a "detailed operating contract". It
is put on a profitable basis and P2,500.00 per month plus 10% of the net was not a contract of agency. Nowhere in the record is it shown that Lepanto considered
profits for a period of five years thereafter. Nielson as its agent and that Lepanto terminated the management contract because it had
lost its trust and confidence in Nielson.
5. That we shall have the option to renew said operating contract for an
additional period of five years, on the same basis as the original contract, The contention of Lepanto that it had terminated the management contract in 1945,
upon the expiration thereof. following the liberation of the mines from Japanese control, because the relation between it
and Nielson was one of agency and as such it could terminate the agency at will, is, therefore,
It is understood that the development and mining operations on said untenable. On the other hand, it can be said that, in asserting that it had terminated or
property, and the erection of the mill thereon, and the expenditures cancelled the management contract in 1945, Lepanto had thereby violated the express terms
therefor shall be subject to the general control of the Board of Directors of of the management contract. The management contract was renewed to last until January 31,

118
MATABABE FILES
Corporation Law

1947, so that the contract had yet almost two years to go upon the liberation of the mines
in 1945. There is no showing that Nielson had ceased to prosecute the operation and We have conscientiously considered the arguments of Lepanto in support of these
development of the mines in good faith and in accordance with approved mining practice three grounds, but We are not persuaded to reconsider the rulings that We made in Our
which would warrant the termination of the contract upon ninety days written notice. In fact decision.
there was no such written notice of termination. It is an admitted fact that Nielson ceased to
operate and develop the mines because of the war a cause beyond the control of Nielson. We want to say a little more on these points, however. Paragraph II of the
Indeed, if the management contract in question was intended to create a relationship of management contract provides as follows:
principal and agent between Lepanto and Nielson, paragraph XI of the contract should not
have been inserted because, as provided in Article 1733 of the old Civil Code, agency is In the event of inundation, flooding of the mine, typhoon, earthquake or any
essentially revocable at the will of the principal that means, with or without cause. But other force majeure, war, insurrection, civil commotion, organized strike, riot, fire,
precisely said paragraph XI was inserted in the management contract to provide for the cause injury to the machinery or other event or cause reasonably beyond the control of
for its revocation. The provision of paragraph XI must be given effect. NIELSON and which adversely affects the work of mining and milling; NIELSON shall
report such fact to LEPANTO and without liability or breach of the terms of this
In the construction of an instrument where there are several provisions or Agreement, the same shall remain in suspense, wholly or partially during the terms of
particulars, such a construction is, if possible, to be adopted as will give effect to all, 3 and if such inability. (Emphasis supplied)
some stipulation of any contract should admit of several meanings, it shall be understood as
bearing that import which is most adequate to render it effectual. 4 A reading of the above-quoted paragraph II cannot but convey the idea that upon the
happening of any of the events enumerated therein, which adversely affects the work of
It is Our considered view that by express stipulation of the parties, the management mining and milling, the agreement is deemed suspended for as long as Nielson is unable to
contract in question is not revocable at the will of Lepanto. We rule that this management perform its work of mining and milling because of the adverse effects of the happening of the
contract is not a contract of agency as defined in Article 1709 of the old Civil Code, but a event on the work of mining and milling. During the period when the adverse effects on the
contract of lease of services as defined in Article 1544 of the same Code. This contract can not work of mining and milling exist, neither party in the contract would be held liable for non-
be unilaterally revoked by Lepanto. compliance of its obligation under the contract. In other words, the operation of the contract
The first ground of the motion for reconsideration should, therefore, be brushed aside. is suspended for as long as the adverse effects of the happening of any of those events had
impeded or obstructed the work of mining and milling. An analysis of the phraseology of the
2. In the second, third and fifth grounds of its motion for reconsideration, Lepanto above-quoted paragraph II of the management contract readily supports the conclusion that
maintains that this Court erred, in holding that paragraph 11 of the management contract it is the agreement, or the contract, that is suspended. The phrase "the same" can refer to no
suspended the period of said contract, in holding that the agreement was not only suspended other than the term "Agreement" which immediately precedes it. The "Agreement" may be
but was extended on account of the war, and in holding that the period of suspension on wholly or partially suspended, and this situation will depend on whether the event wholly or
account of the war lasted from February, 1942 to June 26, 1948. We are going to discuss these partially affected adversely the work of mining and milling. In the instant case, the war had
three grounds together because they are interrelated. adversely affected and wholly at that the work of mining and milling. We have clearly
stated in Our decision the circumstances brought about by the war which caused the whole or
In our decision we have dwelt lengthily on the points that the management contract total suspension of the agreement or of the management contract.
was suspended because of the war, and that the period of the contract was extended for a
period equivalent to the time when Nielson was unable to perform the work of mining and LEPANTO itself admits that the management contract was suspended.
milling because of the adverse effects of the war on the work of mining and milling.
We quote from the brief of LEPANTO:
It is the contention of Lepanto that the happening of those events, and the effects of
those events, simply suspended the performance of the obligations by either party in the Probably, what Nielson meant was, it was prevented by Lepanto to assume again the
contract, but did not suspend the period of the contract, much less extended the period of management of the mine in 1945, at the precise time when defendant was at the
the contract.

119
MATABABE FILES
Corporation Law

feverish phase of rehabilitation and although the contract had already been the war, and this situation continued until June of 1948. Hence, the suspension of the
suspended. (Lepanto's Brief, p. 9). management contract did not end upon the liberation of the mines in August, 1945. The mines
and the mill and the installations, laid waste by the ravages of war, had to be reconstructed
... it was impossible, as a result of the destruction of the mine, for the plaintiff to and rehabilitated, and it can be said that it was only on June 26, 1948 that the adverse effects
manage and operate the same and because, as provided in the agreement, the of the war on the work of mining and milling had ended, because it was on that date that the
contract was suspended by reason of the war (Lepanto's Brief, pp. 9-10). operation of the mines and the mill was resumed. The period of suspension should, therefore,
be reckoned from February 1942 until June 26, 1948, because it was during this period that the
Clause II, by its terms, is clear that the contract is suspended in case fortuitous event war and the adverse effects of the war on the work of mining and milling had lasted. The
or force majeure, such as war, adversely affects the work of mining and milling. mines and the installations had to be rehabilitated because of the adverse effects of the war.
(Lepanto's Brief, p. 49). The work of rehabilitation started soon after the liberation of the mines in August, 1945 and
lasted until June 26, 1948 when, as stated in Lepanto's annual report to its stockholders for
Lepanto is correct when it said that the obligations under the contract were the year 1948, "June 28, 1948 marked the official return to operation of this company at its
suspended upon the happening of any of the events enumerated in paragraph II of the properties at Mankayan, Mountain Province, Philippines" (Exh. F-1).
management contract. Indeed, those obligations were suspended because the contract itself
was suspended. When we talk of a contract that has been suspended we certainly mean that Lepanto would argue that if the management contract was suspended at all the
the contract temporarily ceased to be operative, and the contract becomes operative again suspension should cease in August of 1945, contending that the effects of the war should
upon the happening of a condition or when a situation obtains which warrants the cease upon the liberation of the mines from the enemy. This contention cannot be sustained,
termination of the suspension of the contract. because the period of rehabilitation was still a period when the physical effects of the war
the destruction of the mines and of all the mining installations adversely affected, and
In Our decision We pointed out that the agreement in the management contract made impossible, the work of mining and milling. Hence, the period of the reconstruction and
would be suspended when two conditions concur, namely: (1) the happening of the event rehabilitation of the mines and the installations must be counted as part of the period of
constituting a force majeure that was reasonably beyond the control of Nielson, and (2) that suspension of the contract.
the event constituting the force majeure adversely affected the work of mining and milling.
The suspension, therefore, would last not only while the event constituting the force majeure Lepanto claims that it would not be unfair to end the period of suspension upon the
continued to occur but also for as long as the adverse effects of the force majeure on the liberation of the mines because soon after the liberation of the mines Nielson insisted to
work of mining and milling had not been eliminated. Under the management contract the resume the management work, and that Nielson was under obligation to reconstruct the mill
happening alone of the event constituting the force majeure which did not affect adversely in the same way that it was under obligation to construct the mill in 1937. This contention is
the work of mining and milling would not suspend the period of the contract. It is only when untenable. It is true that Nielson insisted to resume its management work after liberation, but
the two conditions concur that the period of the agreement is suspended. this was only for the purpose of restoring the mines, the mill, and other installations to their
operating and producing condition as of February 1942 when they were ordered destroyed. It
It is not denied that because of the war, in February 1942, the mine, the original mill, is not shown by any evidence in the record, that Nielson had agreed, or would have agreed,
the original power plant, the supplies and equipment, and all installations at the Mankayan that the period of suspension of the contract would end upon the liberation of the mines. This
mines of Lepanto, were destroyed upon order of the United States Army, to prevent their is so because, as found by this Court, the intention of the parties in the management contract,
utilization by the enemy. It is not denied that for the duration of the war Nielson could not and as understood by them, the management contract was suspended for as long as the
undertake the work of mining and milling. When the mines were liberated from the enemy in adverse effects of the force majeure on the work of mining and milling had not been
August, 1945, the condition of the mines, the mill, the power plant and other installations, was removed, and the contract would be extended for as long as it was suspended. Under the
not the same as in February 1942 when they were ordered destroyed by the US army. management contract Nielson had the obligation to erect and operate the mill, but not to
Certainly, upon the liberation of the mines from the enemy, the work of mining and milling erect or reconstruct the mill in case of its destruction by force majeure.
could not be undertaken by Nielson under the same favorable circumstances that obtained
before February 1942. The work of mining and milling, as undertaken by Nielson in January, It is the considered view of this court that it would not be fair to Nielson to consider
1942, could not be resumed by Nielson soon after liberation because of the adverse effects of the suspension of the contract as terminated upon the liberation of the mines because then

120
MATABABE FILES
Corporation Law

Nielson would be placed in a situation whereby it would have to suffer the adverse effects of the work he had accomplished, the force majeure clause is incorporated as a standard clause
the war on the work of mining and milling. The evidence shows that as of January 1942 the in contracts for the management and operation of mines.
operation of the mines under the management of Nielson was already under beneficial
conditions, so much so that dividends were already declared by Lepanto for the years 1939, The nature of the contract for the management and operation of mines justifies the
1940 and 1941. To make the management contract immediately operative after the liberation interpretation of the force majeure clause, that a period equal to the period of suspension
of the mines from the Japanese, at the time when the mines and all its installations were laid due to force majeure should be added to the original term of the contract by way of an
waste as a result of the war, would be to place Nielson in a situation whereby it would lose all extension. We, therefore, reiterate the ruling in Our decision that the management contract
the benefits of what it had accomplished in placing the Lepanto mines in profitable operation in the instant case was suspended from February, 1942 to June 26, 1948, and that from the
before the outbreak of the war in December, 1941. The record shows that Nielson started its latter date the contract had yet five years to go.
management operation way back in 1936, even before the management contract was entered 3. In the fourth ground of its motion for reconsideration, Lepanto maintains that this Court
into. As early as August 1936 Nielson negotiated with Messrs. C. I. Cookes and V. L. Lednicky erred in reversing the finding of the trial court that Nielson's action has prescribed, by
for the operation of the Mankayan mines and it was the result of those negotiations that considering only the first claim and ignoring the prescriptibility of the other claims.
Lepanto was incorporated; that it was Nielson that helped to capitalize Lepanto, and that
after the formation of the corporation (Lepanto) Nielson immediately assumed the This ground of the motion for reconsideration has no merit.
management of the mining properties of Lepanto. It was not until January 30, 1937 when the
management contract in question was entered into between Lepanto and Nielson (Exhibit A). In Our decision We stated that the claims of Nielson are based on a written
A contract for the management and operation of mines calls for a speculative and risky document, and, as such, the cause of action prescribes in ten years. 5 Inasmuch as there are
venture on the part of the manager-operator. The manager-operator invests its technical different claims which accrued on different dates the prescriptive periods for all the claims are
know-how, undertakes back-breaking efforts and tremendous spade-work, so to say, in the not the same. The claims of Nielson that have been awarded by this Court are itemized in the
first years of its management and operation of the mines, in the expectation that the dispositive part of the decision.
investment and the efforts employed might be rewarded later with success. This expected
success may never come. This had happened in the very case of the Mankayan mines where, The first item of the awards in Our decision refers to Nielson's compensation in the
as recounted by Mr. Lednicky of Lepanto, various persons and entities of different sum of P17,500.00, which is equivalent to 10% of the cash dividends declared by Lepanto in
nationalities, including Lednicky himself, invested all their money and failed. The manager- December, 1941. As we have stated in Our decision, this claim accrued on December 31, 1941,
operator may not strike sufficient ore in the first, second, third, or fourth year of the and the right to commence an action thereon started on January 1, 1942. We declared that the
management contract, or he may not strike ore even until the end of the fifth year. Unless the action on this claim did not prescribe although the complaint was filed on February 6, 1958
manager-operator strikes sufficient quantity of ore he cannot expect profits or reward for his or after a lapse of 16 years, 1 month and 5 days because of the operation of the moratorium
investment and efforts. In the case of Nielson, its corps of competent engineers, geologists, law.
and technicians begun working on the Mankayan mines of Lepanto since the latter part of
1936, and continued their work without success and profit through 1937, 1938, and the earlier We declared that under the applicable decisions of this Court 6 the moratorium period
part of 1939. It was only in December of 1939 when the efforts of Nielson started to be of 8 years, 2 months and 8 days should be deducted from the period that had elapsed since
rewarded when Lepanto realized profits and the first dividends were declared. From that time the accrual of the cause of action to the date of the filing of the complaint, so that there is a
on Nielson could expect profit to come to it as in fact Lepanto declared dividends for 1940 period of less than 8 years to be reckoned for the purpose of prescription.
and 1941 if the development and operation of the mines and the mill would continue
unhampered. The operation, and the expected profits, however, would still be subject to This claim of Nielson is covered by Executive Order No. 32, issued on March 10, 1945,
hazards due to the occurrence of fortuitous events, fires, earthquakes, strikes, war, etc., which provides as follows:
constituting force majeure, which would result in the destruction of the mines and the mill.
One of these diverse causes, or one after the other, may consume the whole period of the Enforcement of payments of all debts and other monetary
contract, and if it should happen that way the manager-operator would reap no profit to obligations payable in the Philippines, except debts and other monetary obligations
compensate for the first years of spade-work and investment of efforts and know-how. entered into in any area after declaration by Presidential Proclamation that such area
Hence, in fairness to the manager-operator, so that he may not be deprived of the benefits of has been freed from enemy occupation and control, is temporarily suspended

121
MATABABE FILES
Corporation Law

pending action by the Commonwealth Government. (41 O.G. 56-57; Emphasis the second item in the award, which refers to Nielson's claim for management fee of
supplied) P2,500.00 for January, 1942. Lepanto admits that this second item, like the first, is a monetary
obligation. The right of action of Nielson regarding this claim accrued on January 31, 1942.
Executive Order No. 32 covered all debts and monetary obligation contracted before
the war (or before December 8, 1941) and those contracted subsequent to December 8, 1941 As regards items 3, 4, 5, 6 and 7 in the awards in the decision, the moratorium law is
and during the Japanese occupation. Republic Act No. 342, approved on July 26, 1948, lifted not applicable. That is the reason why in Our decision We did not discuss the question of
the moratorium provided for in Executive Order No. 32 on pre-war (or pre-December 8, 1941) prescription regarding these items. The claims of Nielson involved in these items are based on
debts of debtors who had not filed war damage claims with the United States War Damage the management contract, and Nielson's cause of action regarding these claims prescribes in
Commission. In other words, after the effectivity of Republic Act No. 342, the debt ten years. Corollary to Our ruling that the management contract was suspended from
moratorium was limited: (1) to debts and other monetary obligations which were contracted February, 1942 until June 26, 1948, and that the contract was extended for five years from
after December 8, 1941 and during the Japanese occupation, and (2) to those pre-war (or pre- June 26, 1948, the right of action of Nielson to claim for what is due to it during that period of
December 8, 1941) debts and other monetary obligations where the debtors filed war damage extension accrued during the period from June 26, 1948 till the end of the five-year extension
claims. That was the situation up to May 18, 1953 when this Court declared Republic Act No. period or until June 26, 1953. And so, even if We reckon June 26, 1948 as the starting date of
342 unconstitutional.7 It has been held by this Court, however, that from March 10, 1945 when the ten-year period in connection with the prescriptibility of the claims involved in items 3, 4,
Executive Order No. 32 was issued, to May 18, 1953 when Republic Act No. 342 was declared 5, 6 and 7 of the awards in the decision, it is obvious that when the complaint was filed on
unconstitutional or a period of 8 years, 2 months and 8 days the debt moratorium was in February 6, 1958 the ten-year prescriptive period had not yet lapsed.
force, and had the effect of suspending the period of prescription. 8
In Our decision We have also ruled that the right of action of Nielson against Lepanto
Lepanto is wrong when in its motion for reconsideration it claims that the had not prescribed because of the arbitration clause in the Management contract. We are
moratorium provided for in Executive Order No. 32 was continued by Republic Act No. 342 satisfied that there is evidence that Nielson had asked for arbitration, and an arbitration
"only with respect to debtors of pre-war obligations or those incurred prior to December 8, committee had been constituted. The arbitration committee, however, failed to bring about
1941," and that "the moratorium was lifted and terminated with respect to obligations any settlement of the differences between Nielson and Lepanto. On June 25, 1957 counsel for
incurred after December 8, 1941."9 Lepanto definitely advised Nielson that they were not entertaining any claim of Nielson.

This Court has held that Republic Act No. 342 does not apply to debts contracted The complaint in this case was filed on February 6, 1958.
during the war and did not lift the moratorium in relations thereto. 10 In the case of Abraham,
et al. vs. Intestate Estate of Juan C. Ysmael, et al., L-16741, Jan. 31, 1962, this Court said: 4. In the sixth ground of its motion for reconsideration, Lepanto maintains that this
Court "erred in awarding as damages (a) 10% of the cash dividends declared and paid in
Respondents, however, contend that Republic Act No. 342, which took December, 1941; (b) the management fee of P2,500.00 for the month of January 1942; and (c)
effect on July 26, 1948, lifted the moratorium on debts contracted during the the full contract price for the extended period of 60 months, since the damages were never
Japanese occupation. The court has already held that Republic Act No. 342 did not lift demanded nor proved and, in any case, not allowable under the general law on damages."
the moratorium on debts contracted during the war (Uy vs. Kalaw Katigbak, G.R. No.
L-1830, Dec. 31, 1949) but modified Executive Order No. 32 as to pre-war debts, We have stated in Our decision that the original agreement in the management
making the protection available only to debtors who had war damage claims (Sison v. contract regarding the compensation of Nielson was modified, such that instead of receiving
Mirasol, G.R. No. L-4711, Oct. 3, 1952). a monthly compensation of P2,500.00 plus 10% of the net profits from the operation of the
properties for the preceding month,11 Nielson would receive a compensation of P2,500.00 a
We therefore reiterate the ruling in Our decision that the claim involved in the first month, plus (1) 10% of the dividends declared and paid, when and as paid, during the period of
item awarded to Nielson had not prescribed. the contract, and at the end of each year, (2) 10% of any depletion reserve that may be set up,
and (3) 10% of any amount expended during the year out of surplus earnings for capital
What we have stated herein regarding the non-prescription of the cause of action of account.
the claim involved in the first item in the award also holds true with respect to

122
MATABABE FILES
Corporation Law

It is shown that in December, 1941, cash dividends amounting to P175,000.00 was 5. In the seventh ground of its motion for reconsideration, Lepanto maintains that
declared by Lepanto.12Nielson, therefore, should receive the equivalent of 10% of this amount, this Court erred in ordering Lepanto to issue and deliver to Nielson shares of stock together
or the sum of P17,500.00. We have found that this amount was not paid to Nielson. with fruits thereof.

In its motion for reconsideration, Lepanto inserted a photographic copy of page 127 In Our decision, We declared that pursuant to the modified agreement regarding the
of its cash disbursement book, allegedly for 1941, in an effort to show that this amount of compensation of Nielson which provides, among others, that Nielson would receive 10% of
P17,500.00 had been paid to Nielson. It appears, however, in this photographic copy of page any dividends declared and paid, when and as paid, Nielson should be paid 10% of the stock
127 of the cash disbursement book that the sum of P17,500.00 was entered on October 29 as dividends declared by Lepanto during the period of extension of the contract.
"surplus a/c Nielson & Co. Inc." The entry does not make any reference to dividends or
participation of Nielson in the profits. On the other hand, in the photographic copy of page 89 It is not denied that on November 28, 1949, Lepanto declared stock dividends worth
of the 1941 cash disbursement book, also attached to the motion for reconsideration, there is P1,000,000.00; and on August 22, 1950, it declared stock dividends worth P2,000,000.00). In
an entry for P17,500.00 on April 23, 1941 which states "Accts. Pay. Particip. Nielson & Co. Inc." other words, during the period of extension Lepanto had declared stock dividends worth
This entry for April 23, 1941 may really be the participation of Nielson in the profits based on P3,000,000.00. We held in Our decision that Nielson is entitled to receive l0% of the stock
dividends declared in April 1941 as shown in Exhibit L. But in the same Exhibit L it is not stated dividends declared, or shares of stock worth P300,000.00 at the par value of P0.10 per share.
that any dividend was declared in October 1941. On the contrary it is stated in Exhibit L that We ordered Lepanto to issue and deliver to Nielson those shares of stocks as well as all the
dividends were declared in December 1941. We cannot entertain this piece of evidence for fruits or dividends that accrued to said shares.
several reasons: (1) because this evidence was not presented during the trial in the court
below; (2) there is no showing that this piece of evidence is newly discovered and that In its motion for reconsideration, Lepanto contends that the payment to Nielson of
Lepanto was not in possession of said evidence when this case was being tried in the court stock dividends as compensation for its services under the management contract is a violation
below; and (3) according to Exhibit L cash dividends of P175,000.00 were declared in of the Corporation Law, and that it was not, and it could not be, the intention of Lepanto and
December, 1941, and so the sum of P17,500.00 which appears to have been paid to Nielson in Nielson as contracting parties that the services of Nielson should be paid in shares of
October 1941 could not be payment of the equivalent of 10% of the cash dividends that were stock taken out of stock dividends declared by Lepanto. We have assiduously considered the
later declared in December, 1941. arguments adduced by Lepanto in support of its contention, as well as the answer of Nielson
in this connection, and We have arrived at the conclusion that there is merit in the contention
As regards the management fee of Nielson corresponding to January, 1942, in the of Lepanto.
sum of P2,500.00, We have also found that Nielson is entitled to be paid this amount, and that
this amount was not paid by Lepanto to Nielson. Whereas, Lepanto was able to prove that it Section 16 of the Corporation Law, in part, provides as follows:
had paid the management fees of Nielson for November and December, 1941, 13 it was not able
to present any evidence to show that the management fee of P2,500.00 for January, 1942 had No corporation organized under this Act shall create or issue bills, notes or
been paid. other evidence of debt, for circulation as money, and no corporation shall issue stock
or bonds except in exchange for actual cash paid to the corporation or for: (1)
It having been declared in Our decision, as well as in this resolution, that the property actually received by it at a fair valuation equal to the par or issued value of
management contract had been extended for 5 years, or sixty months, from June 27, 1948 to the stock or bonds so issued; and in case of disagreement as to their value, the same
June 26, 1953, and that the cause of action of Nielson to claim for its compensation during shall be presumed to be the assessed value or the value appearing in invoices or
that period of extension had not prescribed, it follows that Nielson should be awarded the other commercial documents, as the case may be; and the burden or proof that the
management fees during the whole period of extension, plus the 10% of the value of the real present value of the property is greater than the assessed value or value
dividends declared during the said period of extension, the 10% of the depletion reserve that appearing in invoices or other commercial documents, as the case may be, shall be
was set up, and the 10% of any amount expended out of surplus earnings for capital account. upon the corporation, or for (2) profits earned by it but not distributed among its
stockholders or members; Provided, however, That no stock or bond dividend shall be
issued without the approval of stockholders representing not less than two-thirds of

123
MATABABE FILES
Corporation Law

all stock then outstanding and entitled to vote at a general meeting of the been capitalized instead of distributed to the stockholders or retained as surplus available for
corporation or at a special meeting duly called for the purpose. distribution, in money or kind, should opportunity offer. Far from being a realization of profits
xxx xxx xxx for the stockholder, it tends rather to postpone said realization, in that the fund represented
by the new stock has been transferred from surplus to assets and no longer available for
No corporation shall make or declare any dividend except from the surplus actual distribution.17 Thus, it is apparent that stock dividends are issued only to stockholders.
profits arising from its business, or divide or distribute its capital stock or property This is so because only stockholders are entitled to dividends. They are the only ones who
other than actual profits among its members or stockholders until after the payment have a right to a proportional share in that part of the surplus which is declared as dividends.
of its debts and the termination of its existence by limitation or lawful A stock dividend really adds nothing to the interest of the stockholder; the proportional
dissolution: Provided, That banking, savings and loan, and trust corporations may interest of each stockholder remains the same.18If a stockholder is deprived of his stock
receive deposits and issue certificates of deposit, checks, drafts, and bills of dividends - and this happens if the shares of stock forming part of the stock dividends are
exchange, and the like in the transaction of the ordinary business of banking, savings issued to a non-stockholder then the proportion of the stockholder's interest changes
and loan, and trust corporations. (As amended by Act No. 2792, and Act No. 3518; radically. Stock dividends are civil fruits of the original investment, and to the owners of the
Emphasis supplied.) shares belong the civil fruits.19

From the above-quoted provision of Section 16 of the Corporation Law, the The term "dividend" both in the technical sense and its ordinary acceptation, is that
consideration for which shares of stock may be issued are: (1) cash; (2) property; and (3) part or portion of the profits of the enterprise which the corporation, by its governing agents,
undistributed profits. Shares of stock are given the special name "stock dividends" only if they sets apart for ratable division among the holders of the capital stock. It means the fund
are issued in lieu of undistributed profits. If shares of stocks are issued in exchange of cash or actually set aside, and declared by the directors of the corporation as dividends and duly
property then those shares do not fall under the category of "stock dividends". A corporation ordered by the director, or by the stockholders at a corporate meeting, to be divided or
may legally issue shares of stock in consideration of services rendered to it by a person not a distributed among the stockholders according to their respective interests. 20
stockholder, or in payment of its indebtedness. A share of stock issued to pay for services
rendered is equivalent to a stock issued in exchange of property, because services is It is Our considered view, therefore, that under Section 16 of the Corporation Law
equivalent to property.14 Likewise a share of stock issued in payment of indebtedness is stock dividends can not be issued to a person who is not a stockholder in payment of services
equivalent to issuing a stock in exchange for cash. But a share of stock thus issued should be rendered. And so, in the case at bar Nielson can not be paid in shares of stock which form part
part of the original capital stock of the corporation upon its organization, or part of the stocks of the stock dividends of Lepanto for services it rendered under the management contract.
issued when the increase of the capitalization of a corporation is properly authorized. In other We sustain the contention of Lepanto that the understanding between Lepanto and Nielson
words, it is the shares of stock that are originally issued by the corporation and forming part was simply to make the cash value of the stock dividends declared as the basis for
of the capital that can be exchanged for cash or services rendered, or property; that is, if the determining the amount of compensation that should be paid to Nielson, in the proportion of
corporation has original shares of stock unsold or unsubscribed, either coming from the 10% of the cash value of the stock dividends declared. And this conclusion of Ours finds
original capitalization or from the increased capitalization. Those shares of stock may be support in the record.
issued to a person who is not a stockholder, or to a person already a stockholder in exchange
for services rendered or for cash or property. But a share of stock coming from stock We had adverted to in Our decision that in 1940 there was some dispute between
dividends declared cannot be issued to one who is not a stockholder of a corporation. Lepanto and Nielson regarding the application and interpretation of certain provisions of the
original contract particularly with regard to the 10% participation of Nielson in the net profits,
A "stock dividend" is any dividend payable in shares of stock of the corporation so that some adjustments had to be made. In the minutes of the meeting of the Board of
declaring or authorizing such dividend. It is, what the term itself implies, a distribution of the Directors of Lepanto on August 21, 1940, We read the following:
shares of stock of the corporation among the stockholders as dividends. A stock dividend of a
corporation is a dividend paid in shares of stock instead of cash, and is properly payable only The Chairman stated that he believed that it would be better to tie the
out of surplus profits.15 So, a stock dividend is actually two things: (1) a dividend, and (2) the computation of the 10% participation of Nielson & Company, Inc. to the dividend,
enforced use of the dividend money to purchase additional shares of stock at par. 16 When a because Nielson will then be able to definitely compute its net participation by the
corporation issues stock dividends, it shows that the corporation's accumulated profits have amount of the dividends declared. In addition to the dividend, we have been setting

124
MATABABE FILES
Corporation Law

up a depletion reserve and it does not seem fair to burden the 10% participation of November 28, 1949 and on August 20, 1950, together with all the fruits accruing thereto.
Nielson with the depletion reserve, as the depletion reserve should not be Instead, We declare that Nielson is entitled to payment by Lepanto of P300,000.00 in cash,
considered as an operating expense. After a prolonged discussion, upon motion duly which is equivalent to 10% of the money value of the stock dividends worth P3,000,000.00
made and seconded, it was which were declared on November 28, 1949 and on August 20, 1950, with interest thereon at
the rate of 6% from February 6, 1958.
RESOLVED, That the President, be, and he hereby is, authorized to enter into
an agreement with Nielson & Company, Inc., modifying Paragraph V of management 6. In the eighth ground of its motion for reconsideration Lepanto maintains that this
contract of January 30, 1937, effective January 1, 1940, in such a way that Nielson & Court erred in awarding to Nielson an undetermined amount of shares of stock and/or cash,
Company, Inc. shall receive 10% of any dividends declared and paid, when and as paid which award can not be ascertained and executed without further litigation.
during the period of the contract and at the end of each year, 10% of any depletion
reserve that may be set up and 10% of any amount expended during the year out of In view of Our ruling in this resolution that Nielson is not entitled to receive shares of
surplus earnings for capital account. (Emphasis supplied.) stock as stock dividends in payment of its compensation under the management contract, We
do not consider it necessary to discuss this ground of the motion for reconsideration. The
From the sentence, "The Chairman stated that he believed that it would be better to awards in the present case are all reduced to specific sums of money.
tie the computation of the 10% participation of Nielson & Company, Inc., to the dividend,
because Nielson will then be able to definitely compute its net participation by the amount of 7. In the ninth ground of its motion for reconsideration Lepanto maintains that this
the dividends declared" the idea is conveyed that the intention of Lepanto, as expressed by Court erred in rendering judgment or attorney's fees.
its Chairman C. A. DeWitt, was to make the value of the dividends declared whether the The matter of the award of attorney's fees is within the sound discretion of this Court. In Our
dividends were in cash or in stock as the basis for determining the amount of decision We have stated the reason why the award of P50,000.00 for attorney's fees is
compensation that should be paid to Nielson, in the proportion of 10% of the cash value of the considered by this Court as reasonable.
dividends so declared. It does not mean, however, that the compensation of Nielson would
be taken from the amount actually declared as cash dividend to be distributed to the Accordingly, We resolve to modify the decision that We rendered on December 17,
stockholder, nor from the shares of stocks to be issued to the stockholders as stock 1966, in the sense that instead of awarding Nielson shares of stock worth P300,000.00 at the
dividends, but from the other assets or funds of the corporation which are not burdened by par value of ten centavos (P0.10) per share based on the stock dividends declared by Lepanto
the dividends thus declared. In other words, if, for example, cash dividends of P300,000.00 on November 28, 1949 and August 20, 1950, together with their fruits, Nielson should be
are declared, Nielson would be entitled to a compensation of P30,000.00, but this P30,000.00 awarded the sum of P300,000.00 which is an amount equivalent to 10% of the cash value of
should not be taken from the P300,000.00 to be distributed as cash dividends to the the stock dividends thus declared, as part of the compensation due Nielson under the
stockholders but from some other funds or assets of the corporation which are not included management contract. The dispositive portion of the decision should, therefore, be amended,
in the amount to answer for the cash dividends thus declared. This is so because if the to read as follows:
P30,000.00 would be taken out from the P300,000.00 declared as cash dividends, then the
stockholders would not be getting P300,000.00 as dividends but only P270,000.00. There IN VIEW OF THE FOREGOING CONSIDERATIONS, We hereby reverse the decision of
would be a dilution of the dividend that corresponds to each share of stock held by the the court a quo and enter in lieu thereof another, ordering the appellee Lepanto to pay the
stockholders. Similarly, if there were stock dividends worth one million pesos that were appellant Nielson the different amounts as specified hereinbelow:
declared, which means an issuance of ten million shares at the par value of ten centavos per
share, it does not mean that Nielson would be given 100,000 shares. It only means that (1) Seventeen thousand five hundred pesos (P17,500.00), equivalent to 10% of the cash
Nielson should be given the equivalent of 10% of the aggregate cash value of those shares dividends of December, 1941, with legal interest thereon from the date of the filing of the
issued as stock dividends. That this was the understanding of Nielson itself is borne out by the complaint;
fact that in its appeal brief Nielson urged that it should be paid "P300,000.00 being 10% of the
P3,000,000.00 stock dividends declared on November 28, 1949 and August 20, 1950...." 21 (2) Two thousand five hundred pesos (P2,500.00) as management fee for January 1942, with
We, therefore, reconsider that part of Our decision which declares that Nielson is legal interest thereon from the date of the filing of the complaint;
entitled to shares of stock worth P300,000.00 based on the stock dividends declared on

125
MATABABE FILES
Corporation Law

(3) One hundred fifty thousand pesos (P150,000.00), representing management fees for the
sixty-month period of extension of the management contract, with legal interest thereon
from the date of the filing of the complaint;

(4) One million four hundred thousand pesos (P1,400,000.00), equivalent to 10% of the cash
dividends declared during the period of extension of the management contract, with legal
interest thereon from the date of the filing of the complaint;
(5) Three hundred thousand pesos (P300,000.00), equivalent to 10% of the cash value of the
stock dividends declared on November 28, 1949 and August 20, 1950, with legal interest
thereon from the date of the filing of the complaint;

(6) Fifty three thousand nine hundred twenty eight pesos and eighty eight centavos
(P53,928.88), equivalent to 10% of the depletion reserve set up during the period of extension,
with legal interest thereon from the date of the filing of the complaint;

(7) Six hundred ninety four thousand three hundred sixty four pesos and seventy six centavos
(P694,364.76), equivalent to 10% of the expenses for capital account during the period of
extension, with legal interest thereon from the date of the filing of the complaint;

(8) Fifty thousand pesos (P50,000.00) as attorney's fees; and

(9) The costs.

It is so ordered.

Concepcion, C.J., Reyes, J.B.L., Dizon, Makalintal, Sanchez and Castro, JJ., concur.
Fernando, Capistrano, Teehankee and Barredo, JJ., took no part

126
MATABABE FILES
Corporation Law

Republic of the Philippines are not supported by substantial evidence. (Gokongwei, Jr. vs. SEC, 97 SCRA 78). In a long
SUPREME COURT string of cases, the Supreme Court has consistently adhered to the rule that decisions of
Manila administrative officers are not to be disturbed by the courts except when the former have
FIRST DIVISION acted without or in excess of their jurisdiction or with grave abuse of discretion (Sichangco vs.
Board of Commissioners of Immigration, 94 SCRA 61).
G.R. No. L-56655 July 25, 1983
PETITION to review the decision of the Securities and Exchange Commission.
DATU TAGORANAO BENITO, petitioner,
vs. The facts are stated in the opinion of the Court.
SECURITIES AND EXCHANGE COMMISSION and JAMIATUL PHILIPPINE-AL ISLAMIA,
INC., respondents. Datu Tagoranao Benito vs. Securities & Exchange Commission, 123 SCRA 722, No. L-56655 July
25, 1983
Corporation Law; No stockholders meeting or approval is necessary for issuance of
unsubscribed portion of capital stock.As aptly stated by the Securities and Exchange The Solicitor General for respondent.
Commission in its decision: x x x the questioned issuance of the unsubscribed portion of the
capital stock worth P110,980.00 is not invalid even if assuming that it was made without Tacod D. Macaraya for private respondent.
notice to the stockholders as claimed by petitioner. The power to issue shares of stocks in a
corporation is lodged in the board of directors and no stockholders meeting is necessary to RELOVA, J.:
consider it because additional issuance of shares of stocks does not need approval of the
stockholders. The by-laws of the corporation itself states that the Board of Trustees shall, in On February 6, 1959, the Articles of Incorporation of respondent Jamiatul Philippine-
accordance with law, provide for the issue and transfer of shares of stock of the Institute and Al Islamia, Inc. (originally Kamilol Islam Institute, Inc.) were filed with the Securities and
shall prescribe the form of the certificate of stock of the Institute (Art. V, Sec. 1). Exchange Commission (SEC) and were approved on December 14, 1962. The corporation had
an authorized capital stock of P200,000.00 divided into 20,000 shares at a par value of P10.00
Same; A stockholder enjoys no pre-emptive right to buy unissued shares of originally each. Of the authorized capital stock, 8,058 shares worth P80,580.00 were subscribed and
authorized capital stock.Petitioner bewails the fact that in view of the lack of notice to him fully paid for. Herein petitioner Datu Tagoranao Benito subscribed to 460 shares worth
of such subsequent issuance, he was not able to exercise his right of pre-emption over the P4,600.00.
unissued shares. However, the general rule is that pre-emptive right is recognized only with
respect to new issue of shares, and not with respect to additional issues of originally On October 28, 1975, the respondent corporation filed a certificate of increase of its
authorized shares. This is on the theory that when a corporation at its inception offers its first capital stock from P200,000.00 to P1,000,000.00. It was shown in said certificate that
shares, it is presumed to have offered all of those which it is authorized to issue. An original P191,560.00 worth of shares were represented in the stockholders' meeting held on
subscriber is deemed to have taken his shares knowing that they form a definite November 25, 1975 at which time the increase was approved. Thus, P110,980.00 worth of
proportionate part of the whole number of authorized shares. When the shares left shares were subsequently issued by the corporation from the unissued portion of the
unsubscribed are later reoffered, he cannot therefore claim a dilution of interest. (Campos authorized capital stock of P200,000.00. Of the increased capital stock of P1,000,000.00,
and Lopez-Campos Selected Notes and Cases on Corporation Law, p. 855, citing Yasik V. P160,000.00 worth of shares were subscribed by Mrs. Fatima A. Ramos, Mrs. Tarhata A.
Wachtel, 25 Del. Ch. 247, 17A. 2d 308 (1941). (pp. 33-34, Rollo) Lucman and Mrs. Moki-in Alonto.

Same; Administrative Law; Findings of fact of an administrative tribunal, i.e., On November 18, 1976, petitioner Datu Tagoranao filed with respondent Securities
stockholders were notified of the cannot be deemed to have waived his pre-emptive right to and Exchange Commission a petition alleging that the additional issue (worth P110,980.00) of
buy additional shares of stock, are binding on courts.Well-settled is the rule that the findings previously subscribed shares of the corporation was made in violation of his pre-emptive right
of facts of administrative bodies will not be interfered with by the courts in the absence of to said additional issue and that the increase in the authorized capital stock of the corporation
grave abuse of discretion on the part of said agencies, or unless the aforementioned findings from P200,000.00 to P1,000,000.00 was illegal considering that the stockholders of record

127
MATABABE FILES
Corporation Law

were not notified of the meeting wherein the proposed increase was in the agenda. Petitioner Certificate of Stock No. 217, in the name of Moki-in Alonto,
prayed that the additional issue of shares of previously authorized capital stock as well as the upon their presentation by the petitioner and to issue new
shares issued from the increase in capital stock of respondent corporation be cancelled; that certificates corresponding thereto in the name of petitioner
the secretary of respondent corporation be ordered to register the 2,540 shares acquired by herein;
him (petitioner) from Domocao Alonto and Moki-in Alonto; and that the corporation be
ordered to render an accounting of funds to the stockholders. (d) To direct, as it hereby directs, respondent corporation
to religiously comply with the requirement of filing annual
In their answer, respondents denied the material allegations of the petition and, by financial statements under pain of a more drastic action;
way of special defense, claimed that petitioner has no cause of action and that the stock
certificates covering the shares alleged to have been sold to petitioner were only given to him (e) To declare, as it hereby declares, as irregular, the
as collateral for the loan of Domocao Alonto and Moki-in Alonto. election of the nine (9) members of the Board of Trustees
of respondent corporation on October 30, 1976, for which
On July 11, 1980, Hearing Officer Ledor E. Macalalag of the Securities and Exchange reason, respondent corporation is hereby ordered to call a
Commission, after due proceedings, rendered a decision which was affirmed by the stockholders' meeting to elect a new set of five (5)
Commission En Banc during its executive session held on March 9, 1981, as follows: members of the Board of Trustees, unless in the meantime
the said number is accordingly increased and the
RESOLVED, That the decision of the hearing Officer in SEC Case No. requirement of law to make such increase effective have
1392, dated July 11, 1980, the dispositive portion of which reads as follows: been complied with. It is understood that the said
stockholders' meeting be called within thirty (30) days from
WHEREFORE, in view of the foregoing considerations, this the time petitioner shall have subscribed to the increased
Commission hereby rules: capitalization.'

(a) That the issuance by the corporation of its unissued be, as the same is hereby AFFIRMED, the same being in accordance with law
shares was validly made and was not subject to the pre- and the facts of the case. (pp. 28-29, Reno)
emptive rights of stockholders, including the petitioner,
herein; Hence, this petition for review by way of appeal from the aforementioned decision of
the Securities and Exchange Commission, petitioner contending that (1) the issuance of the
(b) That there is no sufficient legal basis to set aside the 11,098 shares without the consent of the stockholders or of the Board of Directors, and in the
certificate issued by this Commission authorizing the absence of consideration, is null and void; (2) the increase in the authorized capital stock from
increase in capital stock of respondent corporation from P200,000.00 to P1,000,000.00 without the consent or express waiver of the stockholders, is
P200,000.00 to Pl,000,000.00. Considering, however, that null and void; (3) he is entitled to attorneys' fees, damages and expenses of litigation in filing
petitioner has not waived his pre-emptive right to subscribe this suit against the directors of respondent corporation.
to the increased capitalization, respondent corporation is
hereby directed to allow petitioner to subscribe thereto, at We are not persuaded. As aptly stated by the Securities and Exchange Commission in
par value, proportionate to his present shareholdings, its decision:
adding thereto the 2,540 shares transferred to him by Mr.
Domocao Alonto and Mrs. Moki-in Alonto; xxx xxx xxx

(c) To direct as it hereby directs, the respondent ... the questioned issuance of the unsubscribed portion of the capital stock
corporation to immediately cancel Certificates of Stock worth P110,980.00 is ' not invalid even if assuming that it was made without
Nos. 216, 223, 302, all in the name of Domocao Alonto, and notice to the stockholders as claimed by petitioner. The power to issue

128
MATABABE FILES
Corporation Law

shares of stocks in a corporation is lodged in the board of directors and no concerned, he had not waived his pre-emptive right to subscribe as he could not have done so
stockholders' meeting is necessary to consider it because additional issuance for the reason that he was not present at the meeting and had not executed a waiver,
of shares of stocks does not need approval of the stockholders. The by-laws thereof. Not having waived such right and for reasons of equity, he may still be allowed to
of the corporation itself states that 'the Board of Trustees shall, in subscribe to the increased capital stock proportionate to his present shareholdings." (pp. 36-
accordance with law, provide for the issue and transfer of shares of stock of 37, Rollo)
the Institute and shall prescribe the form of the certificate of stock of the
Institute. (Art. V, Sec. 1). Well-settled is the rule that the findings of facts of administrative bodies will not be
interfered with by the courts in the absence of grave abuse of discretion on the part of said
Petitioner bewails the fact that in view of the lack of notice to him agencies, or unless the aforementioned findings are not supported by substantial evidence.
of such subsequent issuance, he was not able to exercise his right of pre- (Gokongwei, Jr. vs. SEC, 97 SCRA 78). In a long string of cases, the Supreme Court has
emption over the unissued shares. However, the general rule is that pre- consistently adhered to the rule that decisions of administrative officers are not to be
emptive right is recognized only with respect to new issue of shares, and not disturbed by the courts except when the former have acted without or in excess of their
with respect to additional issues of originally authorized shares. This is on jurisdiction or with grave abuse of discretion (Sichangco vs. Board of Commissioners of
the theory that when a corporation at its inception offers its first shares, it is Immigration, 94 SCRA 61). Thus, in the case of Deluao vs. Casteel ( L-21906, Dec. 24, 1968, 26
presumed to have offered all of those which it is authorized to issue. An SCRA 475, 496, citing Pajo vs. Ago, et al., L-15414, June 30, 1960) and Genitano vs. Secretary of
original subscriber is deemed to have taken his shares knowing that they Agriculture and Natural Resources, et al. (L-2ll67, March 31, 1966), the Supreme Court held that:
form a definite proportionate part of the whole number of authorized
shares. When the shares left unsubscribed are later re-offered, he cannot ... Findings of fact by an administrative board or official, following a hearing,
therefore claim a dilution of interest. (Campos and Lopez-Campos Selected are binding upon the courts and win not be disturbed except where the
Notes and Cases on Corporation Law, p. 855, citing Yasik V. Wachtel 25 Del. board or official has gone beyond his statutory authority, exercised
Ch. 247,17A. 2d 308 (1941). (pp. 33-34, Rollo) unconstitutional powers or clearly acted arbitrarily and without regard to his
With respect to the claim that the increase in the authorized capital stock was duty or with grave abuse of discretion. ...
without the consent, expressed or implied, of the stockholders, it was the finding of the
Securities and Exchange Commission that a stockholders' meeting was held on November ACCORDINGLY, this petition is hereby dismissed for lack of merit.
25,1975, presided over by Mr. Ahmad Domocao Alonto, Chairman of the Board of Trustees SO ORDERED.
and, among the many items taken up then were the change of name of the corporation from
Kamilol Islam Institute Inc. to Jamiatul Philippine-Al Islamia, Inc., the increase of its capital Plana, Escolin and Gutierrez, Jr., JJ., concur.
stock from P200,000.00 to P1,000,000.00, and the increase of the number of its Board of Teehankee, J., concurs in the result.
Trustees from five to nine. "Despite the insistence of petitioner, this Commission is inclined to Melencio-Herrera and Vasquez, JJ., are on leave.
believe that there was a stockholders' meeting on November 25, 1975 which approved the
increase. The petitioner had not sufficiently overcome the evidence of respondents that such
meeting was in fact held. What petitioner successfully proved, however, was the fact that he
was not notified of said meeting and that he never attended the same as he was out of the
country at the time. The documentary evidence of petitioner conclusively proved that he was
attending the Mecca pilgrimage when the meeting was held on November 25, 1975. (Exhs. 'Q',
'Q-14', 'R', 'S' and 'S-l'). While petitioner doubts the authenticity of the alleged minutes of the
proceedings (Exh. '4'), the Commission notes with significance that said minutes contain
numerous details of various items taken up therein that would negate any claim that it was
not authentic. Another thing that petitioner was able to disprove was the allegation in the
certificate of increase (Exh. 'E-l') that all stockholders who did not subscribe to the increase of
capital stock have waived their pre-emptive right to do so. As far as the petitioner is

129
MATABABE FILES
Corporation Law

Republic of the Philippines parlance, ultra vires act refers to one which is not within the corporate powers conferred
SUPREME COURT by the Corporation Code or articles of incorporation or not necessary or incidental in the
Manila exercise of the powers so conferred.
SECOND DIVISION
Same; Same; Providing gratuity pay for its employees is one of the express powers of
G.R. No. 76801 August 11, 1995 the corporation under the Corporation Code.The assailed resolutions before us cover a
subject which concerns the benefit and welfare of the companys employees. To stress,
LOPEZ REALTY, INC., AND ASUNCION LOPEZ GONZALES, petitioners, providing gratuity pay for its employees is one of the express powers of the corporation
vs. under the Corporation Code, hence, petitioners cannot invoke the doctrine of ultra vires to
FLORENTINA FONTECHA, ET AL., AND THE NATIONAL LABOR RELATIONS avoid any liability arising from the issuance of the subject resolutions.
COMMISSION, respondents.
Same; Section 28 1/2 of the Corporation Law (now Section 40 of the Corporation Code)
Actions; Pleadings and Practice; Appeals; It is well settled that questions not raised in requiring authorization of the stockholders of record for action taken by the board of directors
the lower courts cannot be raised for the first time on appeal.Indeed, it would be offensive applies to the sale, lease, exchange or disposition of all or substantially all of the corporations
to the basic rules of fair play and justice to allow petitioners to raise questions which have not assets.Petitioners try to convince us that the subject resolutions had no force and effect in
been passed upon by the labor arbiter and the public respondent NLRC. It is well settled that view of the non-approval thereof during the Annual Stockholders Meeting held on March 1,
questions not raised in the lower courts cannot be raised for the first time on appeal. Hence, 1982. To strengthen their position, petitioners cite section 28 1/2 of the Corporation Law
petitioners may not invoke any other ground, other than those it specified at the labor arbiter (Section 40 of the Corporation Code). We are not persuaded. The cited provision is not
level, to impugn the validity of the subject resolutions. applicable to the case at bench as it refers to the sale, lease, exchange or disposition of all or
substantially all of the corporations assets, including its goodwill. In such a case, the action
Corporation Law; A corporation, through its board of directors, should act in the taken by the board of directors requires the authorization of the stockholders on record.
manner and within the formalities, if any, prescribed by its charter or by the general law.The
general rule is that a corporation, through its board of directors, should act in the manner and Same; Where the stockholders of petitioner, except for one, also sit as members of the
within the formalities, if any, prescribed by its charter or by the general law. Thus, directors board of directors, it will be illogical and superfluous to require the stockholders approval of
must act as a body in a meeting called pursuant to the law or the corporations by-laws, certain resolutions adopted by the board of directors.It will be observed that, except for
otherwise, any action taken therein may be questioned by any objecting director or Arturo Lopez, the stockholders of petitioner corporation also sit as members of the board of
shareholder. directors. Under the circumstances in field, it will be illogical and superfluous to require the
stockholders approval of the subject resolutions. Thus, even without the stockholders
Same; An action of the board of directors during a meeting, which was illegal for lack approval of the subject resolutions, petitioners are still liable to pay private respondents
of notice, may be ratified either expressly, by the action of the directors in subsequent legal gratuity pay.
meeting, or impliedly, by the corporations subsequent course of conduct.Be that as it may,
jurisprudence tells us that an action of the board of directors during a meeting, which was SPECIAL CIVIL ACTION in the Supreme Court. Certiorari.
illegal for lack of notice, may be ratified either expressly, by the action of the directors in
subsequent legal meeting, or impliedly, by the corporations subsequent course of conduct. The facts are stated in the opinion of the Court.

Same; Ultra Vires Acts; Words and Phrases; In legal parlance, ultra vires act refers PUNO, J.:
to one which is not within the corporate powers conferred by the Corporation Code or articles
of incorporation or not necessary or incidental in the exercise of the powers so conferred. The controversy at bench arose from a complaint filed by private
Assuming,arguendo, that there was no notice given to Asuncion Lopez Gonzales during the respondents, 1 namely, Florentina Fontecha, Mila Refuerzo, Marcial Mamaril, Perfecto
special meetings held on August 17, 1981 and September 1, 1981, it is erroneous to state that Bautista, Edward Mamaril, Marissa Pascual and Allan Pimentel, against their employer Lopez
the resolutions passed by the board during the said meetings were ultra vires. In legal Realty Incorporated (petitioner) and its majority stockholder, Asuncion Lopez Gonzales, for

130
MATABABE FILES
Corporation Law

alleged non-payment of their gratuity pay and other benefits. 2 The case was docketed as Benjamin Bernardino, and Leo Rivera, convened a special meeting and passed a
NLRC-NCR Case No. 2-2176-82. resolution which reads:
Lopez Realty, Inc., is a corporation engaged in real estate business, while petitioner Asuncion
Lopez Gonzales is one of its majority shareholders. Her interest in the company vis-a-vis the Resolved, as it is hereby resolved that the gratuity (pay) of the employees be
other shareholders is as follows: given as follows:

1 Asuncion Lopez Gonzales 7831 shares (a) Those who will be laid off be given the full amount of gratuity;
2 Teresita Lopez Marquez 7830 shares
(b) Those who will be retained will receive 25% of their gratuity (pay) due on
3 Arturo F. Lopez 7830 shares September 1, 1981, and another 25% on January 1, 1982, and 50% to be retained
4 Rosendo de Leon 4 shares by the office in the meantime. (emphasis supplied)
5 Benjamin Bernardino 1 share
6 Leo Rivera 1 share Private respondents were the retained employees of petitioner corporation.
In a letter, dated August 31, 1981, private respondents requested for the full payment
of their gratuity pay. Their request was granted in a special meeting held on
Except for Arturo F. Lopez, the rest of the shareholders also sit as members
September 1, 1981. The relevant, portion of the minutes of the said board meeting
of the Board of Directors.
reads:
As found by the Labor arbiter. 3 sometime in 1978, Arturo Lopez submitted a
In view of the request of the employees contained in the letter
proposal relative to the distribution of certain assets of petitioner corporation among
dated August 31, 1981, it was also decided that, all those remaining
its three (3) main shareholders. The proposal had three (3) aspects, viz: (1) the sale of
employees will receive another 25% (of their gratuity) on or before October
assets of the company to pay for its obligations; (2) the transfer of certain assets of
15, 1981 and another 25% on or before the end of November, 1981 of their
the company to its three (3) main shareholders, while some other assets shall remain
respective gratuity.
with the company; and (3) the reduction of employees with provision for their gratuity
pay. The proposal was deliberated upon and approved in a special meeting of the
At that, time, however, petitioner Asuncion Lopez Gonzales was still abroad.
board of directors held on April 17, 1978.
Allegedly, while she was still out of the country, she sent a cablegram to the
corporation, objecting to certain matters taken up by the board in her absence, such
It appears that petitioner corporation approved two (2) resolutions
as the sale of some of the assets of the corporation. Upon her return, she flied a
providing for the gratuity pay of its employees, viz: (a) Resolution No. 6, Series of 1980,
derivative suit with the Securities and Exchange Commission (SEC) against majority
passed by the stockholders in a special meeting held on September 8, 1980, resolving
shareholder Arturo F. Lopez.
to set aside, twice a year, a certain sum of money for the gratuity pay of
its retiring employees and to create a Gratuity Fund for the said contingency; and
Notwithstanding the "corporate squabble" between petitioner Asuncion
(b) Resolution No. 10, Series of 1980, setting aside the amount of P157,750.00 as
Lopez Gonzales and Arturo Lopez, the first two (2) installments of the gratuity pay of
Gratuity Fund covering the period from 1950 up to 1980.
private respondents Florentina Fontecha, Mila Refuerzo, Marcial Mamaril and
Perfecto Bautista were paid by petitioner corporation.
Meanwhile, on July 28, 1981, board member and majority stockholder
Teresita Lopez Marquez died.
Also, petitioner corporation had prepared the cash vouchers and checks for
the third installments of gratuity pay of said private respondents (Florentina
On August 17, 1981, except for Asuncion Lopez Gonzales who was then
Fontecha, Mila Refuerzo, Marcial Mamaril and Perfecto Bautista). For some reason,
abroad, the remaining members of the Board of Directors, namely: Rosendo de Leon,
said vouchers were cancelled by petitioner Asuncion Lopez Gonzales.

131
MATABABE FILES
Corporation Law

Likewise, the first, second and third installments of gratuity pay of the rest respondents' argument allowing gratuities upon retirement of employees.
of private respondents, particularly, Edward Mamaril, Marissa Pascual and Allan Additionally, the proposed distribution of assets (Exh. C-1) filed by Mr. Arturo
Pimentel, were prepared but cancelled by petitioner Asuncion Lopez Gonzales. F. Lopez also made mention of gratuity pay, " . . . (wherein) an employee who
Despite private respondents' repeated demands for their gratuity pay, corporation desires to resign from the LRI will be given the gratuity pay he or she earned."
refused to pay the same. 4 (Emphasis supplied) Let us be reminded, too, that the complainants'
On July 23, 1984, Labor Arbiter Raymundo R. Valenzuela rendered judgment in favor resignation was not voluntary but it was pressurized (sic) due to "power
of private respondents. 5 struggle" which was evident between Arturo Lopez and Asuncion Gonzales.

Petitioners appealed the adverse ruling of the Labor arbiter to public The respondents' (petitioners') contention of a mistake to have
respondent National Labor Relations Commission. The appeal focused on the alleged been committed in granting the first two (2) installments of gratuities to
non-ratification and non-approval of the assailed August 17, 1981 and September 1, complainants Perfecto Bautista, Florentina Fontecha, Marcial Mamaril and
1981 Board Resolutions during the Annual Stockholders' Meeting held on March 1, Mila Refuerzo, (has) no legal leg to stand on. The record is bereft of any
1982. Petitioners further insisted that the payment of the gratuity to some of the evidence that the Board of Directors had passed a resolution nor is there any
private respondents was a mere "mistake" on the part of petitioner corporation minutes of whatever nature proving mistakes in the award of damages (sic).
since, pursuant to Resolution No. 6, dated September 8, 1980, and Resolution No. 10,
dated October 6, 1980, said gratuity pay should be given only upon the employees' With regard to the award of service incentive leave and others, the
retirement. Commission finds no cogent reason to disturb the appealed decision.

On November 20, 1985, public respondent, through its Second Division, We affirm.
dismissed the appeal for lack of merit, the pertinent portion of which states: 6
WHEREFORE, let the appealed decision be, as it is hereby, AFFIRMED and let
We cannot agree with the contention of respondents (petitioners') that the the instant appeal (be) dismissed for lack of merit.
Labor Arbiter a quo committed abuse of discretion in his decision.
SO ORDERED.
Respondents' (petitioners') contention that, the two (2) resolutions dated 17
August 1981 and 1 September 1981 . . . which were not approved in the Petitioners reconsidered. 7 In their motion for reconsideration, petitioners
annual stockholders meeting had no force and effect, deserves scant assailed the validity of the board resolutions passed on August 17, 1981 and
consideration. The records show that the stockholders did not revoke nor September 1, 1981, respectively, and claimed, for the first time, that petitioner
nullify these resolutions granting gratuities to complainants. Asuncion Lopez Gonzales was not notified of the special board meetings held on said
dates. The motion for reconsideration was denied by the Second Division on July 24,
On record, it appears that the said resolutions arose from the 1986.
legitimate creation of the Board of Directors who steered the corporate On September 4, 1986, petitioners filed another motion for reconsideration. Again,
affairs of the corporation. . . . the motion was denied by public respondent in a Minute Resolution dated November
19, 1986. 8
Respondents' (petitioners') allegation that the three (3)
complainants, Mila E. Refuerzo, Marissa S. Pascual and Edward Mamaril, who Hence, the petition. As prayed for, we issued a Temporary Restraining
had resigned after filing the complaint on February 8, 1982, were precluded Order, 9 enjoining public respondent from enforcing or executing the Resolution,
to (sic) receive gratuity because the said resolutions referred to dated November 20, 1986 (sic), in NLRC-NCR-2-2176-82. 10
only retiring employee could not be given credence. A reading of Resolutions
dated 17 August 1981 and 1 September 1981 disclosed that there were periods The sole issue is whether or not public respondent acted with grave abuse
mentioned for the payment of complainants' gratuities. This disproves of discretion in holding that private respondents are entitled to receive their

132
MATABABE FILES
Corporation Law

gratuity pay under the assailed board resolutions dated August 17, 1951 and The general rule is that a corporation, through its board of directors, should
September 1, 1981. act in the manner and within the formalities, if any, prescribed by its charter or by the
general law. 14 Thus, directors must act as a body in a meeting called pursuant to the
Petitioners contend that the board resolutions passed on August 17, 1981 and law or the corporation's by-laws, otherwise, any action taken therein may be
September 1, 1981, granting gratuity pay to their retained employees, are ultra questioned by any objecting director or shareholder. 15
vires on the ground that petitioner Asuncion Lopez Gonzales was not duly notified of
the said special meetings. They aver, further, that said board resolutions were not Be that as it may, jurisprudence 16 tells us that an action of the board of
ratified by the stockholders of the corporation pursuant to Section 28 1/2 of the directors during a meeting, which was illegal for lack of notice, may be ratified either
Corporation Law (Section 40 of the Corporation Code). They also insist that the expressly, by the action of the directors in subsequent legal meeting, or impliedly, by
gratuity pay must be given only to the retiring employees, to the exclusion of the the corporation's subsequent course of conduct. Thus, in one case, 17 it was held:
retained employees or those who voluntarily resigned from their posts.
. . . In 2 Fletcher, Cyclopedia of the Law of Private Corporations (Perm. Ed.)
At the outset, we note that petitioners allegation on lack of notice to sec. 429, at page 290, it is stated:
petitioner Asuncion Lopez Gonzales was raised for the first time in the in their motion
for reconsideration filed before public respondent National Labor Relations Thus, acts of directors at a meeting which was illegal
Commission, or after said public respondent had affirmed the decision of the labor because of want of notice may be ratified by the directors
arbiter. To stress, in their appeal before the NLRC, petitioners never raised the issue at a subsequent legal meeting, or by the corporations
of lack of notice to Asuncion Lopez Gonzales. The appeal dealt with (a) the failure of course of conduct
the stockholders to ratify the assailed resolutions and (b) the alleged "mistake" ...
committed by petitioner corporation in giving the gratuity pay to some of its Fletcher, supra, further states in sec. 762, at page 1073-1074:
employees who are yet to retire from employment.
Ratification by directors may be by an express resolution or
In their comment, 11 private respondents maintain that the new ground of vote to that effect, or it may be implied from adoption of
lack of notice was not raised before the labor arbiter, hence, petitioners are barred the act, acceptance or acquiescence. Ratification may be
from raising the same on appeal. Private respondents claim, further, that such failure effected by a resolution or vote of the board of directors
on the part of petitioners, had deprived them the opportunity to present evidence expressly ratifying previous acts either of corporate officers
that, in a subsequent special board meeting held on September 29, 1981, the subject or agents; but it is not necessary, ordinarily, to show a
resolution dated September 1, 1981, was unanimously approved by the board of meeting and formal action by the board of directors in
directors of petitioner corporation, including petitioner Asuncion Lopez Gonzales. 12 order to establish a ratification.

Indeed, it would be offensive to the basic rules of fair play and justice to In American Casualty Co., v. Dakota Tractor and Equipment Co., 234 F. Supp.
allow petitioners to raise questions which have not been passed upon by the labor 606, 611 (D.N.D. 1964), the court stated:
arbiter and the public respondent NLRC. It is well settled that questions not raised in
the lower courts cannot, be raised for the first time on appeal. 13 Hence, petitioners Moreover, the unauthorized acts of an officer of a
may not invoke any other ground, other than those it specified at the labor arbiter corporation may be ratified by the corporation by conduct
level, to impugn the validity of the subject resolutions. implying approval and adoption of the act in question. Such
ratification may be express or may be inferred from silence
We now come to petitioners' argument that the resolutions passed by the and inaction.
board of directors during the special meetings on August 1, 1981, and September 1,
1981, were ultra vires for lack of notice. In the case at bench, it was established that petitioner corporation did not
issue any resolution revoking nor nullifying the board resolutions granting gratuity

133
MATABABE FILES
Corporation Law

pay to private respondents. Instead, they paid the gratuity pay, particularly, the first Petitioners try to convince us that the subject resolutions had no force and
two (2) installments thereof, of private respondents Florentina Fontecha, Mila effect in view of the non-approval thereof during the Annual Stockholders' Meeting
Refuerzo, Marcial Mamaril and Perfecto Bautista. held on March 1, 1982. To strengthen their position, petitioners cite section 28 1/2 of
the Corporation Law (Section 40 of the Corporation Code). We are not persuaded.
Despite the alleged lack of notice to petitioner Asuncion Lopez Gonzales at
that time the assailed resolutions were passed, we can glean from the records that The cited provision is not applicable to the case at bench as it refers to the
she was aware of the corporation's obligation under the said resolutions. More sale, lease, exchange or disposition of all or substantially all of the corporation's
importantly, she acquiesced thereto. As pointed out by private respondents, assets, including its goodwill. In such a case, the action taken by the board of
petitioner Asuncion Lopez Gonzales affixed her signature on Cash Voucher Nos. 81- directors requires the authorization of the stockholders on record.
10-510 and 81-10-506, both dated October 15, 1981, evidencing the 2nd installment of
the gratuity pay of private respondents Mila Refuerzo and Florentina Fontecha. 18 It will be observed that, except far Arturo Lopez, the stockholders of
petitioner corporation also sit as members of the board of directors. Under the
We hold, therefore, that the conduct of petitioners after the passage of circumstances in field, it will be illogical and superfluous to require the stockholders'
resolutions dated August, 17, 1951 and September 1, 1981, had estopped them from approval of the subject resolutions. Thus, even without the stockholders' approval of
assailing the validity of said board resolutions. the subject resolutions, petitioners are still liable to pay private respondents' gratuity
pay.
Assuming, arguendo, that there was no notice given to Asuncion Lopez
Gonzalez during the special meetings held on August 17, 1981 and September 1, 1981, IN VIEW WHEREOF, the instant petition is DISMISSED for lack of merit and
it is erroneous to state that the resolutions passed by the board during the said the temporary restraining order we issued on February 9, 1987 is LIFTED. Accordingly,
meetings were ultra vires. In legal parlance, "ultra vires" act refers to one which is not the assailed resolution of the National Labor Relations Commission in NLRC-NCR-
within the corporate powers conferred by the Corporation Code or articles of 2176-82 is AFFIRMED. This decision is immediately executory. Costs against
incorporation or not necessary or incidental in the exercise of the powers so petitioners.
conferred. 19
SO ORDERED.
The assailed resolutions before us cover a subject which concerns the
benefit and welfare of the company's employees. To stress, providing gratuity pay Narvasa, C.J., Regalado, Mendoza and Francisco, JJ., concur.
for its employees is one of the express powers of the corporation under the
Corporation Code, hence, petitioners cannot invoke the doctrine of ultra vires to
avoid any liability arising from the issuance the subject resolutions. 20

We reject petitioners' allegation that private respondents, namely, Mila


Refuerzo, Marissa Pascual and Edward Mamaril who resigned from petitioner
corporation after the filing of the case, are precluded from receiving their gratuity
pay. Pursuant to board resolutions dated August 17, 1981 and September 1, 1981,
respectively, petitioner corporation obliged itself to give the gratuity pay of its
retained employees in four (4) installments: on September 1, 1981; October 15, 1981;
November, 1981; and January 1, 1982. Hence, at the time the aforenamed private
respondents tendered their resignation, the aforementioned private respondents
were already entitled to receive their gratuity pay.

134
MATABABE FILES
Corporation Law

Republic of the Philippines and for the year ended December 31, 1977... and the audited financial statements of
SUPREME COURT FARMACOR as of September 30, 1978 being prepared by S[ycip,] G[orres,] V[elayo and Co.]...
Manila fairly present or will present the financial position of FARMACOR and the results of its
THIRD DIVISION operations as of said respective dates; said financial statements show or will show all liabilities
and commitments of FARMACOR, direct or contingent, as of said respective dates . . ."; and
G.R. No. 125778 June 10, 2003 "(v.) [t]he Minimum Guaranteed Net Worth of FARMACOR as of September 30, 1978 shall be
Twelve Million Pesos (P12,000,000.00)."6
INTER-ASIA INVESTMENTS INDUSTRIES, INC., Petitioner,
vs. The Agreement was later amended with respect to the "Closing Date," originally set
COURT OF APPEALS and ASIA INDUSTRIES, INC., Respondents. up at 10:00 a.m. of September 30, 1978, which was moved to October 31, 1978, and to the
mode of payment of the purchase price.7
Corporation Law; An officer of a corporation who is authorized to purchase the stock The Agreement, as amended, provided that pending submission by SGV of FARMACORs
of another corporation has the implied power to perform all other obligations arising audited financial statements as of October 31, 1978, private respondent may retain the sum of
therefrom such as payment of the shares of stock.As correctly argued by private P7,500,000.00 out of the stipulated purchase price of P19,500,000.00; that from this retained
respondent, an officer of a corporation who is authorized to purchase the stock of another amount of P7,500,000.00, private respondent may deduct any shortfall on the Minimum
corporation has the implied power to perform all other obligations arising therefrom, such as Guaranteed Net Worth of P12,000,000.00;8 and that if the amount retained is not sufficient to
payment of the shares of stock. By allowing its president to sign the Agreement on its behalf, make up for the deficiency in the Minimum Guaranteed Net Worth, petitioner shall pay the
petitioner clothed him with apparent capacity to perform all acts which are expressly, difference within 5 days from date of receipt of the audited financial statements.9
impliedly and inherently stated therein.
Respondent paid petitioner a total amount of P 12,000,000.00: P5,000,000.00 upon
PETITION for review on certiorari of the decision and resolution of the Court of Appeals. the signing of the Agreement, and P7,000,000.00 on November 2, 1978. 10

The facts are stated in the opinion of the Court. From the STATEMENT OF INCOME AND DEFICIT attached to the financial
report11 dated November 28, 1978 submitted by SGV, it appears that FARMACOR had, for the
DECISION ten months ended October 31, 1978, a deficit of P11,244,225.00. 12 Since the stockholders
CARPIO-MORALES, J.: equity amounted to P10,000,000.00, FARMACOR had a net worth deficiency of P1,244,225.00.
The guaranteed net worth shortfall thus amounted to P13,244,225.00 after adding the net
The present petition for review on certiorari assails the Court of Appeals Decision1 of worth deficiency of P1,244,225.00 to the Minimum Guaranteed Net Worth of P12,000,000.00.
January 25, 1996 and Resolution2 of July 11, 1996.
The adjusted contract price, therefore, amounted to P6,225,775.00 which is the
The material facts of the case are as follows: difference between the contract price of P19,500,000.00 and the shortfall in the guaranteed
net worth of P13,224,225.00. Private respondent having already paid petitioner
On September 1, 1978, Inter-Asia Industries, Inc. (petitioner), by a Stock Purchase P12,000,000.00, it was entitled to a refund of P5,744,225.00.
Agreement3 (the Agreement), sold to Asia Industries, Inc. (private respondent) for and in
consideration of the sum of P19,500,000.00 all its right, title and interest in and to all the Petitioner thereafter proposed, by letter13 of January 24, 1980, signed by its
outstanding shares of stock of FARMACOR, INC. (FARMACOR). 4 The Agreement was signed president, that private respondents claim for refund be reduced to P4,093,993.00, it
by Leonides P. Gonzales and Jesus J. Vergara, presidents of petitioner and private promising to pay the cost of the Northern Cotabato Industries, Inc. (NOCOSII)
respondent, respectively.5 superstructures in the amount of P759,570.00. To the proposal respondent agreed.
Petitioner, however, weiched on its promise. Petitioners total liability thus stood at
Under paragraph 7 of the Agreement, petitioner as seller made warranties and P4,853,503.00 (P4,093,993.00 plus P759,570.00)14 exclusive of interest.15
representations among which were "(iv.) [t]he audited financial statements of FARMACOR at

135
MATABABE FILES
Corporation Law

On April 5, 1983, private respondent filed a complaint16 against petitioner with the THE RESPONDENT COURT ERRED IN NOT DECLARING THAT THERE IS NO BREACH OF
Regional Trial Court of Makati, one of two causes of action of which was for the recovery of WARRANTIES AND REPRESENTATION AS ALLEGED BY THE PRIVATE RESPONDENT.
above-said amount of P4,853,503.0017 plus interest.
Denying private respondents claim, petitioner countered that private respondent failed to IV
pay the balance of the purchase price and accordingly set up a counterclaim. THE RESPONDENT COURT ERRED IN ORDERING THE PETITIONER TO PAY ATTORNEYS FEES
AND IN SUSTAINING THE DISMISSAL OF THE COUNTERCLAIM.18 (Underscoring in the
Finding for private respondent, the trial court rendered on November 27, 1991 a original)
Decision,18 the dispositive portion of which reads:
WHEREFORE, judgment is rendered in favor of plaintiff and against defendant (a) ordering Petitioner argues that the January 24, 1980 letter-proposal (for the reduction of
the latter to pay to the former the sum of P4,853,503.00 19 plus interest thereon at the legal private respondents claim for refund upon petitioners promise to pay the cost of NOCOSII
rate from the filing of the complaint until fully paid, the sum of P30,000.00 as attorneys fees superstructures in the amount of P759,570.00) which was signed by its president has no legal
and the costs of suit; and (b) dismissing the counterclaim. force and effect against it as it was not authorized by its board of directors, it citing the
Corporation Law which provides that unless the act of the president is authorized by the
SO ORDERED. board of directors, the same is not binding on it.

On appeal to the Court of Appeals, petitioner raised the following errors: This Court is not persuaded.

THE TRIAL COURT ERRED IN HOLDING THE DEFENDANT LIABLE UNDER THE FIRST The January 24, 1980 letter signed by petitioners president is valid and binding. The
CAUSE OF ACTION PLEADED BY THE PLAINTIFF. case of Peoples Aircargo and Warehousing Co., Inc. v. Court of Appeals 19 instructs:

THE TRIAL COURT ERRED IN AWARDING ATTORNEYS FEES AND IN DISMISSING THE The general rule is that, in the absence of authority from the board of directors, no
COUNTERCLAIM. person, not even its officers, can validly bind a corporation. A corporation is a juridical
person, separate and distinct from its stockholders and members, "having x x x powers,
THE TRIAL COURT ERRED IN RENDERING JUDGMENT IN FAVOR OF THE PLAINTIFF, attributes and properties expressly authorized by law or incident to its existence."
THE ALLEGED BREACH OF WARRANTIES AND REPRESENTATION NOT HAVING BEEN
SHOWN, MUCH LESS ESTABLISHED BY THE PLAINTIFF.20 Being a juridical entity, a corporation may act through its board of directors, which
exercises almost all corporate powers, lays down all corporate business policies and is
By Decision of January 25, 1996, the Court of Appeals affirmed the trial courts responsible for the efficiency of management, as provided in Section 23 of the Corporation
decision. Petitioners motion for reconsideration of the decision having been denied by the Code of the Philippines:
Court of Appeals by Resolution of July 11, 1996, the present petition for review on certiorari
was filed, assigning the following errors: SEC. 23. The Board of Directors or Trustees. - Unless otherwise provided in this Code,
I the corporate powers of all corporations formed under this Code shall be exercised, all
THE RESPONDENT COURT ERRED IN NOT HOLDING THAT THE LETTER OF THE PRESIDENT OF business conducted and all property of such corporations controlled and held by the board of
THE PETITIONER IS NOT BINDING ON THE PETITIONER BEING ULTRA VIRES directors or trustees x x x.

II Under this provision, the power and responsibility to decide whether the corporation
THE LETTER CAN NOT BE AN ADMISSION AND WAIVER OF THE PETITIONER AS A should enter into a contract that will bind the corporation is lodged in the board, subject to
CORPORATION. the articles of incorporation, bylaws, or relevant provisions of law. However, just as a natural
person may authorize another to do certain acts for and on his behalf, the board of directors
III may validly delegate some of its functions and powers to officers, committees or
agents. The authority of such individuals to bind the corporation is generally derived from

136
MATABABE FILES
Corporation Law

law, corporate bylaws or authorization from the board, either expressly or impliedly by
habit, custom or acquiescence in the general course of business, viz: xxx

A corporate officer or agent may represent and bind the corporation in transactions (iv) The audited financial statements of FARMACOR as at and for the year ended December
with third persons to the extent that [the] authority to do so has been conferred upon him, 31, 1977 and
and this includes powers as, in the usual course of the particular business, are incidental to, or the audited financial statements of FARMACOR as at September 30, 1978 being prepared by
may be implied from, the powers intentionally conferred, powers added by custom and SGV pursuant toparagraph 6(b) fairly present or will present the financial position of FARM
usage, as usually pertaining to the particular officer or agent, and such apparent powers as ACOR and the results of its operations as of said respective dates; said financial statements
the corporation has caused person dealing with the officer or agent to believe that it has show or will show all liabilities and commitments of FARMACOR, direct or contingent, as of
conferred. said respective dates; and the receivables set forth in said financial statements are fully due
xxx and collectible, free and clear of any set-offs, defenses, claims and other impediments to their
collectibility.
[A]pparent authority is derived not merely from practice. Its existence may be
ascertained through (1) the general manner in which the corporation holds out an officer or (v) The Minimum Guaranteed Net Worth of FARMACOR as of September 30, 1978 shall be
agent as having the power to act or, in other words the apparent authority to act in general, Twelve Million Pesos (P12,000,000.00), Philippine Currency.
with which it clothes him; or (2) the acquiescence in his acts of a particular nature, with
actual or constructive knowledge thereof, within or beyond the scope of his ordinary x x x (Underscoring in the original; emphasis supplied)23
powers. It requires presentation of evidence of similar act(s) executed either in its favor or
in favor of other parties. It is not the quantity of similar acts which establishes True, private respondent accepted as correct the financial statements submitted to it
apparent authority, but the vesting of when the Agreement was executed on September 1, 1978. But petitioner expressly warranted
a corporate officer with power to bind the corporation. that the SGV Reports "fairly present or will present the financial position of FARMACOR." By
such warranty, petitioner is estopped from claiming that the SGV Reports are self-serving and
x x x (Emphasis and underscoring supplied) biased.1wphi1
As correctly argued by private respondent, an officer of a corporation who is As to the claim that the shortfall occurred after the execution of the Agreement, the
authorized to purchase the stock of another corporation has the implied power to perform all declaration of Emmanuel de Asis, supervisor in the Accounting Division of SGV and head of
other obligations arising therefrom, such as payment of the shares of stock. By allowing its the team which conducted the auditing of FARMACOR, that the period covered by the audit
president to sign the Agreement on its behalf, petitioner clothed him with apparent capacity was from January to October 1978 shows that the period before the Agreement was entered
to perform all acts which are expressly, impliedly and inherently stated therein. 21 into (on September 1, 1978) was covered.24

Petitioner further argues that when the Agreement was executed on September 1, As to petitioners assigned error on the award of attorneys fees which, it argues, is
1978, its financial statements were extensively examined and accepted as correct by private bereft of factual, legal and equitable justification, this Court finds the same well-taken.
respondent, hence, it cannot later be disproved "by resorting to some scheme such as future
financial auditing;"22 and that it should not be bound by the SGV Report because it is self- On the matter of attorneys fees, it is an accepted doctrine that the award thereof as
serving and biased, SGV having been hired solely by private respondent, and the alleged an item of damages is the exception rather than the rule, and counsels fees are not to be
shortfall of FARMACOR occurred only after the execution of the Agreement. awarded every time a party wins a suit. The power of the court to award
attorneys fees under Article 2208 of the Civil Code demands
This Court is not persuaded either. factual, legal and equitable justification, without which the award is
a conclusion without a premise, its basis being improperly left
The pertinent provisions of the Agreement read: to speculation and conjecture. In all events, the court must explicitly state in the text of the
decision, and not only in the decretal portion thereof, the legal reason for the award of
7. Warranties and Representations - (a) SELLER warrants and represents as follows: attorneys fees.25

137
MATABABE FILES
Corporation Law

x x x (Emphasis and underscoring supplied; citations omitted)

WHEREFORE, the instant petition is PARTLY GRANTED. The assailed decision of the
Court of Appeals affirming that of the trial court is modified in that the award of attorneys
fees in favor of private respondent is deleted. The decision is affirmed in other respects.

SO ORDERED.

Puno, (Chairman), Panganiban, Sandoval-Gutierrez, and Corona, JJ., concur.

138
MATABABE FILES
Corporation Law

Republic of the Philippines upon mere meeting of the minds. When there is merely an offer by one party without
SUPREME COURT acceptance of the other, there is no contract. When the contract of sale is not perfected, it
Manila cannot, as an independent source of obligation, serve as a binding juridical relation between
FIRST DIVISION the parties.

G.R. No. 166862 December 20, 2006 Same; Same; Stages of a Contract of Sale.In San Miguel Properties Philippines, Inc.
v. Huang, 336 SCRA 737 (2000), the Court ruled that the stages of a contract of sale are as
MANILA METAL CONTAINER CORPORATION, petitioner, follows: (1) negotiation, covering the period from the time the prospective contracting parties
REYNALDO C. TOLENTINO, intervenor, indicate interest in the contract to the time the contract is perfected; (2) perfection, which
vs. takes place upon the concurrence of the essential elements of the sale which are the meeting
PHILIPPINE NATIONAL BANK, respondent, of the minds of the parties as to the object of the contract and upon the price; and (3)
DMCI-PROJECT DEVELOPERS, INC., intervenor. consummation, which begins when the parties perform their respective undertakings under
the contract of sale, culminating in the extinguishment thereof.
Contracts; Requisites; Contracts are perfected by mere consent which is manifested by
the meeting of the offer and the acceptance upon the thing and the cause which are to Same; Same; Same; To convert an offer into a contract, the acceptance must be
constitute the contract.A contract is a meeting of minds between two persons whereby one absolute and must not qualify the terms of the offerit must be plain, unequivocal,
binds himself, with respect to the other, to give something or to render some service. Under unconditional and without variance of any sort from the proposal.A negotiation is formally
Article 1318 of the New Civil Code, there is no contract unless the following requisites concur: initiated by an offer, which, however, must be certain. At any time prior to the perfection of
(1) Consent of the contracting parties; (2) Object certain which is the subject matter of the the contract, either negotiating party may stop the negotiation. At this stage, the offer may
contract; (3) Cause of the obligation which is established. Contracts are perfected by mere be withdrawn; the withdrawal is effective immediately after its manifestation. To convert the
consent which is manifested by the meeting of the offer and the acceptance upon the thing offer into a contract, the acceptance must be absolute and must not qualify the terms of the
and the cause which are to constitute the contract. Once perfected, they bind other offer; it must be plain, unequivocal, unconditional and without variance of any sort from the
contracting parties and the obligations arising therefrom have the form of law between the proposal.
parties and should be complied with in good faith. The parties are bound not only to the
fulfillment of what has been expressly stipulated but also to the consequences which, Same; Same; Same; A counter-offer is considered in law, a rejection of the original
according to their nature, may be in keeping with good faith, usage and law. offer and an attempt to end the negotiation between the parties on a different basis.A
qualified acceptance or one that involves a new proposal constitutes a counter-offer and a
Same; Sales; A definite agreement as to the price is an essential element of a binding rejection of the original offer. A counter-offer is considered in law, a rejection of the original
agreement to sell personal or real property because it seriously affects the rights and offer and an attempt to end the negotiation between the parties on a different basis.
obligations of the parties; When the contract of sale is not perfected, it cannot, as an Consequently, when something is desired which is not exactly what is proposed in the offer,
independent source of obligation, serve as a binding juridical relation between the parties.By such acceptance is not sufficient to guarantee consent because any modification or variation
the contract of sale, one of the contracting parties obligates himself to transfer the from the terms of the offer annuls the offer. The acceptance must be identical in all respects
ownership of and deliver a determinate thing, and the other to pay therefor a price certain in with that of the offer so as to produce consent or meeting of the minds.
money or its equivalent. The absence of any of the essential elements will negate the
existence of a perfected contract of sale. As the Court ruled in Boston Bank of the Philippines Same; Same; Corporation Law; Board of Directors; Contracts or acts of a corporation
v. Manalo, 482 SCRA 108 (2006): A definite agreement as to the price is an essential element must be made either by the board of directors or by a corporate agent duly authorized by the
of a binding agreement to sell personal or real property because it seriously affects the rights boardabsent such valid delegation/authorization, the rule is that the declarations of an
and obligations of the parties. Price is an essential element in the formation of a binding and individual director relating to the affairs of the corporation, but not in the course of, or
enforceable contract of sale. The fixing of the price can never be left to the decision of one of connected with the performance of authorized duties of such director, are held not binding
the contracting parties. But a price fixed by one of the contracting parties, if accepted by the on the corporation.There is no evidence that the SAMD was authorized by respondents
other, gives rise to a perfected sale. A contract of sale is consensual in nature and is perfected Board of Directors to accept petitioners offer and sell the property for P1,574,560.47. Any

139
MATABABE FILES
Corporation Law

acceptance by the SAMD of petitioners offer would not bind respondent. As this Court ruled of P1,000,000.00; and, on November 16, 1973, petitioner executed an Amendment 4 of Real
in AF Realty Development, Inc. vs. Diesehuan Freight Services, Inc., 373 SCRA 385 (2002): Estate Mortgage over its property. On March 31, 1981, petitioner secured another loan
Section 23 of the Corporation Code expressly provides that the corporate powers of all of P653,000.00 from respondent PNB, payable in quarterly installments of P32,650.00, plus
corporations shall be exercised by the board of directors. Just as a natural person may interests and other charges.5
authorize another to do certain acts in his behalf, so may the board of directors of a
corporation validly delegate some of its functions to individual officers or agents appointed On August 5, 1982, respondent PNB filed a petition for extrajudicial foreclosure of the
by it. Thus, contracts or acts of a corporation must be made either by the board of directors real estate mortgage and sought to have the property sold at public auction for P911,532.21,
or by a corporate agent duly authorized by the board. Absent such valid petitioner's outstanding obligation to respondent PNB as of June 30, 1982, 6 plus interests and
delegation/authorization, the rule is that the declarations of an individual director relating to attorney's fees.
the affairs of the corporation, but not in the course of, or connected with the performance of
authorized duties of such director, are held not binding on the corporation. After due notice and publication, the property was sold at public auction on
September 28, 1982 where respondent PNB was declared the winning bidder
Same; Same; Earnest Money; Absent proof of the concurrence of all the essential for P1,000,000.00. The Certificate of Sale 7 issued in its favor was registered with the Office of
elements of a contract of sale, the giving of earnest money cannot establish the existence of a the Register of Deeds of Rizal, and was annotated at the dorsal portion of the title on
perfected contract of sale.The P725,000.00 was merely a deposit to be applied as part of February 17, 1983. Thus, the period to redeem the property was to expire on February 17,
the purchase price of the property, in the event that respondent would approve the 1984.
recommendation of SAMD for respondent to accept petitioners offer to purchase the
property for P1,574,560.47. Unless and until the respondent accepted the offer on these Petitioner sent a letter dated August 25, 1983 to respondent PNB, requesting that it
terms, no perfected contract of sale would arise. Absent proof of the concurrence of all the be granted an extension of time to redeem/repurchase the property. 8 In its reply dated
essential elements of a contract of sale, the giving of earnest money cannot establish the August 30, 1983, respondent PNB informed petitioner that the request had been referred to
existence of a perfected contract of sale. its Pasay City Branch for appropriate action and recommendation.9
PETITION for review on certiorari of the decision and resolution of the Court of Appeals.
In a letter10 dated February 10, 1984, petitioner reiterated its request for a one year
The facts are stated in the opinion of the Court Manila Metal Container Corporation extension from February 17, 1984 within which to redeem/repurchase the property on
vs. Philippine National Bank, 511 SCRA 444, G.R. No. 166862 December 20, 2006 installment basis. It reiterated its request to repurchase the property on
installment.11 Meanwhile, some PNB Pasay City Branch personnel informed petitioner that as a
matter of policy, the bank does not accept "partial redemption." 12
DECISION
CALLEJO, SR., J.: Since petitioner failed to redeem the property, the Register of Deeds cancelled TCT
Before us is a petition for review on certiorari of the Decision1 of the Court of Appeals (CA) in No. 32098 on June 1, 1984, and issued a new title in favor of respondent PNB. 13 Petitioner's
CA-G.R. No. 46153 which affirmed the decision2 of the Regional Trial Court (RTC), Branch 71, offers had not yet been acted upon by respondent PNB.
Pasig City, in Civil Case No. 58551, and its Resolution 3 denying the motion for reconsideration
filed by petitioner Manila Metal Container Corporation (MMCC). Meanwhile, the Special Assets Management Department (SAMD) had prepared a
statement of account, and as of June 25, 1984 petitioner's obligation amounted
The Antecedents to P1,574,560.47. This included the bid price of P1,056,924.50, interest, advances of insurance
premiums, advances on realty taxes, registration expenses, miscellaneous expenses and
Petitioner was the owner of a 8,015 square meter parcel of land located in publication cost.14 When apprised of the statement of account, petitioner
Mandaluyong (now a City), Metro Manila. The property was covered by Transfer Certificate of remitted P725,000.00 to respondent PNB as "deposit to repurchase," and Official Receipt No.
Title (TCT) No. 332098 of the Registry of Deeds of Rizal. To secure a P900,000.00 loan it had 978191 was issued to it.15
obtained from respondent Philippine National Bank (PNB), petitioner executed a real estate
mortgage over the lot. Respondent PNB later granted petitioner a new credit accommodation

140
MATABABE FILES
Corporation Law

In the meantime, the SAMD recommended to the management of respondent PNB of P1,574,560.47 as approved by its SMAD and considering the reliance made by
that petitioner be allowed to repurchase the property for P1,574,560.00. In a letter dated Manila Metal and the long time that has elapsed, the approval of the higher
November 14, 1984, the PNB management informed petitioner that it was rejecting the offer management of the Bank to confirm the agreement of its SMAD is clearly a
and the recommendation of the SAMD. It was suggested that petitioner purchase the potestative condition which cannot legally prejudice Manila Metal which has acted
property for P2,660,000.00, its minimum market value. Respondent PNB gave petitioner until and relied on the approval of SMAD. The Bank cannot take advantage of a condition
December 15, 1984 to act on the proposal; otherwise, its P725,000.00 deposit would be which is entirely dependent upon its own will after accepting and benefiting from the
returned and the property would be sold to other interested buyers. 16 substantial payment made by Manila Metal.

Petitioner, however, did not agree to respondent PNB's proposal. Instead, it wrote 35. PNB approved the repurchase price of P1,574,560.47 for which it
another letter dated December 12, 1984 requesting for a reconsideration. Respondent PNB accepted P725,000.00 from Manila Metal. PNB cannot take advantage of its own
replied in a letter dated December 28, 1984, wherein it reiterated its proposal that petitioner delay and long inaction in demanding a higher amount based on unilateral
purchase the property for P2,660,000.00. PNB again informed petitioner that it would return computation of interest rate without the consent of Manila Metal.
the deposit should petitioner desire to withdraw its offer to purchase the property. 17 On
February 25, 1985, petitioner, through counsel, requested that PNB reconsider its letter dated Petitioner later filed an amended complaint and supported its claim for damages with
December 28, 1984. Petitioner declared that it had already agreed to the SAMD's offer to the following arguments:
purchase the property for P1,574,560.47, and that was why it had paid P725,000.00. Petitioner
warned respondent PNB that it would seek judicial recourse should PNB insist on the 36. That in order to protect itself against the wrongful and malicious acts of the
position.18 defendant Bank, plaintiff is constrained to engage the services of counsel at an
agreed fee of P50,000.00 and to incur litigation expenses of at least P30,000.00,
On June 4, 1985, respondent PNB informed petitioner that the PNB Board of which the defendant PNB should be condemned to pay the plaintiff Manila Metal.
Directors had accepted petitioner's offer to purchase the property, but for P1,931,389.53 in
cash less the P725,000.00 already deposited with it.19 On page two of the letter was a space 37. That by reason of the wrongful and malicious actuations of defendant PNB,
above the typewritten name of petitioner's President, Pablo Gabriel, where he was to affix his plaintiff Manila Metal suffered besmirched reputation for which defendant PNB is
signature. However, Pablo Gabriel did not conform to the letter but merely indicated therein liable for moral damages of at least P50,000.00.
that he had received it.20 Petitioner did not respond, so PNB requested petitioner in a letter
dated June 30, 1988 to submit an amended offer to repurchase. 38. That for the wrongful and malicious act of defendant PNB which are highly
reprehensible, exemplary damages should be awarded in favor of the plaintiff by way
Petitioner rejected respondent's proposal in a letter dated July 14, 1988. It of example or correction for the public good of at least P30,000.00.23
maintained that respondent PNB had agreed to sell the property for P1,574,560.47, and that
since its P725,000.00 downpayment had been accepted, respondent PNB was proscribed Petitioner prayed that, after due proceedings, judgment be rendered in its favor, thus:
from increasing the purchase price of the property.21 Petitioner averred that it had a net
balance payable in the amount of P643,452.34. Respondent PNB, however, rejected a) Declaring the Amended Real Estate Mortgage (Annex "A") null and void and
petitioner's offer to pay the balance of P643,452.34 in a letter dated August 1, 1989.22 without any legal force and effect.

On August 28, 1989, petitioner filed a complaint against respondent PNB for b) Declaring defendant's acts of extra-judicially foreclosing the mortgage over
"Annulment of Mortgage and Mortgage Foreclosure, Delivery of Title, or Specific plaintiff's property and setting it for auction sale null and void.
Performance with Damages." To support its cause of action for specific performance, it
alleged the following: c) Ordering the defendant Register of Deeds to cancel the new title issued in the
name of PNB (TCT NO. 43792) covering the property described in paragraph 4 of the
34. As early as June 25, 1984, PNB had accepted the down payment from Manila Complaint, to reinstate TCT No. 37025 in the name of Manila Metal and to cancel the
Metal in the substantial amount of P725,000.00 for the redemption/repurchase price

141
MATABABE FILES
Corporation Law

annotation of the mortgage in question at the back of the TCT No. 37025 described in for P4,250,000.00 in cash.30The offer was again rejected by respondent PNB on September 13,
paragraph 4 of this Complaint. 1993.31

d) Ordering the defendant PNB to return and/or deliver physical possession of the On May 31, 1994, the trial court rendered judgment dismissing the amended
TCT No. 37025described in paragraph 4 of this Complaint to the plaintiff Manila complaint and respondent PNB's counterclaim. It ordered respondent PNB to refund
Metal. the P725,000.00 deposit petitioner had made.32 The trial court ruled that there was no
perfected contract of sale between the parties; hence, petitioner had no cause of action for
e) Ordering the defendant PNB to pay the plaintiff Manila Metal's actual damages, specific performance against respondent. The trial court declared that respondent had
moral and exemplary damages in the aggregate amount of not less than P80,000.00 rejected petitioner's offer to repurchase the property. Petitioner, in turn, rejected the terms
as may be warranted by the evidence and fixed by this Honorable Court in the and conditions contained in the June 4, 1985 letter of the SAMD. While petitioner had offered
exercise of its sound discretion, and attorney's fees of P50,000.00 and litigation to repurchase the property per its letter of July 14, 1988, the amount of P643,422.34 was way
expenses of at least P30,000.00 as may be proved during the trial, and costs of suit. below the P1,206,389.53 which respondent PNB had demanded. It further declared that
the P725,000.00 remitted by petitioner to respondent PNB on June 4, 1985 was a "deposit,"
Plaintiff likewise prays for such further reliefs which may be deemed just and and not a downpayment or earnest money.
equitable in the premises.24
On appeal to the CA, petitioner made the following allegations:
In its Answer to the complaint, respondent PNB averred, as a special and affirmative
defense, that it had acquired ownership over the property after the period to redeem had I
elapsed. It claimed that no contract of sale was perfected between it and petitioner after the THE LOWER COURT ERRED IN RULING THAT DEFENDANT-APPELLEE'S LETTER
period to redeem the property had expired. DATED 4 JUNE 1985 APPROVING/ACCEPTING PLAINTIFF-APPELLANT'S OFFER TO
During pre-trial, the parties agreed to submit the case for decision, based on their stipulation PURCHASE THE SUBJECT PROPERTY IS NOT VALID AND ENFORCEABLE.
of facts.25 The parties agreed to limit the issues to the following:
II
1. Whether or not the June 4, 1985 letter of the defendant approving/accepting THE LOWER COURT ERRED IN RULING THAT THERE WAS NO PERFECTED CONTRACT
plaintiff's offer to purchase the property is still valid and legally enforceable. OF SALE BETWEEN PLAINTIFF-APPELLANT AND DEFENDANT-APPELLEE.

2. Whether or not the plaintiff has waived its right to purchase the property when it III
failed to conform with the conditions set forth by the defendant in its letter dated THE LOWER COURT ERRED IN RULING THAT PLAINTIFF-APPELLLANT WAIVED ITS
June 4, 1985. RIGHT TO PURCHASE THE SUBJECT PROPERTY WHEN IT FAILED TO CONFORM WITH
CONDITIONS SET FORTH BY DEFENDANT-APPELLEE IN ITS LETTER DATED 4 JUNE
3. Whether or not there is a perfected contract of sale between the parties. 26 1985.

While the case was pending, respondent PNB demanded, on September 20, 1989, IV
that petitioner vacate the property within 15 days from notice, 27 but petitioners refused to do THE LOWER COURT ERRED IN DISREGARDING THE FACT THAT IT WAS THE
so. DEFENDANT-APPELLEE WHICH RENDERED IT DIFFICULT IF NOT IMPOSSIBLE FOR
PLAINTIFF-APPELLANT TO COMPLETE THE BALANCE OF THEIR PURCHASE PRICE.
On March 18, 1993, petitioner offered to repurchase the property
for P3,500,000.00.28 The offer was however rejected by respondent PNB, in a letter dated V
April 13, 1993. According to it, the prevailing market value of the property was THE LOWER COURT ERRED IN DISREGARDING THE FACT THAT THERE WAS NO VALID
approximately P30,000,000.00, and as a matter of policy, it could not sell the property for less RESCISSION OR CANCELLATION OF SUBJECT CONTRACT OF REPURCHASE.
than its market value.29 On June 21, 1993, petitioner offered to purchase the property

142
MATABABE FILES
Corporation Law

VI balance of the repurchase be reduced, however, respondent rejected the proposal in a letter
THE LOWER COURT ERRED IN DECLARING THAT PLAINTIFF FAILED AND REFUSED TO dated August 1, 1989.
SUBMIT THE AMENDED REPURCHASE OFFER.
Petitioner filed a motion for reconsideration, which the CA likewise denied.
VII Thus, petitioner filed the instant petition for review on certiorari, alleging that:
THE LOWER COURT ERRED IN DISMISSING THE AMENDED COMPLAINT OF
PLAINTIFF-APPELLANT. I. THE COURT OF APPEALS ERRED ON A QUESTION OF LAW WHEN IT RULED THAT
THERE IS NO PERFECTED CONTRACT OF SALE BETWEEN THE PETITIONER AND
VIII RESPONDENT.
THE LOWER COURT ERRED IN NOT AWARDING PLAINTIFF-APPELLANT ACTUAL,
MORAL AND EXEMPLARY DAMAGES, ATTOTRNEY'S FEES AND LITIGATION II. THE COURT OF APPEALS ERRED ON A QUESTION OF LAW WHEN IT RULED THAT
EXPENSES.33 THE AMOUNT OF PHP725,000.00 PAID BY THE PETITIONER IS NOT AN EARNEST
MONEY.
Meanwhile, on June 17, 1993, petitioner's Board of Directors approved Resolution No.
3-004, where it waived, assigned and transferred its rights over the property covered by TCT III. THE COURT OF APPEALS ERRED ON A QUESTION OF LAW WHEN IT RULED THAT
No. 33099 and TCT No. 37025 in favor of Bayani Gabriel, one of its Directors. 34 Thereafter, THE FAILURE OF THE PETITIONER-APPELLANT TO SIGNIFY ITS CONFORMITY TO THE
Bayani Gabriel executed a Deed of Assignment over 51% of the ownership and management of TERMS CONTAINED IN PNB'S JUNE 4, 1985 LETTER MEANS THAT THERE WAS NO
the property in favor of Reynaldo Tolentino, who later moved for leave to intervene as VALID AND LEGALLY ENFORCEABLE CONTRACT OF SALE BETWEEN THE PARTIES.
plaintiff-appellant. On July 14, 1993, the CA issued a resolution granting the motion,35 and
likewise granted the motion of Reynaldo Tolentino substituting petitioner MMCC, as plaintiff- IV. THE COURT OF APPEALS ERRED ON A QUESTION OF LAW THAT NON-PAYMENT
appellant, and his motion to withdraw as intervenor.36 OF THE PETITIONER-APPELLANT OF THE BALANCE OF THE OFFERED PRICE IN THE
LETTER OF PNB DATED JUNE 4, 1985, WITHIN SIXTY (60) DAYS FROM NOTICE OF
The CA rendered judgment on May 11, 2000 affirming the decision of the RTC. 37 It APPROVAL CONSTITUTES NO VALID AND LEGALLY ENFORCEABLE CONTRACT OF
declared that petitioner obviously never agreed to the selling price proposed by respondent SALE BETWEEN THE PARTIES.
PNB (P1,931,389.53) since petitioner had kept on insisting that the selling price should be
lowered to P1,574,560.47. Clearly therefore, there was no meeting of the minds between the V. THE COURT OF APPEALS SERIOUSLY ERRED WHEN IT HELD THAT THE LETTERS OF
parties as to the price or consideration of the sale. PETITIONER-APPELLANT DATED MARCH 18, 1993 AND JUNE 21, 1993, OFFERING TO
BUY THE SUBJECT PROPERTY AT DIFFERENT AMOUNT WERE PROOF THAT THERE IS
The CA ratiocinated that petitioner's original offer to purchase the subject property NO PERFECTED CONTRACT OF SALE.38
had not been accepted by respondent PNB. In fact, it made a counter-offer through its June 4,
1985 letter specifically on the selling price; petitioner did not agree to the counter-offer; and The threshold issue is whether or not petitioner and respondent PNB had entered
the negotiations did not prosper. Moreover, petitioner did not pay the balance of the into a perfected contract for petitioner to repurchase the property from respondent.
purchase price within the sixty-day period set in the June 4, 1985 letter of respondent PNB.
Consequently, there was no perfected contract of sale, and as such, there was no contract to Petitioner maintains that it had accepted respondent's offer made through the
rescind. SAMD, to sell the property for P1,574,560.00. When the acceptance was made in its letter
dated June 25, 1984; it then deposited P725,000.00 with the SAMD as partial payment,
According to the appellate court, the claim for damages and the counterclaim were evidenced by Receipt No. 978194 which respondent had issued. Petitioner avers that the
correctly dismissed by the court a quo for no evidence was presented to support it. SAMD's acceptance of the deposit amounted to an acceptance of its offer to repurchase.
Respondent PNB's letter dated June 30, 1988 cannot revive the failed negotiations between Moreover, as gleaned from the letter of SAMD dated June 4, 1985, the PNB Board of Directors
the parties. Respondent PNB merely asked petitioner to submit an amended offer to had approved petitioner's offer to purchase the property. It claims that this was the
repurchase. While petitioner reiterated its request for a lower selling price and that the suspensive condition, the fulfillment of which gave rise to the contract.

143
MATABABE FILES
Corporation Law

considered as the counter-offer since as admitted by petitioner, it was only recommendation


Respondent could no longer unilaterally withdraw its offer to sell the property which was subject to approval of the PNB Board of Directors.
for P1,574,560.47, since the acceptance of the offer resulted in a perfected contract of sale; it
was obliged to remit to respondent the balance of the original purchase price Neither can the receipt by the SAMD of P725,000.00 be regarded as evidence of a
of P1,574,560.47, while respondent was obliged to transfer ownership and deliver the perfected sale contract. As gleaned from the parties' Stipulation of Facts during the
property to petitioner, conformably with Article 1159 of the New Civil Code. proceedings in the court a quo, the amount is merely an acknowledgment of the receipt
Petitioner posits that respondent was proscribed from increasing the interest rate of P725,000.00 as deposit to repurchase the property. The deposit of P725,000.00 was
after it had accepted respondent's offer to sell the property for P1,574,560.00. Consequently, accepted by respondent on the condition that the purchase price would still be approved by
respondent could no longer validly make a counter-offer of P1,931,789.88 for the purchase of its Board of Directors. Respondent maintains that its acceptance of the amount was qualified
the property. It likewise maintains that, although the P725,000.00 was considered as "deposit by that condition, thus not absolute. Pending such approval, it cannot be legally claimed that
for the repurchase of the property" in the receipt issued by the SAMD, the amount respondent is already bound by any contract of sale with petitioner.
constitutes earnest money as contemplated in Article 1482 of the New Civil Code. Petitioner According to respondent, petitioner knew that the SAMD has no capacity to bind respondent
cites the rulings of this Court in Villonco v. Bormaheco39 and Topacio v. Court of Appeals.40 and that its authority is limited to administering, managing and preserving the properties and
other special assets of PNB. The SAMD does not have the power to sell, encumber, dispose
Petitioner avers that its failure to append its conformity to the June 4, 1984 letter of of, or otherwise alienate the assets, since the power to do so must emanate from its Board of
respondent and its failure to pay the balance of the price as fixed by respondent within the Directors. The SAMD was not authorized by respondent's Board to enter into contracts of
60-day period from notice was to protest respondent's breach of its obligation to petitioner. sale with third persons involving corporate assets. There is absolutely nothing on record that
It did not amount to a rejection of respondent's offer to sell the property since respondent respondent authorized the SAMD, or made it appear to petitioner that it represented itself as
was merely seeking to enforce its right to pay the balance of P1,570,564.47. In any event, having such authority.
respondent had the option either to accept the balance of the offered price or to cause the
rescission of the contract. Respondent reiterates that SAMD had informed petitioner that its offer to
repurchase had been approved by the Board subject to the condition, among others, "that
Petitioner's letters dated March 18, 1993 and June 21, 1993 to respondent during the the selling price shall be the total bank's claim as of documentation date x x x payable in cash
pendency of the case in the RTC were merely to compromise the pending lawsuit, they did (P725,000.00 already deposited) within 60 days from notice of approval." A new Statement of
not constitute separate offers to repurchase the property. Such offer to compromise should Account was attached therein indicating the total bank's claim to be P1,931,389.53 less
not be taken against it, in accordance with Section 27, Rule 130 of the Revised Rules of Court. deposit of P725,000.00, or P1,206,389.00. Furthermore, while respondent's Board of Directors
accepted petitioner's offer to repurchase the property, the acceptance was qualified, in that it
For its part, respondent contends that the parties never graduated from the required a higher sale price and subject to specified terms and conditions enumerated
"negotiation stage" as they could not agree on the amount of the repurchase price of the therein. This qualified acceptance was in effect a counter-offer, necessitating petitioner's
property. All that transpired was an exchange of proposals and counter-proposals, nothing acceptance in return.
more. It insists that a definite agreement on the amount and manner of payment of the price
are essential elements in the formation of a binding and enforceable contract of sale. There The Ruling of the Court
was no such agreement in this case. Primarily, the concept of "suspensive condition" signifies
a future and uncertain event upon the fulfillment of which the obligation becomes effective. The ruling of the appellate court that there was no perfected contract of sale
It clearly presupposes the existence of a valid and binding agreement, the effectivity of which between the parties on June 4, 1985 is correct.
is subordinated to its fulfillment. Since there is no perfected contract in the first place, there is
no basis for the application of the principles governing "suspensive conditions." A contract is a meeting of minds between two persons whereby one binds himself,
with respect to the other, to give something or to render some service. 41 Under Article 1318 of
According to respondent, the Statement of Account prepared by SAMD as of June the New Civil Code, there is no contract unless the following requisites concur:
25, 1984 cannot be classified as a counter-offer; it is simply a recital of its total monetary
claims against petitioner. Moreover, the amount stated therein could not likewise be (1) Consent of the contracting parties;

144
MATABABE FILES
Corporation Law

(2) Object certain which is the subject matter of the contract; immediately after its manifestation. To convert the offer into a contract, the acceptance must
(3) Cause of the obligation which is established. be absolute and must not qualify the terms of the offer; it must be plain, unequivocal,
unconditional and without variance of any sort from the proposal. In Adelfa Properties, Inc. v.
Contracts are perfected by mere consent which is manifested by the meeting of the Court of Appeals,51 the Court ruled that:
offer and the acceptance upon the thing and the cause which are to constitute the
contract.42 Once perfected, they bind other contracting parties and the obligations arising x x x The rule is that except where a formal acceptance is so required, although the
therefrom have the form of law between the parties and should be complied with in good acceptance must be affirmatively and clearly made and must be evidenced by some
faith. The parties are bound not only to the fulfillment of what has been expressly stipulated acts or conduct communicated to the offeror, it may be shown by acts, conduct, or
but also to the consequences which, according to their nature, may be in keeping with good words of the accepting party that clearly manifest a present intention or
faith, usage and law.43 determination to accept the offer to buy or sell. Thus, acceptance may be shown by
the acts, conduct, or words of a party recognizing the existence of the contract of
By the contract of sale, one of the contracting parties obligates himself to transfer sale.52
the ownership of and deliver a determinate thing, and the other to pay therefor a price
certain in money or its equivalent.44 The absence of any of the essential elements will negate A qualified acceptance or one that involves a new proposal constitutes a counter-
the existence of a perfected contract of sale. As the Court ruled in Boston Bank of the offer and a rejection of the original offer. A counter-offer is considered in law, a rejection of
Philippines v. Manalo:45 the original offer and an attempt to end the negotiation between the parties on a different
basis.53 Consequently, when something is desired which is not exactly what is proposed in the
A definite agreement as to the price is an essential element of a binding offer, such acceptance is not sufficient to guarantee consent because any modification or
agreement to sell personal or real property because it seriously affects the rights and variation from the terms of the offer annuls the offer.54 The acceptance must be identical in all
obligations of the parties. Price is an essential element in the formation of a binding respects with that of the offer so as to produce consent or meeting of the minds.
and enforceable contract of sale. The fixing of the price can never be left to the In this case, petitioner had until February 17, 1984 within which to redeem the property.
decision of one of the contracting parties. But a price fixed by one of the contracting However, since it lacked the resources, it requested for more time to redeem/repurchase the
parties, if accepted by the other, gives rise to a perfected sale.46 property under such terms and conditions agreed upon by the parties. 55 The request, which
was made through a letter dated August 25, 1983, was referred to the respondent's main
A contract of sale is consensual in nature and is perfected upon mere meeting of the branch for appropriate action.56 Before respondent could act on the request, petitioner again
minds. When there is merely an offer by one party without acceptance of the other, there is wrote respondent as follows:
no contract.47 When the contract of sale is not perfected, it cannot, as an independent source
of obligation, serve as a binding juridical relation between the parties. 48 1. Upon approval of our request, we will pay your goodselves ONE HUNDRED & FIFTY
THOUSAND PESOS (P150,000.00);
In San Miguel Properties Philippines, Inc. v. Huang,49 the Court ruled that the stages of
a contract of sale are as follows: (1) negotiation, covering the period from the time the 2. Within six months from date of approval of our request, we will pay another FOUR
prospective contracting parties indicate interest in the contract to the time the contract is HUNDRED FIFTY THOUSAND PESOS (P450,000.00); and
perfected; (2) perfection, which takes place upon the concurrence of the essential elements
of the sale which are the meeting of the minds of the parties as to the object of the contract 3. The remaining balance together with the interest and other expenses that will be
and upon the price; and (3) consummation, which begins when the parties perform their incurred will be paid within the last six months of the one year grave period
respective undertakings under the contract of sale, culminating in the extinguishment requested for.57
thereof.
When the petitioner was told that respondent did not allow "partial
A negotiation is formally initiated by an offer, which, however, must be certain. 50 At redemption,"58 it sent a letter to respondent's President reiterating its offer to purchase the
any time prior to the perfection of the contract, either negotiating party may stop the property.59 There was no response to petitioner's letters dated February 10 and 15, 1984.
negotiation. At this stage, the offer may be withdrawn; the withdrawal is effective

145
MATABABE FILES
Corporation Law

The statement of account prepared by the SAMD stating that the net claim of We do not agree with petitioner's contention that the P725,000.00 it had remitted
respondent as of June 25, 1984 was P1,574,560.47 cannot be considered an unqualified to respondent was "earnest money" which could be considered as proof of the perfection of
acceptance to petitioner's offer to purchase the property. The statement is but a a contract of sale under Article 1482 of the New Civil Code. The provision reads:
computation of the amount which petitioner was obliged to pay in case respondent would
later agree to sell the property, including interests, advances on insurance premium, advances ART. 1482. Whenever earnest money is given in a contract of sale, it shall be
on realty taxes, publication cost, registration expenses and miscellaneous expenses. considered as part of the price and as proof of the perfection of the contract.

There is no evidence that the SAMD was authorized by respondent's Board of This contention is likewise negated by the stipulation of facts which the parties
Directors to accept petitioner's offer and sell the property for P1,574,560.47. Any acceptance entered into in the trial court:
by the SAMD of petitioner's offer would not bind respondent. As this Court ruled in AF Realty
Development, Inc. vs. Diesehuan Freight Services, Inc.:60 8. On June 8, 1984, the Special Assets Management Department (SAMD) of PNB
prepared an updated Statement of Account showing MMCC's total liability to PNB as
Section 23 of the Corporation Code expressly provides that the corporate powers of of June 25, 1984 to be P1,574,560.47 and recommended this amount as the
all corporations shall be exercised by the board of directors. Just as a natural person repurchase price of the subject property.
may authorize another to do certain acts in his behalf, so may the board of directors
of a corporation validly delegate some of its functions to individual officers or agents 9. On June 25, 1984, MMCC paid P725,000.00 to PNB as deposit to repurchase the
appointed by it. Thus, contracts or acts of a corporation must be made either by the property. The deposit of P725,000 was accepted by PNB on the condition that the
board of directors or by a corporate agent duly authorized by the board. Absent such purchase price is still subject to the approval of the PNB Board.62
valid delegation/authorization, the rule is that the declarations of an individual
director relating to the affairs of the corporation, but not in the course of, or Thus, the P725,000.00 was merely a deposit to be applied as part of the purchase
connected with the performance of authorized duties of such director, are held not price of the property, in the event that respondent would approve the recommendation of
binding on the corporation. SAMD for respondent to accept petitioner's offer to purchase the property for P1,574,560.47.
Unless and until the respondent accepted the offer on these terms, no perfected contract of
Thus, a corporation can only execute its powers and transact its business through its sale would arise. Absent proof of the concurrence of all the essential elements of a contract
Board of Directors and through its officers and agents when authorized by a board resolution of sale, the giving of earnest money cannot establish the existence of a perfected contract of
or its by-laws.61 sale.63

It appears that the SAMD had prepared a recommendation for respondent to accept It appears that, per its letter to petitioner dated June 4, 1985, the respondent had
petitioner's offer to repurchase the property even beyond the one-year period; it decided to accept the offer to purchase the property for P1,931,389.53. However, this
recommended that petitioner be allowed to redeem the property and pay P1,574,560.00 as amounted to an amendment of respondent's qualified acceptance, or an amended counter-
the purchase price. Respondent later approved the recommendation that the property be offer, because while the respondent lowered the purchase price, it still declared that its
sold to petitioner. But instead of the P1,574,560.47 recommended by the SAMD and to which acceptance was subject to the following terms and conditions:
petitioner had previously conformed, respondent set the purchase price at P2,660,000.00. In
fine, respondent's acceptance of petitioner's offer was qualified, hence can be at most 1. That the selling price shall be the total Bank's claim as of documentation date (pls.
considered as a counter-offer. If petitioner had accepted this counter-offer, a perfected see attached statement of account as of 5-31-85), payable in cash (P725,000.00
contract of sale would have arisen; as it turns out, however, petitioner merely sought to have already deposited) within sixty (60) days from notice of approval;
the counter-offer reconsidered. This request for reconsideration would later be rejected by
respondent. 2. The Bank sells only whatever rights, interests and participation it may have in the
property and you are charged with full knowledge of the nature and extent of said
rights, interests and participation and waive your right to warranty against eviction.

146
MATABABE FILES
Corporation Law

3. All taxes and other government imposts due or to become due on the property, as
well as expenses including costs of documents and science stamps, transfer fees,
etc., to be incurred in connection with the execution and registration of all covering
documents shall be borne by you;

4. That you shall undertake at your own expense and account the ejectment of the
occupants of the property subject of the sale, if there are any;

5. That upon your failure to pay the balance of the purchase price within sixty (60)
days from receipt of advice accepting your offer, your deposit shall be forfeited and
the Bank is thenceforth authorized to sell the property to other interested parties.

6. That the sale shall be subject to such other terms and conditions that the Legal
Department may impose to protect the interest of the Bank.64

It appears that although respondent requested petitioner to conform to its amended


counter-offer, petitioner refused and instead requested respondent to reconsider its
amended counter-offer. Petitioner's request was ultimately rejected and respondent offered
to refund its P725,000.00 deposit.

In sum, then, there was no perfected contract of sale between petitioner and
respondent over the subject property.

IN LIGHT OF ALL THE FOREGOING, the petition is DENIED.

The assailed decision is AFFIRMED. Costs against petitioner Manila Metal Container
Corporation.

SO ORDERED.

147
MATABABE FILES
Corporation Law

Republic of the Philippines Same; Notwithstanding the silence of Filports bylaws on the matter, we cannot rule
SUPREME COURT that the creation of the executive committee by the board of directors is illegal or unlawful.
Manila Notwithstanding the silence of Filports bylaws on the matter, we cannot rule that the
FIRST DIVISION creation of the executive committee by the board of directors is illegal or unlawful. One
reason is the absence of a showing as to the true nature and functions of said executive
G.R. No. 161886 March 16, 2007 committee considering that the executive committee, referred to in Section 35 of the
Corporation Code which is as powerful as the board of directors and in effect acting for the
FILIPINAS PORT SERVICES, INC., represented by stockholders, ELIODORO C. CRUZ and board itself, should be distinguished from other committees which are within the competency
MINDANAO TERMINAL AND BROKERAGE SERVICES, INC., Petitioners, of the board to create at anytime and whose actions require ratification and confirmation by
vs. the board. Another reason is that, ratiocinated by both the two (2) courts below, the Board of
VICTORIANO S. GO, ARSENIO LOPEZ CHUA, EDGAR C. TRINIDAD, HERMENEGILDO M. Directors has the power to create positions not provided for in Filports bylaws since the
TRINIDAD, JESUS SYBICO, MARY JEAN D. CO, HENRY CHUA, JOSELITO S. JAYME, ERNESTO S. board is the cor-porations governing body, clearly upholding the power of its board to
JAYME, and ELIEZER B. DE JESUS, Respondents. exercise its prerogatives in managing the business affairs of the corporation.

Corporation Law; Section 23 of the Corporation Code explicitly provides that unless Same; If the cause of the losses is merely error in judgment, not amounting to bad
otherwise provided therein, the corporate powers of all corporations formed under the Code faith or negligence, directors and/or officers are not liable.If the cause of the losses is merely
shall be exercised, all business conducted and all property of the corporation shall be error in business judgment, not amounting to bad faith or negligence, directors and/or
controlled and held by a board of directors.The governing body of a corporation is its board officers are not liable. For them to be held accountable, the mismanagement and the
of directors. Section 23 of the Corporation Code explicitly provides that unless otherwise resulting losses on account thereof are not the only matters to be proven; it is likewise
provided therein, the corporate powers of all corporations formed under the Code shall be necessary to show that the direc-tors and/or officers acted in bad faith and with malice in
exercised, all business conducted and all property of the corporation shall be controlled and doing the assailed acts. Bad faith does not simply connote bad judgment or negligence; it
held by a board of directors. Thus, with the exception only of some powers expressly granted imports a dishonest purpose or some moral obliquity and conscious doing of a wrong, a
by law to stockholders (or members, in case of non-stock corporations), the board of breach of a known duty through some motive or interest or ill-will partaking of the nature of
directors (or trustees, in case of non-stock corporations) has the sole authority to determine fraud. We have searched the records and nowhere do we find a dishonest purpose or
policies, enter into contracts, and conduct the ordinary business of the corporation within the some moral obliquity, or conscious doing of a wrong on the part of the respondents that
scope of its charter, i.e., its articles of incorporation, by-laws and relevant provisions of law. partakes of the nature of fraud.
Verily, the authority of the board of directors is restricted to the management of the regular
business affairs of the corporation, unless more extensive power is expressly conferred. Same; Management Prerogatives; The determination of the necessity for additional
offices and/or positions in a corporation is a management prerogative which courts are not
Same; The raison dtre behind the conferment of corporate powers on the board of wont to review in the absence of any proof that such prerogative was exercised in bad faith or
directors is not lost on the Courtindeed, the concentration in the board of the powers of with malice.The determination of the necessity for additional offices and/or positions in a
control of corporate business and of appointment of corporate officers and managers is corporation is a management prerogative which courts are not wont to review in the absence
necessary for efficiency in any large organization.The raison dtre behind the conferment of any proof that such prerogative was exercised in bad faith or with malice.
of corporate powers on the board of directors is not lost on the Court. Indeed, the
concentration in the board of the powers of control of corporate business and of Same; Under the Corporation Code, where a corporation is an injured party, its power
appointment of corporate officers and managers is necessary for efficiency in any large to sue is lodged with its board of directors or trustees.Under the Corporation Code, where a
organization. Stockholders are too numerous, scattered and unfamiliar with the business of a corporation is an injured party, its power to sue is lodged with its board of directors or
corporation to conduct its business directly. And so the plan of corporate organization is for trustees. But an individual stockholder may be permitted to institute a derivative suit in behalf
the stockholders to choose the directors who shall control and supervise the conduct of of the corporation in order to protect or vindicate corporate rights whenever the officials of
corporate business. the corporation refuse to sue, or when a demand upon them to file the necessary action
would be futile because they are the ones to be sued, or because they hold control of the

148
MATABABE FILES
Corporation Law

corporation. In such actions, the corporation is the real party-in-interest while the suing The relevant facts:
stockholder, in behalf of the corporation, is only a nominal party.
On 4 September 1992, petitioner Eliodoro C. Cruz, Filports president from 1968 until
Same; Derivative Suits; Since the ones to be sued are the directors/officers of the he lost his bid for reelection as Filports president during the general stockholders meeting in
corporation itself, a stockholder, like petitioner Cruz, may validly institute a derivative suit to 1991, wrote a letter2 to the corporations Board of Directors questioning the boards creation
vindicate the alleged corporate injury, in which case Cruz is only a nominal party while Filport is of the following positions with a monthly remuneration of P13,050.00 each, and the election
the real party-in-interest.The action below is principally for damages resulting from alleged thereto of certain members of the board, to wit:
mismanagement of the affairs of Filport by its directors/officers, it being alleged that the acts
of mismanagement are detrimental to the interests of Filport. Thus, the injury complained of Asst. Vice-President for Corporate Planning - Edgar C. Trinidad (Director)
primarily pertains to the corporation so that the suit for relief should be by the corporation. Asst. Vice-President for Operations - Eliezer B. de Jesus (Director)
However, since the ones to be sued are the directors/officers of the corporation itself, a Asst. Vice-President for Finance - Mary Jean D. Co (Director)
stockholder, like petitioner Cruz, may validly institute a derivative suit to vindicate the Asst. Vice-President for Administration - Henry Chua (Director)
alleged corporate injury, in which case Cruz is only a nominal party while Filport is the real Special Asst. to the Chairman - Arsenio Lopez Chua (Director)
party-in-interest. For sure, in the prayer portion of petitioners petition before the SEC, the Special Asst. to the President - Fortunato V. de Castro
reliefs prayed were asked to be made in favor of Filport.
In his aforesaid letter, Cruz requested the board to take necessary action/actions to
PETITION for review on certiorari of a decision of the Court of Appeals. recover from those elected to the aforementioned positions the salaries they have received.

The facts are stated in the opinion of the Court. On 15 September 1992, the board met and took up Cruzs letter. The records do not
show what specific action/actions the board had taken on the letter. Evidently, whatever
DECISION action/actions the board took did not sit well with Cruz.
GARCIA, J.:
On 14 June 1993, Cruz, purportedly in representation of Filport and its stockholders,
Assailed and sought to be set aside in this petition for review on certiorari is the among which is herein co-petitioner Mindanao Terminal and Brokerage Services, Inc.
Decision1 dated 19 January 2004 of the Court of Appeals (CA) in CA-G.R. CV No. 73827, (Minterbro), filed with the SEC a petition3 which he describes as a derivative suit against the
reversing an earlier decision of the Regional Trial Court (RTC) of Davao City and accordingly herein respondents who were then the incumbent members of Filports Board of Directors,
dismissing the derivative suit instituted by petitioner Eliodoro C. Cruz for and in behalf of the for alleged acts of mismanagement detrimental to the interest of the corporation and its
stockholders of co-petitioner Filipinas Port Services, Inc. (Filport, hereafter). shareholders at large, namely:

The case is actually an intra-corporate dispute involving Filport, a domestic 1. creation of an executive committee in 1991 composed of seven (7) members of the
corporation engaged in stevedoring services with principal office in Davao City. It was initially board with compensation of P500.00 for each member per meeting, an office which,
instituted with the Securities and Exchange Commission (SEC) where the case hibernated and to Cruz, is not provided for in the by-laws of the corporation and whose function
remained unresolved for several years until it was overtaken by the enactment into law, on 19 merely duplicates those of the President and General Manager;
July 2000, of Republic Act (R.A.) No. 8799, otherwise known as the Securities Regulation
Code. From the SEC and consistent with R.A. No. 8799, the case was transferred to the RTC of 2. increase in the emoluments of the Chairman, Vice-President, Treasurer and
Manila, Branch 14, sitting as a corporate court. Subsequently, upon respondents motion, the Assistant General Manager which increases are greatly disproportionate to the
case eventually landed at the RTC of Davao City where it was docketed as Civil Case No. volume and character of the work of the directors holding said positions;
28,552-2001. RTC-Davao City, Branch 10, ruled in favor of the petitioners prompting
respondents to go to the CA in CA-G.R. CV No. 73827. This time, the respondents prevailed, 3. re-creation of the positions of Assistant Vice-Presidents (AVPs) for Corporate
hence, this petition for review by the petitioners. Planning, Operations, Finance and Administration, and the election thereto of board

149
MATABABE FILES
Corporation Law

members Edgar C. Trinidad, Eliezer de Jesus, Mary Jean D. Co and Henry Chua, exhaust remedies for redress within the corporation before bringing the suit; and (3) the
respectively; and petition does not show that the stockholders bringing the suit are joined as nominal parties.
In support of their counterclaim, respondents averred that Cruz filed the alleged derivative
4. creation of the additional positions of Special Assistants to the President and the suit in bad faith and purely for harassment purposes on account of his non-reelection to the
Board Chairman, with Fortunato V. de Castro and Arsenio Lopez Chua elected to the board in the 1991 general stockholders meeting.
same, the directors elected/appointed thereto not doing any work to deserve the
monthly remuneration of P13,050.00 each. As earlier narrated, the derivative suit (SEC Case No. 06-93-4491) hibernated with the
SEC for a long period of time. With the enactment of R.A. No. 8799, the case was first turned
In the same petition, docketed as SEC Case No. 06-93-4491, Cruz alleged that despite over to the RTC of Manila, Branch 14, sitting as a corporate court. Thereafter, on respondents
demands made upon the respondent members of the board of directors to desist from motion, it was eventually transferred to the RTC of Davao City whereat it was docketed as
creating the positions in question and to account for the amounts incurred in creating the Civil Case No. 28,552-2001 and raffled to Branch 10 thereof.
same, the demands were unheeded. Cruz thus prayed that the respondent members of the
board of directors be made to pay Filport, jointly and severally, the sums of money variedly On 10 December 2001, RTC-Davao City rendered its decision5 in the case. Even as it
representing the damages incurred as a result of the creation of the offices/positions found that (1) Filports Board of Directors has the power to create positions not provided for
complained of and the aggregate amount of the questioned increased salaries. in the by-laws of the corporation since the board is the governing body; and (2) the increases
in the salaries of the board chairman, vice-president, treasurer and assistant general manager
In their common Answer with Counterclaim,4 the respondents denied the allegations are reasonable, the trial court nonetheless rendered judgment against the respondents by
of mismanagement and materially averred as follows: ordering the directors holding the positions of Assistant Vice President for Corporate
Planning, Special Assistant to the President and Special Assistant to the Board Chairman to
1. the creation of the executive committee and the grant of per diems for the refund to the corporation the salaries they have received as such officers "considering that
attendance of each member are allowed under the by-laws of the corporation; Filipinas Port Services is not a big corporation requiring multiple executive positions" and that
said positions "were just created for accommodation." We quote the fallo of the trial courts
2. the increases in the salaries/emoluments of the Chairman, Vice-President, Treasurer decision.
and Assistant General Manager were well within the financial capacity of the WHEREFORE, judgment is rendered ordering:
corporation and well-deserved by the officers elected thereto; and
Edgar C. Trinidad under the third and fourth causes of action to restore to the
3. the positions of AVPs for Corporate Planning, Operations, Finance and corporation the total amount of salaries he received as assistant vice president for corporate
Administration were already in existence during the tenure of Cruz as president of planning; and likewise ordering Fortunato V. de Castro and Arsenio Lopez Chua under the
the corporation, and were merely recreated by the Board, adding that all those fourth cause of action to restore to the corporation the salaries they each received as special
appointed to said positions of Assistant Vice Presidents, as well as the additional assistants respectively to the president and board chairman. In case of insolvency of any or all
position of Special Assistants to the Chairman and the President, rendered services to of them, the members of the board who created their positions are subsidiarily liable.
deserve their compensation.
The counter claim is dismissed.
In the same Answer, respondents further averred that Cruz and his co-petitioner
Minterbro, while admittedly stockholders of Filport, have no authority nor standing to bring From the adverse decision of the trial court, herein respondents went on appeal to
the so-called "derivative suit" for and in behalf of the corporation; that respondent Mary Jean the CA in CA-G.R. CV No. 73827.
D. Co has already ceased to be a corporate director and so with Fortunato V. de Castro, one of
those holding an assailed position; and that no demand to cease and desist from further In its decision6 of 19 January 2004, the CA, taking exceptions to the findings of the
committing the acts complained of was made upon the board. By way of affirmative trial court that the creation of the positions of Assistant Vice President for Corporate
defenses, respondents asserted that (1) the petition is not duly verified by petitioner Filport Planning, Special Assistant to the President and Special Assistant to the Board Chairman was
which is the real party-in-interest; (2) Filport, as represented by Cruz and Minterbro, failed to merely for accommodation purposes, granted the respondents appeal, reversed and set

150
MATABABE FILES
Corporation Law

aside the appealed decision of the trial court and accordingly dismissed the so-called It is axiomatic that in petitions for review on certiorari under Rule 45 of the Rules of
derivative suit filed by Cruz, et al., thus: Court, only questions of law may be raised and passed upon by the Court. Factual findings of
the CA are binding and conclusive and will not be reviewed or disturbed on appeal. 10 Of
IN VIEW OF ALL THE FOREGOING, the instant appeal is GRANTED, the challenged course, the rule is not cast in stone; it admits of certain exceptions, such as when the findings
decision is REVERSED and SET ASIDE, and a new one entered DISMISSING Civil Case No. of fact of the appellate court are at variance with those of the trial court,11 as here. For this
28,552-2001 with no pronouncement as to costs. reason, and for a proper and complete resolution of the case, we shall delve into the records
SO ORDERED. and reexamine the same.

Intrigued, and quite understandably, by the fact that, in its decision, the CA, before The governing body of a corporation is its board of directors. Section 23 of the
proceeding to address the merits of the appeal, prefaced its disposition with the statement Corporation Code12 explicitly provides that unless otherwise provided therein, the corporate
reading "[T]he appeal is bereft of merit,"7 thereby contradicting the very fallo of its own powers of all corporations formed under the Code shall be exercised, all business conducted
decision and the discussions made in the body thereof, respondents filed with the appellate and all property of the corporation shall be controlled and held by a board of directors. Thus,
court a Motion For Nunc Pro Tunc Order,8 thereunder praying that the phrase "[T]he appeal is with the exception only of some powers expressly granted by law to stockholders (or
bereft of merit," be corrected to read "[T]he appeal is impressed with merit." In its members, in case of non-stock corporations), the board of directors (or trustees, in case of
resolution9 of 23 April 2004, the CA granted the respondents motion and accordingly non-stock corporations) has the sole authority to determine policies, enter into contracts, and
effected the desired correction. conduct the ordinary business of the corporation within the scope of its charter, i.e., its
articles of incorporation, by-laws and relevant provisions of law. Verily, the authority of the
Hence, petitioners present recourse. board of directors is restricted to the management of the regular business affairs of the
corporation, unless more extensive power is expressly conferred.
Petitioners assigned four (4) errors allegedly committed by the CA. For clarity, we
shall formulate the issues as follows: The raison detre behind the conferment of corporate powers on the board of
1. Whether the CA erred in holding that Filports Board of Directors acted within its directors is not lost on the Court. Indeed, the concentration in the board of the powers of
powers in creating the executive committee and the positions of AVPs for Corporate control of corporate business and of appointment of corporate officers and managers is
Planning, Operations, Finance and Administration, and those of the Special Assistants necessary for efficiency in any large organization. Stockholders are too numerous, scattered
to the President and the Board Chairman, each with corresponding remuneration, and unfamiliar with the business of a corporation to conduct its business directly. And so the
and in increasing the salaries of the positions of Board Chairman, Vice-President, plan of corporate organization is for the stockholders to choose the directors who shall
Treasurer and Assistant General Manager; and control and supervise the conduct of corporate business. 13

2. Whether the CA erred in finding that no evidence exists to prove that (a) the In the present case, the boards creation of the positions of Assistant Vice Presidents
positions of AVP for Corporate Planning, Special Assistant to the President and for Corporate Planning, Operations, Finance and Administration, and those of the Special
Special Assistant to the Board Chairman were created merely for accommodation, Assistants to the President and the Board Chairman, was in accordance with the regular
and (b) the salaries/emoluments corresponding to said positions were actually paid business operations of Filport as it is authorized to do so by the corporations by-laws,
to and received by the directors appointed thereto. pursuant to the Corporation Code.

For their part, respondents, aside from questioning the propriety of the instant The election of officers of a corporation is provided for under Section 25 of the Code
petition as the same allegedly raises only questions of fact and not of law, also put in issue the which reads:
purported derivative nature of the main suit initiated by petitioner Eliodoro C. Cruz allegedly
in representation of and in behalf of Filport and its stockholders. Sec. 25. Corporate officers, quorum. Immediately after their election, the directors
of a corporation must formally organize by the election of a president, who shall be a director,
The petition is bereft of merit. a treasurer who may or may not be a director, a secretary who shall be a resident and citizen

151
MATABABE FILES
Corporation Law

of the Philippines, and such other officers as may be provided for in the by-laws. (Emphasis leads to the inevitable conclusion that he did so out of spite and resentment for his non-
supplied.) reelection as president of the corporation.

In turn, the amended Bylaws of Filport14 provides the following: With regard to the increased emoluments of the Board Chairman, Vice-President,
Treasurer and Assistant General Manager which are supposedly disproportionate to the
Officers of the corporation, as provided for by the by-laws, shall be elected by the volume and nature of their work, the Court, after a judicious scrutiny of the increase vis--vis
board of directors at their first meeting after the election of Directors. xxx the value of the services rendered to the corporation by the officers concerned, agrees with
the findings of both the trial and appellate courts as to the reasonableness and fairness
The officers of the corporation shall be a Chairman of the Board, President, a Vice- thereof.
President, a Secretary, a Treasurer, a General Manager and such other officers as the Board of
Directors may from time to time provide, and these officers shall be elected to hold office Continuing, petitioners contend that the CA did not appreciate their evidence as to
until their successors are elected and qualified. (Emphasis supplied.) the alleged acts of mismanagement by the then incumbent board. A perusal of the records,
however, reveals that petitioners merely relied on the testimony of Cruz in support of their
Likewise, the fixing of the corresponding remuneration for the positions in question bold claim of mismanagement. To the mind of the Court, Cruz testimony on the matter of
is provided for in the same by-laws of the corporation, viz: mismanagement is bereft of any foundation. As it were, his testimony consists merely of
xxx The Board of Directors shall fix the compensation of the officers and agents of the insinuations of alleged wrongdoings on the part of the board. Without more, petitioners
corporation. (Emphasis supplied.) posture of mismanagement must fall and with it goes their prayer to hold the respondents
Unfortunately, the bylaws of the corporation are silent as to the creation by its board liable therefor.
of directors of an executive committee. Under Section 35 15 of the Corporation Code, the
creation of an executive committee must be provided for in the bylaws of the corporation. But even assuming, in gratia argumenti, that there was mismanagement resulting to
corporate damages and/or business losses, still the respondents may not be held liable in the
Notwithstanding the silence of Filports bylaws on the matter, we cannot rule that absence, as here, of a showing of bad faith in doing the acts complained of.
the creation of the executive committee by the board of directors is illegal or unlawful. One
reason is the absence of a showing as to the true nature and functions of said executive If the cause of the losses is merely error in business judgment, not amounting to bad
committee considering that the "executive committee," referred to in Section 35 of the faith or negligence, directors and/or officers are not liable.17 For them to be held accountable,
Corporation Code which is as powerful as the board of directors and in effect acting for the the mismanagement and the resulting losses on account thereof are not the only matters to
board itself, should be distinguished from other committees which are within the competency be proven; it is likewise necessary to show that the directors and/or officers acted in bad faith
of the board to create at anytime and whose actions require ratification and confirmation by and with malice in doing the assailed acts. Bad faith does not simply connote bad judgment or
the board.16 Another reason is that, ratiocinated by both the two (2) courts below, the Board negligence; it imports a dishonest purpose or some moral obliquity and conscious doing of a
of Directors has the power to create positions not provided for in Filports bylaws since the wrong, a breach of a known duty through some motive or interest or ill-will partaking of the
board is the corporations governing body, clearly upholding the power of its board to nature of fraud.18 We have searched the records and nowhere do we find a "dishonest
exercise its prerogatives in managing the business affairs of the corporation. purpose" or "some moral obliquity," or "conscious doing of a wrong" on the part of the
respondents that "partakes of the nature of fraud."
As well, it may not be amiss to point out that, as testified to and admitted by
petitioner Cruz himself, it was during his incumbency as Filport president that the executive We thus extend concurrence to the following findings of the CA, affirmatory of those
committee in question was created, and that he was even the one who moved for the of the trial court:
creation of the positions of the AVPs for Operations, Finance and Administration. By his
acquiescence and/or ratification of the creation of the aforesaid offices, Cruz is virtually xxx As a matter of fact, it was during the term of appellee Cruz, as president and
precluded from suing to declare such acts of the board as invalid or illegal. And it makes no director, that the executive committee was created. What is more, it was appellee himself
difference that he sues in behalf of himself and of the other stockholders. Indeed, as his voice who moved for the creation of the positions of assistant vice presidents for operations, for
was not heard in protest when he was still Filports president, raising a hue and cry only now

152
MATABABE FILES
Corporation Law

finance, and for administration. He should not be heard to complain thereafter for similar and the grant of salary increases to officers thereof. Such are corporate and/or business
corporate acts. decisions which only the corporations Board of Directors can determine.

The increase in the salaries of the board chairman, president, treasurer, and assistant So it is that in Philippine Stock Exchange, Inc. v. CA,21 the Court unequivocally held:
general manager are indeed reasonable enough in view of the responsibilities assigned to
them, and the special knowledge required, to be able to effectively discharge their respective Questions of policy or of management are left solely to the honest decision of the
functions and duties. board as the business manager of the corporation, and the court is without authority to
Surely, factual findings of trial courts, especially when affirmed by the CA, are binding and substitute its judgment for that of the board, and as long as it acts in good faith and in the
conclusive on this Court. exercise of honest judgment in the interest of the corporation, its orders are not reviewable
by the courts.
There is, however, a factual matter over which the CA and the trial court parted ways.
We refer to the accommodation angle. In a last-ditch attempt to salvage their cause, petitioners assert that the CA went
The trial court was with petitioner Cruz in saying that the creation of the positions of the beyond the issues raised in the court of origin when it ruled on the absence of receipt of
three (3) AVPs for Corporate Planning, Special Assistant to the President and Special Assistant actual payment of the salaries/emoluments pertaining to the positions of Assistant Vice-
to the Board Chairman, each with a salary of P13,050.00 a month, was merely for President for Corporate Planning, Special Assistant to the Board Chairman and Special
accommodation purposes considering that Filport is not a big corporation requiring multiple Assistant to the President. Petitioners insist that the issue of nonpayment was never raised by
executive positions. Hence, the trial courts order for said officers to return the amounts they the respondents before the trial court, as in fact, the latter allegedly admitted the same in
received as compensation. their Answer With Counterclaim.

On the other hand, the CA took issue with the trial court and ruled that Cruzs We are not persuaded.
accommodation theory is not based on facts and without any evidentiary substantiation.
By claiming that Filport suffered damages because the directors appointed to the
We concur with the line of the appellate court. For truly, aside from Cruzs bare and assailed positions are not doing anything to deserve their compensation, petitioners are
self-serving testimony, no other evidence was presented to show the fact of saddled with the burden of proving that salaries were actually paid. Since the trial court, in
"accommodation." By itself, the testimony of Cruz is not enough to support his claim that effect, found that the petitioners successfully proved payment of the salaries when it directed
accommodation was the underlying factor behind the creation of the aforementioned three the reimbursements of the same, respondents necessarily have to raise the issue on appeal.
(3) positions. And the CA rightly resolved the issue when it found that no evidence of actual payment of the
salaries in question was actually adduced. Respondents alleged admission of the fact of
It is elementary in procedural law that bare allegations do not constitute evidence payment cannot be inferred from a reading of the pertinent portions of the parties
adequate to support a conclusion. It is basic in the rule of evidence that he who alleges a fact respective initiatory pleadings. Respondents allegations in their Answer With Counterclaim
bears the burden of proving it by the quantum of proof required. Bare allegations, that the officers corresponding to the positions created "performed the work called for in
unsubstantiated by evidence, are not equivalent to proof under the Rules of Court. 19 The their positions" or "deserve their compensation," cannot be interpreted to mean that they
party having the burden of proof must establish his case by a preponderance of evidence. 20 were "actually paid" such compensation. Directly put, the averment that "one deserves ones
compensation" does not necessarily carry the implication that "such compensation was
Besides, the determination of the necessity for additional offices and/or positions in a actually remitted or received." And because payment was not duly proven, there is no
corporation is a management prerogative which courts are not wont to review in the absence evidentiary or factual basis for the trial court to direct respondents to make reimbursements
of any proof that such prerogative was exercised in bad faith or with malice. thereof to the corporation.

Indeed, it would be an improper judicial intrusion into the internal affairs of Filport This brings us to the respondents claim that the case filed by the petitioners before
were the Court to determine the propriety or impropriety of the creation of offices therein the SEC, which eventually landed in RTC-Davao City as Civil Case No. 28,552-2001, is not a
derivative suit, as maintained by the petitioners.

153
MATABABE FILES
Corporation Law

We sustain the petitioners. within the corporation itself the redress for his grievances, or actions in conformity to his
wishes, nonetheless, where the corporation is under the complete control of the principal
Under the Corporation Code, where a corporation is an injured party, its power to sue defendants, as here, there is no necessity of making a demand upon the directors. The reason
is lodged with its board of directors or trustees. But an individual stockholder may be is obvious: a demand upon the board to institute an action and prosecute the same effectively
permitted to institute a derivative suit in behalf of the corporation in order to protect or would have been useless and an exercise in futility. In fine, we rule and so hold that the
vindicate corporate rights whenever the officials of the corporation refuse to sue, or when a petition filed with the SEC at the instance of Cruz, which ultimately found its way to the RTC
demand upon them to file the necessary action would be futile because they are the ones to of Davao City as Civil Case No. 28,552-2001, is a derivative suit of which Cruz has the necessary
be sued, or because they hold control of the corporation. 22 In such actions, the corporation is legal standing to institute.
the real party-in-interest while the suing stockholder, in behalf of the corporation, is only a
nominal party.23 WHEREFORE, the petition is DENIED and the challenged decision of the CA is
AFFIRMED in all respects.
Here, the action below is principally for damages resulting from alleged
mismanagement of the affairs of Filport by its directors/officers, it being alleged that the acts No pronouncement as to costs.
of mismanagement are detrimental to the interests of Filport. Thus, the injury complained of
primarily pertains to the corporation so that the suit for relief should be by the corporation. SO ORDERED.
However, since the ones to be sued are the directors/officers of the corporation itself, a
stockholder, like petitioner Cruz, may validly institute a "derivative suit" to vindicate the
alleged corporate injury, in which case Cruz is only a nominal party while Filport is the real
party-in-interest. For sure, in the prayer portion of petitioners petition before the SEC, the
reliefs prayed were asked to be made in favor of Filport.

Besides, the requisites before a derivative suit can be filed by a stockholder are
present in this case, to wit:

a) the party bringing suit should be a shareholder as of the time of the act or
transaction complained of, the number of his shares not being material;

b) he has tried to exhaust intra-corporate remedies, i.e., has made a demand on the
board of directors for the appropriate relief but the latter has failed or refused to
heed his plea; and
c) the cause of action actually devolves on the corporation, the wrongdoing or harm
having been, or being caused to the corporation and not to the particular
stockholder bringing the suit.24

Indisputably, petitioner Cruz (1) is a stockholder of Filport; (2) he sought without


success to have its board of directors remedy what he perceived as wrong when he wrote a
letter requesting the board to do the necessary action in his complaint; and (3) the alleged
wrong was in truth a wrong against the stockholders of the corporation generally, and not
against Cruz or Minterbro, in particular. In the end, it is Filport, not Cruz which directly stands
to benefit from the suit. And while it is true that the complaining stockholder must show to
the satisfaction of the court that he has exhausted all the means within his reach to attain

154
MATABABE FILES
Corporation Law

Republic of the Philippines stockholders. An illustration of a suit of this kind is found in the case of Pascual vs. Del Saz
SUPREME COURT Orozco (19 Phil., 82), decided by this court as early as 1911. In that case, the Banco Espaol-
Manila Filipino suffered heavy losses due to fraudulent connivance between a depositor and an
EN BANC employee of the bank, which losses, it was contended, could have been avoided if the
president and directors had been more vigilant in the administration of the affairs of the bank.
G.R. No. L-43413 August 31, 1937 The stockholders constituting the minority brought a suit in behalf of the bank against the
directors to recover damages, and this over the objection of the majority of the stockholders
HIGINIO ANGELES, JOSE E. LARA and AGUEDO BERNABE, and the directors. This court held that the suit could properly be maintained.
as stockholders for an in behalf and for the benefit of the corporation, Paraaque Rice Mill,
Inc. and the other stockholders who may desire to join, plaintiffs-appellees, 4. ID. ; ID. ; ID. ; RECEIVERSHIP.The action having been properly brought and by the
vs. lower court entertained it was within its power, upon proper showing, to appoint a receiver
TEODORICO B. SANTOS, ESTANISLAO MAYUGA, APOLONIO PASCUAL, and BASILISA of the corporation pendente lite (secs. 173, 174, et seq. Code of Civil Procedure). The
RODRIGUEZ, defendant-appellants. appointment of a receiver upon application of the minority stockholders is a power to be
exercised with great caution. But this does not mean that the rights of the minority
1. CORPORATIONS; BOARD OF DIRECTORS; TRUSTEESHIP.There is ample evidence in stockholders may be entirely disregarded, and where the necessity has arisen, the
the present case to show that the defendants have been guilty of breach of trust as directors appointment of a receiver for a corporation is a matter resting largely in the sound discretion
of the corporation and the lower court so found. The board of directors of a corporation is a of the trial court. Counsel for appellants argue that the appointment of a receiver pendente
creation of the stockholders and controls and directs the affairs of the corporation by lite in the present case has deprived the corporation, Paraaque Rice Mill, Inc. of its property
delegation of the stockholders. But the board of directors, or the majority thereof, in drawing without due process of law, But it is too plain to require argument that the receiver was
to themselves the powers of the corporation, occupies a position of trusteeship in relation to precisely appointed to preserve the properties of the corporation. The receivership in this
the minority of the stock in the sense that the board should exercise good faith, care and case shall continue until a new board of directors shall have been elected and constituted in
diligence in the administration of the affairs of the corporation and should protect not only accordance with law and the by-laws of the corporation.
the interests of the majority but also those of the minority of the stock.
5. ID.; ID.; ACCOUNTING.The lower court in its decision not only orders the
2. ID.; ID.; ACTION FOR THE PROTECTION OF THE RIGHTS OF THE MINORITY defendant S to account for the properties and funds of the corporation, but it also and at the
STOCKHOLDERS.Where a majority of the board of directors wastes or dissipates the funds same time adjudges him to pay an undetermined amount which is made to depend upon the
of the corporation or fraudulently disposes of its properties, or performs ultra vires acts, the result of such accounting. The accounting order was probably intended by the lower court to
court, in the exercise of its equity jurisdiction, and upon showing that intracorporate remedy be filed with it in this proceeding. This requirement will delay the final disposition of the case
is unavailing, will entertain a suit filed by the minority members of the board of directors, for and we are of the opinion that this accounting should better be filed with the new board of
and in behalf of the corporation, to prevent waste and dissipation and the commission of directors whose election has been ordered by the lower court. The decision of the lower court
illegal acts and otherwise redress the injuries of the minority stockholders against the in this respect is therefore modified so that the defendant S shall render a complete
wrongdoing of the majority. The action in such a case is said to be brought derivatively in accounting of all the corporate properties and funds that may have come to his possession
behalf of the corporation to protect the rights of the minority stockholders thereof (7 R. C. L., during the period mentioned in the judgment of the lower court to the new board of directors
pars. 293 and 294, and authorities therein cited; 13 Fletcher, Cyc. of Corp., pars. 593, et seq., to be elected by the stockholders.
and authorities therein cited).
6.ID.; ID.; ELECTION AND REMOVAL OF THE DIRECTORS OF A CORPORATION.The
3. ID. ; ID. ; ID.It is well settled in this jurisdiction that where corporate directors are Corporation Law, as amended, in sections 29 to 34, provide for the election and removal of
guilty of a breach of trustnot of mere error of judgment or abuse of discretionand the directors of a corporation. Our Corporation Law (Act No. 1459, as amended), does not
intracorporate remedy is futile or useless, a stockholder may institute a suit in behalf of confer expressly upon the courts the power to remove a director of a corporation. In some
himself and other stockholders and for the benefit of the corporation, to bring about a jurisdictions, statutes expressly provide a more or less summary method for the confirmation
redress of the wrong inflicted directly upon the corporation and indirectly upon the of the election and for the amotion of the directors of a corporation. This is true in New York,

155
MATABABE FILES
Corporation Law

New Jersey, Virginia and other states of the American Union. There are abundant authorities, of the corporation, refuse to sign and issue the corresponding certificate of stock for the 600
however, which hold that if the court has acquired jurisdiction to appoint a receiver because fully paid-up share of the plaintiff, Higinio Angeles, of the total value of P15,000; ( f ) that
of the mismanagement of directors these may thereafter be removed and others appointed notwithstanding written requests made in conformity with the by-laws of the corporation of
in their place by the court in the exercise of its equity jurisdiction (2 Fletcher, Cyc. of Corp., ftn. three members of the board of directors who are holders of more than one-third of the
sec. 358, pp. 118 and 119). In the present case, however, the properties and assets of the subscribed capital stock of the corporation, the defendant Teodorico B. Santos as president
corporation being amply protected by the appointment of a receiver and in view of the of the corporation refuse to call a meeting of the board of directors and of the stockholers;
statutory provisions above referred to, we are of the opinion that the removal of the directors (g) that in violation of the by-laws of the corporation, the defendant who constitute the
is, under the circumstances, unnecessary and unwarranted. majority of the board of directors refused to hold ordinary monthly meetings of the board
since March, 19332; (h) that Teodorico B. Santos as president of the corporation, in
APPEAL from a judgment of the Court of First Instance of Rizal. Zandueta, J. connivance with his
The facts are stated in the opinion of the court co-defendants, was disposing of the properties and records of the corporation without
authority from the board of directors or the stockholders of the corporation and without
P. Masalin and A. Sta. Maria for appellants. making any report of his acts to the said board of directors or to any other officer of the
Eulogio P. Revilla and Barrera and Reyes for appellees. corporation, and that, to prevent any interferrence with or examination of his arbitrary acts,
he arbitrarily suspended plaintiff Jose de Lara from the office of general manager to which
LAUREL, J.: office the latter had been lawfully elected by the stockholders; and (i) that the corporation
had gained about P4,000 during the first half of the year 1932, but that because of the illegal
The plaintiff and the defenant aree all stockholders and member of the board of and arbitrary acts of the defendants not only the funds but also the books and records of the
directors of the "Paraaque Rice Mill, Inc., "a corporation organized for the purpose of corporation are in danger of disappearing.
operating a rice mill in the municipality of Paraaque, Province of Rizal. On September 6, 1932,
a complaint entitle "Higinio Angeles, Jose de Lara, Aguedo Bernabe, as stockholders, for and The complaint prays: (a) That after the filing of the bond in an amount to be fixed by
in behalf of the corporation, Paraaque Rice Mill, Inc., and other stockholders of said the court, Melchor de Lara of Paraaque, Rizal, be appointed receiver of the properties, funds
corporation who may desire to join, plaintiff, vs. Teodorico B. Santos, Estanislao Mayuga, and business of the Paraaque Rice Mill, Inc., as well as the books and record thereof, with
Apolonio Pascual, and Basilisa Rodriguez, defendant was filed with the Court of First Instance authority to continue the business of the corporation; (b) that the defendant Teodorico B.
of Rizal. After formal allegation relative to age and residence of the parties and the due Santos be ordered to render a detailed accounting of the properties, funds and income of the
incorporation of the Paraaque Rice Mill, Inc., the complaint avers subtantially the following: corporation from the year 1927 to date; (c) that the said defendant be required to pay to the
(a) That the plaintiffs are stockholders and constitute the minority and the defendants are corporation the amount of P10,000 and other amounts which may be found due to the said
also stockholers and constitute the majority of the board of directors of the Paraaque Rice corporation as damages or for my other cause, (d) that said defendant be ordered to sign the
Mill, Inc.; (b) that at an extraordinary meeting held on February 21, 1932, the stockholders certificate of stock subscribed to and paid by the plaintiff Higinio Angeles; and (e) that the
appointed an investigation committee of which the plaintiff Jose de Lara was chairman and members of the board of directors of the Paraaque Rice Mill, Inc., be removed and an
the stockholers Dionisio Tomas and Aguedo Bernabe were members, to investigate and exrtraodinary meeting of the stockholders called for the purpose of electing a new board of
determine the properties, operations, and losses of the corporation as shown in the auditor's directors.
report corresponding to the year 1931, but the defendants, particularly Teodorico B. Santos,
who was the president of the corporation, denied access to the properties, books and record On the date of the filling of the complaint, September 6, 1932, the court issue an ex
of the corporation which were in their possession (c) That the defendant Teodorico B. Santos, parte order of receivership appointing Melchor de Lara as receiver of the corporation upon
in violation of the by-laws of the corporation, had taken possession of the books, vouchers, the filling of a bond of P1,000 by the plaintiffs-appellees. The bond of the receiver was fixed at
and corporate records as well as of the funds and income of the Paraaque Rice Mill, Inc., all P4,000.
of which, according to the by-laws, should be under the exclusive control and possession of
the secretary-treasurer, the plaintiff Aguedo Bernabe; (d) That the said Teodorico B. Santos, Upon an urgent motion of the defendants-appellants setting forth the reasons why
had appropriated to his own benefit properties, funds, and income of the corporation in the Melchor de Lara should not have been appointed receiver, and upon agreement of the
sum of P10,000; (e) that Teodoro B. Santos, for the purpose of illegally controlling the affairs parties, the trial court, by order of September 13, 1932, appointed Benigno Agco, as receiver,

156
MATABABE FILES
Corporation Law

in lieu of Melchor de Lara. About a month after, or on October 14, 1932, the court, after On November 8, 1934, the trial court, having heard the case on its merits rendered a
considering the memoranda filed by both parties revoked its order appointing Agco as decision, the dispositive part of which is as follows:
receiver.
Por todo lo expuesto el Juzgado fall este asunto:
On July 12, 1933, the defendants-appellants presented their amended answer to the
complaint, containing a general and specific denial, and alleging as special defense that the 1. Ordenando al demandado Teodorico B. Santos a rendircuenta ellada de las
defendant Teodorico B. Santos refused to sign the certificate of stock in favor of the plaintiff propiedads, fondos e ingresos dela corporacion Paraaque Rice Mill, Inc., de el ao
Higinio Angeles for 600 shares valued at P15,00, because the board of directors decided to 1931 hasta la fecha;
give Higinio Angeles only 320 shares of stock worth P8,000. The answer contains a counter-
claim for P5,000 alleged illegal and malicious procurement by the plaintiffs of an ex 2. Condenando a dicho demandado a pagar a la corporacion Paraaque Rice Mill, Inc.,
parte order of receivership. Damages in the amount of P2,000 are also alleged to have been cualesquiera cantida o cantidades que resultate en deber a dicha corporacion; de
suffered by the defendants by reason of the failure of the plaintiffs to present their acuerdo con dicha rendicion de cuentas;
grievances to the Board of directors before going to court. The amended answer sets forth,
furthermore, a cross-complaint against the plaintiffs, and in behalf of the Paraaque Rice Mill, 3. Declarando al demanante Higinio M. Angeles con derecho a tener expedido a su
Inc., based on the alleged failure of the plaintiff Higinio Angeles to render a report of his nombre 600 acciones por valor par de P15,000.
administration of the corporation from February 14 to June 30, 1928, during which time the
corporation is alleged to have accrued earnings of approximately P3,000. In both the counter 4. Destituyendo a los demandados de su cargo como directores e la corporacion
claim and cross-complaint Paraaque Rice Mill, Inc. is joined as party defendant. hasta la nueva eleccion por los accionistas que se convocara una vez firme esta
sentencia; y
On July 24, 1934, the plaintiffs-appellees renewed their petition for the appointment 5. Condenando a los demandados a pagar las costas.
of a receiver pendente lite alleging, among other things, that defendant Teodorico B. Santos
was using the funds of the corporation for purely personal ends; that said Teodorico B. Santos On November 21, 1934, the defendants-appellants, moved for reconsideration of the
was managing to the interest of the Corporation and its stockholders; that said defendant did decision and at the same time prayed for the dismissal of the case, because of defect of
not render any account of his management or for the condition of the business of the parties defendant.
corporation; that since 1932 said defendant called no meeting of the board of directors or of
the stockholders thus enabling him to continue holding, without any election, the position of On December 6, 1934, the Paraaque Rice Mill, Inc., thru counsel for the defendants,
present and, finally, that of manager; and that, without the knowledge and consent of the entered a special appearance for the sole purpose of objecting to the order of the court of
stockholders and of the board of directors, the said defendant installed a small rice mill for October 31, 1934, appointing a receiver, on the ground that the Paraaque Rice Mill, Inc., was
converting rice husk into "tiqui-tiqui", the income of which was never turned over or reported not a party to the proceedings. And on December 8, 1934, the defendants excepted to the
to the treasurer of the corporation. decision of the trial court and moved for a new trial on the ground that the evidence
presented was insufficient to justify the decision and that said decision was contrary to law.
The defendant-appellants objected to the petition for the appointment of a receiver The motions for reconsideration and new trial and the special appearance were, by separate
on the ground, among others, that the court had no jurisdiction over the Paraaque Rice Mill, orders bearing date of December 19, 1934, denied by the trial court. The case was finally
Inc., because it had not been include as party defendant in this case and that, therefore the elevated to this court by bill of exceptions.
court could not properly appoint a receiver of the corporation pendente lite.
The defendants-appellants submit the following assignment of errors:
After hearing both parties, the trial court by order of October 31, 1934, appointed
Emilio Figueroa, as receiver of the corporation, after giving a bond in the amount of P2,000. 1. The lower court erred in holding that it has jurisdiction to appoint a receiver o the
An urgent for the reconsideration of this order filed by counsel for the defendant-appellant corporation, "Paraaque Rice Mill, Inc.," on October 31, 1934.
on November 3, 1934, was denied by the court on November 7, 1934.

157
MATABABE FILES
Corporation Law

2. The lower court erred in overruling the motion of the defendants the include the should exercise good faith, care and diligence in the administration of the affairs of the
defendant corporation as party defendant and in holding that it is not a necessary corporation and should protect not only the interest of the majority but also those of the
party. minority of the stock. Where a majority of the board of directors wastes or dissipates the
funds of the corporation or fraudulently disposes of its properties, or performs ultra
3. The lower court erred in not granting a motion for a new trial because there is a vires acts, the court, in the exercise of its equity jurisdiction, and upon showing that
defect of party defendant. intracorporate remedy is unavailing, will entertain a suit filed by the minority members of the
board of directors, for and in behalf of the corporation, to prevent waste and dissipation and
4. The lower court erred in not dismissing the case because a necessary defendant the commission of illegal acts and otherwise redress the injuries of the minority stockholders
was not made a party in the case. against the wrongdoing of the majority. The action in such a case is said to be brought
derivatively in behalf of the corporation to protect the rights of the minority stockholers
5. The lower court erred in ordering the defendant Teodorico B. Santos to render a thereof (7 R. C. L., pars. 293 and 294, and authority therein cited; 13 Fletcher, Cyc. of Corp.,
detailed accounting of the properties, funds and income of the corporation pars. 593, et seq., an authorities therein cite).
"Paraaque Rice Mill, Inc.," from the year 1931 to this date.
It is well settled in this jurisdiction that where corporate directors are guilty of a
6. The lower court erred in condemning the defendant Teodorico B. Santos to pay breach of trust not of mere error of judgment or abuse of discretion and intracorporate
the corporation whatever sum or sums which may be found owing to said remedy is futile or useless, a stockholder may institute a suit in behalf of himself and other
corporation, in accordance with the said accounting to be one by him. stockholders and for the benefit of the corporation, to bring about a redress of the wrong
inflicted directly upon the corporation and indirectly upon the stockholers. An illustration of a
7. The lower court erred in ordering the destitution of the defendants from their suit of this kind is found in the case of Pascual vs. Del Sanz Orozco (19 Phil., 82), decided by this
office as members of the board of directors of the corporation, until the new election court as early as 1911. In that case, the Banco Espaol-Filipino suffered heavy losses due to
of the stockholders which shall be held once the decision has become final. fraudulent connivance between a depositor and an employee of the bank, which losses, it
was contened, could have been avoided if the president and directors has been more vigilant
8. The lower court erred in declaring that Higino Angeles is entitled to have in his in the administration of the affairs of the bank. The stockholers constituting the minority
name 600 shares of stock of the par value of P15,000. brought a suit in behalf of the bank against the directors to recover damages, and this over
the objection of the majority of the stockholers and the directors. This court held that the suit
9. The lower court erred in overruling and denying appellants' motion for the properly be maintained.
reconsideration and the dismissal of the case dated November 21, 1934.
The contention of the defendants in the case at bar that the Paraaque Rice Mill, Inc.,
10. The lower court erred in denying the motion of these appellants for new trial. should have been brought in as necessary party and the action maintained in its name and in
its behalf directly states the general rule, but not the exception recognize by this court in the
In their discussion of the first, second, third, and fourth assignment of error, the case of Everrett vs. Asia Banking Corporation (49 Phil., 512, 527). In that case, upon invocation
defendants-appellants vigorously assert that the Paraaque Rice Mill, Inc., is a necessary party of the general rule by the appellees there, this court said:
in this case, and that not having been made a party, the trial court was without jurisdiction to
appoint a receiver and should have dismissed the case. Invoking the well-known rule that shareholers cannot ordinarily sue in equity
to redress wrong done to the corporation, but that the action must be brought by
There is ample evidence in the present case to show that the defendants have been the board of directors, the appellees argue and the court below held that the
guilty of breach of trust as directors of the corporation and the lower court so found. The corporation Teal & Company is a necessary party plaintiff and that the plaintiff
board of directors of a corporation is a creation of the stockholders and controls and directs stockholder, not having made any demand on the board to bring the action, are not
the affairs of the corporation by allegation of the stockholers. But the board of directors, or the proper parties plaintiff. But, like most rules, the rule in question has its
the majority thereof, in drawing to themselves the power of the corporation, occupies a exceptions. It is alleged in the complaint and, consequently, admitted through the
position of trusteeship in relation to the minority of the stock in the sense that the board demurrer that the corporation Teal & Company is under the complete control of the

158
MATABABE FILES
Corporation Law

principal defendants in the case, and, in these circumstances it is obvious that a In the seventh assignment of error, the appellants contend that the lower court
demand upon the board of directors to institute action and prosecute the same erred in ordering the removal of the defendants from their offices as members of the board
effectively would have been useless, and the law does not require litigants to of directors of the corporation. The Corporation Law, as amended, in section 29 to 34, provide
perform useless acts. (Exchange Bank of Wewoka vs. Bailey, 29 Okla., 246; Fleming for the election and removal of the directors of a corporation. Our Corporation Law (Act No.
and Hewins vs. Black Warrior Copper Co., 15 Ariz., 1; Wickersham vs. Crittenen, 106 1459, as amended), does not confer expressly upon the court the power to remove a director
Cal., 329; Glem vs. Kittanning Brewing Co., 259 Pa., 510; Hawes vs. Contra Costa Water of a corporation. In some jurisdictions, statutes expressly provide a more or less summary
Company, 104 U.S., 450.) method for the confirmation of the election and for the a motion of the directors of a
corporation. This is true in New York, New Jersey, Virginia and other states of the American
The action having been properly brought and by the lower court entertained it was Union. There are abundant authorities, however, which hold that if the court has acquire
within its power, upon proper showing, to appoint a receiver of the corporation pendente jurisdiction to appoint a receiver because of the mismanagement of directors these may
lite (secs. 173, 174, et seq. Code of Civil Procedure). The appointment of a receiver upon thereafter be remove and others appointed in their place by the court in the exercise of its
application of the minority stockholers is power to be exercised with great caution. But this equity jurisdiction (2 Fletcher, Cyc. of Corp., ftn. sec. 358, pp. 18 an 119). In the present case,
does not mean that right of the minority stockholers may be entirely disregarded, and where however, the properties and assets of the corporation being amply protected by the
the necessity has arisen, the appointment of a receiver for a corporation is a matter resting appointment of a receiver and view of the statutory provisions above referred to, we are of
largely in the sound discretion of the trial court. Counsel for appellants argue that the the opinion that the removal of the directors is, under the circumstances, unnecessary and
appointment of a receiver pendente lite in the present case has deprived the corporation, unwarranted. The seventh assignment of error is, therefore, sustained.
Paraaque Rice Mill, Inc., of property without due process of law. But it is too plain to require
argument that the receiver was precisely appointed to preserve the properties of the Under the eighth assignment of error, the appellants argue that the lower court
corporation. The receivership in this case shall continue until a new board of directors shall erred in deciding that the plaintiff Higinio Angeles is entitled to the issuance in his name of a
have been elected and the corporation. certificate covering 600 shares of stock of the total par value of P15,000. A review of the
evidence, oral and documentary, relative to the number of shares of stock to which Higinio
The first, second, third, and fourth assignments of error are, therefore, overruled. Angeles is entitled, shows that Higinio Angeles brought in P15,000 party in money and party in
property, for 600 shares of stock. The very articles of incorporation signed by all the
The appellants contend in their fifth and sixth assignments of error that lower court incorporators, among whom are the defendants, show that Higinio Angeles paid P5,600 on
erred in ordering the defendant, Tedorico B. Santos, to render a detailed accounting of the account of his subscription amounting to P10,000. The amount of P5,600 is the value of
properties, funds and income of the corporation, Paraaque Rice Mill., Inc., from the year 1931 Angeles' cinematograph building in Bacoor, Cavite, which he transferred to the municipality
and in condemning him to pay "the corporation whatever sum or sums which may be found of Paraaque where the same was reconstructed for the use of the corporation. The receipts
owing to said corporation, in accordance with said accounting to be done by him." We note signed by the Philippine Engineering Company and the testimony of Higinio Angeles and
that the lower court in its decision not only orders the defendant Santos to account for the Aguedo Bernabe (secretary-treasurer of the corporation) show that Higinio Angeles paid with
properties and funds of the corporation, but it also and at the same time adjudges him to pay his own funds the sum of P2,750 to the Philippine Engineering Co., as part of the purchase
an undermine amount which is made to depend upon the result of such accounting. The price of the ricemill bought for the corporation. Angeles paid a further sum of P2,397.99 to
accounting order was probably intended by the lower court to be file with it in this the Philippine Engineering Company. It also appears that for the installation of the Rice Mill,
proceeding. This requirement will delay the final disposition of the case and we are of the the construction of camarin, and the cement paving (cementacion) of the whole area of
opinion that this accounting should better be filed with the new board of directors whose two camarines, and for the excavation of a well for the use of the rice mill the plaintiff Higinio
election has been ordered by the lower court. The decision of the lower court in this respect is Angeles paid with his own funds the amount of P7,431.47. Adding all these sums together we
therefore modified so that the defendant Santos shall render a complete accounting of all the have a total of P18, 179.46. At a meeting of the board of directors on December 27, 1931, which
corporate properties and funds that may have come to his possession during the period meeting was convoked by Angeles, it seemed to have been agreed that Angeles was to be
mentioned in the jugment of the lower court to the new board of director to be elected by given shares of stock of the total par value of P15,000. Angeles wanted to have P16,000 worth
the stockholders. of stock to his credit for having made the disbursements mentioned above, but he finally
agreed to accept 600 share worth only P15,000. The certificate of stock, however, was not
issued as disagreement arose between him and the defendant Santos. We, therefore, find no

159
MATABABE FILES
Corporation Law

error in the decision of the lower court ordering the issuance of a certificate for 600 shares of
stock of the total par value of P15,000 to Higinio Angeles.

It is unnecessary to consider the ninth and tenth assignments of error.


In view of the foregoing, we hold:

(1) That the action in the present case was properly instituted by the plaintiff as
stockholders for and in behalf of the corporation Paraaque Rice Mill, Inc., and other
stockholders of the said corporation;
(2) That the lower court committed no reveiwable error in appointing a receiver of
the corporation pendente lite;

(3) That the lower court committed no error in ordering an election of the new board
of directors, which election shall be held within thirty days from the date this decision
becomes final;

(4) That Teodorico B. Santos shall render an accounting of all the properties, funds
and income of the corporation which may have come into his possession to the new
board of directors;

(5) That the receiver, Emilio Figueroa, shall continue in office until the election and
qualification of the members of the new board of directors;

(6) That upon the constitution of the new board of directors, the said receiver shall
turn over all the properties of the corporation in his possession to the corporation, or
such person or persons as may be duly authorized by it; and.

(7) That Higinio Angeles, or his successor in interest, is entitled to 600 shares of stock
at the par value of P15,000 and the lower court committed no error in ordering the
issuance of the corresponding certificate of stock.

On June 10, 1937, counsel for the plaintiff-appellees filed a motion making it appear of
record that Higinio Angeles, one of the plaintiffs and appellees, died on May 4, 1937 and that
one of his daughters, Maura Angeles y Reyes, had been granted letters of administration as
evidenced by the document attached to the motion as Exhibit A, and praying that said Maura
Angeles y Reyes be substituted as one of the plaintiffs and appellees in lieu of Higinio Angeles,
deceased. This motion is hereby granted.

Defendant-appellants shall pay the costs in both instances. So ordered.


Avancea, C.J., Villa-Real, Abad Santos, Imperial, Diaz and Concepcion, JJ., concur.

160
MATABABE FILES
Corporation Law

Republic of the Philippines the corporate affairs.Conformably with the foregoing principles, one of the most important
SUPREME COURT rights of a qualified shareholder or member is the right to voteeither personally or by
Manila proxyfor the directors or trustees who are to manage the corporate affairs. The right to
FIRST DIVISION choose the persons who will direct, manage and operate the corporation is significant,
G.R. No. 153468 August 17, 2006 because it is the main way in which a stockholder can have a voice in the management of
corporate affairs, or in which a member in a nonstock corporation can have a say on how the
PAUL LEE TAN, ANDREW LIUSON, ESTHER WONG, STEPHEN CO, JAMES TAN, JUDITH TAN, purposes and goals of the corporation may be achieved. Once the directors or trustees are
ERNESTO TANCHI JR., EDWIN NGO, VIRGINIA KHOO, SABINO PADILLA JR., EDUARDO P. elected, the stockholders or members relinquish corporate powers to the board in
LIZARES and GRACE CHRISTIAN HIGH SCHOOL, Petitioners, accordance with law.
vs.
PAUL SYCIP and MERRITTO LIM, Respondents. Same; Quorum; In stock corporations, the presence of a quorum is ascertained and
counted on the basis of the outstanding capital stock.In stock corporations, the presence of
Actions; Pleadings and Practice; Forum Shopping; The need for proper verification and a quorum is ascertained and counted on the basis of the outstanding capital stock, as defined
certification against forum shopping are aimed at assuring the truthfulness and correctness of by the Code thus: SEC-TION 137. Outstanding capital stock defined.The term outstanding
the allegations in the Petition and at discouraging forum shopping.The Petition before the capital stock as used in this Code, means the total shares of stock issued under binding
CA was initially flawed, because the Verification and Certification of Non-Forum Shopping subscription agreements to subscribers or stockholders, whether or not fully or partially paid,
were signed by only one, not by all, of the petitioners; further, it failed to show proof that the except treasury shares.
signatory was authorized to sign on behalf of all of them. Subsequently, however, petitioners
submitted a Special Power of Attorney, at-testing that Atty. Padilla was authorized to file the Same; Same; Only stock actually issued and outstanding may be votedneither the
action on their behalf. In the interest of substantial justice, this initial procedural lapse may be stockholders nor the corporation can vote or represent shares that have never passed to the
excused. There appears to be no intention to circumvent the need for proper verification and ownership of stockholders, or, having so passed, have again been purchased by the
certification, which are aimed at assuring the truthfulness and correctness of the allegations corporation.The right to vote is inherent in and incidental to the ownership of corporate
in the Petition for Review and at discouraging forum shopping. More important, the stocks. It is settled that unis-sued stocks may not be voted or considered in determining
substantial merits of petitioners case and the purely legal question involved in the Petition whether a quorum is present in a stockholders meeting, or whether a requisite proportion of
should be considered special circumstances or compelling reasons that justify an exception to the stock of the corporation is voted to adopt a certain measure or act. Only stock actually
the strict requirements of the verification and the certification of non-forum shopping. issued and outstanding may be voted. Under Section 6 of the Corporation Code, each share of
stock is entitled to vote, unless otherwise provided in the articles of incorporation or declared
Corporation Law; Acts of management pertain to the board of directors, and those of delinquent under Section 67 of the Code. Neither the stockholders nor the corporation can
ownership, to the stockholders or members.Under the Corporation Code, stockholders or vote or represent shares that have never passed to the ownership of stockholders; or, having
members periodically elect the board of directors or trustees, who are charged with the so passed, have again been purchased by the corporation. These shares are not to be taken
management of the corporation. The board, in turn, periodically elects officers to carry out into consideration in determining majorities. When the law speaks of a given proportion of
management functions on a day-to-day basis. As owners, though, the stockholders or the stock, it must be construed to mean the shares that have passed from the corporation,
members have residual powers over fundamental and major corporate changes. While and that may be voted.
stockholders and members (in some instances) are entitled to receive profits, the Same; Same; When the principle for determining the quorum for stock corporations is
management and direction of the corporation are lodged with their representatives and applied by analogy to nonstock corporations, only those who are actual members with voting
agentsthe board of directors or trustees. In other words, acts of management pertain to rights should be counted.In nonstock corporations, the voting rights attach to membership.
the board; and those of ownership, to the stockholders or members. In the latter case, the Members vote as persons, in accordance with the law and the bylaws of the corporation.
board cannot act alone, but must seek approval of the stockholders or members. Each member shall be entitled to one vote unless so limited, broadened, or denied in the
articles of incorporation or bylaws. We hold that when the principle for determining the
Same; One of the most important rights of a qualified shareholder or member is the quorum for stock corporations is applied by analogy to non-stock corporations, only those
right to voteeither personally or by proxyfor the directors or trustees who are to manage who are actual members with voting rights should be counted. Under Section 52 of the

161
MATABABE FILES
Corporation Law

Corporation Code, the majority of the members representing the actual number of voting prescribed the specific mode of filling up existing vacancies in its board of directors; that is, by
rights, not the number or numerical constant that may originally be specified in the articles of a majority vote of the remaining members of the board.
incorporation, constitutes the quorum.
Same; Same; There is a well-defined distinction between a corporate act to be done by
Same; Same; In stock corporations, shareholders may generally transfer their shares; the board and that by the constituent members of the corpora-tion.While a majority of the
The determination of whether or not dead members are entitled to exercise their voting remaining corporate members were present, however, the election of the four trustees
rights (through their executor or administrator), depends on the articles of incorporation or cannot be legally upheld for the obvious reason that it was held in an annual meeting of the
bylaws.In stock corporations, shareholders may generally transfer their shares. Thus, on the members, not of the board of trustees. We are not unmindful of the fact that the members of
death of a shareholder, the executor or administrator duly appointed by the Court is vested GCHS themselves also constitute the trustees, but we cannot ignore the GCHS bylaw
with the legal title to the stock and entitled to vote it. Until a settlement and division of the provision, which specifically prescribes that vacancies in the board must be filled up by the
estate is effected, the stocks of the decedent are held by the administrator or executor. On remaining trustees. In other words, these remaining member-trustees must sit as a board in
the other hand, membership in and all rights arising from a nonstock corporation are personal order to validly elect the new ones. Indeed, there is a well-defined distinction between a
and non-transferable, unless the articles of incorporation or the bylaws of the corporation corporate act to be done by the board and that by the constituent members of the
provide otherwise. In other words, the determination of whether or not dead mem-bers corporation. The board of trustees must act, not individually or separately, but as a body in a
are entitled to exercise their voting rights (through their executor or administrator), depends lawful meeting. On the other hand, in their annual meeting, the members may be represented
on those articles of incorporation or bylaws. by their respective proxies, as in the contested annual members meeting of GCHS.

Same; Same; Dead members who are dropped from the membership roster in the PETITION for review on certiorari of the resolutions of the Court of Appeals.
manner and for the cause provided for in the By-Laws of Grace Christian High School, a
nonstock corporation, are not to be counted in determining the requisite vote in corporate DECISION
matters or the requisite quorum.Under the By-Laws of GCHS, membership in the PANGANIBAN, CJ.:
corporation shall, among others, be terminated by the death of the member. Section 91 of the
Corporation Code further provides that termination extinguishes all the rights of a member of For stock corporations, the "quorum" referred to in Section 52 of the Corporation
the corporation, unless otherwise provided in the articles of incorporation or the bylaws. Code is based on the number of outstanding voting stocks. For nonstock corporations, only
Applying Section 91 to the present case, we hold that dead members who are dropped from those who are actual, living members with voting rights shall be counted in determining the
the membership roster in the manner and for the cause provided for in the By-Laws of GCHS existence of a quorum during members meetings. Dead members shall not be counted.
are not to be counted in determining the requisite vote in corporate matters or the requisite
quorum for the annual members meeting. With 11 remaining members, the quorum in the The Case
present case should be 6. Therefore, there being a quorum, the annual members meeting,
conducted with six members present, was valid. The present Petition for Review on Certiorari [1] under Rule 45 of the Rules of Court
seeks the reversal of the January 23 2 and May 7, 2002, 3 Resolutions of the Court of Appeals
Same; Same; Words and Phrases; The phrase may be filled in Section 29 of the (CA) in CA-GR SP No. 68202. The first assailed Resolution dismissed the appeal filed by
Corporation Code shows that the filling of vacancies in the board by the remaining directors or petitioners with the CA. Allegedly, without the proper authorization of the other petitioners,
trustees constituting a quorum is merely permissive, not mandatorycorporations, therefore, the Verification and Certification of Non-Forum Shopping were signed by only one of them --
may choose how vacancies in their respective boards may be filled up.Undoubtedly, trustees Atty. Sabino Padilla Jr. The second Resolution denied reconsideration.
may fill vacancies in the board, provided that those remaining still constitute a quorum. The
phrase may be filled in Section 29 shows that the filling of vacancies in the board by the The Facts
remaining directors or trustees constituting a quorum is merely permissive, not mandatory.
Corporations, therefore, may choose how vacancies in their respective boards may be filled Petitioner Grace Christian High School (GCHS) is a nonstock, non-profit educational
upeither by the remaining directors constituting a quorum, or by the stockholders or corporation with fifteen (15) regular members, who also constitute the board of
members in a regular or special meeting called for the purpose. The By-Laws of GCHS trustees. [4] During the annual members meeting held on April 6, 1998, there were only eleven

162
MATABABE FILES
Corporation Law

(11) [5] living member-trustees, as four (4) had already died. Out of the eleven, seven rights are personal and non-transferable as provided in Sections 90 and 91 of the Corporation
(7) 6 attended the meeting through their respective proxies. The meeting was convened and Code of the Philippines.
chaired by Atty. Sabino Padilla Jr. over the objection of Atty. Antonio C. Pacis, who argued
that there was no quorum. 7 In the meeting, Petitioners Ernesto Tanchi, Edwin Ngo, Virginia "The SEC ruled against the petitioners solely on the basis of a 1989 SEC Opinion that
Khoo, and Judith Tan were voted to replace the four deceased member-trustees. did not even involve a non-stock corporation as petitioner GCHS.
When the controversy reached the Securities and Exchange Commission (SEC), petitioners
maintained that the deceased member-trustees should not be counted in the computation of "The Honorable Court of Appeals on the other hand simply refused to resolve this
the quorum because, upon their death, members automatically lost all their rights (including question and instead dismissed the petition for review on a technicality the failure to timely
the right to vote) and interests in the corporation. submit an SPA from the petitioners authorizing their co-petitioner Padilla, their counsel
and also a petitioner before the Court of Appeals, to sign the petition on behalf of the rest of
SEC Hearing Officer Malthie G. Militar declared the April 6, 1998 meeting null and void the petitioners.
for lack of quorum. She held that the basis for determining the quorum in a meeting of
members should be their number as specified in the articles of incorporation, not simply the "Petitioners humbly submit that the action of both the SEC and the Court of Appeals
number of living members. 8 She explained that the qualifying phrase "entitled to vote" in are not in accord with law particularly the pronouncements of this Honorable Court
Section 24 9 of the Corporation Code, which provided the basis for determining a quorum for in Escorpizo v. University of Baguio (306 SCRA 497), Robern Development Corporation v.
the election of directors or trustees, should be read together with Section 89. 10 Quitain (315 SCRA 150,) and MC Engineering, Inc. v. NLRC, (360 SCRA 183). Due course should
have been given the petition below and the merits of the case decided in petitioners favor." 17
The hearing officer also opined that Article III (2) 11 of the By-Laws of GCHS, insofar as
it prescribed the mode of filling vacancies in the board of trustees, must be interpreted in In sum, the issues may be stated simply in this wise: 1) whether the CA erred in
conjunction with Section 29 12 of the Corporation Code. The SEC en banc denied the appeal of denying the Petition below, on the basis of a defective Verification and Certification; and 2)
petitioners and affirmed the Decision of the hearing officer in toto.13 It found to be untenable whether dead members should still be counted in the determination of the quorum, for
their contention that the word "members," as used in Section 52 14 of the Corporation Code, purposes of conducting the annual members meeting.
referred only to the living members of a nonstock corporation. 15
The Courts Ruling
As earlier stated, the CA dismissed the appeal of petitioners, because the Verification
and Certification of Non-Forum Shopping had been signed only by Atty. Sabino Padilla Jr. No The present Petition is partly meritorious.
Special Power of Attorney had been attached to show his authority to sign for the rest of the
petitioners. Procedural Issue:
Verification and Certification of Non-Forum Shopping
Hence, this Petition. 16
The Petition before the CA was initially flawed, because the Verification and Certification of
Issues Non-Forum Shopping were signed by only one, not by all, of the petitioners; further, it failed
to show proof that the signatory was authorized to sign on behalf of all of them.
Petitioners state the issues as follows: Subsequently, however, petitioners submitted a Special Power of Attorney, attesting that
Atty. Padilla was authorized to file the action on their behalf. 18
"Petitioners principally pray for the resolution of the legal question of whether or not
in NON-STOCK corporations, dead members should still be counted in determination of In the interest of substantial justice, this initial procedural lapse may be
quorum for purposed of conducting the Annual Members Meeting. excused. 19 There appears to be no intention to circumvent the need for proper verification
and certification, which are aimed at assuring the truthfulness and correctness of the
"Petitioners have maintained before the courts below that the DEAD members allegations in the Petition for Review and at discouraging forum shopping. 20More important,
should no longer be counted in computing quorum primarily on the ground that members the substantial merits of petitioners case and the purely legal question involved in the

163
MATABABE FILES
Corporation Law

Petition should be considered special circumstances 21 or compelling reasons that justify an capacities. 31 Generally, the right to be present and to vote in a meeting is determined by the
exception to the strict requirements of the verification and the certification of non-forum time in which the meeting is held. 32
shopping. 22
Section 52 of the Corporation Code states:
Main Issue:
Basis for Quorum "Section 52. Quorum in Meetings. Unless otherwise provided for in this Code or in
the by-laws, a quorum shall consist of the stockholders representing a majority of the
Generally, stockholders or members meetings are called for the purpose of electing outstanding capital stock or a majority of the members in the case of non-stock
directors or trustees 23 and transacting some other business calling for or requiring the action corporations."
or consent of the shareholders or members, 24such as the amendment of the articles of
incorporation and bylaws, sale or disposition of all or substantially all corporate assets, In stock corporations, the presence of a quorum is ascertained and counted on the
consolidation and merger and the like, or any other business that may properly come before basis of the outstanding capital stock, as defined by the Code thus:
the meeting.
"SECTION 137. Outstanding capital stock defined. The term outstanding capital
Under the Corporation Code, stockholders or members periodically elect the board stock as used in this Code, means the total shares of stock issued under binding subscription
of directors or trustees, who are charged with the management of the corporation. 25 The agreements to subscribers or stockholders, whether or not fully or partially paid, except
board, in turn, periodically elects officers to carry out management functions on a day-to-day treasury shares." (Underscoring supplied)
basis. As owners, though, the stockholders or members have residual powers over
fundamental and major corporate changes. The Right to Vote in
Stock Corporations
While stockholders and members (in some instances) are entitled to receive profits,
the management and direction of the corporation are lodged with their representatives and The right to vote is inherent in and incidental to the ownership of corporate
agents -- the board of directors or trustees.26 In other words, acts of management pertain to stocks. 33 It is settled that unissued stocks may not be voted or considered in determining
the board; and those of ownership, to the stockholders or members. In the latter case, the whether a quorum is present in a stockholders meeting, or whether a requisite proportion of
board cannot act alone, but must seek approval of the stockholders or members. 27 the stock of the corporation is voted to adopt a certain measure or act. Only stock actually
issued and outstanding may be voted. 34 Under Section 6 of the Corporation Code, each share
Conformably with the foregoing principles, one of the most important rights of a of stock is entitled to vote, unless otherwise provided in the articles of incorporation or
qualified shareholder or member is the right declared delinquent 35 under Section 67 of the Code.
to vote -- either personally or by proxy -- for the directors or trustees who are to manage the
corporate affairs. 28The right to choose the persons who will direct, manage and operate the Neither the stockholders nor the corporation can vote or represent shares that have
corporation is significant, because it is the main way in which a stockholder can have a voice in never passed to the ownership of stockholders; or, having so passed, have again been
the management of corporate affairs, or in which a member in a nonstock corporation can purchased by the corporation. 36 These shares are not to be taken into consideration in
have a say on how the purposes and goals of the corporation may be achieved. 29 Once the determining majorities. When the law speaks of a
directors or trustees are elected, the stockholders or members relinquish corporate powers given proportion of the stock, it must be construed to mean the shares that have passed from
to the board in accordance with law. the corporation, and that may be voted. 37
Section 6 of the Corporation Code, in part, provides:
In the absence of an express charter or statutory provision to the contrary, the
general rule is that every member of a nonstock corporation, and every legal owner of shares "Section 6. Classification of shares. The shares of stock of stock corporations may
in a stock corporation, has a right to be present and to vote in all corporate meetings. be divided into classes or series of shares, or both, any of which classes or series of shares
Conversely, those who are not stockholders or members have no right to vote. 30Voting may may have such rights, privileges or restrictions as may be stated in the articles of
be expressed personally, or through proxies who vote in their representative incorporation: Provided, That no share may be deprived of voting rights except those

164
MATABABE FILES
Corporation Law

classified and issued as "preferred" or "redeemable" shares, unless otherwise provided in this Under Section 52 of the Corporation Code, the majority of the members representing
Code: Provided, further, that there shall always be a class or series of shares which have the actual number of voting rights, not
complete voting rights. the number or numerical constant that may originally be specified in the articles of
incorporation, constitutes the quorum. 41
xxxxxxxxx
The March 3, 1986 SEC Opinion 42 cited by the hearing officer uses the phrase
"Where the articles of incorporation provide for non-voting shares in the cases allowed by this "majority vote of the members"; likewise Section 48 of the Corporation Code refers to 50
Code, the holders of such shares shall nevertheless be entitled to vote on the following percent of 94 (the number of registered members of the association mentioned therein) plus
matters: one. The best evidence of who are the present members of the corporation is the
"membership book"; in the case of stock corporations, it is the stock and transfer book. 43
1. Amendment of the articles of incorporation;
2. Adoption and amendment of by-laws; Section 25 of the Code specifically provides that a majority of the directors or
3. Sale, lease, exchange, mortgage, pledge or other disposition of all or substantially all of the trustees, as fixed in the articles of incorporation, shall constitute a quorum for the transaction
corporation property; of corporate business (unless the articles of incorporation or the bylaws provide for a greater
4. Incurring, creating or increasing bonded indebtedness; majority). If the intention of the lawmakers was to base the quorum in the meetings of
5. Increase or decrease of capital stock; stockholders or members on their absolute number as fixed in the articles of incorporation, it
6. Merger or consolidation of the corporation with another corporation or other would have expressly specified so. Otherwise, the only logical conclusion is that the
corporations; legislature did not have that intention.
7. Investment of corporate funds in another corporation or business in accordance with this
Code; and Effect of the Death
8. Dissolution of the corporation. of a Member or Shareholder

"Except as provided in the immediately preceding paragraph, the vote necessary to Having thus determined that the quorum in a members meeting is to be reckoned as
approve a particular corporate act as provided in this Code shall be deemed to refer only to the actual number of members of the corporation, the next question to resolve is what
stocks with voting rights." happens in the event of the death of one of them.
In stock corporations, shareholders may generally transfer their shares. Thus, on the death of
Taken in conjunction with Section 137, the last paragraph of Section 6 shows that the a shareholder, the executor or administrator duly appointed by the Court is vested with the
intention of the lawmakers was to base the quorum mentioned in Section 52 on the number legal title to the stock and entitled to vote it. Until a settlement and division of the estate is
of outstanding voting stocks. 38 effected, the stocks of the decedent are held by the administrator or executor. 44

The Right to Vote in On the other hand, membership in and all rights arising from a nonstock corporation
Nonstock Corporations are personal and non-transferable, unless the articles of incorporation or the bylaws of the
corporation provide otherwise. 45 In other words, the determination of whether or not "dead
In nonstock corporations, the voting rights attach to membership. 39 Members vote members" are entitled to exercise their voting rights (through their executor or
as persons, in accordance with the law and the bylaws of the corporation. Each member shall administrator), depends on those articles of incorporation or bylaws.
be entitled to one vote unless so limited, broadened, or denied in the articles of incorporation
or bylaws. 40 We hold that when the principle for determining the quorum for stock Under the By-Laws of GCHS, membership in the corporation shall, among others, be
corporations is applied by analogy to nonstock corporations, only those who are actual terminated by the death of the member. 46 Section 91 of the Corporation Code further
members with voting rights should be counted. provides that termination extinguishes all the rights of a member of the corporation, unless
otherwise provided in the articles of incorporation or the bylaws.

165
MATABABE FILES
Corporation Law

Applying Section 91 to the present case, we hold that dead members who are their annual meeting, the members may be represented by their respective proxies, as in the
dropped from the membership roster in the manner and for the cause provided for in the By- contested annual members meeting of GCHS.
Laws of GCHS are not to be counted in determining the requisite vote in corporate matters or
the requisite quorum for the annual members meeting. With 11 remaining members, the WHEREFORE, the Petition is partly GRANTED.The assailed Resolutions of the Court of
quorum in the present case should be 6. Therefore, there being a quorum, the annual Appeals are hereby REVERSED AND SET ASIDE. The remaining members of the board of
members meeting, conducted with six 47 members present, was valid. trustees of Grace Christian High School (GCHS) may convene and fill up the vacancies in the
board, in accordance with this Decision. No pronouncement as to costs in this instance. SO
Vacancy in the ORDERED.
Board of Trustees

As regards the filling of vacancies in the board of trustees, Section 29 of the


Corporation Code provides:
"SECTION 29. Vacancies in the office of director or trustee. -- Any vacancy occurring in
the board of directors or trustees other than by removal by the stockholders or members or
by expiration of term, may be filled by the vote of at least a majority of the remaining
directors or trustees, if still constituting a quorum; otherwise, said vacancies must be filled by
the stockholders in a regular or special meeting called for that purpose. A director or trustee
so elected to fill a vacancy shall be elected only for the unexpired term of his predecessor in
office."

Undoubtedly, trustees may fill vacancies in the board, provided that those remaining
still constitute a quorum. The phrase "may be filled" in Section 29 shows that the filling of
vacancies in the board by the remaining directors or trustees constituting a quorum is merely
permissive, not mandatory. 48 Corporations, therefore, may choose how vacancies in their
respective boards may be filled up -- either by the remaining directors constituting a quorum,
or by the stockholders or members in a regular or special meeting called for the purpose. 49

The By-Laws of GCHS prescribed the specific mode of filling up existing vacancies in
its board of directors; that is, by a majority vote of the remaining members of the board. 50

While a majority of the remaining corporate members were present, however, the
"election" of the four trustees cannot be legally upheld for the obvious reason that it was
held in an annual meeting of the members, not of the board of trustees. We are not
unmindful of the fact that the members of GCHS themselves also constitute the trustees, but
we cannot ignore the GCHS bylaw provision, which specifically prescribes that vacancies in
the board must be filled up by the remaining trustees. In other words, these remaining
member-trustees must sit as a board in order to validly elect the new ones.

Indeed, there is a well-defined distinction between a corporate act to be done by the


board and that by the constituent members of the corporation. The board of trustees must
act, not individually or separately, but as a body in a lawful meeting. On the other hand, in

166
MATABABE FILES
Corporation Law

Republic of the Philippines Same; No term for life of Board.No time limit has been tacked to the existence of
SUPREME COURT the Board of Liquidators and its function of closing the affairs of various government
Manila corporations. Its term of life is not fixed.
EN BANC
Same; Right of Board of Liquidators to proceed as partyplaintiff; Case at bar.At no
G.R. No. L-18805 August 14, 1967 time had the government withdrawn the property. or the authority to continue the present
suit, from the Board of Liquidators. Hence, the Board can prosecute this case to its final
THE BOARD OF LIQUIDATORS1 representing THE GOVERNMENT OF THE REPUBLIC OF THE conclusion. The provisions of Section 78 of the Corporation Law, the third method of winding
PHILIPPINES, plaintiff-appellant, up corporate affairs, find application. The Board has personality to proceed as party-plaintiff
vs. in this case.
HEIRS OF MAXIMO M. KALAW,2 JUAN BOCAR, ESTATE OF THE DECEASED CASIMIRO
GARCIA,3 and LEONOR MOLL, defendants-appellees. Settlement of decedent's estate; Actions; Actions that survive; Executors and
administrators.The actions that survive against a decedent's executors or administrators
Courts; Judgment; Appeals.An appellate court may base its decision of affirmance are: (1) actions to recover real and personal property from the estate; (2) actions to enforce a
of the judgment below on a point or points ignored by the trial court on which said court was lien thereon; and (3) actions to recover damages for an injury to person or property. A suit to
in error. recover damages, based on the alleged tortious acts of the manager of a government
corporation, survives. It is not a mere money claim that is extinguished upon the death of a
Corporations; Three methods of winding up corporate affairs.Accepted in this party.
jurisdiction are three methods by which a corporation may wind up its affairs: (1) under
Section 3, Rule 104, of the Rules of Court (which superseded Section 66 of the Corporation Corporations; Implied authority of corporate officer to enter into contracts.A
Law), whereby, upon voluntary dissolution of a corporation, the court may direct "such corporate officer, entrusted with the general management and control of its business, has
disposition of its assets as justice requires, and may appoint a receiver to collect such assets implied authority to make any contract or do any other act which is necessary or appropriate
and pay the debts of the corporation"; (2) under Section 77 of the Corporation Law, whereby to the conduct of the ordinary business of the corporation. As such officer, he may, without
a corporation whose corporate existence is terminated, "shall nevertheless be continued as a any special authority from the Board of Directors, perform all acts of an ordinary nature,
body corporate for three years after the time when it would have been so dissolved, for the which by usage or necessity are incident to his office, and may bind the corporation by
purpose of prosecuting and defending suits by or against it and of enabling it gradually to contracts in matters arising in the usual course of business.
settle and close its affairs, to dispose of and convey its property and to divide its capital stock,
but not for the purpose of continuing the business for which it was established"; and (3) Same; Where similar acts of manager were approved by directors.Where similar
under Section 78 of the Corporation Law, by virtue of which the corporation, within the three- acts have been approved by the directors as a matter of general practice, custom, and policy,
year period just mentioned, "is authorized and empowered to convey all of its property to the general manager may bind the company without formal authorization of the board of
trustees for the benefit of members, stockholders, creditors, and others interested," directors. In varying language, existence of such authority is established by proof of the
course of business, the usages and practices of the company and by the knowledge which the
Board of Liquidators; Trustee for government.By Executive Order No. 372, the board of directors has, or must be presumed to have, of acts and doings of its subordinates in
government, the sole stockholder, abolished the National Coconut Corporation (NACOCO) and about the affairs of the corporation. Where the practice of the corporation has been to
and placed its assets in the hands of the Board of Liquidators. The Board thus became the allow its general manager to negotiate and execute contracts in its copra trading activities for
trustee on behalf of the government. It was an express trust. The legal interest became and in Nacoco's behalf without prior board approval, and the board itself, by its acts and
vested in the trustee, the Board of Liquidators. The beneficial interest remained with the sole through acquiescence, practically laid aside the by-law requirement of prior approval, the
stockholder, the government. The Board took the place of the dissolved government contracts of the general manager, under the given circumstances, are valid corporate acts.
corporations after the expiration of the statutory three-year period for the liquidation of their
affairs. Same; Ratification by corporation of unauthorized contract of its officers.
Ratification by a corporation of an unauthorized act or contract by its officers or others

167
MATABABE FILES
Corporation Law

relates back to the time of the act or contract ratified and is equivalent to original authority. NACOCO, after the passage of Republic Act 5, embarked on copra trading activities.
The corporation and the other party to the transaction are in precisely the same position as if Amongst the scores of contracts executed by general manager Kalaw are the disputed
the act or contract had been authorized at the time. The adoption or ratif ication of a contract contracts, for the delivery of copra, viz:
by a corporation is nothing more nor less than the making of an original contract. The theory
of corporate ratification is predicated on the right of a corporation to contract, and any (a) July 30, 1947: Alexander Adamson & Co., for 2,000 long tons, $167.00: per ton, f. o.
ratification or adoption is equivalent to a grant of prior authority. b., delivery: August and September, 1947. This contract was later assigned to Louis
Dreyfus & Co. (Overseas) Ltd.
Contracts; Bad faith.Bad faith does not simply connote bad judgment or
negligence; it imports a dishonest purpose or some moral obliquity and conscious doing of (b) August 14, 1947: Alexander Adamson & Co., for 2,000 long tons $145.00 per long
wrong; it means breach of a known duty through some motive or interest or ill-will; it ton, f.o.b., Philippine ports, to be shipped: September-October, 1947. This contract
partakes of the nature of fraud. was also assigned to Louis Dreyfus & Co. (Overseas) Ltd.

Damages; Damnum absque injuria.The present case is one of damnum absque (c) August 22, 1947: Pacific Vegetable Co., for 3,000 tons, $137.50 per ton, delivery:
injuria. Conjunction of damage and wrong is here absent. There cannot be an actionable September, 1947.
wrong if either one or the other is wanting.
(d) September 5, 1947: Spencer Kellog & Sons, for 1,000 long tons, $160.00 per ton,
APPEAL from a judgment of the Court of First Instance of Manila, Enriquez, J. c.i.f., Los Angeles, California, delivery: November, 1947.

The facts are stated in the opinion of the Court (e) September 9, 1947: Franklin Baker Division of General Foods Corporation, for
1,500 long tons, $164,00 per ton, c.i.f., New York, to be
Simeon M. Gopengco and Solicitor General for plaintiff-appellant. shipped in November, 1947.
L. H. Hernandez, Emma Quisumbing, Fernando and Quisumbing, Jr.; Ponce Enrile, Siguion Reyna,
Montecillo and Belo for defendants-appellees. (f) September 12, 1947: Louis Dreyfus & Co. (Overseas) Ltd., for 3,000 long tons,
$154.00 per ton, f.o.b., 3 Philippine ports, delivery: November, 1947.
SANCHEZ, J.:
(g) September 13, 1947: Juan Cojuangco, for 2,000 tons, $175.00 per ton, delivery:
The National Coconut Corporation (NACOCO, for short) was chartered as a non-profit November and December, 1947. This contract was assigned to Pacific Vegetable Co.
governmental organization on May 7, 1940 by Commonwealth Act 518 avowedly for the
protection, preservation and development of the coconut industry in the Philippines. On (h) October 27, 1947: Fairwood & Co., for 1,000 tons, $210.00 per short ton, c.i.f.,
August 1, 1946, NACOCO's charter was amended [Republic Act 5] to grant that corporation Pacific ports, delivery: December, 1947 and January, 1948. This contract was assigned
the express power "to buy, sell, barter, export, and in any other manner deal in, coconut, to Pacific Vegetable Co.
copra, and dessicated coconut, as well as their by-products, and to act as agent, broker or (i) October 28, 1947: Fairwood & Co., for 1,000 tons, $210.00 per short ton, c.i.f.,
commission merchant of the producers, dealers or merchants" thereof. The charter Pacific ports, delivery: January, 1948. This contract was assigned to Pacific Vegetable
amendment was enacted to stabilize copra prices, to serve coconut producers by securing Co.
advantageous prices for them, to cut down to a minimum, if not altogether eliminate, the
margin of middlemen, mostly aliens.4 An unhappy chain of events conspired to deter NACOCO from fulfilling these
contracts. Nature supervened. Four devastating typhoons visited the Philippines: the first in
General manager and board chairman was Maximo M. Kalaw; defendants Juan Bocar October, the second and third in November, and the fourth in December, 1947. Coconut trees
and Casimiro Garcia were members of the Board; defendant Leonor Moll became director throughout the country suffered extensive damage. Copra production decreased. Prices
only on December 22, 1947. spiralled. Warehouses were destroyed. Cash requirements doubled. Deprivation of export

168
MATABABE FILES
Corporation Law

facilities increased the time necessary to accumulate shiploads of copra. Quick turnovers Case 4322, appealed to this Court in L-2829), P447,908.40. These cases culminated in an out-
became impossible, financing a problem. of-court amicable settlement when the Kalaw management was already out. The corporation
thereunder paid Dreyfus P567,024.52 representing 70% of the total claims. With particular
When it became clear that the contracts would be unprofitable, Kalaw submitted reference to the Dreyfus claims, NACOCO put up the defenses that: (1) the contracts were
them to the board for approval. It was not until December 22, 1947 when the membership void because Louis Dreyfus & Co. (Overseas) Ltd. did not have license to do business here;
was completed. Defendant Moll took her oath on that date. A meeting was then held. Kalaw and (2) failure to deliver was due to force majeure, the typhoons. To project the utter
made a full disclosure of the situation, apprised the board of the impending heavy losses. No unreasonableness of this compromise, we reproduce in haec verba this finding below:
action was taken on the contracts. Neither did the board vote thereon at the meeting of
January 7, 1948 following. Then, on January 11, 1948, President Roxas made a statement that x x x However, in similar cases brought by the same claimant [Louis Dreyfus
the NACOCO head did his best to avert the losses, emphasized that government concerns & Co. (Overseas) Ltd.] against Santiago Syjuco for non-delivery of copra also
faced the same risks that confronted private companies, that NACOCO was recouping its involving a claim of P345,654.68 wherein defendant set up same defenses as above,
losses, and that Kalaw was to remain in his post. Not long thereafter, that is, on January 30, plaintiff accepted a promise of P5,000.00 only (Exhs. 31 & 32 Heirs.) Following the
1948, the board met again with Kalaw, Bocar, Garcia and Moll in attendance. They same proportion, the claim of Dreyfus against NACOCO should have been
unanimously approved the contracts hereinbefore enumerated. compromised for only P10,000.00, if at all. Now, why should defendants be held
liable for the large sum paid as compromise by the Board of Liquidators? This is just a
As was to be expected, NACOCO but partially performed the contracts, as follows: sample to show how unjust it would be to hold defendants liable for the readiness with
which the Board of Liquidators disposed of the NACOCO funds, although there was
much possibility of successfully resisting the claims, or at least settlement for nominal
Buyers Tons Delivered Undelivered
sums like what happened in the Syjuco case.5
Pacific Vegetable Oil 2,386.45 4,613.55
All the settlements sum up to P1,343,274.52.
Spencer Kellog None 1,000
In this suit started in February, 1949, NACOCO seeks to recover the above sum of
Franklin Baker 1,000 500 P1,343,274.52 from general manager and board chairman Maximo M. Kalaw, and directors
Louis Dreyfus 800 2,200 Juan Bocar, Casimiro Garcia and Leonor Moll. It charges Kalaw with negligence under Article
1902 of the old Civil Code (now Article 2176, new Civil Code); and defendant board members,
Louis Dreyfus (Adamson contract of July 30, 1947) 1,150 850 including Kalaw, with bad faith and/or breach of trust for having approved the contracts. The
fifth amended complaint, on which this case was tried, was filed on July 2, 1959. Defendants
Louis Dreyfus (Adamson Contract of August 14, 1947) 1,755 245 resisted the action upon defenses hereinafter in this opinion to be discussed.

The lower court came out with a judgment dismissing the complaint without costs as
TOTALS 7,091.45 9,408.55 well as defendants' counterclaims, except that plaintiff was ordered to pay the heirs of
Maximo Kalaw the sum of P2,601.94 for unpaid salaries and cash deposit due the deceased
The buyers threatened damage suits. Some of the claims were settled, viz: Pacific Kalaw from NACOCO.
Vegetable Oil Co., in copra delivered by NACOCO, P539,000.00; Franklin Baker Corporation, Plaintiff appealed direct to this Court.
P78,210.00; Spencer Kellog & Sons, P159,040.00.
Plaintiff's brief did not, question the judgment on Kalaw's counterclaim for the sum
But one buyer, Louis Dreyfus & Go. (Overseas) Ltd., did in fact sue before the Court of P2,601.94.
of First Instance of Manila, upon claims as follows: For the undelivered copra under the July
30 contract (Civil Case 4459); P287,028.00; for the balance on the August 14 contract (Civil Right at the outset, two preliminary questions raised before, but adversely decided
Case 4398), P75,098.63; for that per the September 12 contract reduced to judgment (Civil by, the court below, arrest our attention. On appeal, defendants renew their bid. And this,

169
MATABABE FILES
Corporation Law

upon established jurisprudence that an appellate court may base its decision of affirmance of Citing Mr. Justice Fisher, defendants proceed to argue that even where it may be
the judgment below on a point or points ignored by the trial court or in which said court was found impossible within the 3 year period to reduce disputed claims to judgment,
in error.6 nonetheless, "suits by or against a corporation abate when it ceases to be an entity capable of
1. First of the threshold questions is that advanced by defendants that plaintiff Board of suing or being sued" (Fisher, The Philippine Law of Stock Corporations, pp. 390-391). Corpus
Liquidators has lost its legal personality to continue with this suit. Juris Secundum likewise is authority for the statement that "[t]he dissolution of a corporation
ends its existence so that there must be statutory authority for prolongation of its life even
Accepted in this jurisdiction are three methods by which a corporation may wind up for purposes of pending litigation"9 and that suit "cannot be continued or revived; nor can a
its affairs: (1) under Section 3, Rule 104, of the Rules of Court [which superseded Section 66 of valid judgment be rendered therein, and a judgment, if rendered, is not only erroneous, but
the Corporation Law]7 whereby, upon voluntary dissolution of a corporation, the court may void and subject to collateral attack." 10 So it is, that abatement of pending actions follows as
direct "such disposition of its assets as justice requires, and may appoint a receiver to collect a matter of course upon the expiration of the legal period for liquidation, 11 unless the statute
such assets and pay the debts of the corporation;" (2) under Section 77 of the Corporation merely requires a commencement of suit within the added time. 12 For, the court cannot
Law, whereby a corporation whose corporate existence is terminated, "shall nevertheless be extend the time alloted by statute. 13
continued as a body corporate for three years after the time when it would have been so
dissolved, for the purpose of prosecuting and defending suits by or against it and of enabling We, however, express the view that the executive order abolishing NACOCO and
it gradually to settle and close its affairs, to dispose of and convey its property and to divide creating the Board of Liquidators should be examined in context. The proviso in Section 1
its capital stock, but not for the purpose of continuing the business for which it was of Executive Order 372, whereby the corporate existence of NACOCO was continued for a
established;" and (3) under Section 78 of the Corporation Law, by virtue of which the period of three years from the effectivity of the order for "the purpose of prosecuting and
corporation, within the three year period just mentioned, "is authorized and empowered to defending suits by or against it and of enabling the Board of Liquidators gradually to settle
convey all of its property to trustees for the benefit of members, stockholders, creditors, and and close its affairs, to dispose of and convey its property in the manner hereinafter
others interested."8 provided", is to be read not as an isolated provision but in conjunction with the whole. So
reading, it will be readily observed that no time limit has been tacked to the existence of the
It is defendants' pose that their case comes within the coverage of the second Board of Liquidators and its function of closing the affairs of the various government owned
method. They reason out that suit was commenced in February, 1949; that by Executive Order corporations, including NACOCO.
372, dated November 24, 1950, NACOCO, together with other government-owned
corporations, was abolished, and the Board of Liquidators was entrusted with the function of By Section 2 of the executive order, while the boards of directors of the various
settling and closing its affairs; and that, since the three year period has elapsed, the Board of corporations were abolished, their powers and functions and duties under existing laws were
Liquidators may not now continue with, and prosecute, the present case to its conclusion, to be assumed and exercised by the Board of Liquidators. The President thought it best to do
because Executive Order 372 provides in Section 1 thereof that away with the boards of directors of the defunct corporations; at the same time, however,
the President had chosen to see to it that the Board of Liquidators step into the vacuum. And
Sec.1. The National Abaca and Other Fibers Corporation, the National Coconut nowhere in the executive order was there any mention of the lifespan of the Board of
Corporation, the National Tobacco Corporation, the National Food Producer Liquidators. A glance at the other provisions of the executive order buttresses our conclusion.
Corporation and the former enemy-owned or controlled corporations or Thus, liquidation by the Board of Liquidators may, under section 1, proceed in accordance with
associations, . . . are hereby abolished. The said corporations shall be liquidated in law, the provisions of the executive order, "and/or in such manner as the President of the
accordance with law, the provisions of this Order, and/or in such manner as the Philippines may direct." By Section 4, when any property, fund, or project is transferred to any
President of the Philippines may direct; Provided, however, That each of the said governmental instrumentality "for administration or continuance of any project," the
corporations shall nevertheless be continued as a body corporate for a period of necessary funds therefor shall be taken from the corresponding special fund created in
three (3) years from the effective date of this Executive Order for the purpose of Section 5. Section 5, in turn, talks of special funds established from the "net proceeds of the
prosecuting and defending suits by or against it and of enabling the Board of liquidation" of the various corporations abolished. And by Section, 7, fifty per centum of the
Liquidators gradually to settle and close its affairs, to dispose of and, convey its fees collected from the copra standardization and inspection service shall accrue "to the
property in the manner hereinafter provided. special fund created in section 5 hereof for the rehabilitation and development of the coconut
industry." Implicit in all these, is that the term of life of the Board of Liquidators is without

170
MATABABE FILES
Corporation Law

time limit. Contemporary history gives us the fact that the Board of Liquidators still exists as Board of Liquidators. If for this reason alone, we cannot stay the hand of the Board of
an office with officials and numerous employees continuing the job of liquidation and Liquidators from prosecuting this case to its final conclusion. 16 The provisions of Section 78 of
prosecution of several court actions. the Corporation Law the third method of winding up corporate affairs find application.

Not that our views on the power of the Board of Liquidators to proceed to the final We, accordingly, rule that the Board of Liquidators has personality to proceed as:
determination of the present case is without jurisprudential support. The first judicial test party-plaintiff in this case.
before this Court is National Abaca and Other Fibers Corporation vs. Pore, L-16779, August 16, 2. Defendants' second poser is that the action is unenforceable against the heirs of Kalaw.
1961. In that case, the corporation, already dissolved, commenced suit within the three-year
extended period for liquidation. That suit was for recovery of money advanced to defendant Appellee heirs of Kalaw raised in their motion to dismiss, 17 which was overruled, and
for the purchase of hemp in behalf of the corporation. She failed to account for that money. in their nineteenth special defense, that plaintiff's action is personal to the deceased Maximo
Defendant moved to dismiss, questioned the corporation's capacity to sue. The lower court M. Kalaw, and may not be deemed to have survived after his death. 18 They say that the
ordered plaintiff to include as co-party plaintiff, The Board of Liquidators, to which the controlling statute is Section 5, Rule 87, of the 1940 Rules of Court. 19 which provides that "[a]ll
corporation's liquidation was entrusted by Executive Order 372. Plaintiff failed to effect claims for money against the decedent, arising from contract, express or implied", must be
inclusion. The lower court dismissed the suit. Plaintiff moved to reconsider. Ground: filed in the estate proceedings of the deceased. We disagree.
excusable negligence, in that its counsel prepared the amended complaint, as directed, and
instructed the board's incoming and outgoing correspondence clerk, Mrs. Receda Vda. de The suit here revolves around the alleged negligent acts of Kalaw for having entered
Ocampo, to mail the original thereof to the court and a copy of the same to defendant's into the questioned contracts without prior approval of the board of directors, to the damage
counsel. She mailed the copy to the latter but failed to send the original to the court. This and prejudice of plaintiff; and is against Kalaw and the other directors for having
motion was rejected below. Plaintiff came to this Court on appeal. We there said that "the subsequently approved the said contracts in bad faith and/or breach of trust." Clearly then,
rule appears to be well settled that, in the absence of statutory provision to the contrary, the present case is not a mere action for the recovery of money nor a claim for money arising
pending actions by or against a corporation are abated upon expiration of the period allowed from contract. The suit involves alleged tortious acts. And the action is embraced in suits filed
by law for the liquidation of its affairs." We there said that "[o]ur Corporation Law contains "to recover damages for an injury to person or property, real or personal", which survive. 20
no provision authorizing a corporation, after three (3) years from the expiration of its lifetime,
to continue in its corporate name actions instituted by it within said period of three (3) The leading expositor of the law on this point is Aguas vs. Llemos, L-18107, August 30,
years." 14 However, these precepts notwithstanding, we, in effect, held in that case that the 1962. There, plaintiffs sought to recover damages from defendant Llemos. The complaint
Board of Liquidators escapes from the operation thereof for the reason that "[o]bviously, the averred that Llemos had served plaintiff by registered mail with a copy of a petition for a writ
complete loss of plaintiff's corporate existence after the expiration of the period of of possession in Civil Case 4824 of the Court of First Instance at Catbalogan, Samar, with
three (3) years for the settlement of its affairs is what impelled the President to create a Board notice that the same would be submitted to the Samar court on February 23, 1960 at 8:00
of Liquidators, to continue the management of such matters as may then be pending." 15 We a.m.; that in view of the copy and notice served, plaintiffs proceeded to the said court of
accordingly directed the record of said case to be returned to the lower court, with Samar from their residence in Manila accompanied by their lawyers, only to discover that no
instructions to admit plaintiff's amended complaint to include, as party plaintiff, the Board of such petition had been filed; and that defendant Llemos maliciously failed to appear in court,
Liquidators. so that plaintiffs' expenditure and trouble turned out to be in vain, causing them mental
anguish and undue embarrassment. Defendant died before he could answer the complaint.
Defendants' position is vulnerable to attack from another direction. Upon leave of court, plaintiffs amended their complaint to include the heirs of the deceased.
The heirs moved to dismiss. The court dismissed the complaint on the ground that the legal
By Executive Order 372, the government, the sole stockholder, abolished NACOCO, representative, and not the heirs, should have been made the party defendant; and that,
and placed its assets in the hands of the Board of Liquidators. The Board of Liquidators thus anyway, the action being for recovery of money, testate or intestate proceedings should be
became the trustee on behalf of the government. It was an express trust. The legal interest initiated and the claim filed therein. This Court, thru Mr. Justice Jose B. L. Reyes, there
became vested in the trustee the Board of Liquidators. The beneficial declared:
interest remained with the sole stockholder the government. At no time had the
government withdrawn the property, or the authority to continue the present suit, from the

171
MATABABE FILES
Corporation Law

Plaintiffs argue with considerable cogency that contrasting the correlated provisions contracts necessary and essential to the proper accomplishment for which the Corporation
of the Rules of Court, those concerning claims that are barred if not filed in the estate was organized."
settlement proceedings (Rule 87, sec. 5) and those defining actions that survive and
may be prosecuted against the executor or administrator (Rule 88, sec. 1), it is Not of de minimis importance in a proper approach to the problem at hand, is the
apparent that actions for damages caused by tortious conduct of a defendant (as in nature of a general manager's position in the corporate structure. A rule that has gained
the case at bar) survive the death of the latter. Under Rule 87, section 5, the actions acceptance through the years is that a corporate officer "intrusted with the general
that are abated by death are: (1) claims for funeral expenses and those for the last management and control of its business, has implied authority to make any contract or do any
sickness of the decedent; (2) judgments for money; and (3) "all claims for money other act which is necessary or appropriate to the conduct of the ordinary business of the
against the decedent, arising from contract express or implied." None of these corporation. 21As such officer, "he may, without any special authority from the Board of
includes that of the plaintiffs-appellants; for it is not enough that the claim against Directors perform all acts of an ordinary nature, which by usage or necessity are incident to
the deceased party be for money, but it must arise from "contract express or his office, and may bind the corporation by contracts in matters arising in the usual course of
implied", and these words (also used by the Rules in connection with attachments business. 22
and derived from the common law) were construed in Leung Ben vs. O'Brien, 38 Phil.
182, 189-194, The problem, therefore, is whether the case at bar is to be taken out of the general
concept of the powers of a general manager, given the cited provision of the NACOCO by-
"to include all purely personal obligations other than those which have their laws requiring prior directorate approval of NACOCO contracts.
source in delict or tort."
The peculiar nature of copra trading, at this point, deserves express articulation.
Upon the other hand, Rule 88, section 1, enumerates actions that survive against a Ordinary in this enterprise are copra sales for future delivery. The movement of the market
decedent's executors or administrators, and they are: (1) actions to recover real and requires that sales agreements be entered into, even though the goods are not yet in the
personal property from the estate; (2) actions to enforce a lien thereon; and (3) hands of the seller. Known in business parlance as forward sales, it is concededly the practice
actions to recover damages for an injury to person or property. The present suit is of the trade. A certain amount of speculation is inherent in the undertaking. NACOCO was
one for damages under the last class, it having been held that "injury to property" is much more conservative than the exporters with big capital. This short-selling was inevitable
not limited to injuries to specific property, but extends to other wrongs by which at the time in the light of other factors such as availability of vessels, the quantity required
personal estate is injured or diminished (Baker vs. Crandall, 47 Am. Rep. 126; also 171 before being accepted for loading, the labor needed to prepare and sack the copra for
A.L.R., 1395). To maliciously cause a party to incur unnecessary expenses, as charged market. To NACOCO, forward sales were a necessity. Copra could not stay long in its hands; it
in this case, is certainly injury to that party's property (Javier vs. Araneta, L-4369, Aug. would lose weight, its value decrease. Above all, NACOCO's limited funds necessitated a quick
31, 1953). turnover. Copra contracts then had to be executed on short notice at times within twenty-
four hours. To be appreciated then is the difficulty of calling a formal meeting of the board.
The ruling in the preceding case was hammered out of facts comparable to those of
the present. No cogent reason exists why we should break away from the views just Such were the environmental circumstances when Kalaw went into copra trading.
expressed. And, the conclusion remains: Action against the Kalaw heirs and, for the matter,
against the Estate of Casimiro Garcia survives. Long before the disputed contracts came into being, Kalaw contracted by himself
alone as general manager for forward sales of copra. For the fiscal year ending June 30,
The preliminaries out of the way, we now go to the core of the controversy. 1947, Kalaw signed some 60 such contracts for the sale of copra to divers parties. During that
period, from those copra sales, NACOCO reaped a gross profit of P3,631,181.48. So pleased
3. Plaintiff levelled a major attack on the lower court's holding that Kalaw justifiedly was NACOCO's board of directors that, on December 5, 1946, in Kalaw's absence, it voted to
entered into the controverted contracts without the prior approval of the corporation's grant him a special bonus "in recognition of the signal achievement rendered by him in putting
directorate. Plaintiff leans heavily on NACOCO's corporate by-laws. Article IV (b), Chapter III the Corporation's business on a self-sufficient basis within a few months after assuming
thereof, recites, as amongst the duties of the general manager, the obligation: "(b) To office, despite numerous handicaps and difficulties."
perform or execute on behalf of the Corporation upon prior approval of the Board, all

172
MATABABE FILES
Corporation Law

These previous contract it should be stressed, were signed by Kalaw without prior
authority from the board. Said contracts were known all along to the board members. When the board met on May 10, 1947, the directors discussed the copra situation:
Nothing was said by them. The aforesaid contracts stand to prove one thing: Obviously, There was a slow downward trend but belief was entertained that the nadir might have
NACOCO board met the difficulties attendant to forward sales by leaving the adoption of already been reached and an improvement in prices was expected. In view thereof, Kalaw
means to end, to the sound discretion of NACOCO's general manager Maximo M. Kalaw. informed the board that "he intends to wait until he has signed contracts to sell before starting
to buy copra."23
Liberally spread on the record are instances of contracts executed by NACOCO's
general manager and submitted to the board after their consummation, not before. These In the board meeting of July 29, 1947, Kalaw reported on the copra price conditions
agreements were not Kalaw's alone. One at least was executed by a predecessor way back in then current: The copra market appeared to have become fairly steady; it was not expected
1940, soon after NACOCO was chartered. It was a contract of lease executed on November 16, that copra prices would again rise very high as in the unprecedented boom during January-
1940 by the then general manager and board chairman, Maximo Rodriguez, and A. Soriano y April, 1947; the prices seemed to oscillate between $140 to $150 per ton; a radical rise or
Cia., for the lease of a space in Soriano Building On November 14, 1946, NACOCO, thru its decrease was not indicated by the trends. Kalaw continued to say that "the Corporation has
general manager Kalaw, sold 3,000 tons of copra to the Food Ministry, London, thru been closing contracts for the sale of copra generally with a margin of P5.00 to P7.00 per
Sebastian Palanca. On December 22, 1947, when the controversy over the present contract hundred kilos." 24
cropped up, the board voted to approve a lease contract previously executed between Kalaw
and Fidel Isberto and Ulpiana Isberto covering a warehouse of the latter. On the same date, We now lift the following excerpts from the minutes of that same board meeting of
the board gave its nod to a contract for renewal of the services of Dr. Manuel L. Roxas. In July 29, 1947:
fact, also on that date, the board requested Kalaw to report for action all copra
contracts signed by him "at the meeting immediately following the signing of the contracts." 521. In connection with the buying and selling of copra the Board inquired whether it
This practice was observed in a later instance when, on January 7, 1948, the board approved is the practice of the management to close contracts of sale first before buying. The
two previous contracts for the sale of 1,000 tons of copra each to a certain "SCAP" and a General Manager replied that this practice is generally followed but that it is not
certain "GNAPO". always possible to do so for two reasons:

And more. On December 19, 1946, the board resolved to ratify the brokerage (1) The role of the Nacoco to stabilize the prices of copra requires that it should not
commission of 2% of Smith, Bell and Co., Ltd., in the sale of 4,300 long tons of copra to the cease buying even when it does not have actual contracts of sale since the
French Government. Such ratification was necessary because, as stated by Kalaw in that same suspension of buying by the Nacoco will result in middlemen taking advantage of the
meeting, "under an existing resolution he is authorized to give a brokerage fee of only 1% on temporary inactivity of the Corporation to lower the prices to the detriment of the
sales of copra made through brokers." On January 15, 1947, the brokerage fee agreements of producers.
1-1/2% on three export contracts, and 2% on three others, for the sale of copra were approved (2) The movement of the market is such that it may not be practical always to wait
by the board with a proviso authorizing the general manager to pay a commission up to the for the consummation of contracts of sale before beginning to buy copra.
amount of 1-1/2% "without further action by the Board." On February 5, 1947, the brokerage fee
of 2% of J. Cojuangco & Co. on the sale of 2,000 tons of copra was favorably acted upon by the The General Manager explained that in this connection a certain amount of
board. On March 19, 1947, a 2% brokerage commission was similarly approved by the board for speculation is unavoidable. However, he said that the Nacoco is much more
Pacific Trading Corporation on the sale of 2,000 tons of copra. conservative than the other big exporters in this respect. 25

It is to be noted in the foregoing cases that only the brokerage fee agreements were Settled jurisprudence has it that where similar acts have been approved by the
passed upon by the board, not the sales contracts themselves. And even those fee directors as a matter of general practice, custom, and policy, the general manager may bind
agreements were submitted only when the commission exceeded the ceiling fixed by the the company without formal authorization of the board of directors. 26 In varying language,
board. existence of such authority is established, by proof of the course of business, the usage and
practices of the company and by the knowledge which the board of directors has, or must
Knowledge by the board is also discernible from other recorded instances.

173
MATABABE FILES
Corporation Law

be presumed to have, of acts and doings of its subordinates in and about the affairs of the 5. It would be difficult, even with hostile eyes, to read the record in terms of "bad faith and/or
corporation. 27So also, breach of trust" in the board's ratification of the contracts without prior approval of the
board. For, in reality, all that we have on the government's side of the scale is that the board
x x x authority to act for and bind a corporation may be presumed from acts of knew that the contracts so confirmed would cause heavy losses.
recognition in other instances where the power was in fact exercised. 28
As we have earlier expressed, Kalaw had authority to execute the contracts without
x x x Thus, when, in the usual course of business of a corporation, an officer has been need of prior approval. Everybody, including Kalaw himself, thought so, and for a long time.
allowed in his official capacity to manage its affairs, his authority to represent the Doubts were first thrown on the way only when the contracts turned out to be unprofitable
corporation may be implied from the manner in which he has been permitted by the for NACOCO.
directors to manage its business.29
Rightfully had it been said that bad faith does not simply connote bad judgment or
In the case at bar, the practice of the corporation has been to allow its general negligence; it imports a dishonest purpose or some moral obliquity and conscious doing of
manager to negotiate and execute contracts in its copra trading activities for and in wrong; it means breach of a known duty thru some motive or interest or ill will; it partakes of
NACOCO's behalf without prior board approval. If the by-laws were to be literally followed, the nature of fraud.34 Applying this precept to the given facts herein, we find that there was
the board should give its stamp of prior approval on all corporate contracts. But that board no "dishonest purpose," or "some moral obliquity," or "conscious doing of wrong," or
itself, by its acts and through acquiescence, practically laid aside the by-law requirement of "breach of a known duty," or "Some motive or interest or ill will" that "partakes of the nature
prior approval. of fraud."

Under the given circumstances, the Kalaw contracts are valid corporate acts. Nor was it even intimated here that the NACOCO directors acted for personal
reasons, or to serve their own private interests, or to pocket money at the expense of the
4. But if more were required, we need but turn to the board's ratification of the corporation. 35 We have had occasion to affirm that bad faith contemplates a "state of mind
contracts in dispute on January 30, 1948, though it is our (and the lower court's) belief that affirmatively operating with furtive design or with some motive of self-interest or ill will or for
ratification here is nothing more than a mere formality. ulterior purposes." 36 Briggs vs. Spaulding, 141 U.S. 132, 148-149, 35 L. ed. 662, 669, quotes with
Authorities, great in number, are one in the idea that "ratification by a corporation of approval from Judge Sharswood (in Spering's App., 71 Pa. 11), the following: "Upon a close
an unauthorized act or contract by its officers or others relates back to the time of the act or examination of all the reported cases, although there are many dicta not easily reconcilable,
contract ratified, and is equivalent to original authority;" and that " [t]he corporation and the yet I have found no judgment or decree which has held directors to account, except when
other party to the transaction are in precisely the same position as if the act or contract had they have themselves been personally guilty of some fraud on the corporation, or have
been authorized at the time." 30 The language of one case is expressive: "The adoption or known and connived at some fraud in others, or where such fraud might have been prevented
ratification of a contract by a corporation is nothing more or less than the making of an had they given ordinary attention to their duties. . . ." Plaintiff did not even dare charge its
original contract. The theory of corporate ratification is predicated on the right of a defendant-directors with any of these malevolent acts.
corporation to contract, and any ratification or adoption is equivalent to a grant of prior
authority." 31 Obviously, the board thought that to jettison Kalaw's contracts would contravene
basic dictates of fairness. They did not think of raising their voice in protest against past
Indeed, our law pronounces that "[r]atification cleanses the contract from all its contracts which brought in enormous profits to the corporation. By the same token, fair
defects from the moment it was constituted." 32 By corporate confirmation, the contracts dealing disagrees with the idea that similar contracts, when unprofitable, should not merit the
executed by Kalaw are thus purged of whatever vice or defect they may have. 33 same treatment. Profit or loss resulting from business ventures is no justification for turning
one's back on contracts entered into. The truth, then, of the matter is that in the words of
In sum, a case is here presented whereunder, even in the face of an express by-law the trial court the ratification of the contracts was "an act of simple justice and fairness to
requirement of prior approval, the law on corporations is not to be held so rigid and inflexible the general manager and the best interest of the corporation whose prestige would have
as to fail to recognize equitable considerations. And, the conclusion inevitably is that the been seriously impaired by a rejection by the board of those contracts which proved
embattled contracts remain valid. disadvantageous." 37

174
MATABABE FILES
Corporation Law

As the trial court correctly observed, this is a case of damnum absque injuria.
The directors are not liable." 38 Conjunction of damage and wrong is here absent. There cannot be an actionable wrong if
either one or the other is wanting. 43
6. To what then may we trace the damage suffered by NACOCO.
7. On top of all these, is that no assertion is made and no proof is presented which
The facts yield the answer. Four typhoons wreaked havoc then on our copra-producing would link Kalaw's acts ratified by the board to a matrix for defraudation of the
regions. Result: Copra production was impaired, prices spiralled, warehouses destroyed. government. Kalaw is clear of the stigma of bad faith. Plaintiff's corporate counsel 44 concedes
Quick turnovers could not be expected. NACOCO was not alone in this misfortune. The record that Kalaw all along thought that he had authority to enter into the contracts, that he did so
discloses that private traders, old, experienced, with bigger facilities, were not spared; also in the best interests of the corporation; that he entered into the contracts in pursuance of an
suffered tremendous losses. Roughly estimated, eleven principal trading concerns did run overall policy to stabilize prices, to free the producers from the clutches of the middlemen.
losses to about P10,300,000.00. Plaintiff's witness Sisenando Barretto, head of the copra The prices for which NACOCO contracted in the disputed agreements, were at a level
marketing department of NACOCO, observed that from late 1947 to early 1948 "there were calculated to produce profits and higher than those prevailing in the local market. Plaintiff's
many who lost money in the trade." 39 NACOCO was not immune from such usual business witness, Barretto, categorically stated that "it would be foolish to think that one would sign
risk. (a) contract when you are going to lose money" and that no contract was executed "at a
price unsafe for the Nacoco." 45 Really, on the basis of prices then prevailing, NACOCO
The typhoons were known to plaintiff. In fact, NACOCO resisted the suits filed by envisioned a profit of around P752,440.00. 46
Louis Dreyfus & Co. by pleading in its answers force majeure as an affirmative defense and
there vehemently asserted that "as a result of the said typhoons, extensive damage was Kalaw's acts were not the result of haphazard decisions either. Kalaw invariably
caused to the coconut trees in the copra producing regions of the Philippines and according consulted with NACOCO's Chief Buyer, Sisenando Barretto, or the Assistant General Manager.
to estimates of competent authorities, it will take about one year until the coconut producing The dailies and quotations from abroad were guideposts to him.
regions will be able to produce their normal coconut yield and it will take some time until the
price of copra will reach normal levels;" and that "it had never been the intention of the Of course, Kalaw could not have been an insurer of profits. He could not be expected
contracting parties in entering into the contract in question that, in the event of a sharp rise in to predict the coming of unpredictable typhoons. And even as typhoons supervened Kalaw
the price of copra in the Philippine market produce by force majeure or by caused beyond was not remissed in his duty. He exerted efforts to stave off losses. He asked the Philippine
defendant's control, the defendant should buy the copra contracted for at exorbitant prices National Bank to implement its commitment to extend a P400,000.00 loan. The bank did not
far beyond the buying price of the plaintiff under the contract." 40 release the loan, not even the sum of P200,000.00, which, in October, 1947, was approved by
the bank's board of directors. In frustration, on December 12, 1947, Kalaw turned to the
A high regard for formal judicial admissions made in court pleadings would suffice to President, complained about the bank's short-sighted policy. In the end, nothing came out of
deter us from permitting plaintiff to stray away therefrom, to charge now that the damage the negotiations with the bank. NACOCO eventually faltered in its contractual obligations.
suffered was because of Kalaw's negligence, or for that matter, by reason of the board's
ratification of the contracts. 41 That Kalaw cannot be tagged with crassa negligentia or as much as simple negligence,
would seem to be supported by the fact that even as the contracts were being questioned in
Indeed, were it not for the typhoons, 42 NACOCO could have, with ease, met its Congress and in the NACOCO board itself, President Roxas defended the actuations of Kalaw.
contractual obligations. Stock accessibility was no problem. NACOCO had 90 buying agencies On December 27, 1947, President Roxas expressed his desire "that the Board of Directors
spread throughout the islands. It could purchase 2,000 tons of copra a day. The various should reelect Hon. Maximo M. Kalaw as General Manager of the National Coconut
contracts involved delivery of but 16,500 tons over a five-month period. Despite the typhoons, Corporation." 47 And, on January 7, 1948, at a time when the contracts had already been
NACOCO was still able to deliver a little short of 50% of the tonnage required under the openly disputed, the board, at its regular meeting, appointed Maximo M. Kalaw as acting
contracts. general manager of the corporation.

Well may we profit from the following passage from Montelibano vs. Bacolod-Murcia
Milling Co., Inc., L-15092, May 18, 1962:

175
MATABABE FILES
Corporation Law

"They (the directors) hold such office charged with the duty to act for the
corporation according to their best judgment, and in so doing they cannot be controlled in
the reasonable exercise and performance of such duty. Whether the business of a corporation
should be operated at a loss during a business depression, or closed down at a smaller loss, is
a purely business and economic problem to be determined by the directors of the
corporation, and not by the court. It is a well known rule of law that questions of policy of
management are left solely to the honest decision of officers and directors of a corporation,
and the court is without authority to substitute its judgment for the judgment of the board of
directors; the board is the business manager of the corporation, and solong as it acts in good
faith its orders are not reviewable by the courts." (Fletcher on Corporations, Vol. 2, p. 390.) 48
Kalaw's good faith, and that of the other directors, clinch the case for defendants. 49

Viewed in the light of the entire record, the judgment under review must be, as it is hereby,
affirmed.

Without costs. So ordered.

Reyes, J.B.L., Makalintal, Bengzon, J.P., Zaldivar, Castro and Angeles, JJ., concur.
Fernando, J., took no part.
Concepcion, C.J. and Dizon, J., are on leave.

176
MATABABE FILES
Corporation Law

Republic of the Philippines Same; Same; Same; Same; Questions of policy and of management are left to the
SUPREME COURT honest decision of the officers and directors of a corporation, and the courts are without
Manila authority to substitute their judgment for that of the board of directorsthe board is the
SECOND DIVISION business manager of the corporation, and so long as it acts in good faith, its orders are not
reviewable by the courts.A corporation is but an association of individuals, allowed to
G.R. No. 125469 October 27, 1997 transact under an assumed corporate name, and with a distinct legal personality. In
organizing itself as a collective body, it waives no constitutional immunities and perquisites
PHILIPPINE STOCK EXCHANGE, INC., petitioner, appropriate to such a body. As to its corporate and management decisions, therefore, the
vs. state will generally not interfere with the same. Questions of policy and of management are
THE HONORABLE COURT OF APPEALS, SECURITIES AND EXCHANGE COMMISSION and left to the honest decision of the officers and directors of a corporation, and the courts are
PUERTO AZUL LAND, INC., respondents. without authority to substitute their judgment for the judgment of the board of directors. The
board is the business manager of the corporation, and so long as it acts in good faith, its
Corporation Law; Securities and Exchange Commission; Stock Exchanges; The SEC is orders are not reviewable by the courts.
the entity with the primary say as to whether or not securities, including shares of stock of a
corporation, may be traded or not in the stock exchange.We affirm that the SEC is the entity Same; Same; Same; Same; Notwithstanding the regulatory power of the SEC over the
with the primary say as to whether or not securities, including shares of stock of a PSE, and the resultant authority to reverse the PSEs decision in matters of application for
corporation, may be traded or not in the stock exchange. This is in line with the SECs mission listing in the market, the SEC may exercise such power only if the PSEs judgment is attended
to ensure proper compliance with the laws, such as the Revised Securities Act and to regulate by bad faith.Thus, notwithstanding the regulatory power of the SEC over the PSE, and the
the sale and disposition of securities in the country. As the appellate court explains: resultant authority to reverse the PSEs decision in matters of application for listing in the
Paramount policy also supports the authority of the public respondent to review petitioners market, the SEC may exercise such power only if the PSEs judgment is attended by bad faith.
denial of the listing. Being a stock exchange, the petitioner performs a function that is vital to In Board of Liquidators vs. Kalaw, it was held that bad faith does not simply connote bad
the national economy, as the business is affected with public interest. As a matter of fact, it judgment or negligence. It imports a dishonest purpose or some moral obliquity and
has often been said that the economy moves on the basis of the rise and fall of stocks being conscious doing of wrong. It means a breach of a known duty through some motive or
traded. By its economic power, the petitioner certainly can dictate which and how many users interest of ill will, partaking of the nature of fraud.
are allowed to sell securities thru the facilities of a stock exchange, if allowed to interpret its
own rules liberally as it may please. Petitioner can either allow or deny the entry to the market Same; Same; Same; Same; As the primary market for securities, the PSE had
of securities. To repeat, the monopoly, unless accompanied by control, becomes subject to established its name and goodwill, and it has the right to protect such goodwill by maintaining
abuse; hence, considering public interest, then it should be subject to government a reasonable standard of propriety in the entities who choose to transact through its facilities;
regulation. The concept of government absolutism is a thing of the past, and should remain so.Also, as
the primary market for securities, the PSE has established its name and goodwill, and it has
Same; Same; Same; Philippine Stock Exchange; The PSEs management prerogatives the right to protect such goodwill by maintaining a reasonable standard of propriety in the
are not under the absolute control of the SEC, for the PSE is, after all, a corporation authorized entities who choose to transact through its facilities. It was reasonable for the PSE, therefore,
by its corporate franchise to engage in its proposed and duly approved business.This is not to exercise its judgment in the manner it deems appropriate for its business identity, as long
to say, however, that the PSEs management prerogatives are under the absolute control of as no rights are trampled upon, and public welfare is safeguarded. In this connection, it is
the SEC. The PSE is, after all, a corporation authorized by its corporate franchise to engage in proper to observe that the concept of government absolutism is a thing of the past, and
its proposed and duly approved business. One of the PSEs main concerns, as such, is still the should remain so.
generation of profit for its stockholders. Moreover, the PSE has all the rights pertaining to Same; Same; Same; Same; The SEC had acted arbitrarily in arrogating unto itself the
corporations, including the right to sue and be sued, to hold property in its own name, to discretion of approving the application for listing of Puerto Azul Land, Inc., since this is a
enter (or not to enter) into contracts with third persons, and to perform all other legal acts matter addressed to the sound discretion of the PSE, a corporate entity, whose business
within its allocated express or implied powers. judgments are respected in the absence of bad faith.In any case, for the purpose of
determining whether PSE acted correctly in refusing the application of PALI, the true

177
MATABABE FILES
Corporation Law

ownership of the properties of PALI need not be determined as an absolute fact. What is PETITION for review on certiorari of a decision of the Court of Appeals.
material is that the uncertainty of the properties ownership and alienability exists, and this
puts to question the qualification of PALIs public offering. In sum, the Court finds that the The facts are stated in the opinion of the Court.
SEC had acted arbitrarily in arrogating unto itself the discretion of approving the application TORRES, JR., J.:
for listing in the PSE of the private respondent PALI, since this is a matter addressed to the
sound discretion of the PSE, a corporate entity, whose business judgments are respected in The Securities and Exchange Commission is the government agency, under the direct
the absence of bad faith. general supervision of the Office of the President, 1 with the immense task of enforcing the
Revised Securities Act, and all other duties assigned to it by pertinent laws. Among its
Same; Same; Same; The question as to what policy is, or should be relied upon in inumerable functions, and one of the most important, is the supervision of all corporations,
approving the registration and sale of securities in the PSE is not for the Supreme Court to partnerships or associations, who are grantees of primary franchise and/or a license or permit
determine, but is left to the sound discretion of the Securities and Exchange Commission.The issued by the government to operate in the Philippines. 2 Just how far this regulatory
question as to what policy is, or should be relied upon in approving the registration and sale authority extends, particularly, with regard to the Petitioner Philippine Stock Exchange, Inc. is
of securities in the PSE is not for the Court to determine, but is left to the sound discretion of the issue in the case at bar.
the Securities and Exchange Commission. In mandating the SEC to administer the Revised
Securities Act, and in performing its other functions under pertinent laws, the Revised In this Petition for Review on Certiorari, petitioner assails the resolution of the
Securities Act, under Section 3 thereof, gives the SEC the power to promulgate such rules and respondent Court of Appeals, dated June 27, 1996, which affirmed the decision of the
regulations as it may consider appropriate in the public interest for the enforcement of the Securities and Exchange Commission ordering the petitioner Philippine Stock Exchange, Inc.
said laws. The second paragraph of Section 4 of the said law, on the other hand, provides that to allow the private respondent Puerto Azul Land, Inc. to be listed in its stock market, thus
no security, unless exempt by law, shall be issued, endorsed, sold, transferred or in any other paving the way for the public offering of PALI's shares.
manner conveyed to the public, unless registered in accordance with the rules and regulations
that shall be promulgated in the public interest and for the protection of investors by the The facts of the case are undisputed, and are hereby restated in sum.
Commission. Presidential Decree No. 902-A, on the other hand, provides that the SEC, as
regulatory agency, has supervision and control over all corporations and over the securities The Puerto Azul Land, Inc. (PALI), a domestic real estate corporation, had sought to
market as a whole, and as such, is given ample authority in determining appropriate policies. offer its shares to the public in order to raise funds allegedly to develop its properties and pay
its loans with several banking institutions. In January, 1995, PALI was issued a Permit to Sell its
Same; Same; Same; The absolute reliance on the full disclosure method in the shares to the public by the Securities and Exchange Commission (SEC). To facilitate the
registration of securities is untenable.A reading of the foregoing grounds reveals the trading of its shares among investors, PALI sought to course the trading of its shares through
intention of the lawmakers to make the registration and issuance of securities dependent, to the Philippine Stock Exchange, Inc. (PSE), for which purpose it filed with the said stock
a certain extent, on the merits of the securities themselves, and of the issuer, to be exchange an application to list its shares, with supporting documents attached.
determined by the Securities and Exchange Commission. This measure was meant to protect
the interests of the investing public against fraudulent and worthless securities, and the SEC is On February 8, 1996, the Listing Committee of the PSE, upon a perusal of PALI's
mandated by law to safeguard these interests, following the policies and rules therefore application, recommended to the PSE's Board of Governors the approval of PALI's listing
provided. The absolute reliance on the full disclosure method in the registration of securities application.
is, therefore, untenable. As it is, the Court finds that the private respondent PALI, on at least
two points (Nos. 1 and 5) has failed to support the propriety of the issue of its shares with On February 14, 1996, before it could act upon PALI's application, the Board of
unfailing clarity, thereby lending support to the conclusion that the PSE acted correctly in Governors of the PSE received a letter from the heirs of Ferdinand E. Marcos, claiming that
refusing the listing of PALI in its stock exchange. This does not discount the effectivity of the late President Marcos was the legal and beneficial owner of certain properties forming
whatever method the SEC, in the exercise of its vested authority, chooses in setting the part of the Puerto Azul Beach Hotel and Resort Complex which PALI claims to be among its
standard for public offerings of corporations wishing to do so. However, the SEC must assets and that the Ternate Development Corporation, which is among the stockholders of
recognize and implement the mandate of the law, particularly the Revised Securities Act, the PALI, likewise appears to have been held and continue to be held in trust by one Rebecco
provisions of which cannot be amended or supplanted by mere administrative issuance.

178
MATABABE FILES
Corporation Law

Panlilio for then President Marcos and now, effectively for his estate, and requested PALI's WHEREFORE, premises considered, and invoking the Commissioner's
application to be deferred. PALI was requested to comment upon the said letter. authority and jurisdiction under Section 3 of the Revised Securities Act, in
conjunction with Section 3, 6(j) and 6(m) of Presidential Decree No. 902-A,
PALI's answer stated that the properties forming part of the Puerto Azul Beach Hotel the decision of the Board of Governors of the Philippine Stock Exchange
and Resort Complex were not claimed by PALI as its assets. On the contrary, the resort is denying the listing of shares of Puerto Azul Land, Inc., is hereby set aside,
actually owned by Fantasia Filipina Resort, Inc. and the Puerto Azul Country Club, entities and the PSE is hereby ordered to immediately cause the listing of the PALI
distinct from PALI. Furthermore, the Ternate Development Corporation owns only 1.20% of shares in the Exchange, without prejudice to its authority to require PALI to
PALI. The Marcoses responded that their claim is not confined to the facilities forming part of disclose such other material information it deems necessary for the
the Puerto Azul Hotel and Resort Complex, thereby implying that they are also asserting legal protection of the investigating public.
and beneficial ownership of other properties titled under the name of PALI.
This Order shall take effect immediately.
On February 20, 1996, the PSE wrote Chairman Magtanggol Gunigundo of the SO ORDERED.
Presidential Commission on Good Government (PCGG) requesting for comments on the
letters of the PALI and the Marcoses. On March 4, 1996, the PSE was informed that the PSE filed a motion for reconsideration of the said order on April 29, 1996, which was,
Marcoses received a Temporary Restraining Order on the same date, enjoining the Marcoses however denied by the Commission in its May 9, 1996 Order which states:
from, among others, "further impeding, obstructing, delaying or interfering in any manner by WHEREFORE, premises considered, the Commission finds no
or any means with the consideration, processing and approval by the PSE of the initial public compelling reason to reconsider its order dated April 24, 1996, and in the
offering of PALI." The TRO was issued by Judge Martin S. Villarama, Executive Judge of the light of recent developments on the adverse claim against the PALI
RTC of Pasig City in Civil Case No. 65561, pending in Branch 69 thereof. properties, PSE should require PALI to submit full disclosure of material facts
and information to protect the investing public. In this regard, PALI is hereby
In its regular meeting held on March 27, 1996, the Board of Governors of the PSE ordered to amend its registration statements filed with the Commission to
reached its decision to reject PALI's application, citing the existence of serious claims, issues incorporate the full disclosure of these material facts and information.
and circumstances surrounding PALI's ownership over its assets that adversely affect the
suitability of listing PALI's shares in the stock exchange. Dissatisfied with this ruling, the PSE filed with the Court of Appeals on May 17, 1996 a
Petition for Review (with Application for Writ of Preliminary Injunction and Temporary
On April 11, 1996, PALI wrote a letter to the SEC addressed to the then Acting Restraining Order), assailing the above mentioned orders of the SEC, submitting the following
Chairman, Perfecto R. Yasay, Jr., bringing to the SEC's attention the action taken by the PSE in as errors of the SEC:
the application of PALI for the listing of its shares with the PSE, and requesting that the SEC,
in the exercise of its supervisory and regulatory powers over stock exchanges under Section I. SEC COMMITTED SERIOUS ERROR AND GRAVE ABUSE OF
6(j) of P.D. No. 902-A, review the PSE's action on PALI's listing application and institute such DISCRETION IN ISSUING THE ASSAILED ORDERS WITHOUT
measures as are just and proper under the circumstances. POWER, JURISDICTION, OR AUTHORITY; SEC HAS NO
POWER TO ORDER THE LISTING AND SALE OF SHARES OF
On the same date, or on April 11, 1996, the SEC wrote to the PSE, attaching thereto PALI WHOSE ASSETS ARE SEQUESTERED AND TO REVIEW
the letter of PALI and directing the PSE to file its comments thereto within five days from its AND SUBSTITUTE DECISIONS OF PSE ON LISTING
receipt and for its authorized representative to appear for an "inquiry" on the matter. On APPLICATIONS;
April 22, 1996, the PSE submitted a letter to the SEC containing its comments to the April 11,
1996 letter of PALI. II. SEC COMMITTED SERIOUS ERROR AND GRAVE ABUSE
OF DISCRETION IN FINDING THAT PSE ACTED IN AN
On April 24, 1996, the SEC rendered its Order, reversing the PSE's decision. The ARBITRARY AND ABUSIVE MANNER IN DISAPPROVING
dispositive portion of the said order reads: PALI'S LISTING APPLICATION;

179
MATABABE FILES
Corporation Law

III. THE ASSAILED ORDERS OF SEC ARE ILLEGAL AND VOID 1. PALI has clearly and admittedly complied with the Listing Rules and full
FOR ALLOWING FURTHER DISPOSITION OF PROPERTIES IN disclosure requirements of the Exchange;
CUSTODIA LEGIS AND WHICH FORM PART OF
NAVAL/MILITARY RESERVATION; AND 2. In applying its clear and reasonable standards on the suitability for listing
of shares, PSE has failed to justify why it acted differently on the application
IV. THE FULL DISCLOSURE OF THE SEC WAS NOT of PALI, as compared to the IPOs of other companies similarly situated that
PROPERLY PROMULGATED AND ITS IMPLEMENTATION were allowed listing in the Exchange;
AND APPLICATION IN THIS CASE VIOLATES THE DUE
PROCESS CLAUSE OF THE CONSTITUTION. 3. It appears that the claims and issues on the title to PALI's properties were
even less serious than the claims against the assets of the other companies
On June 4, 1996, PALI filed its Comment to the Petition for Review and subsequently, in that, the assertions of the Marcoses that they are owners of the disputed
a Comment and Motion to Dismiss. On June 10, 1996, PSE fled its Reply to Comment and properties were not substantiated enough to overcome the strength of a
Opposition to Motion to Dismiss. title to properties issued under the Torrens System as evidence of ownership
thereof;
On June 27, 1996, the Court of Appeals promulgated its Resolution dismissing the
PSE's Petition for Review. Hence, this Petition by the PSE. 4. No action has been filed in any court of competent jurisdiction seeking to
nullify PALI's ownership over the disputed properties, neither has the
The appellate court had ruled that the SEC had both jurisdiction and authority to look government instituted recovery proceedings against these properties. Yet
into the decision of the petitioner PSE, pursuant to Section 3 3 of the Revised Securities Act in the import of PSE's decision in denying PALI's application is that it would be
relation to Section 6(j) and 6(m) 4 of P.D. No. 902-A, and Section 38(b) 5 of the Revised PALI, not the Marcoses, that must go to court to prove the legality of its
Securities Act, and for the purpose of ensuring fair administration of the exchange. Both as a ownership on these properties before its shares can be listed.
corporation and as a stock exchange, the petitioner is subject to public respondent's
jurisdiction, regulation and control. Accepting the argument that the public respondent has In addition, the argument that the PALI properties belong to the Military/Naval
the authority merely to supervise or regulate, would amount to serious consequences, Reservation does not inspire belief. The point is, the PALI properties are now titled. A
considering that the petitioner is a stock exchange whose business is impressed with public property losses its public character the moment it is covered by a title. As a matter of fact, the
interest. Abuse is not remote if the public respondent is left without any system of control. If titles have long been settled by a final judgment; and the final decree having been registered,
the securities act vested the public respondent with jurisdiction and control over all they can no longer be re-opened considering that the one year period has already passed.
corporations; the power to authorize the establishment of stock exchanges; the right to Lastly, the determination of what standard to apply in allowing PALI's application for listing,
supervise and regulate the same; and the power to alter and supplement rules of the whether the discretion method or the system of public disclosure adhered to by the SEC,
exchange in the listing or delisting of securities, then the law certainly granted to the public should be addressed to the Securities Commission, it being the government agency that
respondent the plenary authority over the petitioner; and the power of review necessarily exercises both supervisory and regulatory authority over all corporations.
comes within its authority.
On August 15, 19961 the PSE, after it was granted an extension, filed the instant
All in all, the court held that PALI complied with all the requirements for public listing, Petition for Review on Certiorari, taking exception to the rulings of the SEC and the Court of
affirming the SEC's ruling to the effect that: Appeals. Respondent PALI filed its Comment to the petition on October 17, 1996. On the same
date, the PCGG filed a Motion for Leave to file a Petition for Intervention. This was followed
. . . the Philippine Stock Exchange has acted in an arbitrary and abusive up by the PCGG's Petition for Intervention on October 21, 1996. A supplemental Comment was
manner in disapproving the application of PALI for listing of its shares in the filed by PALI on October 25, 1997. The Office of the Solicitor General, representing the SEC
face of the following considerations: and the Court of Appeals, likewise filed its Comment on December 26, 1996. In answer to the
PCGG's motion for leave to file petition for intervention, PALI filed its Comment thereto on
January 17, 1997, whereas the PSE filed its own Comment on January 20, 1997.

180
MATABABE FILES
Corporation Law

On February 25, 1996, the PSE filed its Consolidated Reply to the comments of PSE, likewise, assails the SEC's and the Court of Appeals reliance on the alleged policy
respondent PALI (October 17, 1996) and the Solicitor General (December 26, 1996). On May of "full disclosure" to uphold the listing of PALI's shares with the PSE, in the absence of a clear
16, 1997, PALI filed its Rejoinder to the said consolidated reply of PSE. mandate for the effectivity of such policy. As it is, the case records reveal the truth that PALI
did not comply with the listing rules and disclosure requirements. In fact, PALI's documents
PSE submits that the Court of Appeals erred in ruling that the SEC had authority to supporting its application contained misrepresentations and misleading statements, and
order the PSE to list the shares of PALI in the stock exchange. Under presidential decree No. concealed material information. The matter of sequestration of PALI's properties and the fact
902-A, the powers of the SEC over stock exchanges are more limited as compared to its that the same form part of military/naval/forest reservations were not reflected in PALI's
authority over ordinary corporations. In connection with this, the powers of the SEC over application.
stock exchanges under the Revised Securities Act are specifically enumerated, and these do
not include the power to reverse the decisions of the stock exchange. Authorities are in It is undeniable that the petitioner PSE is not an ordinary corporation, in that
abundance even in the United States, from which the country's security policies are although it is clothed with the markings of a corporate entity, it functions as the primary
patterned, to the effect of giving the Securities Commission less control over stock channel through which the vessels of capital trade ply. The PSE's relevance to the continued
exchanges, which in turn are given more lee-way in making the decision whether or not to operation and filtration of the securities transactions in the country gives it a distinct color of
allow corporations to offer their stock to the public through the stock exchange. This is in importance such that government intervention in its affairs becomes justified, if not
accord with the "business judgment rule" whereby the SEC and the courts are barred from necessarily. Indeed, as the only operational stock exchange in the country today, the PSE
intruding into business judgments of corporations, when the same are made in good faith. the enjoys a monopoly of securities transactions, and as such, it yields an immense influence upon
said rule precludes the reversal of the decision of the PSE to deny PALI's listing application, the country's economy.
absent a showing of bad faith on the part of the PSE. Under the listing rules of the PSE, to
which PALI had previously agreed to comply, the PSE retains the discretion to accept or reject Due to this special nature of stock exchanges, the country's lawmakers has seen it
applications for listing. Thus, even if an issuer has complied with the PSE listing rules and wise to give special treatment to the administration and regulation of stock exchanges. 6
requirements, PSE retains the discretion to accept or reject the issuer's listing application if
the PSE determines that the listing shall not serve the interests of the investing public. These provisions, read together with the general grant of jurisdiction, and right of
supervision and control over all corporations under Sec. 3 of P.D. 902-A, give the SEC the
Moreover, PSE argues that the SEC has no jurisdiction over sequestered special mandate to be vigilant in the supervision of the affairs of stock exchanges so that the
corporations, nor with corporations whose properties are under sequestration. A reading of interests of the investing public may be fully safeguard.
Republic of the Philippines vs. Sadiganbayan, G.R. No. 105205, 240 SCRA 376, would reveal that
the properties of PALI, which were derived from the Ternate Development Corporation (TDC) Section 3 of Presidential Decree 902-A, standing alone, is enough authority to uphold
and the Monte del Sol Development Corporation (MSDC). are under sequestration by the the SEC's challenged control authority over the petitioner PSE even as it provides that "the
PCGG, and subject of forfeiture proceedings in the Sandiganbayan. This ruling of the Court is Commission shall have absolute jurisdiction, supervision, and control over all corporations,
the "law of the case" between the Republic and TDC and MSDC. It categorically declares that partnerships or associations, who are the grantees of primary franchises and/or a license or
the assets of these corporations were sequestered by the PCGG on March 10, 1986 and April permit issued by the government to operate in the Philippines. . ." The SEC's regulatory
4, 1988. authority over private corporations encompasses a wide margin of areas, touching nearly all
of a corporation's concerns. This authority springs from the fact that a corporation owes its
It is, likewise, intimated that the Court of Appeals' sanction that PALI's ownership existence to the concession of its corporate franchise from the state.
over its properties can no longer be questioned, since certificates of title have been issued to The SEC's power to look into the subject ruling of the PSE, therefore, may be implied from or
PALI and more than one year has since lapsed, is erroneous and ignores well settled be considered as necessary or incidental to the carrying out of the SEC's express power to
jurisprudence on land titles. That a certificate of title issued under the Torrens System is a insure fair dealing in securities traded upon a stock exchange or to ensure the fair
conclusive evidence of ownership is not an absolute rule and admits certain exceptions. It is administration of such exchange. 7 It is, likewise, observed that the principal function of the
fundamental that forest lands or military reservations are non-alienable. Thus, when a title SEC is the supervision and control over corporations, partnerships and associations with the
covers a forest reserve or a government reservation, such title is void.

181
MATABABE FILES
Corporation Law

end in view that investment in these entities may be encouraged and protected, and their collective body, it waives no constitutional immunities and perquisites appropriate to such a
activities for the promotion of economic development. 8 body. 11 As to its corporate and management decisions, therefore, the state will generally not
interfere with the same. Questions of policy and of management are left to the honest
Thus, it was in the alleged exercise of this authority that the SEC reversed the decision of the officers and directors of a corporation, and the courts are without authority to
decision of the PSE to deny the application for listing in the stock exchange of the private substitute their judgment for the judgment of the board of directors. The board is the
respondent PALI. The SEC's action was affirmed by the Court of Appeals. business manager of the corporation, and so long as it acts in good faith, its orders are not
reviewable by the courts. 12
We affirm that the SEC is the entity with the primary say as to whether or not
securities, including shares of stock of a corporation, may be traded or not in the stock Thus, notwithstanding the regulatory power of the SEC over the PSE, and the
exchange. This is in line with the SEC's mission to ensure proper compliance with the laws, resultant authority to reverse the PSE's decision in matters of application for listing in the
such as the Revised Securities Act and to regulate the sale and disposition of securities in the market, the SEC may exercise such power only if the PSE's judgment is attended by bad faith.
country. 9 As the appellate court explains: In Board of Liquidators vs. Kalaw, 13 it was held that bad faith does not simply connote bad
judgment or negligence. It imports a dishonest purpose or some moral obliquity and
Paramount policy also supports the authority of the public conscious doing of wrong. It means a breach of a known duty through some motive or
respondent to review petitioner's denial of the listing. Being a stock interest of ill will, partaking of the nature of fraud.
exchange, the petitioner performs a function that is vital to the national
economy, as the business is affected with public interest. As a matter of fact, In reaching its decision to deny the application for listing of PALI, the PSE considered
it has often been said that the economy moves on the basis of the rise and important facts, which, in the general scheme, brings to serious question the qualification of
fall of stocks being traded. By its economic power, the petitioner certainly PALI to sell its shares to the public through the stock exchange. During the time for receiving
can dictate which and how many users are allowed to sell securities thru the objections to the application, the PSE heard from the representative of the late President
facilities of a stock exchange, if allowed to interpret its own rules liberally as Ferdinand E. Marcos and his family who claim the properties of the private respondent to be
it may please. Petitioner can either allow or deny the entry to the market of part of the Marcos estate. In time, the PCGG confirmed this claim. In fact, an order of
securities. To repeat, the monopoly, unless accompanied by control, sequestration has been issued covering the properties of PALI, and suit for reconveyance to
becomes subject to abuse; hence, considering public interest, then it should the state has been filed in the Sandiganbayan Court. How the properties were effectively
be subject to government regulation. transferred, despite the sequestration order, from the TDC and MSDC to Rebecco Panlilio, and
to the private respondent PALI, in only a short span of time, are not yet explained to the
The role of the SEC in our national economy cannot be minimized. The legislature, Court, but it is clear that such circumstances give rise to serious doubt as to the integrity of
through the Revised Securities Act, Presidential Decree No. 902-A, and other pertinent laws, PALI as a stock issuer. The petitioner was in the right when it refused application of PALI, for a
has entrusted to it the serious responsibility of enforcing all laws affecting corporations and contrary ruling was not to the best interest of the general public. The purpose of the Revised
other forms of associations not otherwise vested in some other government office. 10 Securities Act, after all, is to give adequate and effective protection to the investing public
against fraudulent representations, or false promises, and the imposition of worthless
This is not to say, however, that the PSE's management prerogatives are under the ventures. 14
absolute control of the SEC. The PSE is, alter all, a corporation authorized by its corporate
franchise to engage in its proposed and duly approved business. One of the PSE's main It is to be observed that the U.S. Securities Act emphasized its avowed protection to
concerns, as such, is still the generation of profit for its stockholders. Moreover, the PSE has acts detrimental to legitimate business, thus:
all the rights pertaining to corporations, including the right to sue and be sued, to hold
property in its own name, to enter (or not to enter) into contracts with third persons, and to The Securities Act, often referred to as the "truth in securities" Act,
perform all other legal acts within its allocated express or implied powers. was designed not only to provide investors with adequate information upon
which to base their decisions to buy and sell securities, but also to protect
A corporation is but an association of individuals, allowed to transact under an legitimate business seeking to obtain capital through honest presentation
assumed corporate name, and with a distinct legal personality. In organizing itself as a against competition from crooked promoters and to prevent fraud in the

182
MATABABE FILES
Corporation Law

sale of securities. (Tenth Annual Report, U.S. Securities & Exchange alienability exists, and this puts to question the qualification of PALI's public offering. In sum,
Commission, p. 14). the Court finds that the SEC had acted arbitrarily in arrogating unto itself the discretion of
approving the application for listing in the PSE of the private respondent PALI, since this is a
As has been pointed out, the effects of such an act are chiefly (1) matter addressed to the sound discretion of the PSE, a corporation entity, whose business
prevention of excesses and fraudulent transactions, merely by requirement judgments are respected in the absence of bad faith.
of that their details be revealed; (2) placing the market during the early
stages of the offering of a security a body of information, which operating The question as to what policy is, or should be relied upon in approving the
indirectly through investment services and expert investors, will tend to registration and sale of securities in the SEC is not for the Court to determine, but is left to the
produce a more accurate appraisal of a security, . . . Thus, the Commission sound discretion of the Securities and Exchange Commission. In mandating the SEC to
may refuse to permit a registration statement to become effective if it administer the Revised Securities Act, and in performing its other functions under pertinent
appears on its face to be incomplete or inaccurate in any material respect, laws, the Revised Securities Act, under Section 3 thereof, gives the SEC the power to
and empower the Commission to issue a stop order suspending the promulgate such rules and regulations as it may consider appropriate in the public interest for
effectiveness of any registration statement which is found to include any the enforcement of the said laws. The second paragraph of Section 4 of the said law, on the
untrue statement of a material fact or to omit to state any material fact other hand, provides that no security, unless exempt by law, shall be issued, endorsed, sold,
required to be stated therein or necessary to make the statements therein transferred or in any other manner conveyed to the public, unless registered in accordance
not misleading. (Idem). with the rules and regulations that shall be promulgated in the public interest and for the
protection of investors by the Commission. Presidential Decree No. 902-A, on the other hand,
Also, as the primary market for securities, the PSE has established its name and provides that the SEC, as regulatory agency, has supervision and control over all corporations
goodwill, and it has the right to protect such goodwill by maintaining a reasonable standard and over the securities market as a whole, and as such, is given ample authority in
of propriety in the entities who choose to transact through its facilities. It was reasonable for determining appropriate policies. Pursuant to this regulatory authority, the SEC has
the PSE, therefore, to exercise its judgment in the manner it deems appropriate for its manifested that it has adopted the policy of "full material disclosure" where all companies,
business identity, as long as no rights are trampled upon, and public welfare is safeguarded. listed or applying for listing, are required to divulge truthfully and accurately, all material
information about themselves and the securities they sell, for the protection of the investing
In this connection, it is proper to observe that the concept of government absolutism public, and under pain of administrative, criminal and civil sanctions. In connection with this, a
is a thing of the past, and should remain so. fact is deemed material if it tends to induce or otherwise effect the sale or purchase of its
securities. 15 While the employment of this policy is recognized and sanctioned by the laws,
The observation that the title of PALI over its properties is absolute and can no nonetheless, the Revised Securities Act sets substantial and procedural standards which a
longer be assailed is of no moment. At this juncture, there is the claim that the properties proposed issuer of securities must satisfy. 16 Pertinently, Section 9 of the Revised Securities
were owned by TDC and MSDC and were transferred in violation of sequestration orders, to Act sets forth the possible Grounds for the Rejection of the registration of a security:
Rebecco Panlilio and later on to PALI, besides the claim of the Marcoses that such properties
belong to the Marcos estate, and were held only in trust by Rebecco Panlilio. It is also alleged The Commission may reject a registration statement and refuse to issue a
by the petitioner that these properties belong to naval and forest reserves, and therefore permit to sell the securities included in such registration statement if it finds
beyond private dominion. If any of these claims is established to be true, the certificates of that
title over the subject properties now held by PALI map be disregarded, as it is an established
rule that a registration of a certificate of title does not confer ownership over the properties (1) The registration statement is on its face incomplete or inaccurate in any
described therein to the person named as owner. The inscription in the registry, to be material respect or includes any untrue statement of a material fact or omits to
effective, must be made in good faith. The defense of indefeasibility of a Torrens Title does state a material fact required to be stated therein or necessary to make the
not extend to a transferee who takes the certificate of title with notice of a flaw. statements therein not misleading; or
In any case, for the purpose of determining whether PSE acted correctly in refusing
the application of PALI, the true ownership of the properties of PALI need not be determined (2) The issuer or registrant
as an absolute fact. What is material is that the uncertainty of the properties' ownership and

183
MATABABE FILES
Corporation Law

(i) is not solvent or not in sound financial condition; wishing to do so. However, the SEC must recognize and implement the mandate of the law,
particularly the Revised Securities Act, the provisions of which cannot be amended or
(ii) has violated or has not complied with the provisions of supplanted by mere administrative issuance.
this Act, or the rules promulgated pursuant thereto, or any
order of the Commission; In resume, the Court finds that the PSE has acted with justified circumspection,
discounting, therefore, any imputation of arbitrariness and whimsical animation on its part. Its
(iii) has failed to comply with any of the applicable action in refusing to allow the listing of PALI in the stock exchange is justified by the law and
requirements and conditions that the Commission may, in by the circumstances attendant to this case.
the public interest and for the protection of investors, ACCORDINGLY, in view of the foregoing considerations, the Court hereby GRANTS
impose before the security can be registered; the Petition for Review on Certiorari. The Decisions of the Court of Appeals and the Securities
and Exchange Commission dated July 27, 1996 and April 24, 1996 respectively, are hereby
(iv) has been engaged or is engaged or is about to engage REVERSED and SET ASIDE, and a new Judgment is hereby ENTERED, affirming the decision of
in fraudulent transaction; the Philippine Stock Exchange to deny the application for listing of the private respondent
Puerto Azul Land, Inc.
(v) is in any way dishonest or is not of good repute; or
(vi) does not conduct its business in accordance with law or SO ORDERED.
is engaged in a business that is illegal or contrary to
government rules and regulations. Regalado and Puno, JJ., concur.
Mendoza, J., concurs in the result.
(3) The enterprise or the business of the issuer is not shown to be sound or
to be based on sound business principles;

(4) An officer, member of the board of directors, or principal stockholder of


the issuer is disqualified to be such officer, director or principal stockholder;
or

(5) The issuer or registrant has not shown to the satisfaction of the
Commission that the sale of its security would not work to the prejudice of the
public interest or as a fraud upon the purchasers or investors. (Emphasis Ours)
A reading of the foregoing grounds reveals the intention of the lawmakers to make
the registration and issuance of securities dependent, to a certain extent, on the merits of the
securities themselves, and of the issuer, to be determined by the Securities and Exchange
Commission. This measure was meant to protect the interests of the investing public against
fraudulent and worthless securities, and the SEC is mandated by law to safeguard these
interests, following the policies and rules therefore provided. The absolute reliance on the full
disclosure method in the registration of securities is, therefore, untenable. As it is, the Court
finds that the private respondent PALI, on at least two points (nos. 1 and 5) has failed to
support the propriety of the issue of its shares with unfailing clarity, thereby lending support
to the conclusion that the PSE acted correctly in refusing the listing of PALI in its stock
exchange. This does not discount the effectivity of whatever method the SEC, in the exercise
of its vested authority, chooses in setting the standard for public offerings of corporations

184
MATABABE FILES
Corporation Law

Republic of the Philippines out to the public as possessing the power to do those acts; thus, the corporation will, as
SUPREME COURT against anyone who has in good faith dealt with it through such agent, be estopped from
Manila denying the agents authority.
SECOND DIVISION
Remedial Law; Appeals; Findings of fact of the Court of Appeals are final and
G.R. No. 142435 April 30, 2003 conclusive, and cannot be reviewed on appeal by the Supreme Court, provided they are borne
out by the record or based on substantial evidence.As a general rule, findings of fact of the
ESTELITA BURGOS LIPAT and ALFREDO LIPAT, petitioners, Court of Appeals are final and conclusive, and cannot be reviewed on appeal by the Supreme
vs. Court, provided they are borne out by the record or based on substantial evidence.As noted
PACIFIC BANKING CORPORATION, REGISTER OF DEEDS, RTC EX-OFFICIO SHERIFF OF earlier, BEC merely succeeded BET as petitioners alter ego; hence, petitioners mortgaged
QUEZON CITY and the Heirs of EUGENIO D. TRINIDAD, respondents. property must be held liable for the subsequent loans and credit lines of BEC.

Corporation Law; Piercing the Veil of Corporate Identity; When the corporation is the Same; Same; Basic is the rule that matters not raised in the complaint cannot be raised
mere alter ego or business conduit of a person, the separate personality of the corporation for the first time on appeal.On the third and final issue, petitioners assail the decision of the
may be disregarded.A careful reading of the judgment of the RTC and the resolution of the Court of Appeals for not taking cognizance of the issue on attorneys fees on the ground that
appellate court show that in finding petitioners mortgaged property liable for the obligations it was raised for the first time on appeal. We find the conclusion of the Court of Appeals to be
of BEC, both courts below relied upon the alter ego doctrine or instrumentality rule, rather in accord with settled jurisprudence. Basic is the rule that matters not raised in the complaint
than fraud in piercing the veil of corporate fiction. When the corporation is the mere alter ego cannot be raised for the first time on appeal. A close perusal of the complaint yields no
or business conduit of a person, the separate personality of the corporation may be allegations disputing the attorneys fees imposed under the real estate mortgage and
disregarded. This is commonly referred to as the instrumentality rule or the alter ego petitioners cannot now allege that they have impliedly disputed the same when they sought
doctrine, which the courts have applied in disregarding the separate juridical personality of the annulment of the contract.
corporations. As held in one case, Where one corporation is so organized and controlled and
its affairs are conducted so that it is, in fact, a mere instrumentality or adjunct of the other, PETITION for review on certiorari of the decision and resolution of the Court of Appeals.
the fiction of the corporate entity of the instrumentality may be disregarded. The control
necessary to invoke the rule is not majority or even complete stock control but such The facts are stated in the opinion of the Court.
domination of finances, policies and practices that the controlled corporation has, so to QUISUMBING, J.:
speak, no separate mind, will or existence of its own, and is but a conduit for its principal. x x
x This petition for review on certiorari seeks the reversal of the Decision 1 dated
October 21, 1999 of the Court of Appeals in CA-G.R. CV No. 41536 which dismissed herein
Same; Estoppel; It is a familiar doctrine that if a corporation knowingly permits one of petitioners' appeal from the Decision2 dated February 10, 1993 of the Regional Trial Court
its officers or any other agent to act within the scope of an apparent authority, it holds him out (RTC) of Quezon City, Branch 84, in Civil Case No. Q-89-4152. The trial court had dismissed
to the public as possessing the power to do those acts; thus, the corporation will, as against petitioners' complaint for annulment of real estate mortgage and the extra-judicial
anyone who has in good faith dealt with it through such agent, be estopped from denying the foreclosure thereof. Likewise brought for our review is the Resolution 3 dated February 23,
agents authority.In this case, Teresita Lipat had dealt with Pacific Bank on the mortgage 2000 of the Court of Appeals which denied petitioners' motion for reconsideration.
contract by virtue of a special power of attorney executed by Estelita Lipat. Recall that
Teresita Lipat acted as the manager of both BEC and BET and had been deciding business The facts, as culled from records, are as follows:
matters in the absence of Estelita Lipat. Further, the export bills secured by BEC were for the
benefit of Mystical Fashion owned by Estelita Lipat. Hence, Pacific Bank cannot be faulted Petitioners, the spouses Alfredo Lipat and Estelita Burgos Lipat, owned "Bela's
for relying on the same authority granted to Teresita Lipat by Estelita Lipat by virtue of a Export Trading" (BET), a single proprietorship with principal office at No. 814 Aurora
special power of attorney. It is a familiar doctrine that if a corporation knowingly permits one Boulevard, Cubao, Quezon City. BET was engaged in the manufacture of garments for
of its officers or any other agent to act within the scope of an apparent authority, it holds him domestic and foreign consumption. The Lipats also owned the "Mystical Fashions" in the

185
MATABABE FILES
Corporation Law

United States, which sells goods imported from the Philippines through BET. Mrs. Lipat
designated her daughter, Teresita B. Lipat, to manage BET in the Philippines while she was The promissory notes, export bills, and trust receipt eventually became due and
managing "Mystical Fashions" in the United States. demandable. Unfortunately, BEC defaulted in its payments. After receipt of Pacific Bank's
demand letters, Estelita Lipat went to the office of the bank's liquidator and asked for
In order to facilitate the convenient operation of BET, Estelita Lipat executed on additional time to enable her to personally settle BEC's obligations. The bank acceded to her
December 14, 1978, a special power of attorney appointing Teresita Lipat as her attorney-in- request but Estelita failed to fulfill her promise.
fact to obtain loans and other credit accommodations from respondent Pacific Banking
Corporation (Pacific Bank). She likewise authorized Teresita to execute mortgage contracts Consequently, the real estate mortgage was foreclosed and after compliance with
on properties owned or co-owned by her as security for the obligations to be extended by the requirements of the law the mortgaged property was sold at public auction. On January
Pacific Bank including any extension or renewal thereof. 31, 1989, a certificate of sale was issued to respondent Eugenio D. Trinidad as the highest
bidder.
Sometime in April 1979, Teresita, by virtue of the special power of attorney, was able
to secure for and in behalf of her mother, Mrs. Lipat and BET, a loan from Pacific Bank On November 28, 1989, the spouses Lipat filed before the Quezon City RTC a
amounting to P583,854.00 to buy fabrics to be manufactured by BET and exported to complaint for annulment of the real estate mortgage, extrajudicial foreclosure and the
"Mystical Fashions" in the United States. As security therefor, the Lipat spouses, as certificate of sale issued over the property against Pacific Bank and Eugenio D. Trinidad. The
represented by Teresita, executed a Real Estate Mortgage over their property located at No. complaint, which was docketed as Civil Case No. Q-89-4152, alleged, among others, that the
814 Aurora Blvd., Cubao, Quezon City. Said property was likewise made to secure "other promissory notes, trust receipt, and export bills were all ultra vires acts of Teresita as they
additional or new loans, discounting lines, overdrafts and credit accommodations, of were executed without the requisite board resolution of the Board of Directors of BEC. The
whatever amount, which the Mortgagor and/or Debtor may subsequently obtain from the Lipats also averred that assuming said acts were valid and binding on BEC, the same were the
Mortgagee as well as any renewal or extension by the Mortgagor and/or Debtor of the whole corporation's sole obligation, it having a personality distinct and separate from spouses Lipat.
or part of said original, additional or new loans, discounting lines, overdrafts and other credit It was likewise pointed out that Teresita's authority to secure a loan from Pacific Bank was
accommodations, including interest and expenses or other obligations of the Mortgagor specifically limited to Mrs. Lipat's sole use and benefit and that the real estate mortgage was
and/or Debtor owing to the Mortgagee, whether directly, or indirectly, principal or secondary, executed to secure the Lipats' and BET's P583,854.00 loan only.
as appears in the accounts, books and records of the Mortgagee." 4
In their respective answers, Pacific Bank and Trinidad alleged in common that
On September 5, 1979, BET was incorporated into a family corporation named Bela's petitioners Lipat cannot evade payments of the value of the promissory notes, trust receipt,
Export Corporation (BEC) in order to facilitate the management of the business. BEC was and export bills with their property because they and the BEC are one and the same, the latter
engaged in the business of manufacturing and exportation of all kinds of garments of being a family corporation. Respondent Trinidad further claimed that he was a buyer in good
whatever kind and description5 and utilized the same machineries and equipment previously faith and for value and that petitioners are estopped from denying BEC's existence after
used by BET. Its incorporators and directors included the Lipat spouses who owned a holding themselves out as a corporation.
combined 300 shares out of the 420 shares subscribed, Teresita Lipat who owned 20 shares,
and other close relatives and friends of the Lipats.6 Estelita Lipat was named president of BEC, After trial on the merits, the RTC dismissed the complaint, thus:
while Teresita became the vice-president and general manager.
WHEREFORE, this Court holds that in view of the facts contained in the
Eventually, the loan was later restructured in the name of BEC and subsequent loans record, the complaint filed in this case must be, as is hereby, dismissed. Plaintiffs
were obtained by BEC with the corresponding promissory notes duly executed by Teresita on however has five (5) months and seventeen (17) days reckoned from the finality of
behalf of the corporation. A letter of credit was also opened by Pacific Bank in favor of A. O. this decision within which to exercise their right of redemption. The writ of injunction
Knitting Manufacturing Co., Inc., upon the request of BEC after BEC executed the issued is automatically dissolved if no redemption is effected within that period.
corresponding trust receipt therefor. Export bills were also executed in favor of Pacific Bank
for additional finances. These transactions were all secured by the real estate mortgage over The counterclaims and cross-claim are likewise dismissed for lack of legal and
the Lipats' property. factual basis.

186
MATABABE FILES
Corporation Law

No costs. 3) . . . IN HOLDING THAT "THE IMPOSITION OF 15% ATTORNEY'S FEES IN THE EXTRA-
JUDICIAL FORECLOSURE IS BEYOND THIS COURT'S JURISDICTION FOR IT IS BEING
IT IS SO ORDERED.7 RAISED FOR THE FIRST TIME IN THIS APPEAL."

The trial court ruled that there was convincing and conclusive evidence proving that 4) . . . IN HOLDING PETITIONER ALFREDO LIPAT LIABLE TO PAY THE DISPUTED
BEC was a family corporation of the Lipats. As such, it was a mere extension of petitioners' PROMISSORY NOTES, THE DOLLAR ACCOMMODATIONS AND TRUST RECEIPTS
personality and business and a mere alter ego or business conduit of the Lipats established for DESPITE THE EVIDENT FACT THAT THEY WERE NOT SIGNED BY HIM AND THEREFORE
their own benefit. Hence, to allow petitioners to invoke the theory of separate corporate ARE NOT VALID OR ARE NOT BINDING TO HIM.
personality would sanction its use as a shield to further an end subversive of justice. 8 Thus,
the trial court pierced the veil of corporate fiction and held that Bela's Export Corporation and 5) . . . IN DENYING PETITIONERS' MOTION FOR RECONSIDERATION AND IN HOLDING
petitioners (Lipats) are one and the same. Pacific Bank had transacted business with both BET THAT SAID MOTION FOR RECONSIDERATION IS "AN UNAUTHORIZED MOTION, A
and BEC on the supposition that both are one and the same. Hence, the Lipats were estopped MERE SCRAP OF PAPER WHICH CAN NEITHER BIND NOR BE OF ANY CONSEQUENCE
from disclaiming any obligations on the theory of separate personality of corporations, which TO APPELLANTS."11
is contrary to principles of reason and good faith.
In sum, the following are the relevant issues for our resolution:
The Lipats timely appealed the RTC decision to the Court of Appeals in CA-G.R. CV No.
41536. Said appeal, however, was dismissed by the appellate court for lack of merit. The Court 1. Whether or not the doctrine of piercing the veil of corporate fiction is applicable in this case;
of Appeals found that there was ample evidence on record to support the application of the
doctrine of piercing the veil of corporate fiction. In affirming the findings of the RTC, the 2. Whether or not petitioners' property under the real estate mortgage is liable not only for
appellate court noted that Mrs. Lipat had full control over the activities of the corporation the amount of P583,854.00 but also for the value of the promissory notes, trust receipt, and
and used the same to further her business interests.9 In fact, she had benefited from the loans export bills subsequently incurred by BEC; and
obtained by the corporation to finance her business. It also found unnecessary a board
resolution authorizing Teresita Lipat to secure loans from Pacific Bank on behalf of BEC 3. Whether or not petitioners are liable to pay the 15% attorney's fees stipulated in the deed of
because the corporation's by-laws allowed such conduct even without a board resolution. real estate mortgage.
Finally, the Court of Appeals ruled that the mortgage property was not only liable for the
original loan of P583,854.00 but likewise for the value of the promissory notes, trust receipt, On the first issue, petitioners contend that both the appellate and trial courts erred in
and export bills as the mortgage contract equally applies to additional or new loans, holding them liable for the obligations incurred by BEC through the application of the doctrine
discounting lines, overdrafts, and credit accommodations which petitioners subsequently of piercing the veil of corporate fiction absent any clear showing of fraud on their part.
obtained from Pacific Bank.
The Lipats then moved for reconsideration, but this was denied by the appellate Respondents counter that there is clear and convincing evidence to show fraud on
court in its Resolution of February 23, 2000.10 part of petitioners given the findings of the trial court, as affirmed by the Court of Appeals,
Hence, this petition, with petitioners submitting that the court a quo erred that BEC was organized as a business conduit for the benefit of petitioners.

1) . . . IN HOLDING THAT THE DOCTRINE OF PIERCING THE VEIL OF CORPORATE Petitioners' contentions fail to persuade this Court. A careful reading of the judgment
FICTION APPLIES IN THIS CASE. of the RTC and the resolution of the appellate court show that in finding petitioners'
mortgaged property liable for the obligations of BEC, both courts below relied upon the alter
2) . . . IN HOLDING THAT PETITIONERS' PROPERTY CAN BE HELD LIABLE UNDER THE ego doctrine or instrumentality rule, rather than fraud in piercing the veil of corporate fiction.
REAL ESTATE MORTGAGE NOT ONLY FOR THE AMOUNT OF P583,854.00 BUT ALSO When the corporation is the mere alter ego or business conduit of a person, the separate
FOR THE FULL VALUE OF PROMISSORY NOTES, TRUST RECEIPTS AND EXPORT BILLS personality of the corporation may be disregarded. 12 This is commonly referred to as the
OF BELA'S EXPORT CORPORATION. "instrumentality rule" or the alter ego doctrine, which the courts have applied in disregarding
the separate juridical personality of corporations. As held in one case,

187
MATABABE FILES
Corporation Law

not covered by the mortgage contract of BET which only covered the loan of P583,854.00 and
Where one corporation is so organized and controlled and its affairs are which allegedly had already been paid; and, second, it was secured by Teresita Lipat without
conducted so that it is, in fact, a mere instrumentality or adjunct of the other, the any authorization or board resolution of BEC.
fiction of the corporate entity of the 'instrumentality' may be disregarded. The
control necessary to invoke the rule is not majority or even complete stock control We find petitioners' contention untenable. As found by the Court of Appeals, the
but such domination of finances, policies and practices that the controlled mortgaged property is not limited to answer for the loan of P583,854.00. Thus:
corporation has, so to speak, no separate mind, will or existence of its own, and is
but a conduit for its principal. x x x .13 Finally, the extent to which the Lipats' property can be held liable under the real
estate mortgage is not limited to P583,854.00. It can be held liable for the value of
We find that the evidence on record demolishes, rather than buttresses, petitioners' the promissory notes, trust receipt and export bills as well. For the mortgage was
contention that BET and BEC are separate business entities. Note that Estelita Lipat admitted executed not only for the purpose of securing the Bela's Export Trading's original
that she and her husband, Alfredo, were the owners of BET 14 and were two of the loan of P583,854.00, but also for "other additional or new loans, discounting lines,
incorporators and majority stockholders of BEC. 15 It is also undisputed that Estelita Lipat overdrafts and credit accommodations, of whatever amount, which the Mortgagor
executed a special power of attorney in favor of her daughter, Teresita, to obtain loans and and/or Debtor may subsequently obtain from the mortgagee as well as any renewal
credit lines from Pacific Bank on her behalf.16 Incidentally, Teresita was designated as or extension by the Mortgagor and/or Debtor of the whole or part of said original,
executive-vice president and general manager of both BET and BEC, respectively. 17 We note additional or new loans, discounting lines, overdrafts and other credit
further that: (1) Estelita and Alfredo Lipat are the owners and majority shareholders of BET accommodations, including interest and expenses or other obligations of the
and BEC, respectively;18 (2) both firms were managed by their daughter, Teresita; 19 (3) both Mortgagor and/or Debtor owing to the Mortgagee, whether directly, or indirectly
firms were engaged in the garment business, supplying products to "Mystical Fashion," a U.S. principal or secondary, as appears in the accounts, books and records of the
firm established by Estelita Lipat; (4) both firms held office in the same building owned by the mortgagee.25
Lipats;20 (5) BEC is a family corporation with the Lipats as its majority stockholders; (6) the
business operations of the BEC were so merged with those of Mrs. Lipat such that they were As a general rule, findings of fact of the Court of Appeals are final and conclusive, and
practically indistinguishable; (7) the corporate funds were held by Estelita Lipat and the cannot be reviewed on appeal by the Supreme Court, provided they are borne out by the
corporation itself had no visible assets; (8) the board of directors of BEC was composed of the record or based on substantial evidence.26 As noted earlier, BEC merely succeeded BET as
Burgos and Lipat family members;21 (9) Estelita had full control over the activities of and petitioners' alter ego; hence, petitioners' mortgaged property must be held liable for the
decided business matters of the corporation;22 and that (10) Estelita Lipat had benefited from subsequent loans and credit lines of BEC.
the loans secured from Pacific Bank to finance her business abroad 23 and from the export bills
secured by BEC for the account of "Mystical Fashion."24 It could not have been coincidental Further, petitioners' contention that the original loan had already been paid, hence,
that BET and BEC are so intertwined with each other in terms of ownership, business purpose, the mortgaged property should not be made liable to the loans of BEC, is unsupported by any
and management. Apparently, BET and BEC are one and the same and the latter is a conduit substantial evidence other than Estelita Lipat's self-serving testimony. Two disputable
of and merely succeeded the former. Petitioners' attempt to isolate themselves from and hide presumptions under the rules on evidence weigh against petitioners, namely: (a) that a
behind the corporate personality of BEC so as to evade their liabilities to Pacific Bank is person takes ordinary care of his concerns;27 and (b) that things have happened according to
precisely what the classical doctrine of piercing the veil of corporate entity seeks to prevent the ordinary course of nature and the ordinary habits of life. 28 Here, if the original loan had
and remedy. In our view, BEC is a mere continuation and successor of BET, and petitioners indeed been paid, then logically, petitioners would have asked from Pacific Bank for the
cannot evade their obligations in the mortgage contract secured under the name of BEC on required documents evidencing receipt and payment of the loans and, as owners of the
the pretext that it was signed for the benefit and under the name of BET. We are thus mortgaged property, would have immediately asked for the cancellation of the mortgage in
constrained to rule that the Court of Appeals did not err when it applied the instrumentality the ordinary course of things. However, the records are bereft of any evidence contradicting
doctrine in piercing the corporate veil of BEC. or overcoming said disputable presumptions.

On the second issue, petitioners contend that their mortgaged property should not Petitioners contend further that the mortgaged property should not bind the loans
be made liable for the subsequent credit lines and loans incurred by BEC because, first, it was and credit lines obtained by BEC as they were secured without any proper authorization or

188
MATABABE FILES
Corporation Law

board resolution. They also blame the bank for its laxity and complacency in not requiring a We find no necessity to extensively deal with the liability of Alfredo Lipat for the
board resolution as a requisite for approving the loans. subsequent credit lines of BEC. Suffice it to state that Alfredo Lipat never disputed the validity
of the real estate mortgage of the original loan; hence, he cannot now dispute the
Such contentions deserve scant consideration. subsequent loans obtained using the same mortgage contract since it is, by its very terms, a
continuing mortgage contract.
Firstly, it could not have been possible for BEC to release a board resolution since per
admissions by both petitioner Estelita Lipat and Alice Burgos, petitioners' rebuttal witness, no On the third and final issue, petitioners assail the decision of the Court of Appeals for
business or stockholder's meetings were conducted nor were there election of officers held not taking cognizance of the issue on attorney's fees on the ground that it was raised for the
since its incorporation. In fact, not a single board resolution was passed by the corporate first time on appeal. We find the conclusion of the Court of Appeals to be in accord with
board29 and it was Estelita Lipat and/or Teresita Lipat who decided business matters. 30 settled jurisprudence. Basic is the rule that matters not raised in the complaint cannot be
raised for the first time on appeal.35 A close perusal of the complaint yields no allegations
Secondly, the principle of estoppel precludes petitioners from denying the validity of disputing the attorney's fees imposed under the real estate mortgage and petitioners cannot
the transactions entered into by Teresita Lipat with Pacific Bank, who in good faith, relied on now allege that they have impliedly disputed the same when they sought the annulment of
the authority of the former as manager to act on behalf of petitioner Estelita Lipat and both the contract.
BET and BEC. While the power and responsibility to decide whether the corporation should
enter into a contract that will bind the corporation is lodged in its board of directors, subject In sum, we find no reversible error of law committed by the Court of Appeals in
to the articles of incorporation, by-laws, or relevant provisions of law, yet, just as a natural rendering the decision and resolution herein assailed by petitioners.
person may authorize another to do certain acts for and on his behalf, the board of directors WHEREFORE, the petition is DENIED. The Decision dated October 21, 1999 and the Resolution
may validly delegate some of its functions and powers to officers, committees, or agents. The dated February 23, 2000 of the Court of Appeals in CA-G.R. CV No. 41536 are AFFIRMED. Costs
authority of such individuals to bind the corporation is generally derived from law, corporate against petitioners.
by-laws, or authorization from the board, either expressly or impliedly by habit, custom, or SO ORDERED.
acquiescence in the general course of business.31 Apparent authority, is derived not merely
from practice. Its existence may be ascertained through (1) the general manner in which the Bellosillo, Austria-Martinez and Callejo, Sr., JJ ., concur.
corporation holds out an officer or agent as having the power to act or, in other words, the
apparent authority to act in general, with which it clothes him; or (2) the acquiescence in his
acts of a particular nature, with actual or constructive knowledge thereof, whether within or
beyond the scope of his ordinary powers. 32

In this case, Teresita Lipat had dealt with Pacific Bank on the mortgage contract by
virtue of a special power of attorney executed by Estelita Lipat. Recall that Teresita Lipat
acted as the manager of both BEC and BET and had been deciding business matters in the
absence of Estelita Lipat. Further, the export bills secured by BEC were for the benefit of
"Mystical Fashion" owned by Estelita Lipat.33 Hence, Pacific Bank cannot be faulted for relying
on the same authority granted to Teresita Lipat by Estelita Lipat by virtue of a special power
of attorney. It is a familiar doctrine that if a corporation knowingly permits one of its officers
or any other agent to act within the scope of an apparent authority, it holds him out to the
public as possessing the power to do those acts; thus, the corporation will, as against anyone
who has in good faith dealt with it through such agent, be estopped from denying the agent's
authority.34

189
MATABABE FILES
Corporation Law

Republic of the Philippines


SUPREME COURT Same; Same; Same; Same; Power of Attorney; Powers of attorney are generally
Manila construed strictly and courts will not infer or presume broad powers from deeds which do not
SECOND DIVISION sufficiently include property or subject under which the agent is to deal.Powers of attorney
are generally construed strictly and courts will not infer or presume broad powers from deeds
G.R. No. 140667 August 12, 2004 which do not sufficiently include property or subject under which the agent is to deal.The
general rule is that the power of attorney must be pursued within legal strictures, and the
WOODCHILD HOLDINGS, INC., petitioner, agent can neither go beyond it; nor beside it. The act done must be legally identical with that
vs. authorized to be done.
ROXAS ELECTRIC AND CONSTRUCTION COMPANY, INC., respondent.
Same; Same; Same; Same; The apparent power of an agent is to be determined by the
Corporations; Corporate Officers; Apparent Authority; Agency; The property of the acts of the principal and not by the acts of the agent.It bears stressing that apparent
corporation is not the property of its stockholders or members and may not be sold by the authority is based on estoppel and can arise from two instances: first, the principal may
stockholders or members without express authorization from the corporations board of knowingly permit the agent to so hold himself out as having such authority, and in this way,
directors.A corporation is a juridical person separate and distinct from its stockholders or the principal becomes estopped to claim that the agent does not have such authority; second,
members. Accordingly, the property of the corporation is not the property of its stockholders the principal may so clothe the agent with the indicia of authority as to lead a reasonably
or members and may not be sold by the stockholders or members without express prudent person to believe that he actually has such authority. There can be no apparent
authorization from the corporations board of direc-tors. Section 23 of BP 68, otherwise authority of an agent without acts or conduct on the part of the principal and such acts or
known as the Corporation Code of the Philippines, provides: SEC. 23. The Board of Directors conduct of the principal must have been known and relied upon in good faith and as a result
or Trustees.Unless otherwise provided in this Code, the corporate powers of all of the exercise of reasonable prudence by a third person as claimant and such must have
corporations formed under this Code shall be exercised, all business conducted and all produced a change of position to its detriment. The apparent power of an agent is to be
property of such corporations controlled and held by the board of directors or trustees to be determined by the acts of the principal and not by the acts of the agent.
elected from among the holders of stocks, or where there is no stock, from among the
members of the corporation, who shall hold office for one (1) year and until their successors Same; Same; Same; Elements; For the principle of apparent authority to apply, the
are elected and qualified. Indubitably, a corporation may act only through its board of petitioner was burdened to prove the following.For the principle of apparent authority to
directors or, when authorized either by its by-laws or by its board resolution, through its apply, the petitioner was burdened to prove the following: (a) the acts of the respondent
officers or agents in the normal course of business. The general principles of agency govern justifying belief in the agency by the petitioner; (b) knowledge thereof by the respondent
the relation between the corporation and its officers or agents, subject to the articles of which is sought to be held; and, (c) reliance thereon by the petitioner consistent with ordinary
incorporation, by-laws, or relevant provisions of law. . . . care and prudence.
Same; Same; Same; Implied Ratification; Ratification cannot be inferred from acts that a
Same; Same; Same; Estoppel; Acts done by corporate officers beyond the scope of principal has a right to do independently of the unauthorized act of the agent.For an act of
their authority cannot bind the corporation unless it has ratified such acts expressly or tacitly, the principal to be considered as an implied ratification of an unauthorized act of an agent,
or is estopped from denying them. Generally, the acts of the corporate officers within the such act must be inconsistent with any other hypothesis than that he approved and intended
scope of their authority are binding on the corporation. However, under Article 1910 of the to adopt what had been done in his name. Ratification is based on waiverthe intentional
New Civil Code, acts done by such officers beyond the scope of their authority cannot bind relinquishment of a known right. Ratification cannot be inferred from acts that a principal has
the corporation unless it has ratified such acts expressly or tacitly, or is estopped from a right to do independently of the unauthorized act of the agent. Moreover, if a writing is
denying them: Art. 1910. The principal must comply with all the obligations which the agent required to grant an authority to do a particular act, ratification of that act must also be in
may have contracted within the scope of his authority. As for any obligation wherein the writing.
agent has exceeded his power, the principal is not bound except when he ratifies it expressly
or tacitly. Thus, contracts entered into by corporate officers beyond the scope of authority PETITION for review on certiorari of a decision of the Court of Appeals.
are unenforceable against the corporation unless ratified by the corporation.

190
MATABABE FILES
Corporation Law

The facts are stated in the opinion of the Court. square meter from his current adjacent property to allow the buyer to full access and
full use of the property.5
DECISION
CALLEJO, SR., J.: Roxas indicated his acceptance of the offer on page 2 of the deed. Less than a month
later or on July 1, 1991, Roxas, as President of RECCI, as vendor, and Dy, as President of WHI,
This is a petition for review on certiorari of the Decision 1 of the Court of Appeals in as vendee, executed a contract to sell in which RECCI bound and obliged itself to sell to Dy Lot
CA-G.R. CV No. 56125 reversing the Decision2 of the Regional Trial Court of Makati, Branch 57, No. 491-A-3-B-2 covered by TCT No. 78086 for P7,213,000.6 On September 5, 1991, a Deed of
which ruled in favor of the petitioner. Absolute Sale7 in favor of WHI was issued, under which Lot No. 491-A-3-B-2 covered by TCT No.
78086 was sold for P5,000,000, receipt of which was acknowledged by Roxas under the
The Antecedents following terms and conditions:

The respondent Roxas Electric and Construction Company, Inc. (RECCI), formerly the The Vendor agree (sic), as it hereby agrees and binds itself to give Vendee
Roxas Electric and Construction Company, was the owner of two parcels of land, identified as the beneficial use of and a right of way from Sumulong Highway to the property
Lot No. 491-A-3-B-1 covered by Transfer Certificate of Title (TCT) No. 78085 and Lot No. 491-A- herein conveyed consists of 25 square meters wide to be used as the latter's egress
3-B-2 covered by TCT No. 78086. A portion of Lot No. 491-A-3-B-1 which abutted Lot No. 491-A- from and ingress to and an additional 25 square meters in the corner of Lot No. 491-A-
3-B-2 was a dirt road accessing to the Sumulong Highway, Antipolo, Rizal. 3-B-1, as turning and/or maneuvering area for Vendee's vehicles.

At a special meeting on May 17, 1991, the respondent's Board of Directors approved a The Vendor agrees that in the event that the right of way is insufficient for
resolution authorizing the corporation, through its president, Roberto B. Roxas, to sell Lot the Vendee's use (ex entry of a 45-foot container) the Vendor agrees to sell
No. 491-A-3-B-2 covered by TCT No. 78086, with an area of 7,213 square meters, at a price and additional square meters from its current adjacent property to allow the Vendee full
under such terms and conditions which he deemed most reasonable and advantageous to the access and full use of the property.
corporation; and to execute, sign and deliver the pertinent sales documents and receive the
proceeds of the sale for and on behalf of the company.3
The Vendor hereby undertakes and agrees, at its account, to defend the title
Petitioner Woodchild Holdings, Inc. (WHI) wanted to buy Lot No. 491-A-3-B-2 covered of the Vendee to the parcel of land and improvements herein conveyed, against all
by TCT No. 78086 on which it planned to construct its warehouse building, and a portion of claims of any and all persons or entities, and that the Vendor hereby warrants the
the adjoining lot, Lot No. 491-A-3-B-1, so that its 45-foot container van would be able to readily right of the Vendee to possess and own the said parcel of land and improvements
enter or leave the property. In a Letter to Roxas dated June 21, 1991, WHI President Jonathan thereon and will defend the Vendee against all present and future claims and/or
Y. Dy offered to buy Lot No. 491-A-3-B-2 under stated terms and conditions for P1,000 per action in relation thereto, judicial and/or administrative. In particular, the Vendor shall
square meter or at the price of P7,213,000.4 One of the terms incorporated in Dy's offer was eject all existing squatters and occupants of the premises within two (2) weeks from
the following provision: the signing hereof. In case of failure on the part of the Vendor to eject all occupants
and squatters within the two-week period or breach of any of the stipulations,
5. This Offer to Purchase is made on the representation and warranty of the covenants and terms and conditions herein provided and that of contract to sell
OWNER/SELLER, that he holds a good and registrable title to the property, which dated 1 July 1991, the Vendee shall have the right to cancel the sale and demand
shall be conveyed CLEAR and FREE of all liens and encumbrances, and that the area reimbursement for all payments made to the Vendor with interest thereon at 36% per
of 7,213 square meters of the subject property already includes the area on which the annum.8
right of way traverses from the main lot (area) towards the exit to the Sumulong
Highway as shown in the location plan furnished by the Owner/Seller to the buyer. On September 10, 1991, the Wimbeco Builder's, Inc. (WBI) submitted its quotation for
Furthermore, in the event that the right of way is insufficient for the buyer's P8,649,000 to WHI for the construction of the warehouse building on a portion of the
purposes (example: entry of a 45-foot container), the seller agrees to sell additional property with an area of 5,088 square meters.9 WBI proposed to start the project on October
1, 1991 and to turn over the building to WHI on February 29, 1992.10

191
MATABABE FILES
Corporation Law

6. Defendant Roxas Electric in patent violation of the express and valid terms of the
In a Letter dated September 16, 1991, Ponderosa Leather Goods Company, Inc. Deed of Absolute Sale unjustifiably refused to deliver to Woodchild Holdings the
confirmed its lease agreement with WHI of a 5,000-square-meter portion of the warehouse stipulated beneficial use and right of way consisting of 25 square meters and 55
yet to be constructed at the rental rate of P65 per square meter. Ponderosa emphasized the square meters to the prejudice of the plaintiff.
need for the warehouse to be ready for occupancy before April 1, 1992. 11 WHI accepted the
offer. However, WBI failed to commence the construction of the warehouse in October 1, 1991 7. Similarly, in as much as the 25 square meters and 55 square meters alloted to
as planned because of the presence of squatters in the property and suggested a Woodchild Holdings for its beneficial use is inadequate as turning and/or
renegotiation of the contract after the squatters shall have been evicted.12 Subsequently, the maneuvering area of its 45-foot container van, Woodchild Holdings manifested its
squatters were evicted from the property. intention pursuant to para. 5 of the Deed of Sale to purchase additional square
meters from Roxas Electric to allow it full access and use of the purchased property,
On March 31, 1992, WHI and WBI executed a Letter-Contract for the construction of however, Roxas Electric refused and failed to merit Woodchild Holdings' request
the warehouse building for P11,804,160.13 The contractor started construction in April 1992 contrary to defendant Roxas Electric's obligation under the Deed of Absolute Sale
even before the building officials of Antipolo City issued a building permit on May 28, 1992. (Annex "A").
After the warehouse was finished, WHI issued on March 21, 1993 a certificate of occupancy by
the building official. Earlier, or on March 18, 1993, WHI, as lessor, and Ponderosa, as lessee, 8. Moreover, defendant, likewise, failed to eject all existing squatters and occupants
executed a contract of lease over a portion of the property for a monthly rental of P300,000 of the premises within the stipulated time frame and as a consequence thereof,
for a period of three years from March 1, 1993 up to February 28, 1996. 14 plaintiff's planned construction has been considerably delayed for seven (7) months
due to the squatters who continue to trespass and obstruct the subject property,
In the meantime, WHI complained to Roberto Roxas that the vehicles of RECCI were thereby Woodchild Holdings incurred substantial losses amounting to P3,560,000.00
parked on a portion of the property over which WHI had been granted a right of way. Roxas occasioned by the increased cost of construction materials and labor.
promised to look into the matter. Dy and Roxas discussed the need of the WHI to buy a 500-
square-meter portion of Lot No. 491-A-3-B-1 covered by TCT No. 78085 as provided for in the 9. Owing further to Roxas Electric's deliberate refusal to comply with its obligation
deed of absolute sale. However, Roxas died soon thereafter. On April 15, 1992, the WHI wrote under Annex "A," Woodchild Holdings suffered unrealized income of P300,000.00 a
the RECCI, reiterating its verbal requests to purchase a portion of the said lot as provided for month or P2,100,000.00 supposed income from rentals of the subject property for
in the deed of absolute sale, and complained about the latter's failure to eject the squatters seven (7) months.
within the three-month period agreed upon in the said deed.
10. On April 15, 1992, Woodchild Holdings made a final demand to Roxas Electric to
The WHI demanded that the RECCI sell a portion of Lot No. 491-A-3-B-1 covered by comply with its obligations and warranties under the Deed of Absolute Sale but
TCT No. 78085 for its beneficial use within 72 hours from notice thereof, otherwise the notwithstanding such demand, defendant Roxas Electric refused and failed and
appropriate action would be filed against it. RECCI rejected the demand of WHI. WHI continue to refuse and fail to heed plaintiff's demand for compliance.
reiterated its demand in a Letter dated May 29, 1992. There was no response from RECCI.
Copy of the demand letter dated April 15, 1992 is hereto attached as Annex
On June 17, 1992, the WHI filed a complaint against the RECCI with the Regional Trial "B" and made an integral part hereof.
Court of Makati, for specific performance and damages, and alleged, inter alia, the following
in its complaint: 11. Finally, on 29 May 1991, Woodchild Holdings made a letter request addressed to
Roxas Electric to particularly annotate on Transfer Certificate of Title No. N-78085 the
5. The "current adjacent property" referred to in the aforequoted paragraph of the agreement under Annex "A" with respect to the beneficial use and right of way,
Deed of Absolute Sale pertains to the property covered by Transfer Certificate of however, Roxas Electric unjustifiably ignored and disregarded the same.
Title No. N-78085 of the Registry of Deeds of Antipolo, Rizal, registered in the name
of herein defendant Roxas Electric. Copy of the letter request dated 29 May 1992 is hereto attached as Annex "C" and
made an integral part hereof.

192
MATABABE FILES
Corporation Law

In its amended answer to the complaint, the RECCI alleged that the delay in the
12. By reason of Roxas Electric's continuous refusal and failure to comply with construction of its warehouse building was due to the failure of the WHI's contractor to
Woodchild Holdings' valid demand for compliance under Annex "A," the latter was secure a building permit thereon.18
constrained to litigate, thereby incurring damages as and by way of attorney's fees in
the amount of P100,000.00 plus costs of suit and expenses of litigation.15 During the trial, Dy testified that he told Roxas that the petitioner was buying a
portion of Lot No. 491-A-3-B-1 consisting of an area of 500 square meters, for the price of
The WHI prayed that, after due proceedings, judgment be rendered in its favor, thus: P1,000 per square meter.

WHEREFORE, it is respectfully prayed that judgment be rendered in favor of On November 11, 1996, the trial court rendered judgment in favor of the WHI, the
Woodchild Holdings and ordering Roxas Electric the following: decretal portion of which reads:

a) to deliver to Woodchild Holdings the beneficial use of the stipulated 25 square WHEREFORE, judgment is hereby rendered directing defendant:
meters and 55 square meters;
(1) To allow plaintiff the beneficial use of the existing right of way plus the stipulated
b) to sell to Woodchild Holdings additional 25 and 100 square meters to allow it full 25 sq. m. and 55 sq. m.;
access and use of the purchased property pursuant to para. 5 of the Deed of (2) To sell to plaintiff an additional area of 500 sq. m. priced at P1,000 per sq. m. to
Absolute Sale; allow said plaintiff full access and use of the purchased property pursuant to Par. 5 of
their Deed of Absolute Sale;
c) to cause annotation on Transfer Certificate of Title No. N-78085 the beneficial use
and right of way granted to Woodchild Holdings under the Deed of Absolute Sale; (3) To cause annotation on TCT No. N-78085 the beneficial use and right of way
granted by their Deed of Absolute Sale;
d) to pay Woodchild Holdings the amount of P5,660,000.00, representing actual
damages and unrealized income; (4) To pay plaintiff the amount of P5,568,000 representing actual damages and
plaintiff's unrealized income;
e) to pay attorney's fees in the amount of P100,000.00; and
(5) To pay plaintiff P100,000 representing attorney's fees; and
f) to pay the costs of suit. To pay the costs of suit.

Other reliefs just and equitable are prayed for.16 SO ORDERED.19

In its answer to the complaint, the RECCI alleged that it never authorized its former The trial court ruled that the RECCI was estopped from disowning the apparent
president, Roberto Roxas, to grant the beneficial use of any portion of Lot No. 491-A-3-B-1, nor authority of Roxas under the May 17, 1991 Resolution of its Board of Directors. The court
agreed to sell any portion thereof or create a lien or burden thereon. It alleged that, under the reasoned that to do so would prejudice the WHI which transacted with Roxas in good faith,
Resolution approved on May 17, 1991, it merely authorized Roxas to sell Lot No. 491-A-3-B-2 believing that he had the authority to bind the WHI relating to the easement of right of way,
covered by TCT No. 78086. As such, the grant of a right of way and the agreement to sell a as well as the right to purchase a portion of Lot No. 491-A-3-B-1 covered by TCT No. 78085.
portion of Lot No. 491-A-3-B-1 covered by TCT No. 78085 in the said deed are ultra vires. The
RECCI further alleged that the provision therein that it would sell a portion of Lot No. 491-A-3- The RECCI appealed the decision to the CA, which rendered a decision on November
B-1 to the WHI lacked the essential elements of a binding contract.17 9, 1999 reversing that of the trial court, and ordering the dismissal of the complaint. The CA
ruled that, under the resolution of the Board of Directors of the RECCI, Roxas was merely
authorized to sell Lot No. 491-A-3-B-2 covered by TCT No. 78086, but not to grant right of way
in favor of the WHI over a portion of Lot No. 491-A-3-B-1, or to grant an option to the

193
MATABABE FILES
Corporation Law

petitioner to buy a portion thereof. The appellate court also ruled that the grant of a right of The threshold issues for resolution are the following: (a) whether the respondent is
way and an option to the respondent were so lopsided in favor of the respondent because bound by the provisions in the deed of absolute sale granting to the petitioner beneficial use
the latter was authorized to fix the location as well as the price of the portion of its property and a right of way over a portion of Lot No. 491-A-3-B-1 accessing to the Sumulong Highway
to be sold to the respondent. Hence, such provisions contained in the deed of absolute sale and granting the option to the petitioner to buy a portion thereof, and, if so, whether such
were not binding on the RECCI. The appellate court ruled that the delay in the construction of agreement is enforceable against the respondent; (b) whether the respondent failed to eject
WHI's warehouse was due to its fault. the squatters on its property within two weeks from the execution of the deed of absolute
sale; and, (c) whether the respondent is liable to the petitioner for damages.
The Present Petition
The petitioner now comes to this Court asserting that: On the first issue, the petitioner avers that, under its Resolution of May 17, 1991, the
respondent authorized Roxas, then its president, to grant a right of way over a portion of Lot
I. No. 491-A-3-B-1 in favor of the petitioner, and an option for the respondent to buy a portion of
THE COURT OF APPEALS ERRED IN HOLDING THAT THE DEED OF ABSOLUTE SALE the said property. The petitioner contends that when the respondent sold Lot No. 491-A-3-B-2
(EXH. "C") IS ULTRA VIRES. covered by TCT No. 78086, it (respondent) was well aware of its obligation to provide the
II. petitioner with a means of ingress to or egress from the property to the Sumulong Highway,
THE COURT OF APPEALS GRAVELY ERRED IN REVERSING THE RULING OF THE COURT since the latter had no adequate outlet to the public highway. The petitioner asserts that it
A QUO ALLOWING THE PLAINTIFF-APPELLEE THE BENEFICIAL USE OF THE EXISTING agreed to buy the property covered by TCT No. 78085 because of the grant by the respondent
RIGHT OF WAY PLUS THE STIPULATED 25 SQUARE METERS AND 55 SQUARE METERS of a right of way and an option in its favor to buy a portion of the property covered by TCT No.
BECAUSE THESE ARE VALID STIPULATIONS AGREED BY BOTH PARTIES TO THE DEED 78085. It contends that the respondent never objected to Roxas' acceptance of its offer to
OF ABSOLUTE SALE (EXH. "C"). purchase the property and the terms and conditions therein; the respondent even allowed
Roxas to execute the deed of absolute sale in its behalf. The petitioner asserts that the
III. respondent even received the purchase price of the property without any objection to the
THERE IS NO FACTUAL PROOF OR EVIDENCE FOR THE COURT OF APPEALS TO RULE terms and conditions of the said deed of sale. The petitioner claims that it acted in good faith,
THAT THE STIPULATIONS OF THE DEED OF ABSOLUTE SALE (EXH. "C") WERE and contends that after having been benefited by the said sale, the respondent is estopped
DISADVANTAGEOUS TO THE APPELLEE, NOR WAS APPELLEE DEPRIVED OF ITS from assailing its terms and conditions. The petitioner notes that the respondent's Board of
PROPERTY WITHOUT DUE PROCESS. Directors never approved any resolution rejecting the deed of absolute sale executed by
Roxas for and in its behalf. As such, the respondent is obliged to sell a portion of Lot No. 491-
IV. A-3-B-1 covered by TCT No. 78085 with an area of 500 square meters at the price of P1,000 per
IN FACT, IT WAS WOODCHILD WHO WAS DEPRIVED OF PROPERTY WITHOUT DUE square meter, based on its evidence and Articles 649 and 651 of the New Civil Code.
PROCESS BY THE ASSAILED DECISION.
For its part, the respondent posits that Roxas was not so authorized under the May
V. 17, 1991 Resolution of its Board of Directors to impose a burden or to grant a right of way in
THE DELAY IN THE CONSTRUCTION WAS DUE TO THE FAILURE OF THE APPELLANT favor of the petitioner on Lot No. 491-A-3-B-1, much less convey a portion thereof to the
TO EVICT THE SQUATTERS ON THE LAND AS AGREED IN THE DEED OF ABSOLUTE petitioner. Hence, the respondent was not bound by such provisions contained in the deed of
SALE (EXH. "C"). absolute sale. Besides, the respondent contends, the petitioner cannot enforce its right to
buy a portion of the said property since there was no agreement in the deed of absolute sale
VI. on the price thereof as well as the specific portion and area to be purchased by the petitioner.
THE COURT OF APPEALS GRAVELY ERRED IN REVERSING THE RULING OF THE COURT
A QUO DIRECTING THE DEFENDANT TO PAY THE PLAINTIFF THE AMOUNT OF We agree with the respondent.
P5,568,000.00 REPRESENTING ACTUAL DAMAGES AND PLAINTIFF'S UNREALIZED
INCOME AS WELL AS ATTORNEY'S FEES.20 In San Juan Structural and Steel Fabricators, Inc. v. Court of Appeals,21 we held that:

194
MATABABE FILES
Corporation Law

A corporation is a juridical person separate and distinct from its stockholders In this case, the respondent denied authorizing its then president Roberto B. Roxas
or members. Accordingly, the property of the corporation is not the property of its to sell a portion of Lot No. 491-A-3-B-1 covered by TCT No. 78085, and to create a lien or
stockholders or members and may not be sold by the stockholders or members burden thereon. The petitioner was thus burdened to prove that the respondent so
without express authorization from the corporation's board of directors. Section 23 authorized Roxas to sell the same and to create a lien thereon.
of BP 68, otherwise known as the Corporation Code of the Philippines, provides:
Central to the issue at hand is the May 17, 1991 Resolution of the Board of Directors of
"SEC. 23. The Board of Directors or Trustees. Unless otherwise provided in the respondent, which is worded as follows:
this Code, the corporate powers of all corporations formed under this Code
shall be exercised, all business conducted and all property of such RESOLVED, as it is hereby resolved, that the corporation, thru the President, sell to
corporations controlled and held by the board of directors or trustees to be any interested buyer, its 7,213-sq.-meter property at the Sumulong Highway,
elected from among the holders of stocks, or where there is no stock, from Antipolo, Rizal, covered by Transfer Certificate of Title No. N-78086, at a price and on
among the members of the corporation, who shall hold office for one (1) terms and conditions which he deems most reasonable and advantageous to the
year and until their successors are elected and qualified." corporation;

Indubitably, a corporation may act only through its board of directors or, FURTHER RESOLVED, that Mr. ROBERTO B. ROXAS, President of the corporation, be,
when authorized either by its by-laws or by its board resolution, through its officers as he is hereby authorized to execute, sign and deliver the pertinent sales documents
or agents in the normal course of business. The general principles of agency govern and receive the proceeds of sale for and on behalf of the company. 25
the relation between the corporation and its officers or agents, subject to the articles
of incorporation, by-laws, or relevant provisions of law. 22 Evidently, Roxas was not specifically authorized under the said resolution to grant a
right of way in favor of the petitioner on a portion of Lot No. 491-A-3-B-1 or to agree to sell to
Generally, the acts of the corporate officers within the scope of their authority are the petitioner a portion thereof. The authority of Roxas, under the resolution, to sell Lot No.
binding on the corporation. However, under Article 1910 of the New Civil Code, acts done by 491-A-3-B-2 covered by TCT No. 78086 did not include the authority to sell a portion of the
such officers beyond the scope of their authority cannot bind the corporation unless it has adjacent lot, Lot No. 491-A-3-B-1, or to create or convey real rights thereon. Neither may such
ratified such acts expressly or tacitly, or is estopped from denying them: authority be implied from the authority granted to Roxas to sell Lot No. 491-A-3-B-2 to the
petitioner "on such terms and conditions which he deems most reasonable and
Art. 1910. The principal must comply with all the obligations which the agent advantageous." Under paragraph 12, Article 1878 of the New Civil Code, a special power of
may have contracted within the scope of his authority. attorney is required to convey real rights over immovable property. 26 Article 1358 of the New
Civil Code requires that contracts which have for their object the creation of real rights over
As for any obligation wherein the agent has exceeded his power, the immovable property must appear in a public document. 27 The petitioner cannot feign
principal is not bound except when he ratifies it expressly or tacitly. ignorance of the need for Roxas to have been specifically authorized in writing by the Board
of Directors to be able to validly grant a right of way and agree to sell a portion of Lot No. 491-
Thus, contracts entered into by corporate officers beyond the scope of A-3-B-1. The rule is that if the act of the agent is one which requires authority in writing, those
authority are unenforceable against the corporation unless ratified by the dealing with him are charged with notice of that fact.28
corporation.23 Powers of attorney are generally construed strictly and courts will not infer or presume broad
powers from deeds which do not sufficiently include property or subject under which the
In BA Finance Corporation v. Court of Appeals,24 we also ruled that persons dealing agent is to deal.29 The general rule is that the power of attorney must be pursued within legal
with an assumed agency, whether the assumed agency be a general or special one, are bound strictures, and the agent can neither go beyond it; nor beside it. The act done must be legally
at their peril, if they would hold the principal liable, to ascertain not only the fact of agency identical with that authorized to be done.30 In sum, then, the consent of the respondent to
but also the nature and extent of authority, and in case either is controverted, the burden of the assailed provisions in the deed of absolute sale was not obtained; hence, the assailed
proof is upon them to establish it. provisions are not binding on it.

195
MATABABE FILES
Corporation Law

We reject the petitioner's submission that, in allowing Roxas to execute the contract inferred from acts that a principal has a right to do independently of the unauthorized act of
to sell and the deed of absolute sale and failing to reject or disapprove the same, the the agent. Moreover, if a writing is required to grant an authority to do a particular act,
respondent thereby gave him apparent authority to grant a right of way over Lot No. 491-A-3- ratification of that act must also be in writing.37 Since the respondent had not ratified the
B-1 and to grant an option for the respondent to sell a portion thereof to the petitioner. unauthorized acts of Roxas, the same are unenforceable.38 Hence, by the respondent's
Absent estoppel or ratification, apparent authority cannot remedy the lack of the written retention of the amount, it cannot thereby be implied that it had ratified the unauthorized
power required under the statement of frauds. 31 In addition, the petitioner's fallacy is its acts of its agent, Roberto Roxas.
wrong assumption of the unproved premise that the respondent had full knowledge of all the
terms and conditions contained in the deed of absolute sale when Roxas executed it. On the last issue, the petitioner contends that the CA erred in dismissing its
complaint for damages against the respondent on its finding that the delay in the
It bears stressing that apparent authority is based on estoppel and can arise from construction of its warehouse was due to its (petitioner's) fault. The petitioner asserts that
two instances: first, the principal may knowingly permit the agent to so hold himself out as the CA should have affirmed the ruling of the trial court that the respondent failed to cause
having such authority, and in this way, the principal becomes estopped to claim that the the eviction of the squatters from the property on or before September 29, 1991; hence, was
agent does not have such authority; second, the principal may so clothe the agent with the liable for P5,660,000. The respondent, for its part, asserts that the delay in the construction
indicia of authority as to lead a reasonably prudent person to believe that he actually has such of the petitioner's warehouse was due to its late filing of an application for a building permit,
authority.32 There can be no apparent authority of an agent without acts or conduct on the only on May 28, 1992.
part of the principal and such acts or conduct of the principal must have been known and
relied upon in good faith and as a result of the exercise of reasonable prudence by a third The petitioner's contention is meritorious. The respondent does not deny that it
person as claimant and such must have produced a change of position to its detriment. The failed to cause the eviction of the squatters on or before September 29, 1991. Indeed, the
apparent power of an agent is to be determined by the acts of the principal and not by the respondent does not deny the fact that when the petitioner wrote the respondent
acts of the agent.33 demanding that the latter cause the eviction of the squatters on April 15, 1992, the latter were
still in the premises. It was only after receiving the said letter in April 1992 that the respondent
For the principle of apparent authority to apply, the petitioner was burdened to caused the eviction of the squatters, which thus cleared the way for the petitioner's
prove the following: (a) the acts of the respondent justifying belief in the agency by the contractor to commence the construction of its warehouse and secure the appropriate
petitioner; (b) knowledge thereof by the respondent which is sought to be held; and, (c) building permit therefor.
reliance thereon by the petitioner consistent with ordinary care and prudence.34 In this case,
there is no evidence on record of specific acts made by the respondent 35 showing or The petitioner could not be expected to file its application for a building permit
indicating that it had full knowledge of any representations made by Roxas to the petitioner before April 1992 because the squatters were still occupying the property. Because of the
that the respondent had authorized him to grant to the respondent an option to buy a respondent's failure to cause their eviction as agreed upon, the petitioner's contractor failed
portion of Lot No. 491-A-3-B-1 covered by TCT No. 78085, or to create a burden or lien thereon, to commence the construction of the warehouse in October 1991 for the agreed price of
or that the respondent allowed him to do so. P8,649,000. In the meantime, costs of construction materials spiraled. Under the
construction contract entered into between the petitioner and the contractor, the petitioner
The petitioner's contention that by receiving and retaining the P5,000,000 purchase was obliged to pay P11,804,160,39including the additional work costing P1,441,500, or a net
price of Lot No. 491-A-3-B-2, the respondent effectively and impliedly ratified the grant of a increase of P1,712,980.40 The respondent is liable for the difference between the original cost
right of way on the adjacent lot, Lot No. 491-A-3-B-1, and to grant to the petitioner an option of construction and the increase thereon, conformably to Article 1170 of the New Civil Code,
to sell a portion thereof, is barren of merit. It bears stressing that the respondent sold Lot No. which reads:
491-A-3-B-2 to the petitioner, and the latter had taken possession of the property. As such, the
respondent had the right to retain the P5,000,000, the purchase price of the property it had Art. 1170. Those who in the performance of their obligations are guilty of
sold to the petitioner. For an act of the principal to be considered as an implied ratification of fraud, negligence, or delay and those who in any manner contravene the tenor
an unauthorized act of an agent, such act must be inconsistent with any other hypothesis thereof, are liable for damages.
than that he approved and intended to adopt what had been done in his name. 36 Ratification
is based on waiver the intentional relinquishment of a known right. Ratification cannot be

196
MATABABE FILES
Corporation Law

The petitioner, likewise, lost the amount of P3,900,000 by way of unearned income
from the lease of the property to the Ponderosa Leather Goods Company. The respondent is,
thus, liable to the petitioner for the said amount, under Articles 2200 and 2201 of the New Civil
Code:

Art. 2200. Indemnification for damages shall comprehend not only the value
of the loss suffered, but also that of the profits which the obligee failed to obtain.

Art. 2201. In contracts and quasi-contracts, the damages for which the
obligor who acted in good faith is liable shall be those that are the natural and
probable consequences of the breach of the obligation, and which the parties have
foreseen or could have reasonably foreseen at the time the obligation was
constituted.

In case of fraud, bad faith, malice or wanton attitude, the obligor shall be
responsible for all damages which may be reasonably attributed to the non-
performance of the obligation.

In sum, we affirm the trial court's award of damages and attorney's fees to the
petitioner.

IN LIGHT OF ALL THE FOREGOING, judgment is hereby rendered AFFIRMING the


assailed Decision of the Court of Appeals WITH MODIFICATION. The respondent is ordered to
pay to the petitioner the amount of P5,612,980 by way of actual damages and P100,000 by
way of attorney's fees. No costs.

197
MATABABE FILES
Corporation Law

Republic of the Philippines Same; Same; Same; Maxim that the one who made it pos-sible for a wrong to be done
SUPREME COURT should suffer.The equitable maxim that between two innocent parties the one who made it
Manila possi-ble for the wrong to be done should be the one to bear the re-sulting loss, applies when
EN BANC as in the instant case a cor-poration allows one of its officers, now alleged to be without
the proper authority, to send a telegram binding the corporation.
G.R. No. L-18287 March 30, 1963
Damages; Breach of contract; Moral damages not warranted if breach is not malicious
TRINIDAD J. FRANCISCO, plaintiff-appellee, or fraudulent.Award of moral dam-ages under Article 2220 of the Civil Code is not
vs. warranted if the breach of contract is not malicious or fraudulent (Ventanilla vs. Centeno, L-
GOVERNMENT SERVICE INSURANCE SYSTEM, defendant-appellant. 14333, 28 Jan. 1961; Fores vs. Miranda, L-12163, 4 March 1959).
-----------------------------
G.R. No. L-18155 March 30, 1963 Same; Same; When exemplary damages allowed.Exem-plary damages are only
allowed in addition to moral, temperate, liquidated, or compensatory damages (Art. 2234,
TRINIDAD J. FRANCISCO, plaintiff-appellant, Civil Code; Velayo vs. Shell Co. of P.I., L-7817, Res. July 30, 1957; Singson, et al. vs. Aragon and
vs. Lorza, L-5164, Jan. 27, 1953, 49 0.G. No. 2, 515).
GOVERNMENT SERVICE INSURANCE SYSTEM, defendant-appellee.
Attorneys fees; Award essentially discretionary with trial court.The award of
Corporations; Binding effect of acts of corporate officers.A corporation cannot attorneys fees is essentially discretionary with the trial court, and no abuse of discretion is
evade the binding effect produced by a telegram sent by its board secretary, and the committed when the court refuses to make an award because of the ab-sence of gross and
addressee of such telegram cannot be blamed for relying upon it, because if every person evident bad faith in defendants refusal to satisfy plaintiffs claim, or of any of the other
dealing with a corporation were held duty-bound to dis-believe every act of its responsible grounds enu-merated in Article 2208 of the Civil Code.
officers no matter how reg-ular it should appear on its face, corporate transactions would
speedily come to a standstill. APPEAL from a decision of the Court of First Instance of Rizal. Mojica, J.

Same; Same; When corporation estopped to deny apparent authority of its officers. The facts are stated in the opinion of the Court
If a private corporation intentional-ly or negligently clothes its officers or agents with
apparent power to perform acts for it, the corporation will be estopped to deny that such Vicente J. Francisco for plaintiff-appellee.
apparent authority is real, as to innocent third persons dealing in good faith with such officers
or agents. (2 Fletchers Encyclopedia, Priv. Corp. 255, Perm. Ed.) The Government Corporate Counsel for defendant-appellant.

Same; Same; Same; When notice of lands by a corporate officer is notice to REYES, J.B.L., J.:
corporation.Knowledge of facts acquired or possessed by an officer or agent of a
corporation in the course of his employment, and in relation to matters within the scope of Appeal by the Government Service Insurance System from the decision of the Court
his authority, is notice to the corporation, whether he communicates such knowledge or not. of First Instance of Rizal (Hon. Angel H. Mojica, presiding), in its Civil Case No. 2088-P, entitled
(Ballentine, Law on Cor-porations, section 112.) "Trinidad J. Francisco, plaintiff, vs. Government Service Insurance System, defendant", the
dispositive part of which reads as follows:
Same; Same; Same; Silence of corporation as ratification of agreement.The silence
of the corporation, taken together with the unconditional acceptance of three subsequent WHEREFORE, judgment is hereby rendered: (a) Declaring null and void the
remit-tances from plaintiff, constitutes a binding ratification of the original agreement consolidation in the name of the defendant, Government Service Insurance System,
between them (Civil Code, Article 1393). of the title of the VIC-MARI Compound; said title shall be restored to the plaintiff; and
all payments made by the plaintiff, after her offer had been accepted by the

198
MATABABE FILES
Corporation Law

defendant, must be credited as amortizations on her loan; and (b) Ordering the It will be noted that the collectible income each month from the
defendant to abide by the terms of the contract created by plaintiff's offer and it's mortgaged property, which as I said consists of installments amounting to
unconditional acceptance, with costs against the defendant. about P5,000, is more than enough to cover the monthly amortization on
Miss Francisco's loan. Indeed, had she not encountered difficulties, due to
The plaintiff, Trinidad J. Francisco, likewise appealed separately (L-18155), because unforeseen circumstances, in collecting the said installments, she could have
the trial court did not award the P535,000.00 damages and attorney's fees she claimed. Both paid the amortizations as they fell due and there would have been really no
appeals are, therefore, jointly treated in this decision. need for the GSIS to resort to foreclosure.
The proposed administration by the GSIS of the mortgaged property will
The following facts are admitted by the parties: On 10 October 1956, the plaintiff, continue even after Miss Francisco's account shall have been kept up to
Trinidad J. Francisco, in consideration of a loan in the amount of P400,000.00, out of which date. However, once the arrears shall have been paid, whatever amount of
the sum of P336,100.00 was released to her, mortgaged in favor of the defendant, the monthly installments collected in excess of the amortization due on the
Government Service Insurance System (hereinafter referred to as the System) a parcel of land loan will be turned over to Miss Francisco.
containing an area of 18,232 square meters, with twenty-one (21) bungalows, known as Vic-
Mari Compound, located at Baesa, Quezon City, payable within ten (10) years in monthly I make the foregoing proposal to show Francisco's sincere desire to
installments of P3,902.41, and with interest of 7% per annum compounded monthly. work out any fair arrangement for the settlement of her obligation. I trust
that the GSIS, under the broadminded policies of your administration, would
On 6 January 1959, the System extrajudicially foreclosed the mortgage on the ground give it serious consideration.
that up to that date the plaintiff-mortgagor was in arrears on her monthly installments in the
amount of P52,000.00. Payments made by the plaintiff at the time of foreclosure amounted Sincerely,
to P130,000.00. The System itself was the buyer of the property in the foreclosure sale.
s/ Vicente J. Francisco
On 20 February 1959, the plaintiff's father, Atty. Vicente J. Francisco, sent a letter to t/ VICENTE J. FRANCISCO
the general manager of the defendant corporation, Mr. Rodolfo P. Andal, the material portion
of which recited as follows: On the same date, 20 February 1959, Atty. Francisco received the
following telegram:.
Yesterday, I was finally able to collect what the Government owed VICENTE FRANCISCO
me and I now propose to pay said amount of P30,000 to the GSIS if it would SAMANILLO BLDG. ESCOLTA.
agree that after such payment the foreclosure of my daughter's mortgage GSIS BOARD APPROVED YOUR REQUEST RE REDEMPTION OF
would be set aside. I am aware that the amount of P30,000 which I offer to FORECLOSED PROPERTY OF YOUR DAUGHTER
pay will not cover the total arrearage of P52,000 but as regards the balance, ANDAL"
I propose this arrangement: for the GSIS to take over the administration of
the mortgaged property and to collect the monthly installments, amounting On 28 February 1959, Atty. Francisco remitted to the System, through Andal, a check
to about P5,000, due on the unpaid purchase price of more than 31 lots and for P30,000.00, with an accompanying letter, which reads:
houses therein and the monthly installments collected shall be applied to the
payment of Miss Francisco's arrearage until the same is fully covered. It is I am sending you herewith BPI Check No. B-299484 for Thirty Thousand
requested, however, that from the amount of the monthly installments Pesos (P30,000.00) in accordance with my letter of February 20th and your reply
collected, the sum of P350.00 be deducted for necessary expenses, such as thereto of the same date, which reads:
to pay the security guard, the street-caretaker, the Meralco Bill for the street
lights and sundry items. GSIS BOARD APPROVED YOUR REQUEST RE REDEMPTION OF FORECLOSED
PROPERTY OF YOUR DAUGHTER
xxx xxx xxx

199
MATABABE FILES
Corporation Law

redemption has expired, said defendant, on 5 July 1960, consolidated the title to the
The defendant received the amount of P30,000.00, and issued therefor its official compound in its name, and gave notice thereof to the plaintiff on 26 July 1960 and to each
receipt No. 1209874, dated 4 March 1959. It did not, however, take over the administration of occupant of the compound.
the compound. In the meantime, the plaintiff received the monthly payments of some of the
occupants thereat; then on 4 March 1960, she remitted, through her father, the amount of Hence, the plaintiff instituted the present suit, for specific performance and
P44,121.29, representing the total monthly installments that she received from the occupants damages. The defendant answered, pleading that the binding acceptance of Francisco's offer
for the period from March to December 1959 and January to February 1960, minus expenses was the resolution of the Board, and that Andal's telegram, being erroneous, should be
and real estate taxes. The defendant also received this amount, and issued the corresponding disregarded. After trial, the court below found that the offer of Atty. Francisco, dated 20
official receipt. February 1959, made on behalf of his daughter, had been unqualifiedly accepted, and was
binding, and rendered judgment as noted at the start of this opinion.
Remittances, all accompanied by letters, corresponding to the months of March,
April, May, and June, 1960 and totalling P24,604.81 were also sent by the plaintiff to the The defendant-appellant corporation assigns six (6) errors allegedly committed by
defendant from time to time, all of which were received and duly receipted for. the lower court, all of which, however, are resolvable on the single issue as to whether or not
the telegram generated a contract that is valid and binding upon the parties.
Then the System sent three (3) letters, one dated 29 January 1960, which was signed
by its assistant general manager, and the other two letters, dated 19 and 26 February 1960, Wherefore, the parties respectfully pray that the foregoing stipulation of facts be
respectively, which were signed by Andal, asking the plaintiff for a proposal for the payment admitted and approved by this Honorable Court, without prejudice to the parties adducing
of her indebtedness, since according to the System the one-year period for redemption had other evidence to prove their case not covered by this stipulation of facts.
expired. We find no reason for altering the conclusion reached by the court below that the
offer of compromise made by plaintiff in the letter, Exhibit "A", had been validly accepted,
In reply, Atty. Francisco sent a letter, dated 11 March 1960, protesting against the and was binding on the defendant. The terms of the offer were clear, and over the signature
System's request for proposal of payment and inviting its attention to the concluded contract of defendant's general manager, Rodolfo Andal, plaintiff was informed telegraphically that
generated by his offer of 20 February 1959, and its acceptance by telegram of the same date, her proposal had been accepted. There was nothing in the telegram that hinted at any
the compliance of the terms of the offer already commenced by the plaintiff, and the anomaly, or gave ground to suspect its veracity, and the plaintiff, therefore, can not be
misapplication by the System of the remittances she had made, and requesting the proper blamed for relying upon it. There is no denying that the telegram was within Andal's apparent
corrections. authority, but the defense is that he did not sign it, but that it was sent by the Board Secretary
in his name and without his knowledge. Assuming this to be true, how was appellee to know
By letter, dated 31 May 1960, the defendant countered the preceding protest that, by it? Corporate transactions would speedily come to a standstill were every person dealing with
all means, the plaintiff should pay attorney's fees of P35,644.14, publication expenses, filing a corporation held duty-bound to disbelieve every act of its responsible officers, no matter
fee of P301.00, and surcharge of P23.64 for the foreclosure work done; that the telegram how regular they should appear on their face. This Court has observed in Ramirez vs.
should be disregarded in view of its failure to express the contents of the board resolution Orientalist Co., 38 Phil. 634, 654-655, that
due to the error of its minor employees in couching the correct wording of the telegram. A
copy of the excerpts of the resolution of the Board of Directors (No. 380, February 20, 1959) In passing upon the liability of a corporation in cases of this kind it is always
was attached to the letter, showing the approval of Francisco's offer well to keep in mind the situation as it presents itself to the third party with whom
the contract is made. Naturally he can have little or no information as to what occurs
... subject to the condition that Mr. Vicente J. Francisco shall pay all expenses in corporate meetings; and he must necessarily rely upon the external manifestations
incurred by the GSIS in the foreclosure of the mortgage. of corporate consent. The integrity of commercial transactions can only be
maintained by holding the corporation strictly to the liability fixed upon it by its
Inasmuch as, according to the defendant, the remittances previously made by Atty. agents in accordance with law; and we would be sorry to announce a doctrine which
Francisco were allegedly not sufficient to pay off her daughter's arrears, including attorney's would permit the property of a man in the city of Paris to be whisked out of his hands
fees incurred by the defendant in foreclosing the mortgage, and the one-year period for and carried into a remote quarter of the earth without recourse against the

200
MATABABE FILES
Corporation Law

corporation whose name and authority had been used in the manner disclosed in this while it accepted the various other remittances, starting on 28 February 1959, sent by the
case. As already observed, it is familiar doctrine that if a corporation knowingly plaintiff to it in compliance with her performance of her part of the new contract.
permits one of its officers, or any other agent, to do acts within the scope of an
apparent authority, and thus holds him out to the public as possessing power to do The inequity of permitting the System to deny its acceptance become more patent
those acts, the corporation will, as against any one who has in good faith dealt with when account is taken of the fact that in remitting the payment of P30,000 advanced by her
the corporation through such agent, be estopped from denying his authority; and father, plaintiff's letter to Mr. Andal quoted verbatim the telegram of acceptance. This was in
where it is said "if the corporation permits" this means the same as "if the thing is itself notice to the corporation of the terms of the allegedly unauthorized telegram, for as
permitted by the directing power of the corporation." Ballentine says:

It has also been decided that Knowledge of facts acquired or possessed by an officer or agent of a
corporation in the course of his employment, and in relation to matters within the
A very large part of the business of the country is carried on by corporations. scope of his authority, is notice to the corporation, whether he communicates such
It certainly is not the practice of persons dealing with officers or agents who assume knowledge or not. (Ballentine, Law on Corporations, section 112.)
to act for such entities to insist on being shown the resolution of the board of
directors authorizing the particular officer or agent to transact the particular since a corporation cannot see, or know, anything except through its officers.
business which he assumes to conduct. A person who knows that the officer or agent Yet, notwithstanding this notice, the defendant System pocketed the amount, and kept silent
of the corporation habitually transacts certain kinds of business for such corporation about the telegram not being in accordance with the true facts, as it now alleges. This silence,
under circumstances which necessarily show knowledge on the part of those taken together with the unconditional acceptance of three other subsequent remittances
charged with the conduct of the corporate business assumes, as he has the right to from plaintiff, constitutes in itself a binding ratification of the original agreement (Civil Code,
assume, that such agent or officer is acting within the scope of his authority. (Curtis Art. 1393).
Land & Loan Co. vs. Interior Land Co., 137 Wis. 341, 118 N.W. 853, 129 Am. St. Rep.
1068; as cited in 2 Fletcher's Encyclopedia, Priv. Corp. 263, perm. Ed.) ART. 1393. Ratification may be effected expressly or tacitly. It is understood
that there is a tacit ratification if, with knowledge of the reason which renders the
Indeed, it is well-settled that contract voidable and such reason having ceased, the person who has a right to
invoke it should execute an act which necessarily implies an intention to waive his
If a private corporation intentionally or negligently clothes its officers or right.
agents with apparent power to perform acts for it, the corporation will be estopped
to deny that such apparent authority is real, as to innocent third persons dealing in Nowhere else do the circumstances call more insistently for the application of the
good faith with such officers or agents. (2 Fletcher's Encyclopedia, Priv. Corp. 255, equitable maxim that between two innocent parties, the one who made it possible for the
Perm. Ed.) wrong to be done should be the one to bear the resulting loss..

Hence, even if it were the board secretary who sent the telegram, the corporation The defendant's assertion that the telegram came from it but that it was incorrectly
could not evade the binding effect produced by the telegram.. worded renders unnecessary to resolve the other point on controversy as to whether the said
telegram constitutes an actionable document..
The defendant-appellant does not disown the telegram, and even asserts that it
came from its offices, as may be gleaned from the letter, dated 31 May 1960, to Atty. Since the terms offered by the plaintiff in the letter of 20 February 1959 (Exhibit "A")
Francisco, and signed "R. P. Andal, general manager by Leovigildo Monasterial, legal counsel", provided for the setting aside of the foreclosure effected by the defendant System, the
wherein these phrases occur: "the telegram sent ... by this office" and "the telegram we sent acceptance of the offer left the account of plaintiff in the same condition as if no foreclosure
your" (emphasis supplied), but it alleges mistake in couching the correct wording. This alleged had taken place. It follows, as the lower court has correctly held, that the right of the System
mistake cannot be taken seriously, because while the telegram is dated 20 February 1959, the to collect attorneys' fees equivalent to 10% of the due (P35,694.14) and the expenses and
defendant informed Atty. Francisco of the alleged mistake only on 31 May 1960, and all the

201
MATABABE FILES
Corporation Law

charges of P3,300.00 may no longer be enforced, since by the express terms of the mortgage precludes a recovery. The award of attorneys' fees is essentially discretionary in the trial
contract, these sums were collectible only "in the event of foreclosure." court, and no abuse of discretion has been shown.
FOR THE FOREGOING REASONS, the appealed decision is hereby affirmed, with costs
The court a quo also called attention to the unconscionability of defendant's charging against the defendant Government Service Insurance System, in G.R. No.L-18287.
the attorney's fees, totalling over P35,000.00; and this point appears well-taken, considering
that the foreclosure was merely extra-judicial, and the attorneys' work was limited to Bengzon, C.J., Padilla, Bautista Angelo, Labrador, Concepcion, Barrera, Paredes, Dizon, Regala
requiring the sheriff to effectuate the foreclosure. However, in view of the parties' agreement and Makalintal, JJ., concur.
to set the same aside, with the consequential elimination of such incidental charges, the
matter of unreasonableness of the counsel fees need not be labored further.

Turning now to the plaintiff's separate appeal (Case G.R. No. L-18155): Her prayer for
an award of actual or compensatory damages for P83,333.33 is predicated on her alleged
unrealized profits due to her inability to sell the compound for the price of P750,000.00
offered by one Vicente Alunan, which sale was allegedly blocked because the System
consolidated the title to the property in its name. Plaintiff reckons the amount of P83,333.33
by placing the actual value of the property at P666,666.67, a figure arrived at by assuming
that the System's loan of P400,000.00 constitutes 60% of the actual value of the security. The
court a quo correctly refused to award such actual or compensatory damages because it could
not determine with reasonable certainty the difference between the offered price and the
actual value of the property, for lack of competent evidence. Without proof we cannot
assume, or take judicial notice, as suggested by the plaintiff, that the practice of lending
institutions in the country is to give out as loan 60% of the actual value of the collateral. Nor
should we lose sight of the fact that the price offered by Alunan was payable in installments
covering five years, so that it may not actually represent true market values.

Nor was there error in the appealed decision in denying moral damages, not only on
account of the plaintiff's failure to take the witness stand and testify to her social humiliation,
wounded feelings, anxiety, etc., as the decision holds, but primarily because a breach of
contract like that of defendant, not being malicious or fraudulent, does not warrant the
award of moral damages under Article 2220 of the Civil Code (Ventanilla vs. Centeno, L-14333,
28 Jan. 1961; Fores vs. Miranda, L-12163, 4 March 1959).

There is no basis for awarding exemplary damages either, because this species of
damages is only allowed in addition to moral, temperate, liquidated, or compensatory
damages, none of which have been allowed in this case, for reasons herein before discussed
(Art. 2234, Civil Code; Velayo vs. Shell Co. of P.I., L-7817, Res. July 30, 1957; Singson, et al. vs.
Aragon and Lorza, L-5164, Jan. 27, 1953, 49 O.G. No. 2, 515).

As to attorneys' fees, we agree with the trial court's stand that in view of the absence
of gross and evident bad faith in defendant's refusal to satisfy the plaintiff's claim, and there
being none of the other grounds enumerated in Article 2208 of the Civil Code, such absence

202
MATABABE FILES
Corporation Law

Republic of the Philippines


SUPREME COURT Same; Same; Same.Granting arguendo that the "dealership agreement" involved
Manila here would be valid and enforceable if entered into with a person other than a director or
SECOND DIVISION officer of the corporation, the fact that the other party to the contract was a Director and
Auditor of the petitioner corporation changes the whole situation. First of all, We believe that
G.R. No. L-68555 March 19, 1993 the contract was neither fair nor reasonable. The "dealership agreement" entered into in July,
1969, was to sell and supply to respondent Te 20,000 bags of white cement per month, for
PRIME WHITE CEMENT CORPORATION, petitioner, five years starting September, 1970, at the fixed price of P9.70 per bag. Respondent Te is a
vs. businessman himself and must have known, or at least must be presumed to know, that at
HONORABLE INTERMEDIATE APPELLATE COURT and ALEJANDRO TE, respondents. that time, prices of commodities in general, and white cement in particular, were not stable
and were expected to rise. At the time of the contract, petitioner corporation had not even
Corporation Law; Contracts; When contracts signed by corporate officers binding on commenced the manufacture of white cement, the reason why delivery was not to begin until
corporation.Under the Corporation Law, which was then in force at the time this case 14 months later. He must have known that within that period of six years, there would be a
arose, as well as under the present Corporation Code, all corporate powers shall be exercised considerable rise in the price of white cement. In fact, respondent Te's own Memorandum
by the Board of Directors, except as otherwise provided by law. Although it cannot shows that in September, 1970, the price per bag was P 14.50, and by the middle of 1975, it
completely abdicate its power and responsibility to act for the juridical entity, the Board may was already P37.50 per bag. Despite this, no provision was made in the "dealership
expressly delegate specific powers to its President or any of its officers. In the absence of agreement" to allow for an increase in price mutually acceptable to the parties. Instead, the
such express delegation, a contract entered into by its President, on behalf of the price was pegged at P9.70 per bag for the whole five years of the contract. Fairness on his
corporation, may still bind the corporation if the board should ratify the same expressly or part as a director of the corporation from whom he was to buy the cement, would require
impliedly. Implied ratification may take various formslike silence or acquiescence; by acts such a provision. In fact, this unfairness in the contract is also a basis which renders a contract
showing approval or adoption of the contract; or by acceptance and retention of benefits entered into by the President, without authority from the Board of Directors, void or voidable,
flowing therefrom. Furthermore, even in the absence of express or implied authority by although it may have been in the ordinary course of business. We believe that the fixed price
ratification, the President as such may, as a general rule, bind the corporation by a contract in of P9.70 per bag for a period of five years was not fair and reasonable. Respondent Te,
the ordinary course of business, provided the same is reasonable under the circumstances. himself, when he subsequently entered into contracts to resell the cement to his "new
These rules are basic, but are all general and thus quite flexible. They apply where the dealers" Henry Wee and Gaudencio Galang stipulated as follows: The price of white cement
President or other officer, purportedly acting for the corporation, is dealing with a third shall be mutually determined by us but in no case shall the same be less than P14.00 per bag
person, i.e., a person outside the corporation. (94 Ibs)."

Same; Same; A board director or other corporate officer cannot readily enter into a Same; Same; Damages; No moral damages for lost goodwill are awardable to a
contract with his own corporation; Exceptions.A director of a corporation holds a position corporation.As a result of this action which has been proven to be without legal basis,
of trust and as such, he owes a duty of loyalty to his corporation. In case his interests conflict petitioner corporation's reputation and goodwill have been prejudiced. However, there can
with those of the corporation, he cannot sacrifice the latter to his own advantage and benefit. be no award for moral damages under Article 2217 and succeeding articles on Section 1 of
As corporate managers, directors are committed to seek the maximum amount of profits for Chapter 3 of Title XVIII of the Civil Code in favor of a corporation.
the corporation. This trust relationship "is not a matter of statutory or technical law. It springs
from the fact that directors have the control and guidance of corporate affairs and property PETITION for review on certiorari of the decision of the then Intermediate Appellate
and hence of the property interests of the stockholders." Court.

Same; Same; Same.On the other hand, a director's contract with his corporation is The facts are stated in the opinion of the Court. Prime White Cement Corp. vs.
not in all instances void or voidable. If the contract is fair and reasonable under the Intermediate Appellate Court, 220 SCRA 103, G.R. No. 68555 March 19, 1993
circumstances, it may be ratified by the stockholders provided a full disclosure of his adverse
interest is made. De Jesus & Associates for petitioner.

203
MATABABE FILES
Corporation Law

Padlan, Sutton, Mendoza & Associates for private respondent. Right after the plaintiff entered into the aforesaid dealership
agreement, he placed an advertisement in a national, circulating newspaper
CAMPOS, JR., J.: the fact of his being the exclusive dealer of the defendant corporation's
white cement products in Mindanao area, more particularly, in the Manila
Before Us is a Petition for Review on Certiorari filed by petitioner Prime White Chronicle dated August 16, 1969 (Exhibits R and R-1) and was even
Cement Corporation seeking the reversal of the decision * of the then Intermediate Appellate congratulated by his business associates, so much so, he was asked by some
Court, the dispositive portion of which reads as follows: of his businessmen friends and close associates if they can be his sub-dealer
in the Mindanao area.
WHEREFORE, in view of the foregoing, the judgment appealed from
is hereby affirmed in toto. 1 Relying heavily on the dealership agreement, plaintiff sometime in
the months of September, October, and December, 1969, entered into a
The facts, as found by the trial court and as adopted by the respondent Court are written agreement with several hardware stores dealing in buying and
hereby quoted, to wit: selling white cement in the Cities of Davao and Cagayan de Oro which would
thus enable him to sell his allocation of 20,000 bags regular supply of the
On or about the 16th day of July, 1969, plaintiff and defendant said commodity, by September, 1970 (Exhibits O, O-1, O-2, P, P-1, P-2, Q, Q-1
corporation thru its President, Mr. Zosimo Falcon and Justo C. Trazo, as and Q-2). After the plaintiff was assured by his supposed buyer that his
Chairman of the Board, entered into a dealership agreement (Exhibit A) allocation of 20,000 bags of white cement can be disposed of, he informed
whereby said plaintiff was obligated to act as the exclusive dealer and/or the defendant corporation in his letter dated August 18, 1970 that he is
distributor of the said defendant corporation of its cement products in the making the necessary preparation for the opening of the requisite letter of
entire Mindanao area for a term of five (5) years and proving (sic) among credit to cover the price of the due initial delivery for the month of
others that: September, 1970 (Exhibit B), looking forward to the defendant corporation's
duty to comply with the dealership agreement. In reply to the aforesaid
a. The corporation shall, commencing September, letter of the plaintiff, the defendant corporation thru its corporate secretary,
1970, sell to and supply the plaintiff, as dealer with 20,000 replied that the board of directors of the said defendant decided to impose
bags (94 lbs/bag) of white cement per month; the following conditions:

b. The plaintiff shall pay the defendant corporation a. Delivery of white cement shall commence at the end of
P9.70, Philippine Currency, per bag of white cement, FOB November, 1970;
Davao and Cagayan de Oro ports;
b. Only 8,000 bags of white cement per month for only a
c. The plaintiff shall, every time the defendant period of three (3) months will be delivered;
corporation is ready to deliver the good, open with any
bank or banking institution a confirmed, unconditional, and c. The price of white cement was priced at P13.30 per bag;
irrevocable letter of credit in favor of the corporation and
that upon certification by the boat captain on the bill of d. The price of white cement is subject to readjustment
lading that the goods have been loaded on board the vessel unilaterally on the part of the defendant;
bound for Davao the said bank or banking institution shall
release the corresponding amount as payment of the e. The place of delivery of white cement shall be Austurias
goods so shipped. (sic);

204
MATABABE FILES
Corporation Law

f. The letter of credit may be opened only with the to bind the corporation as this avoidance is barred by the principle of
Prudential Bank, Makati Branch; estoppel. 3

g. Payment of white cement shall be made in advance and In this petition for review, petitioner Prime White Cement Corporation made the
which payment shall be used by the defendant as guaranty following assignment of errors. 4
in the opening of a foreign letter of credit to cover costs
and expenses in the procurement of materials in the I
manufacture of white cement. (Exhibit C). THE DECISION AND RESOLUTION OF THE INTERMEDIATE APPELLATE
xxx xxx xxx COURT ARE UNPRECEDENTED DEPARTURES FROM THE CODIFIED
PRINCIPLE THAT CORPORATE OFFICERS COULD ENTER INTO CONTRACTS IN
Several demands to comply with the dealership agreement (Exhibits BEHALF OF THE CORPORATION ONLY WITH PRIOR APPROVAL OF THE
D, E, G, I, R, L, and N) were made by the plaintiff to the defendant, however, BOARD OF DIRECTORS.
defendant refused to comply with the same, and plaintiff by force of
circumstances was constrained to cancel his agreement for the supply of II
white cement with third parties, which were concluded in anticipation of, THE DECISION AND RESOLUTION OF THE INTERMEDIATE APPELLATE
and pursuant to the said dealership agreement. COURT ARE CONTRARY TO THE ESTABLISHED JURISPRUDENCE, PRINCIPLE
AND RULE ON FIDUCIARY DUTY OF DIRECTORS AND OFFICERS OF THE
Notwithstanding that the dealership agreement between the CORPORATION.
plaintiff and defendant was in force and subsisting, the defendant
corporation, in violation of, and with evident intention not to be bound by III
the terms and conditions thereof, entered into an exclusive dealership THE DECISION AND RESOLUTION OF THE INTERMEDIATE APPELLATE
agreement with a certain Napoleon Co for the marketing of white cement in COURT DISREGARDED THE PRINCIPLE AND JURISPRUDENCE, PRINCIPLE
Mindanao (Exhibit T) hence, this suit. (Plaintiff's Record on Appeal, pp. 86- AND RULE ON UNENFORCEABLE CONTRACTS AS PROVIDED IN ARTICLE 1317
90). 2 OF THE NEW CIVIL CODE.

After trial, the trial court adjudged the corporation liable to Alejandro Te in the IV
amount of P3,302,400.00 as actual damages, P100,000.00 as moral damages, and P10,000.00 THE DECISION AND RESOLUTION OF THE INTERMEDIATE APPELLATE
as and for attorney's fees and costs. The appellate court affirmed the said decision mainly on COURT DISREGARDED THE PRINCIPLE AND JURISPRUDENCE AS TO WHEN
the following basis, and We quote: AWARD OF ACTUAL AND MORAL DAMAGES IS PROPER.

There is no dispute that when Zosimo R. Falcon and Justo B. Trazo V


signed the dealership agreement Exhibit "A", they were the President and IN NOT AWARDING PETITIONER'S CAUSE OF ACTION AS STATED IN ITS
Chairman of the Board, respectively, of defendant-appellant corporation. ANSWER WITH SPECIAL AND AFFIRMATIVE DEFENSES WITH
Neither is the genuineness of the said agreement contested. As a matter of COUNTERCLAIM THE INTERMEDIATE APPELLATE COURT HAS CLEARLY
fact, it appears on the face of the contract itself that both officers were duly DEPARTED FROM THE ACCEPTED USUAL, COURSE OF JUDICIAL
authorized to enter into the said agreement and signed the same for and in PROCEEDINGS.
behalf of the corporation. When they, therefore, entered into the said
transaction they created the impression that they were duly clothed with the There is only one legal issue to be resolved by this Court: whether or not the
authority to do so. It cannot now be said that the disputed agreement which "dealership agreement" referred by the President and Chairman of the Board of petitioner
possesses all the essential requisites of a valid contract was never intended corporation is a valid and enforceable contract. We do not agree with the conclusion of the
respondent Court that it is.

205
MATABABE FILES
Corporation Law

Under the Corporation Law, which was then in force at the time this case arose, 5 as On the other hand, a director's contract with his corporation is not in all instances
well as under the present Corporation Code, all corporate powers shall be exercised by the void or voidable. If the contract is fair and reasonable under the circumstances, it may be
Board of Directors, except as otherwise provided by law. 6Although it cannot completely ratified by the stockholders provided a full disclosure of his adverse interest is made. Section
abdicate its power and responsibility to act for the juridical entity, the Board may expressly 32 of the Corporation Code provides, thus:
delegate specific powers to its President or any of its officers. In the absence of such express
delegation, a contract entered into by its President, on behalf of the corporation, may still Sec. 32. Dealings of directors, trustees or officers with the
bind the corporation if the board should ratify the same expressly or impliedly. Implied corporation. A contract of the corporation with one or more of its
ratification may take various forms like silence or acquiescence; by acts showing approval directors or trustees or officers is voidable, at the option of such
or adoption of the contract; or by acceptance and retention of benefits flowing corporation, unless all the following conditions are present:
therefrom. 7 Furthermore, even in the absence of express or implied authority by ratification,
the President as such may, as a general rule, bind the corporation by a contract in the ordinary 1. That the presence of such director or trustee in the board meeting
course of business, provided the same is reasonable under the circumstances. 8 These rules in which the contract was approved was not necessary to constitute a
are basic, but are all general and thus quite flexible. They apply where the President or other quorum for such meeting;
officer, purportedly acting for the corporation, is dealing with a third person, i. e., a
person outside the corporation. 2. That the vote of such director or trustee was not necessary for
the approval of the contract;
The situation is quite different where a director or officer is dealing with his own
corporation. In the instant case respondent Te was not an ordinary stockholder; he was a 3. That the contract is fair and reasonable under the circumstances;
member of the Board of Directors and Auditor of the corporation as well. He was what is and
often referred to as a "self-dealing" director.
A director of a corporation holds a position of trust and as such, he owes a duty of loyalty to 4. That in the case of an officer, the contract with the officer has
his corporation. 9 In case his interests conflict with those of the corporation, he cannot been previously authorized by the Board of Directors.
sacrifice the latter to his own advantage and benefit. As corporate managers, directors are
committed to seek the maximum amount of profits for the corporation. This trust relationship Where any of the first two conditions set forth in the preceding
"is not a matter of statutory or technical law. It springs from the fact that directors have the paragraph is absent, in the case of a contract with a director or trustee, such
control and guidance of corporate affairs and property and hence of the property interests of contract may be ratified by the vote of the stockholders representing at
the stockholders." 10 In the case of Gokongwei v. Securities and Exchange Commission, this least two-thirds (2/3) of the outstanding capital stock or of two-thirds (2/3)
Court quoted with favor from Pepper v. Litton, 11 thus: of the members in a meeting called for the purpose: Provided, That full
disclosure of the adverse interest of the directors or trustees involved is
. . . He cannot by the intervention of a corporate entity violate the made at such meeting: Provided, however, That the contract is fair and
ancient precept against serving two masters. . . . He cannot utilize his inside reasonable under the circumstances.
information and his strategic position for his own preferment. He cannot
violate rules of fair play by doing indirectly through the corporation what he Although the old Corporation Law which governs the instant case did not contain a
could not do directly. He cannot use his power for his personal advantage similar provision, yet the cited provision substantially incorporates well-settled principles in
and to the detriment of the stockholders and creditors no matter how corporate law. 12
absolute in terms that power may be and no matter how meticulous he is to
satisfy technical requirements. For that power is at all times subject to the Granting arguendo that the "dealership agreement" involved here would be valid and
equitable limitation that it may not be exercised for the aggrandizement, enforceable if entered into with a person other than a director or officer of the corporation,
preference, or advantage of the fiduciary to the exclusion or detriment of the fact that the other party to the contract was a Director and Auditor of the petitioner
the cestuis. . . . . corporation changes the whole situation. First of all, We believe that the contract was neither

206
MATABABE FILES
Corporation Law

fair nor reasonable. The "dealership agreement" entered into in July, 1969, was to sell and As a result of this action which has been proven to be without legal basis, petitioner
supply to respondent Te 20,000 bags of white cement per month, for five years starting corporation's reputation and goodwill have been prejudiced. However, there can be no award
September, 1970, at the fixed price of P9.70 per bag. Respondent Te is a businessman himself for moral damages under Article 2217 and succeeding articles on Section 1 of Chapter 3 of Title
and must have known, or at least must be presumed to know, that at that time, prices of XVIII of the Civil Code in favor of a corporation.
commodities in general, and white cement in particular, were not stable and were expected
to rise. At the time of the contract, petitioner corporation had not even commenced the In view of the foregoing, the Decision and Resolution of the Intermediate Appellate
manufacture of white cement, the reason why delivery was not to begin until 14 months later. Court dated March 30, 1984 and August 6, 1984, respectively, are hereby SET ASIDE. Private
He must have known that within that period of six years, there would be a considerable rise in respondent Alejandro Te is hereby ordered to pay petitioner corporation the sum of
the price of white cement. In fact, respondent Te's own Memorandum shows that in P20,000.00 for attorney's fees, plus the cost of suit and expenses of litigation.
September, 1970, the price per bag was P14.50, and by the middle of 1975, it was already
P37.50 per bag. Despite this, no provision was made in the "dealership agreement" to allow SO ORDERED.
for an increase in price mutually acceptable to the parties. Instead, the price was pegged at
P9.70 per bag for the whole five years of the contract. Fairness on his part as a director of the Narvasa, C.J., Padilla, Regalado and Nocon, JJ., concur.
corporation from whom he was to buy the cement, would require such a provision. In fact,
this unfairness in the contract is also a basis which renders a contract entered into by the
President, without authority from the Board of Directors, void or voidable, although it may
have been in the ordinary course of business. We believe that the fixed price of P9.70 per bag
for a period of five years was not fair and reasonable. Respondent Te, himself, when he
subsequently entered into contracts to resell the cement to his "new dealers" Henry
Wee 13 and Gaudencio Galang 14 stipulated as follows:

The price of white cement shall be mutually determined by us but in


no case shall the same be less than P14.00 per bag (94 lbs).

The contract with Henry Wee was on September 15, 1969, and that with Gaudencio
Galang, on October 13, 1967. A similar contract with Prudencio Lim was made on December
29, 1969. 15 All of these contracts were entered into soon after his "dealership agreement"
with petitioner corporation, and in each one of them he protected himself from any increase
in the market price of white cement. Yet, except for the contract with Henry Wee, the
contracts were for only two years from October, 1970. Why did he not protect the
corporation in the same manner when he entered into the "dealership agreement"? For that
matter, why did the President and the Chairman of the Board not do so either? As director,
specially since he was the other party in interest, respondent Te's bounden duty was to act in
such manner as not to unduly prejudice the corporation. In the light of the circumstances of
this case, it is to Us quite clear that he was guilty of disloyalty to the corporation; he was
attempting in effect, to enrich himself at the expense of the corporation. There is no showing
that the stockholders ratified the "dealership agreement" or that they were fully aware of its
provisions. The contract was therefore not valid and this Court cannot allow him to reap the
fruits of his disloyalty.

207
MATABABE FILES
Corporation Law

Republic of the Philippines such express, but limited, delegated authority. Neither can it be logically claimed that any
SUPREME COURT power greater than that expressly conferred is inherent in Mr. Maglana's position as
Manila president and chairman of the corporation.
THIRD DIVISION
Same; Same; Same.Petitioner's last refuge then is his alternative proposition,
G.R. No. L-53820 June 15, 1992 namely, that private respondent had clothed Mr. Maglana with the apparent power to act for
it and had caused persons dealing with it to believe that he was conferred with such power.
YAO KA SIN TRADING, owned and operated by YAO KA SIN, petitioner, The rule is of course settled that "[a]lthough an officer or agent acts without, or in excess of,
vs. his actual authority if he acts within the scope of an apparent authority with which the
HONORABLE COURT OF APPEALS and PRIME WHITE CEMENT CORPORATION, represented corporation has clothed him by holding him out or permitting him to appear as having such
by its President-Chairman, CONSTANCIO B. MALAGNA, respondents. authority, the corporation is bound thereby in favor of a person who deals with him in good
faith in reliance on such apparent authority, as where an officer is allowed to exercise a
Actions; A sole proprietorship does not have legal capacity to sue. Its owner shall be particular authority with respect to the business, or a particular branch of it, continuously and
deemed the plaintiff.The complaint then should have been amended to implead Yao Ka Sin publicly, for a considerable time." Also, "if a private corporation intentionally or negligently
as plaintiff in substitution of Yao Ka Sin Trading. However, it is now too late in the history of clothes its officers or agents with apparent power to perform acts for it, the corporation will
this case to dismiss this petition and, in effect, nullify all proceedings had before the trial court be estopped to deny that such apparant authority is real, as to innocent third persons dealing
and the respondent Court on the sole ground of petitioner's lack of capacity to sue, in good faith with such officers or agents." This "apparent authority may result from (1) the
Considering that private respondent did not pursue this issue before the respondent Court general manner by which the corporation holds out an officer or agent as having power to act
and this Court; that, as We held in Juasing, the defect is merely formal and not substantial, or, in other words, the apparent authority with which it clothes him to act in general, or (2)
and an amendment to cure such defect is expressly authorized by Section 4, Rule 10 of the the acquiescence in his acts of a particular nature, with actual or constructive knowledge
Rules of Court which provides that "[a] defect in the designation of the parties may be thereof, whether within or without the scope of his ordinary powers."
summarily corrected at any stage of the action provided no prejudice is caused thereby to the
adverse party;" and that "[a] sole proprietorship does not, of course, possess any juridical Same; Same; Petitioner failed to prove President of herein corporation clothe with
personality separate and apart from the personality of the owner of the enterprise and the apparent authority to constract with it.lt was incumbent upon the petitioner to prove that
personality of the persons acting in the name of such proprietorship," We hold and declare indeed the private respondent had clothed Mr. Maglana with the apparent power to execute
that Yao Ka Sin should be deemed as the plaintiff in Civil Case No. 5064 and the petitioner in Exhibit "A" or any similar contract. This could have been easily done by evidence of similar
the instant case. acts executed either in its favor or in favor of other parties. Petitioner miserably failed to do
that. Upon the other hand, private respondent's evidence overwhelmingly shows that no
Corporations; Contracts; A contract signed by the President and Board Chairman contract can be signed by the president without first being approved by the Board of
without authority from the Board of Directors is void; Exceptions.While there can be no Directors; such approval may only be given after the contract passes through, at least, the
question that Mr, Maglana was an officerthe President and Chairmanof private comptroller, who is the NIDC representative, and the legal counsel.
respondent corporation at the time he signed Exhibit "A", the above provisions of said private
respondent's By-Laws do not in any way confer upon the President the authority to enter into Same; Same; Acceptance of goods and receipt therefor without protest, resulted in a
contracts for the corporation independently of the Board of Directors. That power is new transaction.The second ground is based on a wrong premise. It assumes, contrary to
exclusively lodged in the latter. Nevertheless, to expedite or facilitate the execution of the Our conclusion above, that Exhibit "A" is a valid contract binding upon the private
contract, only the Presidentand not all the members of the Board, or so much thereof as respondent. It was effectively disapproved and rejected by the Board of Directors which, at
are required for the actshall sign it for the corporation. This is the import of the words the same time, considered the amount of P243,000.00 received by Maglana as payment for
through the president in Exhibit "8-A" and the clear intent of the power of the chairman "to 10,000 bags of white cement, treated as an entirely different contract, and forthwith notified
execute and sign for and in behalf of the corporation all contracts and agreements which the petitioner of its decision that "If within ten (10) days from date hereof we will not hear from
corporation may enter into" in Exhibit "1-1". Both powers presuppose a prior act of the you but you will withdraw cement at P24.30 per bag from our plant, then we will deposit your
corporation exercised through the Board of Directors. No greater power can be implied from check of P243,000.00 dated June 7, 1973 issued by the Producers Bank of the Philippines, per

208
MATABABE FILES
Corporation Law

instruction of the Board." Petitioner received a copy of this notification and thereafter Yao Ka Sin
accepted without any protest the Delivery Receipt covering the 10,000 bags and the Official Tacloban City
Receipt for the P243,000.00. The respondent Court thus correctly ruled that petitioner had in
fact agreed to a new transaction involving only 10,000 bags of white cement. Gentlemen:

Same; Same; Option given without consideration is void.The third ground must We have the pleasure to submit hereby our firm offer to you under the following quotations,
likewise fail. Exhibit "A" being unenforceable, the option to renew it would have no leg to terms, and conditions, to wit:
stand on. The river cannot rise higher than its source. In any event, the option granted in this
case is without any consideration. Article 1324 of the Civil Code expressly provides that: 1). Commodity Prime White Cement
"When the offerer has allowed the offeree a certain period to accept, the offer may be
withdrawn at any time before acceptance by communicating such withdrawal, except when 2). Price At your option: a) P24.30 per 94 lbs. bag net, FOB Cebu City; and b) P23.30 per 94
the option is founded upon a consideration, as something paid or promised." lbs. bag net, FOB Asturias Cebu.

Actions; Sec. 8, Rule 8 of the Rules of Court on how to contest genuineness of a 3). Quality As fully specified in certificate No. 224-73 by Bureau of Public Works, Republic of
document does not apply to a person not privy thereto.lt is clear that the petitioner is not a the Philippines.
party to any of the documents attached to the private respondent's Answer. Thus, the above
quoted rule is not applicable. While the respondent Court erred in holding otherwise, the 4). Quantity Forty-five Thousand (45,000) bags at 94 lbs. net per bag withdrawable in
challenged decision must, nevertheless, stand in view of the above disquisitions on the first to guaranteed monthly quantity of Fifteen Thousand (15,000) bags minimum effective from
the third grounds of the petition. June, 1973 to August 1973.

PETITION for review from the decision of the Court of Appeals. San Diego, J. 5). Delivery Schedule Shipment be made within four (4) days upon receipt of your shipping
instruction.
The facts are stated in the opinion of the Court
6). Bag/Container a) All be made of Standard Kraft (water resistant paper, 4 ply, with
DECISION bursting strength of 220 pounds, and b) Breakage allowance additional four percent (4%)
DAVIDE, JR., J.: over the quantity of each shipment.

Assailed in this petition for review is the decision of the respondent Court of Appeals 7). Terms of Payment Down payment of PESOS: TWO HUNDRED FORTY THREE THOUSAND
in C.A.-G.R. No. 61072-R, 1 promulgated on 21 December 1979, reversing the decision 2 of the (P243,000.00) payable on the signing of this contract and the balance to be paid upon
then Court of First Instance (now Regional Trial Court) of Leyte dated 20 November 1975 in presentation of corresponding shipping documents.
Civil Case No. 5064 entitled Yao Ka Sin Trading versus Prime White Cement Corporation. It is understood that in the event of a delay in our shipment, you hold the option to discount
any price differential resulting from a lower market price vis-a-vis the contract price. In
The root of this controversy is the undated letter-offer of Constancio B. Maglana, addition, grant (sic) you the option to extend this contract until the complete delivery of Forty
President and Chairman of the Board of private respondent Prime White Cement Corporation, Five Thousand (45,000) bags of 94 lbs. each is made by us. You are also hereby granted the
hereinafter referred to as PWCC, to Yao Ka Sin Trading, hereinafter referred to as YKS, which option to renew this contract under the same price, terms and conditions.
describes itself as a business concern of single proprietorship, 3 and is represented by its
manager, Mr. Henry Yao; the letter reads as follows: Please countersign on the space provided for below as your acknowledgement and
confirmation of the above transaction. Thank You.
PRIME WHITE CEMENT CORPORATION
602 Cardinal Life Building Very truly yours,
Herran Street, Manila

209
MATABABE FILES
Corporation Law

PRIME WHITE CEMENT CORPORATION On 5 July 1973, PWCC wrote a letter (Exhibit 1) to YKS informing it of the
BY: (SGD) CONSTANCIO B. MAGLANA disapproval of Exhibit A. Pursuant, however, to its decision with respect to the 10,000 bags
President & Chairman of cement, it is issued the corresponding Delivery Order (Exhibit 4) and Official Receipt No.
0394 (Exhibit 5) for the payment of the same in the amount of P243,000.00 This is the same
CONFORME: amount received and acknowledged by Maglana in Exhibit A.

YAO KA SIN TRADING YKS accepted without protest both the Delivery and Official Receipts. While YKS
denied having received a copy of Exhibit 1, it was established that the original thereof was
BY: (SGD) HENRY YAO shown to Mr. Henry Yao; since no one would sign a receipt for it, the original was left at the
latters office and this fact was duly noted in Exhibit 1 (Exhibit l-A).
WITNESSES:
On 4 August 1973, PWCC wrote a letter (Exhibit 2) to YKS in answer to the latters 4
(SGD) T. CATINDIG (SGD) ERNESTO LIM August 1973 letter stating that it is withdrawing or taking delivery of not less than 10,000
RECEIVED from Mr. Henry Yao of Yao Ka Sin Trading, in pursuance of the above offer, the sum bags of white cement on August 6-7, 1973 at Asturias, Cebu, thru M/V Taurus. In said reply,
of Pesos: TWO HUNDRED FORTY THREE THOUSAND ONLY (P243,000.00) in the form of PWCC reminded YKS of its (PWCCs) 5 July 1973 letter (Exhibit 1) and told the latter that
Producers Bank of the Philippines Check No. C-153576 dated June 7, 1973. PWCC only committed to you and which you correspondingly paid 10,000 bags of white
cement of which 4,150 bags were already delivered to you as of August 11,
PRIME WHITE CEMENT CORPORATION 1973. 6 Unfortunately, no copy of the said 4 August 1973 letter of YKS was presented in
BY: evidence.
(SGD) CONSTANCIO B. MAGLANA
On 21 August 1973, PWCC wrote another letter (Exhibit 3) 7 to YKS in reply to the
4
President & Chairman latters letter of 15 August 1973. Enclosed in the reply was a copy of Exhibit 2. While the
records reveal that YKS received this reply also on 21 August 1973 (Exhibit 3 A), 8 it still
This letter-offer, hereinafter referred to as Exhibit A, was prepared, typed and denied having received it. Likewise, no copy of the so-called 15 August 1973 letter was
signed on 7 June 1973 in the office of Mr. Teodoro Catindig, Senior Vice-President of the presented in evidence.
Consolidated Bank and Trust Corporation (Solid Bank). 5 On 10 September 1973, YKS, through Henry Yao, wrote a letter 9 to PWCC as a follow-
up to the letter of 15 August 1973; YKS insisted on the delivery of 45,030 bags of white
The principal issue raised in this case is whether or not the aforesaid letter-offer, as cement. 10
accepted by YKS, is a contract that binds the PWCC. The trial court rule in favor of the
petitioner, but the respondent Court held otherwise. On 12 September 1973, Henry Yao sent a letter (Exhibit G) to PWCC calling the
latters attention to the statement of delivery dated 24 August 1973, particularly the price
The records disclose the following material operative facts: change from P23.30 to P24.30 per 94 lbs. bag net FOB Asturias, Cebu. 11

In its meeting in Cebu City on 30 June 1973, or twenty-three (23) days after the On 2 November 1973, YKS sent a telegram (Exhibit C) 12 to PWCC insisting on the
signing of Exhibit A, the Board of Directors of PWCC disapproved the same; the rejection is full compliance with the terms of Exhibit A and informing the latter that it is exercising the
evidenced by the following Minutes (Exhibit 10): the 10,000 bags of white cement sold to option therein stipulated.
Yao Ka Sin Trading is sold not because of the alleged letter-contract adhered to by them, but
must be understood as a new and separate contract, and has in no way to do with the letter- On 3 November 1973, YKS sent to PWCC a letter (Exhibit D) as a follow-up to the 2
offer which they (sic) as consummated is by this resolution totally disapproved and is November 1973 telegram, but this was returned to sender as unclaimed. 13
unacceptable to the corporation.
As of 7 December 1973, PWCC had delivered only 9,775 bags of white cement.

210
MATABABE FILES
Corporation Law

corporation should meet the approval of the NIDC and/or the PNB Board because of an
On 9 February 1974, YKS wrote PWCC a letter (Exhibit H) requesting, for the last exposure and financial involvement of around P10 million therein. 23
time, compliance by the latter with its obligation under
Exhibit A. 14 During the trial, PWCC presented evidence to prove that Exhibit A is not binding
upon it because Mr. Maglana was not authorized to make the offer and sign the contract in
On 27 February 1974, PWCC sent an answer (Exhibit 7) to the aforementioned behalf of the corporation. Per its By-Laws (Exhibit 8), only the Board of Directors has the
letter of 9 February 1974; PWCC reiterated the unenforceability of Exhibit A. 15 power . . . (7) To enter into (sic) agreement or contract of any kind with any person in the
name and for and in behalf of the corporation through its President, subject only to the
On 4 March 1974, YKS filed with the then Court of First Instance of Leyte a complaint declared objects and purpose of the corporation and the existing provisions of law. 24 Among
for Specific Performance with Damages against PWCC. The complaint 16 was based on Exhibit the powers of the President is to operate and conduct the business of the corporation
A and was docketed as Civil Case No. 5064. according to his own judgment and discretion, whenever the same is not expressly limited by
In its Answer with Counterclaim 17 filed on 1 July 1974, PWCC denied under oath the material such orders, directives or resolutions. 25 Per standard practice of the corporation, contracts
averments in the complaint and alleged that: (a) YKS has no legal personality to sue having should first pass through the marketing and intelligence unit before they are finalized.
no legal personality even by fiction to represent itself; (b) Mr. Maglana, its President and Because of its interest in the PWCC, the NIDC, through its comptroller, goes over contracts
Chairman, was lured into signing Exhibit A; (c) such signing was subject to the condition involving funds of and white cement produced by the PWCC. Finally, among the duties of its
that Exhibit A be approved by the Board of Directors of PWCC, as corporate commitments legal counsel is to review proposed contracts before they are submitted to the Board. While
are made through it; (d) the latter disapproved it, hence Exhibit A was never consummated the president. may be tasked with the preparation of a contract, it must first pass through the
and is not enforceable against PWCC; (e) it agreed to sell 10,000 bags of white cement, not legal counsel and the comptroller of the corporation. 26
under Exhibit A, but under a separate contract prepared by the Board; (f) the rejection by
the Board of Exhibit A was made known to YKS through various letters sent to it, copies of On 20 November 1975, after trial on the merits, the court handed down its decision in
which were attached to the Answer as Annexes 1, 2 and 3; 18 (g) YKS knew, per Delivery favor of herein petitioner, the dispositive portion of which reads:
Order 19 and Official Receipt 20 issued by PWCC, that only 10,000 bags were sold to it without
any terms or conditions, at P24.30 per bag FOB Asturias, Cebu; (h) YKS is solely to blame for WHEREFORE, in view of the foregoing, judgment is hereby rendered:
the failure to take complete delivery of 10,000 bags for it did not send its boat or truck to
PWCCs plant; and (i) YKS has, therefore, no cause of action. (1) Ordering defendant: to complete the delivery of 45,000 bags of prime white cement at 94
lbs. net per bag at the price agreed, with a breakage allowance of empty bags at 4% over the
In its Counterclaim, PWCC asks for moral damages in the amount of not less than quantity agreed;
P10,000.00, exemplary damages in the sum of P500,000.00 and attorneys fees in the sum of
P10,000.00. (2) Ordering defendant to pay P50,000.00, as moral damages; P5,000.00 as exemplary
damages; P3,000.00 as attorneys fees; and the costs of these proceedings.
On 24 July 1974, YKS filed its Answer to the Counterclaim. 21
SO ORDERED. 27
22
Issues having been joined, the trial court conducted a pre-trial. On that occasion,
the parties admitted that according to the By-Laws of PWCC, the Chairman of the Board, who In disregarding PWCCs theory, the trial court interpreted the provision of the By-
is also the President of the corporation, has the power to execute and sign, for and in behalf Laws granting its Board of Directors the power to enter into an agreement or contract of
of the corporation, all contracts or agreements which the corporation enters into, subject to any kind with any person through the President, to mean that the latter may enter into such
the qualification that all the presidents actuations, prior to and after he had signed and contract or agreement at any time and that the same is not subject to the ratification of the
executed said contracts, shall be given to the board of directors of defendant Corporation. board of directors but subject only to the declared objects and purpose of the corporation
Furthermore, it was likewise stated for the record that the corporation is a semi-subsidiary of and existing laws. It then concluded:
the government because of the NIDC participation in the same, and that all contracts of the

211
MATABABE FILES
Corporation Law

It is obvious therefore, that it is not the whole membership of the board of directors
who actually enters into any contract with any person in the name and for and in behalf of the I
corporation, but only its president. It is likewise crystal clear that this automatic THE TRIAL COURT ERRED IN HOLDING THAT EXHIBIT A IS A VALID CONTRACT OR
representation of the board by the president is limited only by the declared objects and PLAINTIFF CAN CLAIM THAT THE PROPOSED LETTER-CONTRACT, EXHIBIT A IS LEGALLY
purpose of the corporation and existing provisions of law. 28 ENFORCEABLE, AS THE SAME IS A MERE UNACCEPTED PROPOSAL, NOT HAVING BEEN
PREVIOUSLY AUTHORIZED TO BE ENTERED INTO OR LATER ON RATIFIED BY THE
It likewise interpreted the provision on the power of the president to operate and DEFENDANTS BOARD OF DIRECTORS; IN FACT EXHIBIT A WAS TOTALLY REJECTED AND
conduct the business of the corporation according to the orders, directives or resolutions of DISAPPROVED IN TOTO BY THE DEFENDANTS BOARD OF DIRECTORS IN CLEAR, PLAIN
the board of directors and according to his own judgment and discretion whenever the same LANGUAGE AND DULY INFORMED AND TRANSMITTED TO PLAINTIFF.
is not expressly limited by such orders, directives and resolutions, to mean that the president
can operate and conduct the business of the corporation according to his own judgment and II
discretion as long as it is not expressly limited by the orders, directives or resolutions of the THE TRIAL COURT ERRED IN HOLDING THAT PLAINTIFF CAN LEGALLY UTILIZE THE COURTS
board of directors. 29 The trial court found no evidence that the board had set a prior AS THE FORUM TO GIVE LIFE AND VALIDITY TO A TOTALLY UNENFORCEABLE OR NON-
limitation upon the exercise of such judgment and discretion; it further ruled that the By- EXISTING CONTRACT.
Laws, does not require that Exhibit A be approved by the Board of Directors. Finally, in the III
light of the Chairmans power to execute and sign for and in behalf of the corporation all THE TRIAL COURT ERRED IN ALLOWING YAO KA SIN TO IMPUGN AND CONTRADICT HIS VERY
contracts or agreements which the corporation may enter into (Exhibit I-1), it concluded OWN ACTUATIONS AND REPUDIATE HIS ACCEPTANCE AND RECEIPTS OF BENEFITS FROM
that Mr. Maglana merely followed the By-Laws presumably both as president and chairman THE COUNTER-OFFER OF DEFENDANT FOR 10,000 BAGS OF CEMENT ONLY, UNDER THE
of the board thereof. 30 Hence, Exhibit A was validly entered into by Maglana and thus PRICE, TERMS AND CONDITIONS TOTALLY FOREIGN TO AND WHOLLY DIFFERENT FROM
binds the corporation. THOSE WHICH APPEAR IN EXHIBIT A.

The trial court, however, ruled that the option to sell is not valid because it is not IV
supported by any consideration distinct from the price; it was exercised before compliance THE TRIAL COURT ERRED IN DISMISSING DEFENDANTS COUNTER-CLAIMS AS THE SAME ARE
with the original contract by PWCC; and the repudiation of the original contract by PWCC was DULY SUPPORTED BY CLEAR AND INDUBITABLE EVIDENCE. 32
deemed a withdrawal of the option before acceptance by the petitioner.
In its decision 33 promulgated on 21 December 1979, the respondent Court reversed
Both parties appealed from the said decision to the respondent Court of Appeals the decision of the trial court, thus:
before which petitioner presented the following Assignment of Errors:
WHEREFORE, the judgment appealed from is REVERSED and set aside, Plaintiffs complaint is
I dismissed with costs. Plaintiff is ordered to pay defendant corporation P25,000.00 exemplary
THE TRIAL COURT ERRED IN HOLDING THAT THE OPTION TO RENEW THE CONTRACT OF damages, and P10,000.00 attorneys fees.
SALE IS NOT ENFORCEABLE BECAUSE THE OPTION WAS MADE EVEN BEFORE THE
COMPLIANCE OF (sic) THE ORIGINAL CONTRACT BY DEFENDANT AND THAT DEFENDANTS SO ORDERED.
PROMISE TO SELL IS NOT SUPPORTED BY ANY CONSIDERATION DISTINCT FROM THE PRICE.
Such conclusion is based on its findings, to wit:
II
THE TRIAL COURT ERRED IN NOT AWARDING TO THE PLAINTIFF ACTUAL DAMAGES, Before resolving the issue, it is helpful to bring out some preliminary facts. First, the
SUFFICIENT EXEMPLARY DAMAGES AND ATTORNEYS FEES AS ALLEGED IN THE COMPLAINT defendant corporation is supervised and principally financed by the National Investment and
AND PROVEN DURING THE TRIAL. 31 Development Corporation (NIDC), a subsidiary investment of the Philippine National Bank
(PNB), with cash financial exposure of some P10,000,000.00. PNB is a government financial
while the private respondent cited the following errors: institution whose Board is chairmaned (sic) by the Minister of National Defense. This fact is

212
MATABABE FILES
Corporation Law

very material to the issue of whether defendant corporations president can bind the 1403 (1), Civil Code). And because Maglana was not authorized by the Board of Directors of
corporation with his own act. defendant corporation nor was his, actuation ratified by the Board, the agreement is
unenforceable (Art. 1403 (1), Civil Code; Raquiza et al. vs. Lilles et al., 13 CA Rep. 343; Gana vs.
Second, for failure to deny under oath the following actionable documents in support Archbishop of Manila, 43 O-G. 3224).
of defendants counterclaim:
While it may be true that Maglana is President of defendant corporation nowhere in
1. The resolution contained in defendants letter to plaintiff dated July 5, 1973, on the the Articles of Incorporation nor in the By-Laws of said corporation was he empowered to
10,000 bags of white cement delivered to plaintiff was not by reason of the letter contract, enter into any contract all by himself and bind the corporation without first securing the
Exhibit A, which was totally disapproved by defendant corporations board of directors, authority and consent of the Board of Directors. Whatever authority Maglana may have must
clearly stating that If within ten (10) days from date hereof, we will not hear from you but be derived from the Board of Directors of defendant corporation. A corporate officers power
you will withdraw cement at P24.30 per bag from our plant, then we will deposit your check as an agent must be sought from the law, the articles of incorporation and the By-Laws or
of P243,000.00 dated June 7, 1973 issued by the Producers Bank of the Philippines, per from a resolution of the Board (Vicente vs. Geraldez, 52 SCRA 227, Board of Liquidators vs.
instruction of the Board. (Annex I to defendants Answer). Kalaw, 20 SCRA 987).

2. Letter of defendant to plaintiff dated August 4, 1973 that defendant only It clearly results from the foregoing that the judgment appealed from is untenable.
committed to you and which you accordingly paid 10,000 bags of white cement of which 4,150 Having no cause of action against defendant corporation, plaintiff is not entitled to any relief.
bags were already delivered to you as of August 1, 1973 (Annex 2 of defendants Answer). We see no justification, therefore, for the court a quos awards in its favor. . . . 34
3. Letter dated August 21, 1973 to plaintiff reiterating defendants letter of August 4,
1973 (Annex 3 to defendants Answer). Its motion for reconsideration having been denied by the respondent Court in its
resolution 35 dated 15 April 1980, petitioner filed the instant petition based on the following
4. Letter to stores dated August 21, 1973, grounds:

5. Receipt from plaintiff (sic) P243,000.00 in payment of 10,000 bags of white cement 1. That the contract (Exh. A) entered into by the President and Chairman of the
at P24.30 per bag (Annex 5, to defendants Answer) plaintiff is deemed to have admitted, Board of Directors Constancio B. Maglana in behalf of the respondent corporation binds the
not only the due execution and genuineness (sic) of said documents, (Rule 8 Sec. 8, Rules of said corporation.
Court) but also the allegations therein (Rule 9, Sec. 1, Rules of Court). All of the foregoing
documents tend to prove that the letter-offer, Exhibit A, was rejected by defendant 2. That the contract (Exh. A) was never novated nor superseded (sic) by a
corporations Board of Directors and plaintiff was duly notified thereof and that the subsequent contract.
P243,000.00 check was considered by both parties as payment of the 10,000 bags of cement
under a separate transaction. As proof of which plaintiff did not complain nor protest until 3. That the option to renew the contract as contained in Exhibit A is enforceable.
February 9, 1974, when he threatened legal action.
4. That Sec. 8, Rule 8 of the Rules of Court only applies when the adverse party
Third, Maglanas signing the letter-offer prepared for him in the Solidbank was made appear (sic) to be a party to the instrument but not to one who is not a party to the
clearly upon the condition that it was subject to the approval of the board of directors of instrument and Sec. 1, Rule 9 of the said Rules with regards (sic) to denying under oath refers
defendant corporation. We find consistency herein because according to the Corporation only to allegations of
Law, and the By-Laws of defendant corporation, all corporate commitments and business are usury. 36
conducted by, and contracts entered into through, the express authority of the Board of
Directors (Sec. 28. Corp. Law, Exh I or 8). We gave due course 37 to the petition after private respondent filed its
Comment 38 and required the parties to submit simultaneously their Memoranda, which the
Fourth, What Henry Yao and Maglana agreed upon as embodied in Exhibit A, parties subsequently complied with. 39
insofar as defendant corporation is concerned, was an unauthorized contract (Arts. 1317 and

213
MATABABE FILES
Corporation Law

Before going any further, this Court must first resolve an issue which, although raised acting in the name of such proprietorship, 47 We hold and declare that Yao Ka Sin should be
in the Answer of private respondent, was neither pursued in its appeal before the respondent deemed as the plaintiff in Civil Case No. 5064 and the petitioner in the instant case. As this
Court nor in its Comment and Memorandum in this case. It also eluded the attention of the Court stated nearly eighty (80) years ago in Alonso vs. Villamor: 48
trial court and the respondent Court. The issue, which is of paramount importance, concerns
the lack of capacity of plaintiff/petitioner to sue. In the caption of both the complaint and the No one has been misled by the error in the name of the party plaintiff. If we should
instant petition, the plaintiff and the petitioner, respectively, is: by reason of this error send this case back for amendment and new trial, there would be on
the retrial the same complaint, the same answer, the same defense, the same interests, the
YAO KA SIN TRADING, same witnesses, and the same evidence. The name of the plaintiff would constitute the only
owned and operated by difference between the old trial and the new. In our judgment there is not enough in a name
YAO KA SIN. 40 to justify such action.

and is described in the body thereof as a business concern of single proprietorship And now to the merits of the petition.
owned and operated by Yao Ka Sin. 41 In the body of the petition, it is described as a single
proprietorship business concern. 42 It also appears that, as gathered from the decision of the The respondent Court correctly ruled that Exhibit A is not binding upon the private
trial court, no Yao Ka Sin testified. Instead, one Henry Yao took the witness stand and testified respondent. Mr. Maglana, its President and Chairman, was not empowered to execute it.
that he is the manager of Yao Ka Sin Trading and it was in representation of the plaintiff Petitioner, on the other hand, maintains that it is a valid contract because the Maglana has
that he signed Exhibit A43 Under Section 1, Rule 3 of the Rules of Court, only natural or the power to enter into contracts for the corporation as implied from the following provisions
juridical persons or entities authorized by law may be parties in a civil action. In Juasing of the By-Laws of private respondent:
Hardware vs. Mendoza, 44 this Court held that a single proprietorship is neither a natural
person nor a juridical person under Article 44 of the Civil Code; it is not an entity authorized by a) The power of the Board of Directors to . . . enter into (sic) agreement or contract
law to bring suit in court: of any kind with any person in the name and for and in behalf of the corporation through its
President, subject only to the declared objects and purpose of the corporation and the
The law merely recognizes the existence of a sole proprietorship as a form of existing provisions of law. (Exhibit 8-A); and
business organization conducted for profit by a single individual, and requires the proprietor
or owner thereof to secure licenses and permits, register the business name, and pair taxes to b) The power of the Chairman of the Board of Directors to execute and sign, for and
the national government. It does not vest juridical or legal personality upon the sole in behalf of the corporation, all contracts or agreements which the corporation may enter
proprietorship nor empower it to file or defend an action in court. 45 into (Exhibit I-1).

Accordingly, the proper party plaintiff/petitioner should be YAO KA SIN. 46 And even admitting, for the sake of argument, that Mr. Maglana was not so
authorized under the By-Laws, the private respondent, pursuant to the doctrine laid down by
The complaint then should have been amended to implead Yao Ka Sin as plaintiff in this Court in Francisco vs. Government Service Insurance System 49 and Board of Liquidators vs.
substitution of Yao Ka Sin Trading. However, it is now too late in the history of this case to Kalaw, 50 is still bound by his act for clothing him with apparent authority.
dismiss this petition and, in effect, nullify all proceedings had before the trial court and the
respondent Court on the sole ground of petitioners lack of capacity to sue. Considering that We are not persuaded.
private respondent did not pursue this issue before the respondent Court and this Court; that,
as We held in Juasing, the defect is merely formal and not substantial, and an amendment to Since a corporation, such as the private respondent, can act only through its officers
cure such defect is expressly authorized by Section 4, Rule 10 of the Rules of Court which and agents, all acts within the powers of said corporation may be performed by agents of its
provides that [a] defect in the designation of the parties may be summarily corrected at any selection; and, except so far as limitations or restrictions may be imposed by special charter,
stage of the action provided no prejudice is caused thereby to the adverse party; and that by-law, or statutory provisions, the same general principles of law which govern the relation
[a] sole proprietorship does not, of course, possess any juridical personality separate and of agency for a natural person govern the officer or agent of a corporation, of whatever
apart from the personality of the owner of the enterprise and the personality of the persons status or rank, in respect to his power to act for the corporation; and agents when once

214
MATABABE FILES
Corporation Law

appointed, or members acting in their stead, are subject to the same rules, liabilities and he acts within the scope of an apparent authority with which the corporation has clothed him
incapacities as are agents of individuals and private persons. 51 Moreover, . . . a corporate by holding him out or permitting him to appear as having such authority, the corporation is
officer or agent may represent and bind the corporation in transactions with third persons to bound thereby in favor of a person who deals with him in good faith in reliance on such
the extent that authority to do so has been conferred upon him, and this includes powers apparent authority, as where an officer is allowed to exercise a particular authority with
which have been intentionally conferred, and also such powers as, in the usual course of the respect to the business, or a particular branch of it, continuously and publicly, for a
particular business, are incidental to, or may be implied from, the powers intentionally considerable time. 54 Also, if a private corporation intentionally or negligently clothes its
conferred, powers added by custom and usage, as usually pertaining to the particular officer officers or agents with apparent power to perform acts for it, the corporation will be
or agent, and such apparent powers as the corporation has caused persons dealing with the estopped to deny that such apparent authority in real, as to innocent third persons dealing in
officer or agent to believe that it has conferred. 52 good faith with such officers or agents. 55 This apparent authority may result from (1) the
general manner, by which the corporation holds out an officer or agent as having power to
While there can be no question that Mr. Maglana was an officer the President and act or, in other words, the apparent authority with which it clothes him to act in general or (2)
Chairman of private respondent corporation at the time he signed Exhibit A, the above acquiescence in his acts of a particular nature, with actual or constructive knowledge thereof,
provisions of said private respondents By-Laws do not in any way confer upon the President whether within or without the scope of his ordinary powers. 56
the authority to enter into contracts for the corporation independently, of the Board of
Directors. That power is exclusively lodged in the latter. Nevertheless, to expedite or facilitate It was incumbent upon the petitioner to prove that indeed the private respondent
the execution of the contract, only the President and not all the members of the Board, or had clothed Mr. Maglana with the apparent power to execute Exhibit A or any similar
so much thereof as are required for the act shall sign it for the corporation. This is the contract. This could have been easily done by evidence of similar acts executed either in its
import of the words through the president in Exhibit 8-A and the clear intent of the power favor or in favor of other parties. Petitioner miserably failed to do that. Upon the other hand,
of the chairman to execute and sign for and in behalf of the corporation all contracts and private respondents evidence overwhelmingly shows that no contract can be signed by the
agreements which the corporation may enter into in Exhibit I-1. Both powers presuppose president without first being approved by the Board of Directors; such approval may only be
a prior act of the corporation exercised through the Board of Directors. No greater power can given after the contract passes through, at least, the comptroller, who is the NIDC
be implied from such express, but limited, delegated authority. Neither can it be logically representative, and the legal counsel.
claimed that any power greater than that expressly conferred is inherent in Mr. Maglanas
position as president and chairman of the corporation. The cases then of Francisco vs. GSIS and Board of Liquidators vs. Kalaw are hopelessly
unavailing to the petitioner. In said cases, this Court found sufficient evidence, based on the
Although there is authority that if the president is given general control and conduct and actuations of the corporations concerned, of apparent authority conferred upon
supervision over the affairs of the corporation, it will be presumed that he has authority to the officer involved which bound the corporations on the basis of ratification. In the first case,
make contract and do acts within the course of its ordinary business, 53 We find such it was established that the offer of compromise made by plaintiff in the letter, Exhibit A,
inapplicable in this case. We note that the private corporation has a general manager who, was validly accepted by the GSIS. The terms of the trial offer were clear, and over the
under its By-Laws has, inter alia, the following powers: (a) to have the active and direct signature of defendants general manager Rodolfo Andal, plaintiff was informed
management of the business and operation of the corporation, conducting the same telegraphically that her proposal had been accepted. It was sent by the GSIS Board Secretary
accordingly to the order, directives or resolutions of the Board of Directors or of the and defendant did not disown the same. Moreover, in a letter remitting the payment of
president. It goes without saying then that Mr. Maglana did not have a direct and active P30,000 advanced by her father, plaintiff quoted verbatim the telegram of acceptance. This
hand in the management of the business and operations of the corporation. Besides, no was in itself notice to the corporation of the terms of the allegedly unauthorized telegram.
evidence was adduced to show that Mr. Maglana had, in the past, entered into contracts Notwithstanding this notice, GSIS pocketed the amount and kept silent about the telegram.
similar to that of Exhibit A either with the petitioner or with other parties. This Court then ruled that:

Petitioners last refuge then is his alternative proposition, namely, that private This silence, taken together with the unconditional acceptance of three other
respondent had clothed Mr. Maglana with the apparent power to act for it and had caused subsequent remittances from plaintiff, constitutes in itself a binding ratification of the original
persons dealing with it to believe that he was conferred with such power. The rule is of course agreement (Civil Code, Art. 1393).
settled that [a]lthough an officer or agent acts without, or in excess of, his actual authority if

215
MATABABE FILES
Corporation Law

Art. 1393. Ratification may be effected expressly or tacitly it is understood that there event, the option granted in. this case is without any consideration Article 1324 of the Civil
is a tacit ratification if, with knowledge of the reason which renders the contract voidable and Code expressly provides that:
such reason having ceased, the person who has a right to invoke it should execute an act
which necessarily implies an intention to waive his right When the offerer has allowed the offeree a certain period to accept, the offer may be
withdrawn at any time before acceptance by communicating such withdrawal, except when
In the second case, this Court found: the option is founded upon a consideration, as something paid or promised.

In the case at bar, the practice of the corporation has been to allow its general while Article 1749 of the same Code provides:
manager to negotiate and execute contracts in its copra trading activities for and in
NACOCOs behalf without prior board approval. If the by-laws were to be literally followed, A promise to buy and sell a determinate thing for a price certain is reciprocally
the board should give its stamp of prior approval on all corporate contracts. But that board demandable.
itself, by its acts and through acquiescence, practically laid aside the by-laws requirement of
prior approval. An accepted unilateral promise to buy or to sell a determinate thing for a price
certain is binding upon the promissor if the promise is supported by a consideration distinct
Under the given circumstances, the Kalaw contracts are valid corporate acts. from the price.

The inevitable conclusion then is that Exhibit A is an unenforceable contract under Accordingly, even if it were accepted, it cannot validly bind the private respondent. 58
Article 1317 of the Civil Code which provides as follows:
The fourth ground is, however, meritorious.
Art. 1317. No one may contract in the name of another without being authorized by
the latter, or unless he has by law a right to represent him. Section 8, Rule 8 of the Rules of Court provides:

A contract entered into in the name of another by one who has no authority or legal Sec. 8. How to contest genuineness of such documents When an action or defense is
representation, or who has acted beyond his powers, shall be unenforceable, unless it is founded upon a written instrument, copied in or attached in the corresponding pleading as
ratified, expressly or impliedly, by the person on whose behalf it, has been execrated, before provided in the preceding section, the genuineness and due execution of the instrument shall
it is revoked by the other contracting party. be deemed admitted unless the adverse party, under oath, specifically denies them, and sets
The second ground is based on a wrong premise. It assumes, contrary to Our forth what he claims to be the facts; but this provision does not apply when the adverse party
conclusion above, that Exhibit A is a valid contract binding upon the private respondent. It does not appear, to be a party to the instrument or when compliance with an order for an
was effectively disapproved and rejected by the Board of Directors which, at the same time, inspection of the original instrument is refused.
considered the amount of P243,000.00 received Mr. Maglana as payment for 10,000 bags of
white cement, treated as an entirely different contract, and forthwith notified petitioner of its It is clear that the petitioner is not a party to any of the documents attached to the
decision that If within ten (10) days from date hereof we will not hear from you but you will private respondents Answer. Thus, the above quoted rule is not applicable. 59 While the
withdraw cement at P24.30 per bag from our plant, then we will deposit your check of respondent Court, erred in holding otherwise, the challenged decision must, nevertheless,
P243,000.00 dated June 7, 1973 issued by the Producers Bank of the Philippines, per stand in view of the above disquisitions on the first to the third grounds of the petition.
instruction of the Board. 57 Petitioner received the copy of this notification and thereafter
accepted without any protest the Delivery Receipt covering the 10,000 bags and the Official WHEREFORE, judgment is hereby rendered AFFIRMING the decision of respondent
Receipt for the P243,000.00. The respondent Court thus correctly ruled that petitioner had in Court of Appeals in C.A. G.R. No. 61072-R promulgated on 21 December 1979.
fact agreed to a new transaction involving only 10,000 bags of white cement.

The third ground must likewise fail. Exhibit A being unenforceable, the option to
renew it would have no leg to stand on. The river cannot rise higher than its source. In any

216
MATABABE FILES
Corporation Law

Republic of the Philippines unequivocal terms as novation is never presumed (Mondragon v. Intermediate Appellate
SUPREME COURT Court, G.R. No. 71889, April 17, 1990; Caneda, Jr. v. Court of Appeals, G.R. No. 81322, February
Manila 5, 1990). Secondly, the old and the new obligations must be incompatible on every point. The
SECOND DIVISION test of incompatibility is whether or not the two obligations can stand together, each one
having its independent existence. If they cannot, they are incompatible and the latter
G.R. No. 71694 August 16, 1991 obligation novates the first (Mondragon v. Intermediate Appellate Court, supra; Caneda, Jr. v.
Court of Appeals, supra). In the instant case, there was no express agreement that BA
NYCO SALES CORPORATION, petitioner, Finances acceptance of the SBTC check will discharge Nyco from liability. Neither is there
vs. incompatibility because both checks were given precisely to terminate a single obligation
BA FINANCE CORPORATION, JUDGE ROSALIO A. DE LEONREGIONAL TRIAL COURT, BR. II, arising from Nycos sale of credit to BA Finance. As novation speaks of two distinct
INTERMEDIATE APPELLATE COURT, FIRST CIVIL CASES DIVISION, respondents. obligations, such is inapplicable to this case.

Civil Law; Assignment; Case at bar; According to Art. 1628 of the Civil Code, the Same; Corporation Law; Estoppel in pais.Nyco is estopped from denying Rufino
assignor-vendor warrants both the credit itself and the person of the debtor, if so stipulated, as Yaos authority as far as the latters transactions with BA Finance are concerned.
in the case at bar.An assignment of credit is the process of transferring the right of the
assignor to the assignee, who would then be allowed to proceed against the debtor. It may PETITION for certiorari to review the decision of the then Intermediate Appellate
be done either gratuitously or onerously, in which case, the assignment has an effect similar Court. Caquioa, J.
to that of a sale. According to Article 1628 of the Civil Code, the assignor-vendor warrants
both the credit itself (its existence and legality) and the person of the debtor (his solvency), if The facts are stated in the opinion of the Court.
so stipulated, as in the case at bar. Consequently, if there be any breach of the above
warranties, the assignor-vendor should be held answerable therefor. There is no question ABC Law Offices for petitioner.
then that the assignor-vendor is indeed liable for the invalidity of whatever he assigned to the Valera, Urmeneta & Associates for private respondent.
assignee-vendee.
PARAS, J.:
Same; Same; Negotiable Instruments Law; Notice of Dishonor; The dishonor of an
assigned check simply stresses its liability and the failure to give a notice of dishonor will not In this petition for review on certiorari, petitioner challenges the April 22, 1985
discharge it from such liability.Nycos pretension that it had not been notified of the fact of decision * and the July 16, 1985 resolution * of the then Intermediate Appellate Court in AC-
dishonor is belied not only by the formal demand letter but also by the findings of the trial G.R. CV No. 02553 entitled "BA Finance Corporation v. Nyco Sales Corporation, et al." which
court that Rufino Yao of Nyco and the Fernandez Brothers of Sanshell had frequent contacts affirmed with modification the July 20, 1983 decision ** of the Regional Trial Court, National
before, during and after the dishonor (Rollo, p. 40). More importantly, it fails to realize that Capital Region, Manila, Branch II in the same case docketed as Civil Case No. 125909 ordering
for as long as the credit remains outstanding, it shall continue to be liable to BA Finance as its petitioner to pay respondent the amount of P60,000.00 as principal obligation plus
assignor. The dishonor of an assigned check simply stresses its liability and the failure to give a corresponding interest, the sum of P10,000.00 as and for, attomey's fees and 1/3 of the costs
notice of dishonor will not discharge it from such liability. This is because the cause of action of suit.
stems from the breach of the warranties embodied in the Deed of Assignment, and not from
the dishonoring of the check alone (See Art. 1628, Civil Code). It appears on record that petitioner Nyco Sales Corporation (hereinafter referred to
as Nyco) whose president and general manager is Rufino Yao, is engaged in the business of
Same; Novation; In order to produce the effect of extinguishing obligation, novation selling construction materials with principal office in Davao City. Sometime in 1978, the
must (a) be explicitly stated and declared in unequivocal terms, and (b) the old and the new brothers Santiago and Renato Fernandez (hereinafter referred to as the Fernandezes), both
obligations must be incompatible on every point.There are only two ways which indicate the acting in behalf of Sanshell Corporation, approached Rufino Yao for credit accommodation.
presence of novation and thereby produce the effect of extinguishing an obligation by They requested Nyco, thru Yao, to grant Sanshell discounting privileges which Nyco had with
another which substitutes the same. First, novation must be explicitly stated and declared in BA Finance Corporation (hereinafter referred to as BA Finance). Yao apparently acquiesced,

217
MATABABE FILES
Corporation Law

hence on or about November 15, 1978, the Fernandezes went to Yao for the purpose of
discounting Sanshell's post-dated check which was a BPI-Davao Branch Check No. 499648 The cross-claim of defendant Nyco Sales Corporation against
dated February 17, 1979 for the amount of P60,000.00. The said check was payable to Nyco. codefendants Santiago B. Fernandez and Renato B. Fernandez is hereby
Following the discounting process agreed upon, Nyco, thru Yao, endorsed the check in favor denied, as there is no showing that Nyco's Answer with cross-claim dated
of BA Finance. Thereafter, BA Finance issued a check payable to Nyco which endorsed it in May 29, 1980 was ever received by said Fernandez brothers, even as it is
favor of Sanshell. Sanshell then made use of and/or negotiated the check. Accompanying the noted that the latter have not been declared in default with respect to said
exchange of checks was a Deed of Assignment executed by Nyco in favor of BA Finance with cross-claim, nor were evidence adduced in connection therewith.
the conformity of Sanshell. Nyco was represented by Rufino Yao, while Sanshell was
represented by the Fernandez brothers. Under the said Deed, the subject of the discounting As to the would-be litigant Sanshell Construction and Development
was the aforecited check (Rollo, pp- 26-28). At the back thereof and of every deed of Corporation, defendant Nyco Sales Corporation did not properly implead
assignment was the Continuing Suretyship Agreement whereby the Fernandezes said corporation which should have been by way of a third-party complaint
unconditionally guaranteed to BA Finance the full, faithful and prompt payment and discharge instead of a mere cross-claim. The same observations are noted as regard
of any and all indebtedness of Nyco (Ibid., pp. 36, 46). The BPI check, however, was this cross-claim against Sanshell as those made with respect to the
dishonored by the drawee bank upon presentment for payment. BA Finance immediately Fernandez brothers.
reported the matter to the Fernandezes who thereupon issued a substitute check dated
February 19,1979 for the same amount in favor of BA Finance. It was a Security Bank and Trust SO ORDERED.
Company check bearing the number 183157, which was again dishonored when it was
presented for payment. Despite repeated demands, Nyco and the Fernandezes failed to settle On appeal, the appellate court also upheld BA Finance but modified the lower court's
the obligation with BA Finance, thus prompting the latter to institute an action in court (Ibid., decision by ordering that the interest should run from February 19, 1979 until paid and not
p 28). Nyco and the Fernandezes, despite having been served with summons and copies of from February 1, 1979. Nyco's subsequent motion for reconsideration was denied (Ibid., pp.
the complaint, failed to file their answer and were consequently declared in default. On May 33, 62). Hence, the present recourse.
16, 1980, the lower court ruled in favor of BA Finance ordering them to pay the former jointly
and severally, the sum of P65,536.67 plus 14% interest per annum from July 1, 1979 and The crux of the controversy is whether or not the assignor is liable to its assignee for
attorney's fees in the amount of P3, 000. 00 as well as the costs of suit (Rollo, pp. 51-52). its dishonored checks.
Nyco, however, moved to set aside the order of default, to have its answer admitted and to
be able to implead Sanshell. The prayer was granted through an order dated June 23, 1980, For its defense, Nyco anchors its arguments on the following premises:
wherein the decision of the court was set aside only as regards Nyco. Trial ensued once more
until the court reached a second decision which states: a) that the appellate court erred in affirming its liability for the BPI check despite a
similar finding of liability for the SBTC check rendered by the same lower court;
WHEREFORE, judgment is hereby rendered in favor of the plaintiff
and against the defendant Nyco Sales Corporation by ordering the latter to b) that it was actually discharged of its liability over the SBTC check when BA Finance
pay the former the following: failed to give it a notice of dishonor; c) that there was novation when BA Finance accepted
the SBTC check in replacement of the BPI check; and d) that it cannot be held liable for its
1) P60,000.00 as principal obligation, plus interest thereon at the rate of 14% Presidents unauthorized acts.
per annum from February 1, 1979 until fully paid;
2) The amount of P100,000.00 as and for attorney's fees; and The petition is devoid of merit.

3) One-third (1/3) of the costs of this suit. An assignment of credit is the process of transferring the right of the assignor to the
assignee, who would then be allowed to proceed against the debtor. It may be done either
With respect to defendants Santiago and Renato Fernandez, the gratuitously or generously, in which case, the assignment has an effect similar to that of a
decision of May 16, 1980 stands. sale.

218
MATABABE FILES
Corporation Law

Intermediate Appellate Court, supra; Caneda Jr. v. Court of Appeals, supra). In the instant
According to Article 1628 of the Civil Code, the assignor-vendor warrants both the case, there was no express agreement that BA Finance's acceptance of the SBTC check will
credit itself (its existence and legality) and the person of the debtor (his solvency), if so discharge Nyco from liability. Neither is there incompatibility because both checks were given
stipulated, as in the case at bar. Consequently, if there be any breach of the above warranties, precisely to terminate a single obligation arising from Nyco's sale of credit to BA Finance. As
the assignor-vendor should be held answerable therefor. There is no question then that the novation speaks of two distinct obligations, such is inapplicable to this case.
assignor-vendor is indeed liable for the invalidity of whatever he as signed to the assignee-
vendee. Finally, Nyco disowns its President's acts claiming that it never authorized Rufino Yao
(Nyco's President) to even apply to BA Finance for credit accommodation. It supports its
Considering now the facts of the case at bar, it is beyond dispute that Nyco executed argument with the fact that it did not issue a Board resolution giving Yao such authority.
a deed of assignment in favor of BA Finance with Sanshell Corporation as the debtor-obligor. However, the very evidence on record readily belies Nyco's contention. Its corporate By-Laws
BA Finance is actually enforcing said deed and the check covered thereby is merely an clearly provide for the powers of its President, which include, inter alia, executing contracts
incidental or collateral matter. This particular check merely evidenced the credit which was and agreements, borrowing money, signing, indorsing and delivering checks, all in behalf of
actually assigned to BA Finance. Thus, the designation is immaterial as it could be any other the corporation. Furthermore, the appellate court correctly adopted the lower court's
check. Both the lower and the appellate courts recognized this and so it is utterly misplaced observation that there was already a previous transaction of discounting of checks involving
to say that Nyco is being held liable for both the BPI and the SBTC checks. It is only what is the same personalities wherein any enabling resolution from Nyco was dispensed with and
represented by the said checks that Nyco is being asked to pay. Indeed, nowhere in the yet BA Finance was able to collect from Nyco and Sanshell was able to discharge its own
dispositive parts of the decisions of the courts can it be gleaned that BA Finance may recover undertakings. Such effectively places Nyco under estoppel in pais which arises when one, by
from the two checks. his acts, representations or admissions, or by his silence when he ought to speak out,
intentionally or through culpable negligence, induces another to believe certain facts to exist
Nyco's pretension that it had not been notified of the fact of dishonor is belied not and such other rightfully relies and acts on such belief, so that he will be prejudiced if the
only by the formal demand letter but also by the findings of the trial court that Rufino Yao of former is permitted to deny the existence of such facts (Panay Electric Co., Inc. v. Court of
Nyco and the Fernandez Brothers of Sanshell had frequent contacts before, during and after Appeals, G.R. No. 81939, June 29,1989). Nyco remained silent in the course of the transaction
the dishonor (Rollo, p. 40). More importantly, it fails to realize that for as long as the credit and spoke out only later to escape liability. This cannot be countenanced. Nyco is estopped
remains outstanding, it shall continue to be liable to BA Finance as its assignor. The dishonor from denying Rufino Yao's authority as far as the latter's transactions with BA Finance are
of an assigned check simply stresses its liability and the failure to give a notice of dishonor will concerned.
not discharge it from such liability. This is because the cause of action stems from the breach
of the warranties embodied in the Deed of Assignment, and not from the dishonoring of the PREMISES CONSIDERED, the decision appealed from is AFFIRMED.
check alone (See Art. 1628, Civil Code).
SO ORDERED.
Novation is the third defense set up by petitioner Nyco. It insists that novation took
place when BA Finance accepted the SBTC check in replacement of the BPI cheek. Such is Melencio-Herrera (Chairperson), Padilla, Sarmiento and Regalado, JJ., concur.
manifestly untenable.

There are only two ways which indicate the presence of novation and thereby
produce the effect of extinguishing an obligation by another which substitutes the same.
First, novation must be explicitly stated and declared in unequivocal terms as novation is
never presumed (Mondragon v. Intermediate Appellate Court, G.R. No. 71889, April 17, 1990;
Caneda Jr. v. Court of Appeals, G.R. No. 81322, February 5, 1990). Secondly, the old and the
new obligations must be incompatible on every point. The test of incompatibility is whether
or not the two obligations can stand together, each one having its independent existence If
they cannot, they are incompatible and the latter obligation novates the first (Mondragon v.

219
MATABABE FILES
Corporation Law

Republic of the Philippines with special reference to banks, had long been recognized in this jurisdiction. Apparent
SUPREME COURT authority is derived not merely from practice. Its existence may be ascertained through 1) the
Manila general manner in which the corporation holds out an officer or agent as having the power to
THIRD DIVISION act, or in other words, the apparent authority to act in general, with which it clothes him; or 2)
the acquiescence in his acts of a particular nature, with actual or constructive knowledge
G.R. No. 148444 July 14, 2008 thereof, within or beyond the scope of his ordinary powers.

ASSOCIATED BANK (now UNITED OVERSEAS BANK [PHILS.]), Petitioner, Same; Same; Same; It is not the quantity of similar acts which establishes apparent
vs. authority, but the vesting of a corporate officer with the power to bind the corporationthe
SPOUSES RAFAEL and MONALIZA PRONSTROLLER, Respondents. third person has little or no information as to what occurs in corporate meetings; and he must
necessarily rely upon the external manifestations of corporate consent; The integrity of
Actions; Appeals; Well-settled is the rule that the findings of the RTC, as affirmed by commercial transactions can only be maintained by holding the corporation strictly to the
the appellate court, are binding on the Supreme Court.Well-settled is the rule that the liability fixed upon it by its agents in accordance with law.The authority to act for and to
findings of the RTC, as affirmed by the appellate court, are binding on this Court. In a petition bind a corporation may be presumed from acts of recognition in other instances, wherein the
for review on certiorari under Rule 45 of the Rules of Court, as in this case, this Court may not power was exercised without any objection from its board or shareholders. Undoubtedly,
review the findings of fact all over again. It must be stressed that this Court is not a trier of petitioner had previously allowed Atty. Soluta to enter into the first agreement without a
facts, and it is not its function to re-examine and weigh anew the respective evidence of the board resolution expressly authorizing him; thus, it had clothed him with apparent authority
parties. The findings of the CA are conclusive on the parties and carry even more weight when to modify the same via the second letter-agreement. It is not the quantity of similar acts
these coincide with the factual findings of the trial court, unless the factual findings are not which establishes apparent authority, but the vesting of a corporate officer with the power to
supported by the evidence on record. Petitioner failed to show why the above doctrine bind the corporation. Naturally, the third person has little or no information as to what occurs
should not be applied to the instant case. in corporate meetings; and he must necessarily rely upon the external manifestations of
corporate consent. The integrity of commercial transactions can only be maintained by
Corporation Law; Board of Directors; Doctrine of Apparent Authority; Words and holding the corporation strictly to the liability fixed upon it by its agents in accordance with
Phrases; While it is a general rule that, in the absence of authority from the board of directors, law. What transpires in the corporate board room is entirely an internal matter. Hence,
no person, not even its officers, can validly bind a corporation, the Board may validly delegate petitioner may not impute negligence on the part of the respondents in failing to find out the
some of its functions and powers to officers, committees and agents; The doctrine of scope of Atty. Solutas authority. Indeed, the public has the right to rely on the
apparent authority, with special reference to banks, had long been recognized in the trustworthiness of bank officers and their acts.
Philippine jurisdiction; Apparent authority is derived not only from practiceits existence may
be ascertained through (1) the general manner in which the corporation holds out an officer or Contracts; Mutuality of Contracts; Unilateral Rescission; Basic is the rule that a
agent as having the power to act, with which it clothes him, or, (2) the acquiescence in his acts contract constitutes the law between the parties; Parties may validly stipulate the unilateral
of a particular nature, with actual or constructive knowledge thereof, within or beyond the rescission of a contract.Basic is the rule that a contract constitutes the law between the
scope of his ordinary powers.The general rule is that, in the absence of authority from the parties. Concededly, parties may validly stipulate the unilateral rescission of a contract. This is
board of directors, no person, not even its officers, can validly bind a corporation. The power usually in the form of a stipulation granting the seller the right to forfeit installments or
and responsibility to decide whether the corporation should enter into a contract that will deposits made by the buyer in case of the latters failure to make full payment on the
bind the corporation is lodged in the board of directors. However, just as a natural person stipulated date. While the petitioner in the instant case may have the right, under the March
may authorize another to do certain acts for and on his behalf, the board may validly delegate 18 agreement, to unilaterally rescind the contract in case of respondents failure to comply
some of its functions and powers to officers, committees and agents. The authority of such with the terms of the contract, the execution of the July 14 Agreement prevented petitioner
individuals to bind the corporation is generally derived from law, corporate bylaws or from exercising the right to rescind. This is so because there was in the first place, no breach
authorization from the board, either expressly or impliedly, by habit, custom, or of contract, as the date of full payment had already been modified by the later agreement.
acquiescence, in the general course of business. The authority of a corporate officer or agent
in dealing with third persons may be actual or apparent. The doctrine of apparent authority,

220
MATABABE FILES
Corporation Law

Same; Valid Offer; It is contrary to human experience that parties would abandon timely registration of a notice of lis pendens.This registration, therefore, gives the court
their right to a contract and subject property, absent any form of protection, considering that clear authority to cancel the title of the spouses Vaca, since the sale of the subject property
they have a continuing interest over the said property and when they have the capacity to pay was made after the notice of lis pendens. Settled is the rule that the notice is not considered a
for the same.Neither can the July 14, 1993 agreement be considered abandoned by collateral attack on the title, for the indefeasibility of the title shall not be used to defraud
respondents act of making a new offer, which was unfortunately rejected by petitioner. A another especially if the latter performs acts to protect his rights such as the timely
careful reading of the June 6, 1994 letter of respondents impels this Court to believe that such registration of a notice of lis pendens.
offer was made only to demonstrate their capacity to purchase the subject property. Besides,
even if it was a valid new offer, they did so only due to the fraudulent misrepresentation Damages; Article 2220 of the New Civil Code allows the recovery of moral damages in
made by petitioner that their earlier contracts had already been rescinded. Considering breaches of contract where the party acted fraudulently and in bad faith.As to the liability
respondents capacity to pay and their continuing interest in the subject property, to abandon for moral damages, attorneys fees and expenses of litigation, we affirm in toto the appellate
their right to the contract and to the property, absent any form of protection, is contrary to courts conclusion. Article 2220 of the New Civil Code allows the recovery of moral damages in
human nature. The presumption that a person takes ordinary care of his concerns applies and breaches of contract where the party acted fraudulently and in bad faith. As found by the CA,
remains unrebutted. Obviously therefore, respondents made the new offer without petitioner undoubtedly acted fraudulently and in bad faith in breaching the letter-
abandoning the previous contract. Since there was never a perfected new contract, the July agreements. Despite the pendency of the case in the RTC, it sold the subject property to the
14, 1993 agreement was still in effect and there was no abandonment to speak of. spouses Vaca and allowed the demolition of the house even if there was already a writ of
preliminary injunction lawfully issued by the court. This is apart from its act of unilaterally
Actions; Lis Pendens; Effect of Filing; Words and Phrases; Lis Pendens, Defined; rescinding the subject contract. Clearly, petitioners acts are brazen attempts to frustrate the
Founded upon public policy and necessity, lis pendens is intended to keep the properties in decision that the court may render in favor of respondents. It is, likewise, apparent that
litigation within the power of the court until the litigation is terminated, and to prevent the because of petitioners acts, respondents were compelled to litigate justifying the award of
defeat of the judgment or decree by subsequent alienation; The filing of a notice of lis pendens attorneys fees and expenses of litigation.
has a twofold effect(1) to keep the subject matter of the litigation within the power of the
court until the entry of the final judgment to prevent the defeat of the final judgment by PETITION for review on certiorari of the decision and resolution of the Court of Appeals.
successive alienations; and (2) to bind a purchaser, bona fide or not, of the land subject of the
litigation to the judgment or decree that the court will promulgate subsequently.Lis The facts are stated in the opinion of the Court.
pendens, which literally means pending suit, refers to the jurisdiction, power or control which
a court acquires over property involved in a suit, pending the continuance of the action, and DECISION
until final judgment. Founded upon public policy and necessity, lis pendens is intended to NACHURA, J.:
keep the properties in litigation within the power of the court until the litigation is
terminated, and to prevent the defeat of the judgment or decree by subsequent alienation. This is a Petition for Review on Certiorari under Rule 45 of the Rules of Court filed by
Its notice is an announcement to the whole world that a particular property is in litigation and petitioner Associated Bank (now United Overseas Bank [Phils.]) assailing the Court of Appeals
serves as a warning that one who acquires an interest over said property does so at his own (CA) Decision1 dated February 27, 2001, which in turn affirmed the Regional Trial Court 2 (RTC)
risk or that he gambles on the result of the litigation over said property. The filing of a notice Decision3 dated November 14, 1997 in Civil Case No. 94-3298 for Specific Performance.
of lis pendens has a twofold effect: (1) to keep the subject matter of the litigation within the Likewise assailed is the appellate courts Resolution4 dated May 31, 2001 denying petitioners
power of the court until the entry of the final judgment to prevent the defeat of the final motion for reconsideration.
judgment by successive alienations; and (2) to bind a purchaser, bona fide or not, of the land
subject of the litigation to the judgment or decree that the court will promulgate The facts of the case are as follows:
subsequently.
On April 21, 1988, the spouses Eduardo and Ma. Pilar Vaca (spouses Vaca) executed a
Same; Same; Land Titles and Deeds; Settled is the rule that the notice of lis pendens is Real Estate Mortgage (REM) in favor of the petitioner5 over their parcel of residential land
not considered a collateral attack on the title, for the indefeasibility of the title shall not be with an area of 953 sq. m. and the house constructed thereon, located at No. 18, Lovebird
used to defraud another especially if the latter performs acts to protect his rights such as the Street, Green Meadows Subdivision 1, Quezon City (herein referred to as the subject

221
MATABABE FILES
Corporation Law

property). For failure of the spouses Vaca to pay their obligation, the subject property was receipt of a final order from this Court (in the Vaca case) and/or the delivery of the property to
sold at public auction with the petitioner as the highest bidder. Transfer Certificate of Title them free from occupants.14
(TCT) No. 254504, in the name of spouses Vaca, was cancelled and a new one --TCT No. 52593--
was issued in the name of the petitioner.6 Towards the end of 1993, or in early 1994, petitioner reorganized its management.
Atty. Braulio Dayday (Atty. Dayday) became petitioners Assistant Vice-President and Head of
The spouses Vaca, however, commenced an action for the nullification of the real the Documentation Section, while Atty. Soluta was relieved of his responsibilities. Atty.
estate mortgage and the foreclosure sale. Petitioner, on the other hand, filed a petition for Dayday reviewed petitioners records of its outstanding accounts and discovered that
the issuance of a writ of possession which was denied by the RTC. Petitioner, thereafter, respondents failed to deposit the balance of the purchase price of the subject property. He,
obtained a favorable judgment when the CA granted its petition but the spouses Vaca likewise, found that respondents requested for an extension of time within which to pay. The
questioned the CA decision before this Court in the case docketed as G.R. No. 109672.7 matter was then resubmitted to the ARRMC during its meeting on March 4, 1994, and it was
disapproved. ARRMC, thus, referred the matter to petitioners Legal Department for
During the pendency of the aforesaid cases, petitioner advertised the subject rescission or cancellation of the contract due to respondents breach thereof.15
property for sale to interested buyers for P9,700,000.00.8 Respondents Rafael and Monaliza
Pronstroller offered to purchase the property for P7,500,000.00. Said offer was made On May 5, 1994, Atty. Dayday informed respondents that their request for extension
through Atty. Jose Soluta, Jr. (Atty. Soluta), petitioners Vice-President, Corporate Secretary was disapproved by ARRMC and, in view of their breach of the contract, petitioner was
and a member of its Board of Directors.9 Petitioner accepted respondents offer of P7.5 rescinding the same and forfeiting their deposit. Petitioner added that if respondents were
million. Consequently, respondents paid petitioner P750,000.00, or 10% of the purchase price, still interested in buying the subject property, they had to submit their new
as down payment.10 proposal.16 Respondents went to the petitioners office, talked to Atty. Dayday and gave him
the Letter-Agreement of July 14, 1993 to show that they were granted an extension. However,
On March 18, 1993, petitioner, through Atty. Soluta, and respondents, executed a Atty. Dayday claimed that the letter was a mistake and that Atty. Soluta was not authorized to
Letter-Agreement setting forth therein the terms and conditions of the sale, to wit: give such extension.17

1. Selling price shall be at P7,500,000.00 payable as follows: On June 6, 1994, respondents proposed to pay the balance of the purchase price as
a. 10% deposit and balance of P6,750,000.00 to be deposited under escrow follows: P3,000,000.00 upon the approval of their proposal and the balance after six (6)
agreement. Said escrow deposit shall be applied as payment upon delivery of the months.18 However, the proposal was disapproved by the petitioners President. In a letter
aforesaid property to the buyers free from occupants. dated June 9, 1994, petitioner advised respondents that the former would accept the latters
b. The deposit shall be made within ninety (90) days from date hereof.Any interest proposal only if they would pay interest at the rate of 24.5% per annum on the unpaid balance.
earned on the aforesaid investment shall be for the buyers account. However, the Petitioner also allowed respondents a refund of their deposit of P750,000.00 if they would
10% deposit is non-interest earning.11 not agree to petitioners new proposal.19

Prior to the expiration of the 90-day period within which to make the escrow deposit, For failure of the parties to reach an agreement, respondents, through their counsel,
in view of the pendency of the case between the spouses Vaca and petitioner involving the informed petitioner that they would be enforcing their agreement dated July 14,
subject property,12 respondents requested that the balance of the purchase price be made 1993.20 Petitioner countered that it was not aware of the existence of the July 14 agreement
payable only upon service on them of a final decision or resolution of this Court affirming and that Atty. Soluta was not authorized to sign for and on behalf of the bank. It, likewise,
petitioners right to possess the subject property. Atty. Soluta referred respondents proposal reiterated the rescission of their previous agreement because of the breach committed by
to petitioners Asset Recovery and Remedial Management Committee (ARRMC) but the latter respondents.21
deferred action thereon.13
On July 14, 1994, in the Vaca case, this Court upheld petitioners right to possess the
On July 14, 1993, a month after they made the request and after the payment subject property.
deadline had lapsed, respondents and Atty. Soluta, acting for the petitioner, executed
another Letter-Agreement allowing the former to pay the balance of the purchase price upon

222
MATABABE FILES
Corporation Law

On July 28, 1994, respondents commenced the instant suit by filing a Complaint for eventually rescind contracts entered into by previous officers. As to whether respondents
Specific Performance before the RTC of Antipolo, Rizal. 22 The case was raffled to Branch 72 were estopped from enforcing the July 14, 1993 Letter-Agreement, the appellate court ruled
and was docketed as Civil Case No. 94-3298. Respondents prayed that petitioner be ordered in the negative. It found, instead, that petitioners were estopped from questioning the
to sell the subject property to them in accordance with their letter-agreement of July 14, 1993. efficacy of the July 14 agreement because of its failure to repudiate the same for a period of
They, likewise, caused the annotation of a notice of lis pendens at the dorsal portion of TCT one year.29 Thus, the court said in its decision:
No. 52593.
1. The Appellant (Westmont Bank) is hereby ordered to execute a "Deed of Absolute
For its part, petitioner contended that their contract had already been rescinded Sale" in favor of the Appellees over the property covered by Transfer Certificate of
because of respondents failure to deposit in escrow the balance of the purchase price within Title No. 52593, including the improvement thereon, and secure, from the Register of
the stipulated period.23 Deeds, a Torrens Title over the said property free from all liens, claims or
encumbrances upon the payment by the Appellees of the balance of the purchase
During the pendency of the case, petitioner sold the subject property to the spouses price of the property in the amount of P6,750,000.00;
Vaca, who eventually registered the sale; and on the basis thereof, TCT No. 52593 was 2. The Register of Deeds is hereby ordered to cancel Transfer Certificate of Title No.
cancelled and TCT No. 158082 was issued in their names. 24 As new owners, the spouses Vaca 158082 under the names of the Spouses Eduardo [and Ma. Pilar] Vaca and to issue
started demolishing the house on the subject property which, however, was not completed another under the names of the Appellees as stated in the preceding paragraph;
by virtue of the writ of preliminary injunction issued by the court.25
3. The appellant is hereby ordered to pay to the appellee Rafael Pronstroller the
On November 14, 1997, the trial court finally resolved the matter in favor of amount of P100,000.00 as and by way of moral damages and to pay to the Appellees
respondents, disposing, as follows: the amount of P30,000.00 as and by way of attorneys fees and the amount
of P20,000.00 for litigation expense.
WHEREFORE, premises considered, the Court finds defendants rescission of the
Agreement to Sell to be null and void for being contrary to law and public policy. 4. The counterclaims of the Appellant are dismissed.

ACCORDINGLY, defendant bank is hereby ordered to accept plaintiffs payment of SO ORDERED.30


the balance of the purchase price in the amount of Six Million Seven Hundred Fifty Thousand
Pesos (P6,750,000.00) and to deliver the title and possession to subject property, free from Petitioners motion for reconsideration was denied on May 31, 2001. Hence, the
all liens and encumbrances upon receipt of said payment. Likewise, defendant bank is ordered present petition raising the following issues:
to pay plaintiffs moral damages and attorneys fees in the amount of One Hundred Thirty
Thousand Pesos (P130,000.00) and expenses of litigation in the amount of Twenty Thousand I.
Pesos (P20,000.00). THE NARRATION OR STATEMENT OF THE FACTS OF THE CASE BY THE HONORABLE COURT
OF APPEALS IS TOTALLY BEREFT OF EVIDENTIARY SUPPORT, CONTRARY TO THE EVIDENCE
SO ORDERED.26 ON RECORD AND PURELY BASED ON ERRONEOUS ASSUMPTIONS, PRESUMPTIONS,
SURMISES, AND CONJECTURES.
Applying the rule of "apparent authority,"27 the court upheld the validity of the July
14, 1993 Letter-Agreement where the respondents were given an extension within which to II.
make payment. Consequently, respondents did not incur in delay, and thus, the court THE HONORABLE COURT OF APPEALS GROSSLY ERRED IN MERELY RELYING UPON THE
concluded that the rescission of the contract was without basis and contrary to law. 28 MANIFESTLY ERRONEOUS FINDING OF THE HONORABLE TRIAL COURT ON THE ALLEGED
APPARENT AUTHORITY OF ATTY. JOSE SOLUTA, JR. IN THAT THE LATTERS FINDING IS
On appeal, the CA affirmed the RTC decision and upheld Atty. Solutas authority to CONTRARY TO THE UNDISPUTED FACTS AND THE EVIDENCE ON RECORD.
represent the petitioner. It further ruled that petitioner had no right to unilaterally rescind the
contract; otherwise, it would give the bank officers license to continuously review and III.

223
MATABABE FILES
Corporation Law

THE HONORABLE COURT OF APPEALS OWN FINDING THAT ATTY. JOSE SOLUTA, JR. HAD THE HONORABLE COURT OF APPEALS ERRED IN HOLDING PETITIONER ESTOPPED FROM
AUTHORITY TO SELL THE SUBJECT PROPERTY ON HIS OWN (EVEN WITHOUT THE QUESTIONING THE VALIDITY OF THE JULY 14, 1993 LETTER SIGNED BY ATTY. JOSE SOLUTA,
COMMITTEES APPROVAL) IS LIKEWISE GROSSLY ERRONEOUS, FINDS NO EVIDENTIARY JR.
SUPPORT AND IS EVEN CONTRARY TO THE EVIDENCE ON RECORD IN THAT
IX.
A.) AT NO TIME DID PETITIONER ADMIT THAT ATTY. JOSE SOLUTA, JR. IS THE HONORABLE COURT OF APPEALS GROSSLY ERRED IN HOLDING THAT PETITIONER
AUTHORIZED TO SELL THE SUBJECT PROPERTY ON HIS OWN; ALLEGEDLY ACTED FRAUDULENTLY AND IN BAD FAITH IN ITS DEALINGS WITH
RESPONDENTS.
B.) THE AUTHORITY OF ATTY. JOSE SOLUTA, JR. CANNOT BE PRESUMED FROM HIS
DESIGNATIONS OR TITLES; AND X.
THE ORDER OF THE HONORABLE COURT OF APPEALS TO CANCEL TCT NO. 158082 UNDER
C.) RESPONDENTS FULLY KNEW OR HAD KNOWLEDGE OF THE LACK OF AUTHORITY THE NAMES OF SPS. VACA IS A COLLATERAL ATTACK AGAINST THE SAID CERTIFICATE OF
OF ATTY. JOSE SOLUTA, JR. TO SELL THE SUBJECT PROPERTY ON HIS OWN. TITLE WHICH IS PROSCRIBED BY SECTION 48 OF P.D. 1529.

IV. XI.
THE HONORABLE TRIAL COURT AND THE HONORABLE COURT OF APPEALS GROSSLY THE HONORABLE COURT OF APPEALS ERRED IN AWARDING MORAL DAMAGES, ATTORNEYS
MISAPPLIED THE DOCTRINE OF APPARENT AUTHORITY IN THE PRESENT CASE. FEES, AND EXPENSES OF LITIGATION IN FAVOR OF RESPONDENTS.31

V. Reduced to bare essentials, the decision on the instant petition hinges on the
THE HONORABLE TRIAL COURT AND THE HONORABLE COURT OF APPEALS GROSSLY ERRED resolution of the following specific questions: 1) Is the petitioner bound by the July 14, 1993
IN NOT HOLDING THAT THE CONTRACT TO SELL CONTAINED IN THE MARCH 18, 1993 LETTER Letter-Agreement signed by Atty. Soluta under the doctrine of apparent authority? 2) Was
WAS VALIDLY RESCINDED BY PETITIONER. there a valid rescission of the March 18, 1993 and/or July 14, 1993 Letter-Agreement? 3) Are the
respondents estopped from enforcing the July 14 Letter-Agreement because of their June 6,
VI. 1994 "new" proposal? 4) Is the petitioner estopped from questioning the validity of the July 14
THE HONORABLE COURT OF APPEALS GROSSLY ERRED IN NOT HOLDING RESPONDENTS letter because of its failure to repudiate the same and 5) Is the instant case a collateral attack
ESTOPPED FROM DENYING THE VALIDITY OF THE RESCISSION OF THE CONTRACT TO SELL AS on TCT No. 158082 in the name of the spouses Vaca?
EMBODIED IN THE MARCH 18, 1993 LETTER AND THE LACK OF AUTHORITY OF ATTY. SOLUTA,
JR. TO GRANT THE EXTENSION AS CONTAINED IN HIS LETTER OF JULY 14, 1993 AFTER THEY The petition is unmeritorious.
VOLUNTARILY SUBMITTED WITH FULL KNOWLEDGE OF ITS IMPORT AND IMPLICATION A
NEW OFFER TO PURCHASE THE SUBJECT PROPERTY CONTAINED IN THEIR LETTER DATED Well-settled is the rule that the findings of the RTC, as affirmed by the appellate
JUNE 6, 1994. court, are binding on this Court. In a petition for review on certiorari under Rule 45 of the
Rules of Court, as in this case, this Court may not review the findings of fact all over again. It
VII. must be stressed that this Court is not a trier of facts, and it is not its function to re-examine
IN ANY EVENT, THE HONORABLE COURT OF APPEALS ERRED IN NOT HOLDING THAT THE and weigh anew the respective evidence of the parties.32 The findings of the CA are conclusive
CONTRACT TO SELL UNDER THE LETTER OF MARCH 18, 1993 AND THE LETTER OF JULY 14, on the parties and carry even more weight when these coincide with the factual findings of
1993 HAD BEEN VACATED WHEN RESPONDENTS VOLUNTARILY SUBMITTED WITH FULL the trial court, unless the factual findings are not supported by the evidence on
KNOWLEDGE OF ITS IMPORT AND IMPLICATION THEIR NEW OFFER CONTAINED IN THEIR record.33 Petitioner failed to show why the above doctrine should not be applied to the
LETTER OF JUNE 6, 1994 WITHOUT ANY CONDITION OR RESERVATION WHATSOEVER. instant case.

VIII. Contrary to petitioners contention that the CAs factual findings are not supported
by the evidence on record, the assailed decision clearly shows that the appellate court not

224
MATABABE FILES
Corporation Law

only relied on the RTCs findings but made its own analysis of the record of the case. The CA objection from its board or shareholders. Undoubtedly, petitioner had previously allowed
decision contains specific details drawn from the contents of the pleadings filed by both Atty. Soluta to enter into the first agreement without a board resolution expressly authorizing
parties, from the testimonies of the witnesses and from the documentary evidence him; thus, it had clothed him with apparent authority to modify the same via the second
submitted. It was from all these that the appellate court drew its own conclusion using letter-agreement. It is not the quantity of similar acts which establishes apparent authority,
applicable legal principles and jurisprudential rules. but the vesting of a corporate officer with the power to bind the corporation. 37

The Court notes that the March 18, 1993 Letter-Agreement was written on a paper Naturally, the third person has little or no information as to what occurs in corporate
with petitioners letterhead. It was signed by Atty. Soluta with the conformity of respondents. meetings; and he must necessarily rely upon the external manifestations of corporate
The authority of Atty. Soluta to act for and on behalf of petitioner was not reflected in said consent. The integrity of commercial transactions can only be maintained by holding the
letter or on a separate paper attached to it. Yet, petitioner recognized Atty. Solutas authority corporation strictly to the liability fixed upon it by its agents in accordance with law.38 What
to sign the same and, thus, acknowledged its binding effect. On the other hand, the July 14, transpires in the corporate board room is entirely an internal matter. Hence, petitioner may
1993 letter was written on the same type of paper with the same letterhead and of the same not impute negligence on the part of the respondents in failing to find out the scope of Atty.
form as the earlier letter. It was also signed by the same person with the conformity of the Solutas authority. Indeed, the public has the right to rely on the trustworthiness of bank
same respondents. Again, nowhere in said letter did petitioner specifically authorize Atty. officers and their acts.39
Soluta to sign it for and on its behalf. This time, however, petitioner questioned the validity
and binding effect of the agreement, arguing that Atty. Soluta was not authorized to modify As early as June 1993, or prior to the 90-day period within which to make the full
the earlier terms of the contract and could not in any way bind the petitioner. payment, respondents already requested a modification of the earlier agreement such that
the full payment should be made upon receipt of this Courts decision confirming petitioners
We beg to differ. right to the subject property. The matter was brought to the petitioners attention and was in
fact discussed by the members of the Board. Instead of acting on said request (considering
The general rule is that, in the absence of authority from the board of directors, no that the 90-day period was about to expire), the board deferred action on the request. It was
person, not even its officers, can validly bind a corporation. The power and responsibility to only after one year and after the banks reorganization that the board rejected respondents
decide whether the corporation should enter into a contract that will bind the corporation is request. We cannot therefore blame the respondents in relying on the July 14, 1993 Letter-
lodged in the board of directors. However, just as a natural person may authorize another to Agreement. Petitioners inaction, coupled with the apparent authority of Atty. Soluta to act
do certain acts for and on his behalf, the board may validly delegate some of its functions and on behalf of the corporation, validates the July 14 agreement and thus binds the corporation.
powers to officers, committees and agents. The authority of such individuals to bind the All these taken together, lead to no other conclusion than that the petitioner attempted to
corporation is generally derived from law, corporate bylaws or authorization from the board, defraud the respondents. This is bolstered by the fact that it forged another contract
either expressly or impliedly, by habit, custom, or acquiescence, in the general course of involving the same property, with another buyer, the spouses Vaca, notwithstanding the
business.34 pendency of the instant case.

The authority of a corporate officer or agent in dealing with third persons may be We would like to emphasize that if a corporation knowingly permits its officer, or any
actual or apparent. The doctrine of "apparent authority," with special reference to banks, had other agent, to perform acts within the scope of an apparent authority, holding him out to
long been recognized in this jurisdiction.35 Apparent authority is derived not merely from the public as possessing power to do those acts, the corporation will, as against any person
practice. Its existence may be ascertained through 1) the general manner in which the who has dealt in good faith with the corporation through such agent, be estopped from
corporation holds out an officer or agent as having the power to act, or in other words, the denying such authority.40
apparent authority to act in general, with which it clothes him; or 2) the acquiescence in his Petitioner further insists that specific performance is not available to respondents
acts of a particular nature, with actual or constructive knowledge thereof, within or beyond because the Letter-Agreements had already been rescinded --- the March 18 agreement
the scope of his ordinary powers.36 because of the breach committed by the respondents; and the July 14 letter because of the
new offer of the respondents which was not approved by petitioner.
Accordingly, the authority to act for and to bind a corporation may be presumed
from acts of recognition in other instances, wherein the power was exercised without any Again, the argument is misplaced.

225
MATABABE FILES
Corporation Law

serves as a warning that one who acquires an interest over said property does so at his own
Basic is the rule that a contract constitutes the law between the parties. Concededly, risk or that he gambles on the result of the litigation over said property.46
parties may validly stipulate the unilateral rescission of a contract. 41 This is usually in the form
of a stipulation granting the seller the right to forfeit installments or deposits made by the The filing of a notice of lis pendens has a twofold effect: (1) to keep the subject
buyer in case of the latters failure to make full payment on the stipulated date. While the matter of the litigation within the power of the court until the entry of the final judgment to
petitioner in the instant case may have the right, under the March 18 agreement, to prevent the defeat of the final judgment by successive alienations; and (2) to bind a
unilaterally rescind the contract in case of respondents failure to comply with the terms of purchaser, bona fide or not, of the land subject of the litigation to the judgment or decree that
the contract,42 the execution of the July 14 Agreement prevented petitioner from exercising the court will promulgate subsequently.47
the right to rescind. This is so because there was in the first place, no breach of contract, as
the date of full payment had already been modified by the later agreement. This registration, therefore, gives the court clear authority to cancel the title of the
spouses Vaca, since the sale of the subject property was made after the notice of lis pendens.
Neither can the July 14, 1993 agreement be considered abandoned by respondents Settled is the rule that the notice is not considered a collateral attack on the title, 48 for the
act of making a new offer, which was unfortunately rejected by petitioner. A careful reading indefeasibility of the title shall not be used to defraud another especially if the latter performs
of the June 6, 1994 letter of respondents impels this Court to believe that such offer was acts to protect his rights such as the timely registration of a notice of lis pendens.
made only to demonstrate their capacity to purchase the subject property. 43 Besides, even if it
was a valid new offer, they did so only due to the fraudulent misrepresentation made by As to the liability for moral damages, attorneys fees and expenses of litigation, we
petitioner that their earlier contracts had already been rescinded. Considering respondents affirm in toto the appellate courts conclusion. Article 2220 49 of the New Civil Code allows the
capacity to pay and their continuing interest in the subject property, 44 to abandon their right recovery of moral damages in breaches of contract where the party acted fraudulently and in
to the contract and to the property, absent any form of protection, is contrary to human bad faith. As found by the CA, petitioner undoubtedly acted fraudulently and in bad faith in
nature. The presumption that a person takes ordinary care of his concerns applies and breaching the letter-agreements. Despite the pendency of the case in the RTC, it sold the
remains unrebutted.45 Obviously therefore, respondents made the new offer without subject property to the spouses Vaca and allowed the demolition of the house even if there
abandoning the previous contract. Since there was never a perfected new contract, the July was already a writ of preliminary injunction lawfully issued by the court. This is apart from its
14, 1993 agreement was still in effect and there was no abandonment to speak of. act of unilaterally rescinding the subject contract. Clearly, petitioners acts are brazen
attempts to frustrate the decision that the court may render in favor of respondents. 50 It is,
In its final attempt to prevent respondents from attaining a favorable result, likewise, apparent that because of petitioners acts, respondents were compelled to litigate
petitioner argues that the instant case should not prosper because the cancellation of TCT No. justifying the award of attorneys fees and expenses of litigation.
158082 is a collateral attack on the title which is proscribed by law.
WHEREFORE, premises considered, the petition is DENIED. The Decision of the Court
Such contention is baseless. of Appeals dated February 27, 2001 and its Resolution dated May 31, 2001 in CA-G.R. CV No.
60315 are AFFIRMED.
Admittedly, during the pendency of the case, respondents timely registered a notice
of lis pendens to warn the whole world that the property was the subject of a pending SO ORDERED.
litigation.

Lis pendens, which literally means pending suit, refers to the jurisdiction, power or
control which a court acquires over property involved in a suit, pending the continuance of
the action, and until final judgment. Founded upon public policy and necessity, lis pendens is
intended to keep the properties in litigation within the power of the court until the litigation is
terminated, and to prevent the defeat of the judgment or decree by subsequent alienation.
Its notice is an announcement to the whole world that a particular property is in litigation and

226
MATABABE FILES
Corporation Law

Republic of the Philippines the contrary, it is manifestly clear from the terms of the voting trust agreement between
SUPREME COURT ALFA and the DBP that the duration of the agreement is contingent upon the fulfillment of
Manila certain obligations of ALFA with the DBP.
THIRD DIVISION
Remedial Law; Civil Procedure; Service of summons; If the defendant is a corporation
G.R. No. 93695 February 4, 1992 organized under the laws of the Philippines, service may be made on the president, manager,
secretary, cashier, agent or any of its directors.It is a basic principle in Corporation Law that
RAMON C. LEE and ANTONIO DM. LACDAO, petitioners, a corporation has a personality separate and distinct from the officers or members who
vs. compose it. (See Sulo ng Bayan Inc. v. Araneta, Inc., 72 SCRA 347 [1976]; Osias Academy v.
THE HON. COURT OF APPEALS, SACOBA MANUFACTURING CORP., PABLO GONZALES, JR. Department of Labor and Employment, et al., G.R. Nos. 83257-58, December 21, 1990). Thus,
and THOMAS GONZALES, respondents. the above rule on service of processes on a corporation enumerates the representatives of a
corporation who can validly receive court processes on its behalf. Not every stockholder or
Mercantile Law; Corporation Code; Every director must own at least one (1) share of officer can bind the corporation considering the existence of a corporate entity separate from
the capital stock of the corporation of which he is a director which share shall stand in his those who compose it. The rationale of the aforecited rule is that service must be made on a
name on the books of the corporation. Any director who ceases to be the owner of at least one representative so integrated with the corporation sued as to make it a priori supposable that
(1) share of the capital stock of the corporation of which he is a director shall thereby cease to he will realize his responsibilities and know what he should do with any legal papers served on
be a director.Under the old Corporation Code, the eligibility of a director, strictly speaking, him. (Far Corporation v. Francisco, 146 SCRA 197 [1986] citing Villa Rey Transit, Inc. v. Far East
cannot be adversely affected by the simple act of such director being a party to a voting trust Motor Corp., 81 SCRA 303 [1978]).
agreement inasmuch as he remains owner (although beneficial or equitable only) of the
shares subject of the voting trust agreement pursuant to which a transfer of the PETITION for certiorari to review the decision and resolution of the Court of Appeals.
stockholder's shares in favor of the trustee is required (section 36 of the old Corporation
Code). No disqualification arises by virtue of the phrase "in his own right" provided under the The facts are stated in the opinion of the Court.
old Corporation Code. With the omission of the phrase "in his own right" the election of
trustees and other persons who in fact are not the beneficial owners of the shares registered Cayanga, Zuniga & Angel Law Offices for petitioners.
in their names on the books of the corporation becomes formally legalized (see Campos and Timbol & Associates for private respondents.
Lopez-Campos, supra, p. 296) Hence, this is a clear indication that in order to be eligible as a
director, what is material is the legal title to, not beneficial ownership of, the stock as GUTIERREZ, JR., J.:
appearing on the books of the corporation (2 Fletcher, Cyclopedia of the Law of Private
Corporations, section 300, p. 92 [1969] citing People v. Lihme, 269111. 351, 109 N.E. 1051). What is the nature of the voting trust agreement executed between two parties in
this case? Who owns the stocks of the corporation under the terms of the voting trust
Same; Same; Voting Trusts; A voting trust agreement results in the separation of agreement? How long can a voting trust agreement remain valid and effective? Did a director
the voting rights of a stockholder from his other rights such as the right to receive dividends of the corporation cease to be such upon the creation of the voting trust agreement? These
and other rights to which a stockholder may be entitled until the liquidation of the are the questions the answers to which are necessary in resolving the principal issue in this
corporation.There can be no reliance on the inference that the five-year period of the petition for certiorari whether or not there was proper service of summons on Alfa
voting trust agreement in question had lapsed in 1986 so that the legal title to the stocks Integrated Textile Mills (ALFA, for short) through the petitioners as president and vice-
covered by the said voting trust agreement ipso facto reverted to the petitioners as beneficial president, allegedly, of the subject corporation after the execution of a voting trust
owners pursuant to the 6th paragraph of section 59 of the new Corporation Code which agreement between ALFA and the Development Bank of the Philippines (DBP, for short).
reads: "Unless expressly renewed, all rights granted in a voting trust agreement shall
automatically expire at the end of the agreed period, and the voting trust certificates as well From the records of the instant case, the following antecedent facts appear:
as the certificates of stock in the name of the trustee or trustees shall thereby be deemed
cancelled and new certificates of stock shall be reissued in the name of the transferors." On

227
MATABABE FILES
Corporation Law

On November 15, 1985, a complaint for a sum of money was filed by the International stockholders of ALFA (the petitioners included), on the one hand, and the DBP, on the other
Corporate Bank, Inc. against the private respondents who, in turn, filed a third party hand, whereby the management and control of ALFA became vested upon the DBP.
complaint against ALFA and the petitioners on March 17, 1986.
On September 17, 1987, the petitioners filed a motion to dismiss the third party complaint On April 25, 1989, the trial court reversed itself by setting aside its previous Order
which the Regional Trial Court of Makati, Branch 58 denied in an Order dated June 27, 1988. dated January 2, 1989 and declared that service upon the petitioners who were no longer
corporate officers of ALFA cannot be considered as proper service of summons on ALFA.
On July 18, 1988, the petitioners filed their answer to the third party complaint.
On May 15, 1989, the private respondents moved for a reconsideration of the above
Meanwhile, on July 12, 1988, the trial court issued an order requiring the issuance of Order which was affirmed by the court in its Order dated August 14, 1989 denying the private
an alias summons upon ALFA through the DBP as a consequence of the petitioner's letter respondent's motion for reconsideration.
informing the court that the summons for ALFA was erroneously served upon them
considering that the management of ALFA had been transferred to the DBP. On September 18, 1989, a petition for certiorari was belatedly submitted by the private
respondent before the public respondent which, nonetheless, resolved to give due course
In a manifestation dated July 22, 1988, the DBP claimed that it was not authorized to thereto on September 21, 1989.
receive summons on behalf of ALFA since the DBP had not taken over the company which has
a separate and distinct corporate personality and existence. On October 17, 1989, the trial court, not having been notified of the pending petition
for certiorari with public respondent issued an Order declaring as final the Order dated April
On August 4, 1988, the trial court issued an order advising the private respondents to 25, 1989. The private respondents in the said Order were required to take positive steps in
take the appropriate steps to serve the summons to ALFA. prosecuting the third party complaint in order that the court would not be constrained to
On August 16, 1988, the private respondents filed a Manifestation and Motion for the dismiss the same for failure to prosecute. Subsequently, on October 25, 1989 the private
Declaration of Proper Service of Summons which the trial court granted on August 17, 1988. respondents filed a motion for reconsideration on which the trial court took no further action.
On March 19, 1990, after the petitioners filed their answer to the private
On September 12, 1988, the petitioners filed a motion for reconsideration submitting respondents' petition for certiorari, the public respondent rendered its decision, the
that Rule 14, section 13 of the Revised Rules of Court is not applicable since they were no dispositive portion of which reads:
longer officers of ALFA and that the private respondents should have availed of another
mode of service under Rule 14, Section 16 of the said Rules, i.e., through publication to effect WHEREFORE, in view of the foregoing, the orders of respondent
proper service upon ALFA. judge dated April 25, 1989 and August 14, 1989 are hereby SET ASIDE and
In their Comment to the Motion for Reconsideration dated September 27, 1988, the private respondent corporation is ordered to file its answer within the reglementary
respondents argued that the voting trust agreement dated March 11, 1981 did not divest the period. (CA Decision, p. 8; Rollo, p. 24)
petitioners of their positions as president and executive vice-president of ALFA so that service
of summons upon ALFA through the petitioners as corporate officers was proper. On April 11, 1990, the petitioners moved for a reconsideration of the decision of the
public respondent which resolved to deny the same on May 10, 1990. Hence, the petitioners
On January 2, 1989, the trial court upheld the validity of the service of summons on filed this certiorari petition imputing grave abuse of discretion amounting to lack of
ALFA through the petitioners, thus, denying the latter's motion for reconsideration and jurisdiction on the part of the public respondent in reversing the questioned Orders dated
requiring ALFA to filed its answer through the petitioners as its corporate officers. April 25, 1989 and August 14, 1989 of the court a quo, thus, holding that there was proper
service of summons on ALFA through the petitioners.
On January 19, 1989, a second motion for reconsideration was filed by the petitioners
reiterating their stand that by virtue of the voting trust agreement they ceased to be officers In the meantime, the public respondent inadvertently made an entry of judgment on
and directors of ALFA, hence, they could no longer receive summons or any court processes July 16, 1990 erroneously applying the rule that the period during which a motion for
for or on behalf of ALFA. In support of their second motion for reconsideration, the reconsideration has been pending must be deducted from the 15-day period to appeal.
petitioners attached thereto a copy of the voting trust agreement between all the However, in its Resolution dated January 3, 1991, the public respondent set aside the

228
MATABABE FILES
Corporation Law

aforestated entry of judgment after further considering that the rule it relied on applies to Under Section 59 of the new Corporation Code which expressly recognizes voting
appeals from decisions of the Regional Trial Courts to the Court of Appeals, not to appeals trust agreements, a more definitive meaning may be gathered. The said provision partly
from its decision to us pursuant to our ruling in the case of Refractories Corporation of the reads:
Philippines v. Intermediate Appellate Court, 176 SCRA 539 [1989]. (CA Rollo, pp. 249-250)
Sec. 59. Voting Trusts One or more stockholders of a stock corporation
In their memorandum, the petitioners present the following arguments, to wit: may create a voting trust for the purpose of conferring upon a trustee or
trustees the right to vote and other rights pertaining to the share for a
(1) that the execution of the voting trust agreement by a stockholders period rights pertaining to the shares for a period not exceeding five (5)
whereby all his shares to the corporation have been transferred to the years at any one time: Provided, that in the case of a voting trust specifically
trustee deprives the stockholders of his position as director of the required as a condition in a loan agreement, said voting trust may be for a
corporation; to rule otherwise, as the respondent Court of Appeals did, period exceeding (5) years but shall automatically expire upon full payment
would be violative of section 23 of the Corporation Code ( Rollo, pp. 270- of the loan. A voting trust agreement must be in writing and notarized, and
3273); and shall specify the terms and conditions thereof. A certified copy of such
agreement shall be filed with the corporation and with the Securities and
(2) that the petitioners were no longer acting or holding any of the positions Exchange Commission; otherwise, said agreement is ineffective and
provided under Rule 14, Section 13 of the Rules of Court authorized to unenforceable. The certificate or certificates of stock covered by the voting
receive service of summons for and in behalf of the private domestic trust agreement shall be cancelled and new ones shall be issued in the name
corporation so that the service of summons on ALFA effected through the of the trustee or trustees stating that they are issued pursuant to said
petitioners is not valid and ineffective; to maintain the respondent Court of agreement. In the books of the corporation, it shall be noted that the
Appeals' position that ALFA was properly served its summons through the transfer in the name of the trustee or trustees is made pursuant to said
petitioners would be contrary to the general principle that a corporation can voting trust agreement.
only be bound by such acts which are within the scope of its officers' or By its very nature, a voting trust agreement results in the separation of the voting
agents' authority (Rollo, pp. 273-275) rights of a stockholder from his other rights such as the right to receive dividends, the right to
In resolving the issue of the propriety of the service of summons in the instant case, inspect the books of the corporation, the right to sell certain interests in the assets of the
we dwell first on the nature of a voting trust agreement and the consequent effects upon its corporation and other rights to which a stockholder may be entitled until the liquidation of
creation in the light of the provisions of the Corporation Code. the corporation. However, in order to distinguish a voting trust agreement from proxies and
other voting pools and agreements, it must pass three criteria or tests, namely: (1) that the
A voting trust is defined in Ballentine's Law Dictionary as follows: voting rights of the stock are separated from the other attributes of ownership; (2) that the
voting rights granted are intended to be irrevocable for a definite period of time; and (3) that
(a) trust created by an agreement between a group of the stockholders of a the principal purpose of the grant of voting rights is to acquire voting control of the
corporation and the trustee or by a group of identical agreements between corporation. (5 Fletcher, Cyclopedia of the Law on Private Corporations, section 2075 [1976] p.
individual stockholders and a common trustee, whereby it is provided that 331 citing Tankersly v. Albright, 374 F. Supp. 538)
for a term of years, or for a period contingent upon a certain event, or until
the agreement is terminated, control over the stock owned by such Under section 59 of the Corporation Code, supra, a voting trust agreement may
stockholders, either for certain purposes or for all purposes, is to be lodged confer upon a trustee not only the stockholder's voting rights but also other rights pertaining
in the trustee, either with or without a reservation to the owners, or persons to his shares as long as the voting trust agreement is not entered "for the purpose of
designated by them, of the power to direct how such control shall be used. circumventing the law against monopolies and illegal combinations in restraint of trade or
(98 ALR 2d. 379 sec. 1 [d]; 19 Am J 2d Corp. sec. 685). used for purposes of fraud." (section 59, 5th paragraph of the Corporation Code) Thus, the
traditional concept of a voting trust agreement primarily intended to single out a
stockholder's right to vote from his other rights as such and made irrevocable for a limited

229
MATABABE FILES
Corporation Law

duration may in practice become a legal device whereby a transfer of the stockholder's shares some purposes the depositing stockholder holding voting trust certificates
is effected subject to the specific provision of the voting trust agreement. in lieu of his stock and being the beneficial owner thereof, remains and is
treated as a stockholder. It seems to be deducible from the case that he may
The execution of a voting trust agreement, therefore, may create a dichotomy sue as a stockholder if the suit is in equity or is of an equitable nature, such
between the equitable or beneficial ownership of the corporate shares of a stockholders, on as, a technical stockholders' suit in right of the corporation. [Commercial
the one hand, and the legal title thereto on the other hand. Laws of the Philippines by Agbayani, Vol. 3 pp. 492-493, citing 5 Fletcher 326,
327] (Rollo, p. 291)
The law simply provides that a voting trust agreement is an agreement in writing
whereby one or more stockholders of a corporation consent to transfer his or their shares to We find the petitioners' position meritorious.
a trustee in order to vest in the latter voting or other rights pertaining to said shares for a
period not exceeding five years upon the fulfillment of statutory conditions and such other Both under the old and the new Corporation Codes there is no dispute as to the most
terms and conditions specified in the agreement. The five year-period may be extended in immediate effect of a voting trust agreement on the status of a stockholder who is a party to
cases where the voting trust is executed pursuant to a loan agreement whereby the period is its execution from legal titleholder or owner of the shares subject of the voting trust
made contingent upon full payment of the loan. agreement, he becomes the equitable or beneficial owner. (Salonga, Philippine Law on Private
Corporations, 1958 ed., p. 268; Pineda and Carlos, The Law on Private Corporations and
In the instant case, the point of controversy arises from the effects of the creation of Corporate Practice, 1969 ed., p. 175; Campos and Lopez-Campos, The Corporation Code;
the voting trust agreement. The petitioners maintain that with the execution of the voting Comments, Notes & Selected Cases, 1981, ed., p. 386; Agbayani, Commentaries and
trust agreement between them and the other stockholders of ALFA, as one party, and the Jurisprudence on the Commercial Laws of the Philippines, Vol. 3, 1988 ed., p. 536). The
DBP, as the other party, the former assigned and transferred all their shares in ALFA to DBP, penultimate question, therefore, is whether the change in his status deprives the stockholder
as trustee. They argue that by virtue to of the voting trust agreement the petitioners can no of the right to qualify as a director under section 23 of the present Corporation Code which
longer be considered directors of ALFA. In support of their contention, the petitioners invoke deletes the phrase "in his own right." Section 30 of the old Code states that:
section 23 of the Corporation Code which provides, in part, that:
Every director must own in his own right at least one share of the
Every director must own at least one (1) share of the capital stock of capital stock of the stock corporation of which he is a director, which stock
the corporation of which he is a director which share shall stand in his name shall stand in his name on the books of the corporation. A director who
on the books of the corporation. Any director who ceases to be the owner of ceases to be the owner of at least one share of the capital stock of a stock
at least one (1) share of the capital stock of the corporation of which he is a corporation of which is a director shall thereby cease to be a director . . .
director shall thereby cease to be director . . . (Rollo, p. 270) (Emphasis supplied)

The private respondents, on the contrary, insist that the voting trust agreement Under the old Corporation Code, the eligibility of a director, strictly speaking, cannot
between ALFA and the DBP had all the more safeguarded the petitioners' continuance as be adversely affected by the simple act of such director being a party to a voting trust
officers and directors of ALFA inasmuch as the general object of voting trust is to insure agreement inasmuch as he remains owner (although beneficial or equitable only) of the
permanency of the tenure of the directors of a corporation. They cited the commentaries by shares subject of the voting trust agreement pursuant to which a transfer of the
Prof. Aguedo Agbayani on the right and status of the transferring stockholders, to wit: stockholder's shares in favor of the trustee is required (section 36 of the old Corporation
Code). No disqualification arises by virtue of the phrase "in his own right" provided under the
The "transferring stockholder", also called the "depositing old Corporation Code.
stockholder", is equitable owner for the stocks represented by the voting With the omission of the phrase "in his own right" the election of trustees and other persons
trust certificates and the stock reversible on termination of the trust by who in fact are not beneficial owners of the shares registered in their names on the books of
surrender. It is said that the voting trust agreement does not destroy the the corporation becomes formally legalized (see Campos and Lopez-Campos, supra, p. 296)
status of the transferring stockholders as such, and thus render them Hence, this is a clear indication that in order to be eligible as a director, what is material is the
ineligible as directors. But a more accurate statement seems to be that for legal title to, not beneficial ownership of, the stock as appearing on the books of the

230
MATABABE FILES
Corporation Law

corporation (2 Fletcher, Cyclopedia of the Law of Private Corporations, section 300, p. 92 Considering that the voting trust agreement between ALFA and the DBP transferred
[1969] citing People v. Lihme, 269 Ill. 351, 109 N.E. 1051). legal ownership of the stock covered by the agreement to the DBP as trustee, the latter
became the stockholder of record with respect to the said shares of stocks. In the absence of
The facts of this case show that the petitioners, by virtue of the voting trust a showing that the DBP had caused to be transferred in their names one share of stock for the
agreement executed in 1981 disposed of all their shares through assignment and delivery in purpose of qualifying as directors of ALFA, the petitioners can no longer be deemed to have
favor of the DBP, as trustee. Consequently, the petitioners ceased to own at least one share retained their status as officers of ALFA which was the case before the execution of the
standing in their names on the books of ALFA as required under Section 23 of the new subject voting trust agreement. There appears to be no dispute from the records that DBP
Corporation Code. They also ceased to have anything to do with the management of the has taken over full control and management of the firm.
enterprise. The petitioners ceased to be directors. Hence, the transfer of the petitioners'
shares to the DBP created vacancies in their respective positions as directors of ALFA. The Moreover, in the Certification dated January 24, 1989 issued by the DBP through one
transfer of shares from the stockholder of ALFA to the DBP is the essence of the subject Elsa A. Guevarra, Vice-President of its Special Accounts Department II, Remedial Management
voting trust agreement as evident from the following stipulations: Group, the petitioners were no longer included in the list of officers of ALFA "as of April
1982." (CA Rollo, pp. 140-142)
1. The TRUSTORS hereby assign and deliver to the TRUSTEE the certificate of
the shares of the stocks owned by them respectively and shall do all things Inasmuch as the private respondents in this case failed to substantiate their claim
necessary for the transfer of their respective shares to the TRUSTEE on the that the subject voting trust agreement did not deprive the petitioners of their position as
books of ALFA. directors of ALFA, the public respondent committed a reversible error when it ruled that:

2. The TRUSTEE shall issue to each of the TRUSTORS a trust certificate for . . . while the individual respondents (petitioners Lee and Lacdao)
the number of shares transferred, which shall be transferrable in the same may have ceased to be president and vice-president, respectively, of the
manner and with the same effect as certificates of stock subject to the corporation at the time of service of summons on them on August 21, 1987,
provisions of this agreement; they were at least up to that time, still directors . . .

3. The TRUSTEE shall vote upon the shares of stock at all meetings of ALFA, The aforequoted statement is quite inaccurate in the light of the express terms of
annual or special, upon any resolution, matter or business that may be Stipulation No. 4 of the subject voting trust agreement. Both parties, ALFA and the DBP, were
submitted to any such meeting, and shall possess in that respect the same aware at the time of the execution of the agreement that by virtue of the transfer of shares of
powers as owners of the equitable as well as the legal title to the stock; ALFA to the DBP, all the directors of ALFA were stripped of their positions as such.

4. The TRUSTEE may cause to be transferred to any person one share of There can be no reliance on the inference that the five-year period of the voting trust
stock for the purpose of qualifying such person as director of ALFA, and agreement in question had lapsed in 1986 so that the legal title to the stocks covered by the
cause a certificate of stock evidencing the share so transferred to be issued said voting trust agreement ipso facto reverted to the petitioners as beneficial owners
in the name of such person; pursuant to the 6th paragraph of section 59 of the new Corporation Code which reads:

xxx xxx xxx Unless expressly renewed, all rights granted in a voting trust
agreement shall automatically expire at the end of the agreed period, and
9. Any stockholder not entering into this agreement may transfer his shares the voting trust certificate as well as the certificates of stock in the name of
to the same trustees without the need of revising this agreement, and this the trustee or trustees shall thereby be deemed cancelled and new
agreement shall have the same force and effect upon that said stockholder. certificates of stock shall be reissued in the name of the transferors.
(CA Rollo, pp. 137-138; Emphasis supplied)
On the contrary, it is manifestly clear from the terms of the voting trust agreement
between ALFA and the DBP that the duration of the agreement is contingent upon the

231
MATABABE FILES
Corporation Law

fulfillment of certain obligations of ALFA with the DBP. This is shown by the following Under section 13, Rule 14 of the Revised Rules of Court, it is provided that:
portions of the agreement. Sec. 13. Service upon private domestic corporation or partnership.
If the defendant is a corporation organized under the laws of the Philippines
WHEREAS, the TRUSTEE is one of the creditors of ALFA, and its or a partnership duly registered, service may be made on the president,
credit is secured by a first mortgage on the manufacturing plant of said manager, secretary, cashier, agent or any of its directors.
company;
WHEREAS, ALFA is also indebted to other creditors for various It is a basic principle in Corporation Law that a corporation has a personality separate
financial accomodations and because of the burden of these obligations is and distinct from the officers or members who compose it. (See Sulo ng Bayan Inc. v. Araneta,
encountering very serious difficulties in continuing with its operations. Inc., 72 SCRA 347 [1976]; Osias Academy v. Department of Labor and Employment, et al., G.R.
Nos. 83257-58, December 21, 1990). Thus, the above rule on service of processes of a
WHEREAS, in consideration of additional accommodations from the corporation enumerates the representatives of a corporation who can validly receive court
TRUSTEE, ALFA had offered and the TRUSTEE has accepted participation in processes on its behalf. Not every stockholder or officer can bind the corporation considering
the management and control of the company and to assure the aforesaid the existence of a corporate entity separate from those who compose it.
participation by the TRUSTEE, the TRUSTORS have agreed to execute a The rationale of the aforecited rule is that service must be made on a representative so
voting trust covering their shareholding in ALFA in favor of the TRUSTEE; integrated with the corporation sued as to make it a priori supposable that he will realize his
responsibilities and know what he should do with any legal papers served on him. (Far
AND WHEREAS, DBP is willing to accept the trust for the purpose Corporation v. Francisco, 146 SCRA 197 [1986] citing Villa Rey Transit, Inc. v. Far East Motor
aforementioned. Corp. 81 SCRA 303 [1978]).

NOW, THEREFORE, it is hereby agreed as follows: The petitioners in this case do not fall under any of the enumerated officers. The
service of summons upon ALFA, through the petitioners, therefore, is not valid. To rule
xxx xxx xxx otherwise, as correctly argued by the petitioners, will contravene the general principle that a
corporation can only be bound by such acts which are within the scope of the officer's or
6. This Agreement shall last for a period of Five (5) years, and is agent's authority. (see Vicente v. Geraldez, 52 SCRA 210 [1973]).
renewable for as long as the obligations of ALFA with DBP, or any portion WHEREFORE, premises considered, the petition is hereby GRANTED. The appealed
thereof, remains outstanding; (CA Rollo, pp. 137-138) decision dated March 19, 1990 and the Court of Appeals' resolution of May 10, 1990 are SET
ASIDE and the Orders dated April 25, 1989 and October 17, 1989 issued by the Regional Trial
Had the five-year period of the voting trust agreement expired in 1986, the DBP Court of Makati, Branch 58 are REINSTATED.
would not have transferred all its rights, titles and interests in ALFA "effective June 30, 1986"
to the national government through the Asset Privatization Trust (APT) as attested to in a SO ORDERED.
Certification dated January 24, 1989 of the Vice President of the DBP's Special Accounts
Department II. In the same certification, it is stated that the DBP, from 1987 until 1989, had Feliciano, Bidin, Davide, Jr. and Romero, JJ., concur.
handled APT's account which included ALFA's assets pursuant to a management agreement
by and between the DBP and APT (CA Rollo, p. 142) Hence, there is evidence on record that at
the time of the service of summons on ALFA through the petitioners on August 21, 1987, the
voting trust agreement in question was not yet terminated so that the legal title to the stocks
of ALFA, then, still belonged to the DBP.

In view of the foregoing, the ultimate issue of whether or not there was proper
service of summons on ALFA through the petitioners is readily answered in the negative.

232
MATABABE FILES
Corporation Law

Republic of the Philippines this [sic] constitutional rights, and that, an appeal to this Court, we would, therefore, have to
SUPREME COURT set aside the judgment of conviction of the lower court. This would, obviously, be most unfair
Manila and unjust. Under the circumstances obtaining in the present case, the flaw in the procedure
SECOND DIVISION followed by petitioner herein may be overlooked, in the interest of a more enlightened and
substantial justice." Thus, where there is patent grave abuse of discretion, in denying the
G.R. No. L-34192 June 30, 1988 motion to disrniss, as in the present case, this Court may entertain the petition for certiorari
interposed by the party against whom the said order is issued.
NATIONAL INVESTMENT AND DEVELOPMENT CORPORATION, EUSEBIO VILLATUYA MARIO
Y. CONSING and ROBERTO S. BENEDICTO, petitioners, Same; Same; Same; Jurisdiction; Jurisdiction of CFI to issue a writ of preliminary or
vs. permanent injunction is confmed within the province where the land in question is situated.
HON. BENJAMIN AQUINO, in his official capacity as Presiding Judge of Branch VIII of the Anent the first ground, it is a well-settled rule that the jurisdiction of a Court of First Instance
Court of First Instance of Rizal, BATJAK INC., GRACIANO A. GARCIA and MARCELINO to issue a writ of preliminary permanent injunction is confmed within the boundaries of the
CALINAWAN JR., respondents. province where the land in controversy is situated. The petition for mandamus of Batjak
prayed that NIDC and PNB be ordered to surrender, relinquish and turnover to Batjak the
G.R. No. L-34213 June 30, 1988 assets, management and operation of Batjak particularly the three (3) oil mills located in Sasa,
Davao City, Jimenez, Misamis Occidental and Tanauan, Leyte.
PHILIPPINE NATIONAL BANK, petitioner,
vs. Same; Same; Same; Venue; Respondent Batjak's complaint should have been filed in
HON. BENJAMIN H. AQUINO, in his capacity as Presiding Judge of the Court of First Instance the provinces where the oil mills are located pursuajit to Sec. 2, Rule 4, par. A ofRules
of Rizal, Branch VIII and BATJAK INCORPORATED, respondents. ofCourt.On the matter of proper venue, Batjak's complaint should have been filed in the
provinces where said oil mills are located. Under Rule 4, Sec. 2, paragraph A of the Rules of
Remedial Law; Certiorari; Mjotion to Quash; General Rule; An order denying a motion Court, "actions affecting title to, or for recovery of possession, or for partition or
to quash or to dismiss is interlocutory and cannot be subject ofa petition for certiorari; condemnation of, or foreclosure of mortgage on, real property, shall be commenced and
Remedies ofthe aggrieved party; Exceptions to the general ruJe.As a general rule, an order tried in the province where the property or any part thereof lies."
denying a motion to quash or to dismiss is interlocutory and cannot be the subject of a
petition for certiorari. The remedy of the aggrieved party in a denied motion to dismiss is to Same; Same; Same;Actions; Every action must beprosecuted and defended in the
file an answer and interpose, as defense or defenses, the objection or objections raised by name ofthe real party in interest.In support of the third ground of their motion to dismiss,
him in said motion to dismiss, then proceed to trial and, in case of adverse decision, to elevate PNB and NIDC contend that Batjak's complaint for mandamus is based 011 its claim or right to
the entire case by appeal in due course. However, under certain situations, recourse to the recovery of possession of the three (3) oil mills, on the ground of an alleged breach of
extraordinary legal remedies of certiorari, prohibition and mandamus to question the denial fiduciary relationship. Noteworthy is the fact that, in the Voting Trust Agreement, the parties
of a motion to dismiss or quash is considered proper, in the interest of more enlightened and thereto were NIDC and certain stockholders of Batjak. Batjak itself was not a signatory
substantial justice. As the Court said in Pineda andAmpil Manufacturing Co. vs. Bartolome, 95 thereto. Under Sec. 2, Rule 3 of the Rules of Court, every action must be prosecuted and
Phil. 930, 938: "For analogous reasons it may be said that the petition for certiorari interposed defended in the name of the real party in interest. Applying the rule in the present case, the
by the accused against the order of the court a quo denying the motion to quash may be action should have been filed by the stockholders of Batjak, who executed the Voting Trust
entertained, not only because it was rendered in a criminal case, but because it was rendered, Agreement with NIDC; and not by Batjak itself which is not a party to said agreement, and
as claimed, with grave abuse of discretion, as found by the Court of Appeals, xxx." and therefore, not the real party in interest in the suit to enforce the same.
reiterated in Mead v. Argel citing Yap v. Lutero (105 Phil. 1307): "However, were we to require
adherence to this pretense, the case at bar would have to be dismissed and petitioner Same; Same; Same; Mandamus, nature of; Legal Right, defined in Sec. 3, Rule 65
required to go through the inconvenience, not to say the mental agony the torture, of ofRules of Court.Moreover, the action instituted by Batjak before the respondent court was
submitting himself to trial on the merits in Case No. 166443, apart from the expenses a special civil action for mandamus with prayer for preliminajy mandatory injunction.
incidental thereto, despite the fact that his trial and conviction therein would violate one of Generally, mandamus is not a writ of righiand its allowance or refusal is a matter of discretion

233
MATABABE FILES
Corporation Law

to be exercised on equitable principles and in accordance with well-settled rules of law, and being lost, removed or materially injured unless a receiver is appointed to guard and preserve
that it should never be used to effectuate an injustice, but only to prevent a failure of justice. it.
The writ does not issue as a matter of course. It will issue only where there is a clear legal
right sought to be enforced. It will not issue to enforce a doubtful right. A clear legal right Corporations; Voting Trust Agreement; A voting trust transfers only voting or other
within the meaning of Sec. 3, Rule 65 of the Rules of Court means a right clearly founded in or rights pertaining to the shares subject of the agreement or control over the stock.ln any
granted by law, a right which is enforceable as a matter of law. event, a voting trust transfers only voting or other rights pertaining to the shares subject of
the agreement, or control over the stock. The law on the matter is Section 59, paragraph 1 of
Same; Same; Same; Same; Writ of mandamus will not issue to give the applicant the Corporation Code (BP 68) which provides: "Sec. 59. Voting TrustsOne or more
anything to which he is not entitled by law; Case at bar.Applymg the above-cited principles stockholders of a stock corporation may create a voting trust for the purpose of confering
of law in the present case, the Court fmds no clear right in Batjak to be entitled to the writ upon a trustee or trusties the right to vote and other rights pertaining to the shares for a
prayed for. It should be noted that the petition for mandamus filed by it prayed that NIDC and period not exceeding five (5) years at any one time: x x x"
PNB be ordered to surrender, relinquish and turn-over to Batjak the assets, management, and
operation of Batjak particularly the three (3) oil mills and to make the order permanent, after PETITIONS for certiorari and prohibition with preliminary injunction to review the orders of
trial, and ordering NIDC and PNB to submit a complete accounting of the assets, management the Court of First Instance of Rizal, Br. VIII. Aquino, J.
and operation of Batjak from 1965. In effect, what Batjak seeks to recover is title to, or
possession of, real property (the three (3) oil mills which really made up the assets of Batjak) The facts are stated in the opinion of the Court.
but which the records show already belong to NIDC. It is not disputed that the mortgages on
the three (3) oil mills were foreclosed by PNB and NIDC and acquired by them as the highest Cruz, Palafox, Alfonso and Associates for petitioner NIDC in G.R. No. 34192.
bidder in the appropriate foreclosure sales. Ownership thereto was subsequently The Chief Legal Counsel for petitioner PNB in G.R. No. 34213.
consolidated by PNB and NIDC, after Batjak failed to exercise its right of redemption. The Reyes and Sundiam Law Office for respondent Batjak, Inc.
three (3) oil mills are now titled in the name of NIDC. From the foregoing, it is evident that Duran, Chuanico Oebanda, Benemerito & Associates for private respondents in G.R. Nos. 34192 &
Batjak had no clear right to be entitled to the writ prayed for. In Lamb vs. Philippines (22 Phil. 34213.
456) citing the case of Gonzales V. Salazar vs. The Board ofPharmacy, 20 Phil. 367, the Court Tolentino, Garcia, Cruz & Reyes for movant in G.R. No. L-34192.
said that the writ of mandamus will not issue to give to the applicant anything to which he is
not entitled by law. PADILLA, J.:

Same; Same; Same; Receivership; A receiver ofproperty subject of the action may be These two (2) separate petitions for certiorari and prohibition, with preliminary
appointed by the court when the party applying for the appointment of a receiver has an injunction, seek to annul and set aside the orders of respondent judge, dated 16 August 1971
interest in said property.A receiver of real or personal property, which is the subject of the and 30 September 1971, in Civil Case No. 14452 of the Court of First Instance of Rizal, entitled
action, may be appointed by the court when it appears from the pleadings that the party Batjak Inc. vs. NIDC et al." The order of 16 August 1971 1 granted the alternative petition of
applying for the appointment of receiver has an interest in said property. The right, interest, private respondent Batjak, Inc. Batjak for short) for the appointment of receiver and denied
or claim in property, to entitle one to a receiver over it, must be present and existing. petitioners' motion to dismiss the complaint of said private respondent. The order dated 30
September 1971 2 denied petitioners' motion for reconsideration of the order dated 16 August
Same; Same; Same; Same; Prevention of imminent danger to property, the guiding 1971.
principle that governs courts in appointing receivers.Moreover, the prevention of imminent
danger to property is the guiding principle that governs courts in the matter of appointing The herein petitions likewise seek to prohibit the respondent judge from hearing
receivers. Under Sec. l(b), Rule 59 of the Rules of Court, it is necessary in granting the relief of and/or conducting any further proceedings in Civil Case No. 14452 of said court.
receivership that the property or fund be in danger of loss, removal or material injury. In the
case at bar, Batjak in its petition for receivership, or in its amended petition therefor, failed to Batjak, (Basic Agricultural Traders Jointly Administered Kasamahan) is a Filipino-
present any evidence to establish the requisite condition that the property is in danger of American corporation organized under the laws of the Philippines, primarily engaged in the
manufacture of coconut oil and copra cake for export. In 1965, Batjak's financial condition

234
MATABABE FILES
Corporation Law

deteriorated to the point of bankruptcy. As of that year, Batjak's indebtedness to some 1) That NIDC shall invest P6,722,500.00 in the form of preferred shares of
private banks and to the Philippine National Bank (PNB) amounted to P11,915,000.00, shown stocks at 9% cumulative, participating and convertible within 5 years at par
as follows: into common stocks to liquidate your accounts with the Republic Bank,
Manufacturers Bank & Trust Company and the PCIB which, however, shall be
Republic Bank P 2,324,000.00 applied to the latter three (3) banks accounts with the Loans & Discounts
Philippine Commercial and Dept. NIDC shall match your P 10 million subscription by an additional
Industrial Bank 1,346,000.00 investment of P3,277,500 within a period of one to two years at NIDC's
Manila Banking Corporation 2,000,000.00 option;
Manufacturers Bank 440,000.00
Hongkong and Shanghai 2) That NIDC will guaranty for five (5) years your account with the Manila
Banking Corporation 250,000.00 Banking Corporation;
Foreign Export Advances
(against immediate shipment) 555,000.00 3) That the above banks (Republic Bank, PCIB, MBTC and Manila Banking
PNB export advance line Corp.) shall release in favor of PNB the first and any mortgage they hold on
(against immediate shipment) 5,000,000.00 your properties;
TOTAL 11,915,000.00
4) That you shall exercise (execute) a first mortgage on all your properties
As security for the payment of its obligations and advances against shipments, Batjak located at Sasa, Davao City; Jimenez, Misamis Occidental; and Tanauan,
mortgaged its three (3) coco-processing oil mills in Sasa, Davao City, Jimenez, Misamis Leyte and assign leasehold rights on the property on which your plant at
Occidental and Tanauan, Leyte to Manila Banking Corporation (Manila Bank), Republic Bank Sasa, Davao City is erected in favor of PNB;
(RB), and Philippine Commercial and Industrial Bank (PCIB), respectively. In need for
additional operating capital to place the three (3) coco-processing mills at their optimum 5) That a voting trust agreement for five (5) years over 60% of the
capacity and maximum efficiency and to settle, pay or otherwise liquidate pending financial oustanding paid up and subscribed shares shall be executed by your
obligations with the different private banks, Batjak applied to PNB for additional financial stockholders in favor of NIDC;
assistance. On 5 October 1965, a Financial Agreement was submitted by PNB to Batjak for
acceptance. The Financial Agreement reads: 6) That this accomodation shall be secured by the joint and several
signatures of officers and directors;
PHILIPPINE NATIONAL BANK
Manila, Philippines 7) That the number of the Board of Directors shall be increased to seven (7),
International Department three (3) from your firm and the other four (4) from the PNB-NIDC;
5
BATJAK, INCORPORATED 8) That a comptroller, at your expense, shall be appointed by PNB-NIDC to
3rd Floor, G. Puyat Bldg. supervise the financial management of your firm;
Escolta, Manila
9) That the past due accounts of P 5 million with the International
Attn.: Mr. CIRIACO B. MENDOZA Department of the PNB shall be transferred to the Loans & Discount
Vice-President & General Manager Department and to be treated as a Demand Loan;
Gentlemen:
We are pleased to advise that our Board of Directors 10) That any excess of NIDC investment as required in Condition 1 after
approved for you the following: payment of the obligations to three (3) Banks (RB, MBTC, & PCIB) shall be
applied to reduce the above Demand Loan of P 5 million;

235
MATABABE FILES
Corporation Law

accepting the conditions enumerated above authorizing the above


11) That we shall grant you an export advance of P3 million to be used for transactions and the officer or officers to sign on behalf of the corporation.
copra purchases, subject to the following conditions:
Thank you.
a) That the line shall expire on September 30, 1966 but
revocable at the Bank(s) option;
The terms and conditions of the Financial Agreement were duly accepted by Batjak.
b) That drawings against the line shall be allowed only Under said Agreement, NIDC would, as it actually did, invest P6,722,500.00 in Batjak in the
when an irrevocable export L/C for coconut products has form of preferred shares of stock convertible within five (5) years at par into common stock,
been established or assigned in your favor and you shall to liquidate Batjak's obligations to Republic Bank (RB), Manufacturers Bank and Trust
assign to us all proceeds of negotiations to be received Company (MBTC) and Philippine Commercial & Industrial Bank (PCIB), and the balance of the
from your letters of credit; investment was to be applied to Batjak's past due account of P 5 million with the PNB.

c) That drawings against the line be limited to 60% of the Upon receiving payment, RB, PCIB, and MBTC released in favor of PNB the first and
peso value of the export letters of credit computed at any mortgages they held on the properties of Batjak.
P3.50 per $1.00 but total drawings shall not in any event
exceed P3,000,000.00; As agreed, PNB also granted Batjak an export-advance line of P 3 million, later
d) That release or releases against the line shall be covered increased to P 5million, and a standby letter of credit facility in the amount of P5,850,000.00.
by promissory note or notes for 90 days but not beyond the As of 29 September 1966, the financial accomodation that had been extended by PNB to
expiry dates of the coveting L/C and proceeds of said L/C Batjak amounted to a total of P 14,207,859.51.
shall first be applied to the correspondent drawings on the
line; As likewise agreed, Batjak executed a first mortgage in favor of PNB on all its
properties located at Jimenez, Misamis Occidental and Tanauan, Leyte. Batjak's plant in Sasa,
e) That drawings against the line shall be charged interest Davao City was mortgaged to the Manila Bank which, in 1967, instituted foreclosure
at the rate of 9% per annum and subject to 1/2% penalty proceedings against the same but which were aborted by the payment by Batjak of the sum
charge on all drawings not paid or extended on maturity of P2,400,000.00 to Manila Bank, and which amount was advanced to Batjak by NIDC, a
date; and wholly-owned subsidiary of PNB. To secure the advance, Batjak mortgaged the oil mill in Sasa,
Davao City to NIDC. 4
f) That within 90 days from date of release against the line,
you shall negotiate with us on equivalent amount in export Next, a Voting Trust Agreement was executed on 26 October 1965 in favor of NIDC by
bills, otherwise, the line shag be temporarily suspended the stockholders representing 60% of the outstanding paid-up and subscribed shares of
until the outstanding export advance is fully liquidated. Batjak. This agreement was for a period of five (5) years and, upon its expiration, was to be
subject to negotiation between the parties.
We are writing the National Investment & Development
Corporation, the Republic Bank, the Philippine Commercial & Industrial Bank The voting Trust Agreement reads:
and the Manufacturers Bank & Trust Company and the Manila Banking
Corporation regarding the above. VOTING TRUST AGREEMENT

In connection with the above, kindly submit to us two (2) copies of KNOW ALL MEN BY THESE PRESENTS:
your board resolution certified to under oath by your corporate secretary

236
MATABABE FILES
Corporation Law

This AGREEMENT made and executed by the undersigned CBM FINANCE & INVESTMENT
stockholders of BATJAK, INC., a corporation duly organized and existing CORP. (C.B. Mendoza, Pres.) 5,000 shares
under the laws of the Philippines, whose names are hereinbelow subscribed ALEJANDRO G. BELTRAN 4,000 shares
hereinafter caged the SUBSCRIBERS, and the NATIONAL INVESTMENT AND ESPERANZA A. ZAMORA 3,000 shares
DEVELOPMENT CORPORATION, hereinafter referred to as the trustee. CIRIACO B. MENDOZA 2,000 shares
FIDELA DE GUZMAN 2,000 shares
WITNESSETH: LLOYD D. COMBS 2,000 shares
RENATO B. BEJAR 200 shares
WHEREAS, the SUBSCRIBERS are owners respectively of the capital TOTAL 60,000 shares
stock of the BATJAK, INC. (hereinafter called the CORPORATION) in the
amounts represented by the number of shares set fort opposite their to the TRUSTEE by virtue of the provisions hereof and do hereby
respective names hereunder; authorize the Secretary of the CORPORATION to issue the corresponding
certificate directly in the name of the TRUSTEE and on which certificates it
AND WHEREAS, with a view or establishing a safe and competent shall appear that they have been issued pursuant to this Voting Trust
management to operate the corporation for the best interest of all the Agreement and the said TRUSTEE shall hold in escrow all such certificates
stockholders thereof, and as mutually agreed between the SUBSCRIBERS during the term of the Agreement. In turn, the TRUSTEE shall deliver to the
and the TRUSTEE, this Voting Trust Agreement has been executed under the undersigned stockholders the corresponding Voting Trust certificates
following terms and conditions. provided for in Sec. 36 of Act No. 1459.

NOW THEREFORE, the undersigned stockholders, in consideration 3. VOTING POWER OF TRUSTEE The TRUSTEE and its successors in trust, if
of the premises and of the mutual covenants and agreements herein anym shall have the power and it shall be its duty to vote the shares of the
contained and to carry out the foregoing purposes in order to vest in the undersigned subject hereof and covered by this Agreement at all annual,
TRUSTEE the voting rights of the shares of stock held by the undersigned in adjourned and special meetings of the CORPORATION on all questions,
the CORPORATION as hereinafter stated it is mutually agreed as follows: motions, resolutions and matters including the election of directors and such
matters on which the stockholders, by virtue of the by-laws of the
1. PERIOD OF DESIGNATION For a period of five (5) years from CORPORATION and of the existing legislations are entitled to vote, which
and after date hereof, without power of revocation on the part of the may be voted upon at any and all said meetings and shall also have the
SUBSCRIBERS, the TRUSTEE designated in the manner herein provided is power to execute and acknowledge any agreements or documents that may
hereby made, constituted and appointed as a VOTING TRUSTEE to act for be necessary in its opinion to express the consent or assent of all or any of
and in the name of the SUBSCRIBERS, it being understood, however, that the stockholders of the CORPORATION with respect to any matter or thing
this Voting Trust Agreement shall, upon its expiration be subject to a re- to which any consent or assent of the stockholders may be necessary,
negotiation between the parties, as may be warranted by the balance and proper or convenient.
attending circumstance of the loan investment of the TRUSTEE or otherwise
in the CORPORATION. 4. FILING of AGREEMENT An executed copy of this Agreement shall be
filed with the CORPORATION at its office in the City of Manila wherever it
2. ASSIGNMENT OF STOCK CERTIFICATES UPON ISSUANCE The may be transfered therefrom and shall constitute irrevocable authority and
undersigned stockholders hereby transfer and assign their common shares absolute direction of the officers of the CORPORATION whose duty is to sign
to the capital stock of the CORPORATION to the extent shown hereunder: and deliver stock certificates to make delivery only to said voting trustee of
the shares and certificates of stock subject to the provisions of this
JAMES A. KEISTER 21,500 shares Agreement as aforesaid. Such copy of this Agreement shall at all times be
JOHNNY LIEUSON 20,300 shares open to inspection by any stockholder, as provided by law.

237
MATABABE FILES
Corporation Law

9. TERMINATION Upon termination of this Agreement as heretofore


5. DIVIDEND the full and absolute beneficial interest in the shares subject provided, the certificates delivered to the TRUSTEE by virtue hereof shall be
of this Agreement shall remain with the stockholders executing the same returned and delivered to the undersigned stockholders as the absolute
and any all dividends which may be declared by the CORPORATION shall owners thereof, upon surrender of their respective voting trust certificates,
belong and be paid to them exclusively in accordance with their and the duties of the TRUSTEE shall cease and terminate.
stockholdings after deducting therefrom or applying the same to whatever
liabilities the stockholders may have in favor of the TRUSTEE by virtue of any 10. ACCEPTANCE OF TRUST The TRUSTEE hereby accepts the trust
Agreement or Contract that may have been or will be executed by and created by this Agreement under the signature of its duly authorized
between the TRUSTEE and the CORPORATION or between the former and representative affixed hereinbelow and agrees to perform the same in
the undersigned stockholders. accordance with the term/s hereof.
IN WITNTESS HEREOF, the undersigned stockholders and the TRUSTEE by its
6 COMPENSATION; IMMUNITY The TRUSTEE or its successor in trust shall representatives, have hereunto affixed their signatures this 26 day of
not receive any compensation for its serviceexcept perhaps that which the October, 1965 in the City of Manila, Philippines.
CORPORATION may grant to the TRUSTEE's authorized representative, if
any. Expenses costs, champs, and other liabilities incurred in the carrying out (SGD) JAMES A. KEISER (SGD) JOHNNY LIEUSON
of the but herein established or by reason thereof, shall be paid for with the Stockholder Stockholder
funds of the CORPORATION. The TRUSTEE or any of its duly authorized CBM FINANCE & INVESTMENT CORPORATION
representative shall incur no liability by reason of any error of law or of any
matter or thing done or omitted under this Agreement, except for his own By: (SGD) C.B. MENDOZA
individual malfeasance. President

7. REPRESENTATION The TRUSTEE, being a corporation and a juridical ESPERANZA A. ZAMORA (SGD) ALEJANDRO G. BELTRAN
person shall accomplish the foregoing objectives and perform its functions By: (SGD) MARIANO ZAMORA Stockholder
under this Agreement as well as enjoy and exercise the powers, privileges, ESPERANZA A. ZAMORA
rights and interests herein established through its duly authorized and
accredited re resentatives . p with full authority under the specific (SGD) FIDELA DE GUZMAN (SGD) CIRIACO B. MENDOZA
appointment or designation or Proxy. Stockholder Stockholder
(SGD) RENATO B. BEJAR (SGD) LLOYD D. COMBS
8. IRREVOCABILITY This Agreement shall during its 5-year term or any Stockholder Stockholder
extension thereof be binding upon and inure to the benefit of the
undersigned stockholders and their respective legal representatives, INVESTMENT AND
pledges, transferees, and/or assigns and shall be irrevocable during the said DEVELOPMENT CORPORATION
terms and/or its extension pursuant to the provisions of paragraph 1 hereof. By:
It is hereby understood and the undersigned stockholders have bound as (SGD) IGNACIO DEBUQUE JRVice-President 5
they hereby bind themselves to make a condition of every pledge, transfer
of assignment of their interests in the CORPORATION that the interests and In July 1967, forced by the insolvency of Batjak, PNB instituted extrajudicial
participation so pledged, transferred or assigned is evidenced by foreclosure proceedings against the oil mills of Batjak located in Tanauan, Leyte and Jimenez,
annotations in the certificates of stocks or in the books of the corporation, Misamis Occidental. The properties were sold to PNB as the highest bidder. One year
shall be subject to this Agreement and the same shall be binding upon the thereafter, or in September 1968, final Certificates of Sale were issued by the provincial
pledgees, transferees and assigns while the trust herein created still subsists. sheriffs of Leyte 6 and Misamis Occidental 7 for the two (2) oil mills in Tanauan and Jimenez in
favor of PNB, after Batjak failed to exercise its right to redeem the foreclosed properties

238
MATABABE FILES
Corporation Law

within the allowable one year period of redemption. Subsequently, PNB transferred the On 24 April 1971, NIDC and PNB filed an opposition to the ex parte application for the
ownership of the two (2) oil mills to NIDC which, as aforestated, was a wholly-owned PNB issuance of a writ of preliminary prohibitory and mandatory injunction and a motion to set
subsidiary. aside restraining order.

As regards the oil mill located at Sasa, Davao City, the same was similarly foreclosed Before the court could act on the said motion, private respondent Batjak filed on 3
extrajudicial by NIDC. It was sold to NIDC as the highest bidder. After Batjak failed to redeem May 1971 a petition for receivership as alternative to writ of preliminary prohibitory and
the property, NIDC consolidated its ownership of the oil mill. 8 mandatory injunction. 16 This was opposed by PNB and NIDC . 17

Three (3) years thereafter, or on 31 August 1970, Batjak represented by majority On 8 May 1971., NIDC and PNB filed a motion to dismiss Batjak's complaints. 18
stockholders, through Atty. Amado Duran, legal counsel of private respondent Batjak, wrote a
letter to NIDC inquiring if the latter was still interested in negotiating the renewal of the On 16 August 1971, respondent judge issued the now assailed order denying
Voting Trust Agreement. 9 On 22 September 1970, legal counsel of Batjak wrote another letter petitioners' motion to dismiss and appointing a set of three (3) receivers. 19 NIDC moved for
to NIDC informing the latter that Batjak would now safely assume that NIDC was no longer reconsideration of the aforesaid order. 20 On 30 September 1971, respondent judge denied the
interested in the renewal of said Voting Trust Agreement and, in view thereof, requested for motion for reconsideration. 21
the turn-over and transfer of all Batjak assets, properties, management and operations. 10
Hence, these two (2) petitions, which have been consolidated, as they involve a
On 23 September 1970, legal counsel of Batjak sent stin another letter to NIDC, this resolution of the same issues. In their manifestation with motion for early decision, dated 25
time asking for a complete accounting of the assets, properties, management and operation August 1986, private respondent, Batjak contends that the NIDC has already been abolished
of Batjak, preparatory to their turn-over and transfer to the stockholders of Batjak. 11 or scrapped by its parent company, the PNB.

NIDC replied, confirming the fact that it had no intention whatsoever to comply with After a careful study and examination of the records of the case, the Court finds and
the demands of Batjak. 12 holds for the petitioners.

On 24 February 1971, Batjak filed before the Court of First Instance of Rizal a special 1. On the denial of petitioners' motion to dismiss.
civil action for mandamus with preliminary injunction against herein petitioners docketed as
Civil Case No. 14452. 13 As a general rule, an order denying a motion to quash or to dismiss is interlocutory
and cannot be the subject of a petition for certiorari. The remedy of the aggrieved party in a
On 14 April 1971, in said Civil Case No. 14452, Batjak filed an urgent ex parte motion for denied motion to dismiss is to file an answer and interpose, as defense or defenses, the
the issuance of a writ of preliminary prohibitory and mandatory injunction. 14 On the same day, objection or objections raised by him in said motion to dismiss, then proceed to trial and, in
respondent judge issued a restraining order "prohibiting defendants (herein petitioners) from case of adverse decision, to elevate the entire case by appeal in due course. However, under
removing any record, books, commercial papers or cash, and leasing, renting out, disposing of certain situations, recourse to the extraordinary legal remedies of certiorari, prohibition and
or otherwise transferring any or all of the properties, machineries, raw materials and finished mandamus to question the denial of a motion to dismiss or quash is considered proper, in the
products and/or by-products thereof now in the factory sites of the three (3) modem coco interest of more enlightened and substantial justice. As the court said in Pineda and Ampil
milling plants situated in Jimenez, Misamis Occidental, Sasa, Davao City, and Tanauan, Manufacturing Co. vs. Bartolome, 95 Phil. 930,938
Leyte." 15 For analogous reasons it may be said that the petition for certiorari
interposed by the accused against the order of the court a quo denying the
The order of 14 April 1971 was subsequently amended by respondent judge upon motion to quash may be entertained, not only because it was rendered in a
an ex parte motion of private respondent Batjak so as to include the premises of NIDC in criminal case, but because it was rendered, as claimed, with grave abuse of
Makati and those of PNB in Manila, as among the premises which private respondent Batjak discretion, as found by the Court of Appeals. ..
was authorized to enter in order to conduct an inventory.
and reiterated in Mead v. Argel 22 citing Yap v. Lutero (105 Phil. 1307):

239
MATABABE FILES
Corporation Law

However, were we to require adherence to this pretense, the case at bar


would have to be dismissed and petitioner required to go through the In support of the third ground of their motion to dismiss, PNB and NIDC contend that
inconvenience, not to say the mental agony and torture, of submitting Batjak's complaint for mandamus is based on its claim or right to recovery of possession of
himself to trial on the merits in Case No. 166443, apart from the expenses the three (3) oil mills, on the ground of an alleged breach of fiduciary relationship.
incidental thereto, despite the fact that his trial and conviction therein would Noteworthy is the fact that, in the Voting Trust Agreement, the parties thereto were NIDC
violate one of this [sic] constitutional rights, and that, an appeal to this and certain stockholders of Batjak. Batjak itself was not a signatory thereto. Under Sec. 2,
Court, we would, therefore, have to set aside the judgment of conviction of Rule 3 of the Rules of Court, every action must be prosecuted and defended in the name of
the lower court. This would, obviously, be most unfair and unjust. Under the the real party in interest. Applying the rule in the present case, the action should have been
circumstances obtaining the present case, the flaw in the procedure filed by the stockholders of Batjak, who executed the Voting Trust Agreement with NIDC, and
followed by petitioner herein may be overlooked, in the interest of a more not by Batjak itself which is not a party to said agreement, and therefore, not the real party in
enlightened and substantial justice. interest in the suit to enforce the same.

Thus, where there is patent grave abuse of discretion, in denying the motion to In addition, PNB claims that Batjak has no cause of action and prays that the petition
dismiss, as in the present case, this Court may entertain the petition for certiorari interposed for mandamus be dismissed. A careful reading of the Voting Trust Agreement shows that PNB
by the party against whom the said order is issued. was really not a party thereto. Hence, mandamus will not lie against PNB.

In their motion to dismiss Batjaks complaint, in Civil Case No. 14452, NIDC and PNB Moreover, the action instituted by Batjak before the respondent court was a special
raised common grounds for its allowance, to wit: civil action for mandamus with prayer for preliminary mandatory injunction. Generally,
mandamus is not a writ of right and its allowance or refusal is a matter of discretion to be
1. This Honorable Court (the trial court) has no jurisdiction over the subject exercised on equitable principles and in accordance with well-settled rules of law, and that it
of the action or suit; should never be used to effectuate an injustice, but only to prevent a failure of justice. 24 The
2. The venue is improperly laid; and writ does not issue as a matter of course. It will issue only where there is a clear legal right
3. Plaintiff has no legal capacity to sue. sought to be enforced. It will not issue to enforce a doubtful right. A clear legal right within
the meaning of Sec. 3, Rule 65 of the Rules of Court means a right clearly founded in or
In addition, PNB contended that the complaint states no cause of action (Rule 16, granted by law, a right which is enforceable as a matter of law.
Sec. 1, Par. a, c, d & g, Rules of Court).
Anent the first ground, it is a well-settled rule that the jurisdiction of a Court of First Applying the above-cited principles of law in the present case, the Court finds no
Instance to issue a writ of preliminary or permanent injunction is confined within the clear right in Batjak to be entitled to the writ prayed for. It should be noted that the petition
boundaries of the province where the land in controversy is situated. 23 The petition for for mandamus filed by it prayed that NIDC and PNB be ordered to surrender, relinquish and
mandamus of Batjak prayed that NIDC and PNB be ordered to surrender, relinquish and turn-over to Batjak the assets, management, and operation of Batjak particularly the three (3)
turnover to Batjak the assets, management and operation of Batjak particularly the three (3) oil mills and to make the order permanent, after trial, and ordering NIDC and PNB to submit a
oil mills located in Sasa, Davao City, Jimenez, Misamis Occidental and Tanauan, Leyte. complete accounting of the assets, management and operation of Batjak from 1965. In effect,
what Batjak seeks to recover is title to, or possession of, real property (the three (3) oil mills
Clearly, what Batjak asked of respondent court was the exercise of power or which really made up the assets of Batjak) but which the records show already belong to
authority outside its jurisdiction. NIDC. It is not disputed that the mortgages on the three (3) oil mills were foreclosed by PNB
and NIDC and acquired by them as the highest bidder in the appropriate foreclosure sales.
On the matter of proper venue, Batjak's complaint should have been filed in the Ownership thereto was subsequently consolidated by PNB and NIDC, after Batjak failed to
provinces where said oil mills are located. Under Rule 4, Sec. 2, paragraph A of the Rules of exercise its right of redemption. The three (3) oil mills are now titled in the name of NIDC.
Court, "actions affecting title to, or for recovery of possession, or for partition or From the foregoing, it is evident that Batjak had no clear right to be entitled to the writ
condemnation of, or foreclosure of mortgage on, real property, shall be commenced and prayed for. In Lamb vs. Philippines (22 Phil. 456) citing the case of Gonzales V. Salazar vs. The
tried in the province where the property or any part thereof lies."

240
MATABABE FILES
Corporation Law

Board of Pharmacy, 20 Phil. 367, the Court said that the writ of mandamus will not issue to the parties, as may be warranted by the balance and attending circumstance
give to the applicant anything to which he is not entitled by law. of the loan investment of the TRUSTEE or otherwise in the CORPORATION.
and No. 3 thereof reads:
2. On the appointment of receiver.
3. VOTING POWER OF TRUSTEE The TRUSTEE and its successors
A receiver of real or personal property, which is the subject of the action, may be in trust, if any, shall have the power and it shall be its duty to vote the shares
appointed by the court when it appears from the pleadings that the party applying for the of the undersigned subject hereof and covered by this Agreement at all
appointment of receiver has an interest in said property. 25 The right, interest, or claim in annual, adjourned and special meetings of the CORPORATION on all
property, to entitle one to a receiver over it, must be present and existing. questions, motions, resolutions and matters including the election of
directors and all such matters on which the stockholders, by virtue of the by-
As borne out by the records of the case, PNB acquired ownership of two (2) of the laws of the CORPORATION and of the existing legislations are entitled to
three (3) oil mills by virtue of mortgage foreclosure sales. NIDC acquired ownership of the vote, which may be voted upon at any and all said meetings and shall also
third oil mill also under a mortgage foreclosure sale. Certificates of title were issued to PNB have the power to execute and acknowledge any agreements or documents
and NIDC after the lapse of the one (1) year redemption period. Subsequently, PNB that may be necessary in its opinion to express the consent or assent of all or
transferred the ownership of the two (2) oil mills to NIDC. There can be no doubt, therefore, any of the stockholders of the CORPORATION with respect to any matter or
that NIDC not only has possession of, but also title to the three (3) oil mills formerly owned by thing to which any consent or assent of the stockholders may be necessary,
Batjak. The interest of Batjak over the three (3) oil mills ceased upon the issuance of the proper or convenient.
certificates of title to PNB and NIDC confirming their ownership over the said properties.
More so, where Batjak does not impugn the validity of the foreclosure proceedings. Neither From the foregoing provisions, it is clear that what was assigned to NIDC was the
Batjak nor its stockholders have instituted any legal proceedings to annul the mortgage power to vote the shares of stock of the stockholders of Batjak, representing 60% of Batjak's
foreclosure aforementioned. outstanding shares, and who are the signatories to the agreement. The power entrusted to
NIDC also included the authority to execute any agreement or document that may be
Batjak premises its right to the possession of the three (3) off mills on the Voting necessary to express the consent or assent to any matter, by the stockholders. Nowhere in
Trust Agreement, claiming that under said agreement, NIDC was constituted as trustee of the the said provisions or in any other part of the Voting Trust Agreement is mention made of any
assets, management and operations of Batjak, that due to the expiration of the Voting Trust transfer or assignment to NIDC of Batjak's assets, operations, and management. NIDC was
Agreement, on 26 October 1970, NIDC should tum over the assets of the three (3) oil mills to constituted as trustee only of the voting rights of 60% of the paid-up and outstanding shares
Batjak. The relevant provisions of the Voting Trust Agreement, particularly paragraph 4 & No. of stock in Batjak. This is confirmed by paragraph No. 9 of the Voting Trust Agreement, thus:
1 thereof, are hereby reproduced:
9. TERMINATION Upon termination of this Agreement as heretofore
NOW THEREFORE, the undersigned stockholders, in consideration of the provided, the certificates delivered to the TRUSTEE by virtue hereof shall be
premises and of the mutual covenants and agreements herein contained and returned and delivered to the undersigned stockholders as the absolute
to carry out the foregoing purposes in order to vest in the TRUSTEE the owners thereof, upon surrender of their respective voting trust certificates,
voting right.8 of the shares of stock held by the undersigned in the and the duties of the TRUSTEE shall cease and terminate.-
CORPORATION as hereinafter stated it is mutually agreed as follows:
Under the aforecited provision, what was to be returned by NIDC as trustee to
1. PERIOD OF DESIGNATION For a period of five (5) years from and after Batjak's stockholders, upon the termination of the agreement, are the certificates of shares of
date hereof, without power of revocation on the part of the SUBSCRIBERS, stock belonging to Batjak's stockholders, not the properties or assets of Batjak itself which
the TRUSTEE designated in the manner herein provided is hereby made, were never delivered, in the first place to NIDC, under the terms of said Voting Trust
constituted and appointed as a VOTING TRUSTEE to act for and in the name Agreement.
of the SUBSCRIBERS, it being understood, however, that this Voting Trust
Agreement shall, upon its expiration be subject to a re-negotiation between

241
MATABABE FILES
Corporation Law

In any event, a voting trust transfers only voting or other rights pertaining to the
shares subject of the agreement or control over the stock. The law on the matter is Section
59, Paragraph 1 of the Corporation Code (BP 68) which provides:

Sec. 59. Voting Trusts One or more stockholders of a stock corporation


may create a voting trust for the purpose of confering upon a trustee or
trusties the right to vote and other rights pertaining to the shares for a
period not exceeding five (5) years at any one time: ... 26

The acquisition by PNB-NIDC of the properties in question was not made or effected
under the capacity of a trustee but as a foreclosing creditor for the purpose of recovering on
a just and valid obligation of Batjak.

Moreover, the prevention of imminent danger to property is the guiding principle


that governs courts in the matter of appointing receivers. Under Sec. 1 (b), Rule 59 of the
Rules of Court, it is necessary in granting the relief of receivership that the property or fired
be in danger of loss, removal or material injury.

In the case at bar, Batjak in its petition for receivership, or in its amended petition
therefor, failed to present any evidence, to establish the requisite condition that the property
is in danger of being lost, removed or materially injured unless a receiver is appointed to
guard and preserve it.
WHEREFORE, the petitions are GRANTED. The orders of the respondent judge, dated
16 August 1971 and 30 September 1971, are hereby ANNULLED and SET ASIDE. The respondent
judge and/or his successors are ordered to desist from hearing and/or conducting any further
proceedings in Civil Case No. 14452, except to dismiss the same. With costs against private
respondent.

242
MATABABE FILES
Corporation Law

Republic of the Philippines previous agreement to dissolve the corporation does not affect or render illegal the said
SUPREME COURT order issued by the court.
Manila
EN BANC ORIGINAL ACTION in the Supreme Court. Certiorari.

G.R. No. L-5883 November 28, 1953 The facts are stated in the opinion of the Court. Ponce vs. Encarnacion etc., and Gapol, 94
Phil., 81, No. L-5883 November 28, 1953
DOMINGO PONCE AND BUHAY L. PONCE, petitioners,
vs. Marcelino Lontok for petitioners.
DEMETRIO B. ENCARNACION, Judge of the Court of First Instance of Manila, Branch I, and Zavalla, Bautista and Nuevas for respondents.
POTENCIANO GAPOL, respondents.
PADILLA, J.:
1. CORPORATION LAW; STOCKHOLDERS' MEETING TO ELECT A NEW BOARD OF
DlRECTORSJ CALL OF MEETING BY A STOCKHOLDER ON COURT'S AUTHORITY.Under and This is a petition for a writ of certiorari to annul an order of the respondent court
pursuant to section 26 of Act No. 1459, on the showing of good cause therefor the court may granting Potenciano Gapol authority, pursuant to section 26, Act No. 1459, otherwise known
authorize a stockholder to call a meeting and to preside thereat until the majority as the Corporation Law, to call a meeting of the stockholders of the Dagunoy Enterprises, Inc.
stockholders representing a majority of the stockholders present and permitted to be voted and to preside at such meeting by giving proper notice to the stockholders, as required by law
shall have chosen one among them to preside it. And this showing of good cause therefor or by laws of the corporation, until after the majority of the stockholders present and
exists when the court is apprised of the fact that the by-laws of the corporation require the qualified to vote shall have chosen one of them to act as presiding officer of the meeting;
calling of a general meeting of 'the stockholders to elect the board of directors but the call for another order denying a motion of the petitioners to have the previous order set aside; and a
such meeting has not been done. third order denying a motion to the same effect as the one previously filed.

2. ID.; ID.; ID.; PETITION FOR SUCH PURPOSE NEED NOT BE SET FOR HEARING.The The petitioners aver that the Daguhoy Enterprises, Inc., was duly registered as such
requirement that "on the showing of good cause therefor," the court may grant to a on 24 June 1948; that on 16 April 1951 at a meeting duly called, the voluntary dissolution of the
stockholder the authority to call such meeting and to preside thereat does not mean that the corporation and the appointment of Potenciano Gapol as receiver were agreed upon and to
petition for such authority must be set for hearing with notice served upon the board of that end a petitioner Domingo Ponce; that instead of filing the petition for voluntary
directors. It may be likened to a writ of preliminary injunction or of attachment which may be dissolution of the of the corporation as agreed upon, the respondent Potenciano Gapol, who
issued ex-parte upon compliance with the requirements of the rules and upon the court being is the largest stockholder, charged his mind and filed a complaint in the Court of First Instance
satisfied that the same should issue. Such provisional reliefs have not been deemed and held of Manila (civil No. 13753) to compel the petitioners to render an accounting of the funds and
as violative of the due process of law clause of the Constitution. assets of the corporation, to reimburse it, jointly and severally, in the sum of P4,500, the
purchase price of a parcel of land acquired by the corporation; P6,190 loaned to the wife of
3. ID.; ID.; ID.; "QUO WARRANTO" TO QUESTION AN ILLEGALITY IN THE ELECTION OF A petitioner Domingo Ponce; and P8,000 spent by the latter in his trip to the United States, or a
MEMBER OF THE BOARD OF DIRECTORS.The alleged illegality of the election of one member total sum of P18,690, plus interest, or such sum as may be found after the accounting shall
of the board of directors at the meeting called as authorized by the court being subsequent have been rendered to have been misspent, misapplied, missappropriated and converted by
to the order complained of cannot affect the validity and legality of the order. If it be true that the petitioner Domingo Ponce to his own use and benefit; that on 18 May 1951 the plaintiff in
one of the directors elected at such meeting was not qualified in accordance with the that case, the respondent Potenciano Gapol in this case, filed a motion praying that the
provisions of the by-laws, the remedy of an aggrieved party would be quo warranto. petitioners be removed as members of the board of directors which was denied by the court;
that on 3 January 1952 respondent Potenciano Gapol filed a petition (civil No. 15445, Exhibit
4.ID.; ID.; ID.; AGREEMENT TO DISSOLVE CORPORATION, IS NO HINDRANCE TO THE L), praying for an order directing him to a call a meeting of the stockholders of the
COURT'S POWER TO AUTHORIZE STOCKHOLDER TO CALL SUCH MEETING.An alleged corporation and to preside at such meeting in accordance with section 26 of the Corporation
law; that two days later, without notice to the petitioners and to the other members of the

243
MATABABE FILES
Corporation Law

board of directors and in violation of the Rules of Court which require that the adverse parties
be notified of the hearing of the motion three days in advance, the respondent court issued The Board of Directors shall compose of five (5) members who shall be elected by the
the order as prayed for (Exhibit M); that the petitioners learned only of this order of the court stockholders in a general meeting called for that purpose which shall be held every
on 27 February, when the Bank of America refused to recognize the new board of directors even year during the month of January.
elected at such meeting and returned the checks drawn upon it by the said board of directors;
that the election of Juanito R. Tianzon as member of the board of directors of the corporation Article 20 of the by-laws in part provides:
he must be a member of the Legionarios del Trabajo, as required and provided for in article 7
of the by-laws of the corporation; that on 5 March the petitioners filed a petition in the . . . Regular general meetings are those which shall be called for every even year . . . .
respondent court to have the order of 5 January set aside but on April, the date set for the
hearing of the petition, as the respondent judge was on leave vacation judge directed its The requirement that "on the showing of good cause therefor," the court may grant
transfer to the branch of the respondent judge; that without having set the motion for to a stockholder the authority to call such meeting and to preside thereat does not mean that
hearing, the respondent court denied the motion of 5 March in its order of 7 May; that on 14 the petition must be set for hearing with notice served upon the board of directors. The
May the petitioners filed another motion inviting the attention of the respondent court to the respondent court was satisfied that there was a showing of good cause for authorizing the
irregularity and illegality of its procedure and setting the motion for hearing on 21 May, but respondent Potenciano Gapol to call a meeting of the stockholders for the purpose of
the court denied the motion by its order of 13 June. electing the board of directors as required and provided for in the by-laws, because the
chairman of the board of directors called upon to do so had failed, neglected, or refused to
The only question to determine in this case is whether under and pursuant to section perform his duty. It may be likened to a writ of preliminary injunction or of attachment which
26 of Act No. 1459, known as the Corporation law, the respondent court may issue the order may be issued ex-parte upon compliance with the requirements of the rules and upon the
complained of. Said section provides: court being satisfied that the same should be issue. Such provisional reliefs have not been
deemed and held as violative of the due process of law clause of the Constitution.
Whenever, from any cause, there is no person authorized to call a meeting,
or when the officer authorized to do so refuses, fails or neglects to call a meeting, In several state of the Union1 the remedy which may be availed of our resorted to in a
any judge of a Court of First Instance on the showing of good cause therefor, may situation such as the one brought about in this case is mandamus to compel the officer or
issue an order to any stockholder or member of a corporation, directing him to call a incumbent board of directors to perform a duties specifically enjoined by law or by-laws, to
meeting of the corporation by giving the proper notice required by this Act or by- wit: to call a meeting of the stockholders. Dela ware is the estate that has a law similar to ours
laws; and if there be no person legally authorized to preside at such meeting, the and there the chancellor of a chancery court may summarily issue or enter an order
judge of the Court of First Instance may direct the person calling the meeting to authorizing a stockholder to call a meeting of the stockholders of the corporation and preside
preside at the same until a majority of the members or stockholders representing a thereat.2 It means that the chancellor may issue such order without notice and hearing.
majority of the stock members or stockholders presenting a majority of the stock
present and permitted by law to be voted have chosen one of their number to act as That the relief granted by the respondent court lies within its jurisdiction is not
presiding officer for the purposes of the meeting. disputed. Having the authority to grant the relief, the respondent court did not exceed its
jurisdiction; nor did it abuse its discretion in granting it.
On the showing of good cause therefor, the court may authorize a stockholder to call
a meeting and to preside threat until the majority stockholders representing a majority With persistency petitioners claim that they have been deprived of their right without
strockholders representing a majority of the stock present and permitted to be voted shall due process of law. They had no right to continue as directors of the corporation unless
have chosen one among them to preside it. And this showing of good cause therefor exists reflected by the stockholders in a meeting called for that purpose every even year. They had
when the court is apprised of the fact that the by-laws of the corporation require the calling no right to a hold-over brought about by the failure to perform the duty incumbent upon one
of a general meeting of the stockholders to elect the board of directors but call for such of them. If they felt that they were sure to be reelected, why did they fail, neglect, or refuse
meeting has not been done. to call the meeting to elect the members of the board? Or, why did they not seek their
reelection at the meeting called to elect the directors pursuant to the order of the respondent
Article 9 of the by-laws of the Daguhoy Enterprises, Inc., provides: court.

244
MATABABE FILES
Corporation Law

The alleged illegality of the election of one member of the board of directors at the
meeting called by the respondent Potenciano Gapol as authorized by the court being
subsequent to the order complained of cannot affect the validity and legality of the order. If it
be true that one of the directors elected at the meting called by the respondent Potenciano
Gapol, as authorized by the order of the court complained of, was not qualified in accordance
with the provisions of the by-laws, the remedy of an aggrieved party would be quo a
warranto. Also, the alleged previous agreement to dissolve the corporation does not affect or
render illegal the order issued by the respondent court.

The petition is denied, with costs against the petitioners.

Paras, C.J., Pablo, Bengzon, Tuason, Montemayor, Reyes, Jugo, Bautista Angelo and Labrador,
JJ., concur.

245

Вам также может понравиться